RESPIRATORY LJ FINAL

अब Quizwiz के साथ अपने होमवर्क और परीक्षाओं को एस करें!

A 48-year-old patient has just been orally intubated prior to being placed on volume-controlled ventilation. The respiratory therapist should do which of the following to best confirm proper placement of the endotracheal tube?

A. examine tube markings near the lips B. obtain a chest radiograph C. assess chest rise symmetry D. auscultate breath sounds B. Although there are many ways to determine the location of an endotracheal tube inside the patient's airway, there is only one way to confirm its location - a chest x-ray or radiograph. Other methods include auscultation of breath sounds and observation of chest rise and symmetry. While these methods are quick and helpful they do not provide CONFIRMING evidence of the ET tube's location.

Intermittent breaths by mouthpiece with a pressure-cycled ventilator are being administered to a patient who is post bariatric surgery. The respiratory therapist notices the ventilator cycles into exhalation immediately after the inhalation phase begins, causing a rapid cycle of the machine. Which of the following could be the cause?

A. excessive inspiratory pressure settings B. occlusion of the mouthpiece C. faulty air-entrainment valve D. inadequate flow rate B.

An adult patient with a history congestive heart failure and who was involved in a motor vehicle accident is in the intensive care unit receiving mechanical ventilatory support. The following data is available: CVP 4 cm H2O mPAP 12 mm Hg PCWP 5 mm Hg PvO2 44 torr Urine output 5 mL/hr The respiratory therapist should conclude the patient is

A. exhibiting reduced tissue oxygen consumption B. hypovolemic C. in cardiogenic shock D. experiencing fluid overload B. Examination of this hemodynamic data reveals that the central venous pressure, mean pulmonary artery pressure, pulmonary capillary wedge pressure are all excessively low. This is most consistent with dehydration or hypovolemia. Additionally, urine output supports this supposition.

Which of the following would be used to lengthen the expiratory time and alter the I:E ratio for a patient receiving PEP therapy?

A. expiratory resistance valve B. injector line C. tidal volume control D. pressure control knob A. To answer this question you have to be familiar with a PEP therapy device. To lengthen the expiratory time on this device one should alter the expiratory resistance valve.

Administration of a 60/40% helium-oxygen mixture requires which of the following masks?

A. face tent B. air-entrainment C. nonrebreather D. aerosol C. Because helium oxygen therapy consists of administration of predetermined mixtures of gases, additional air-entrainment is not desirable. Therefore, use of a nonrebreather mask is most appropriate.

A patient in the emergency room demonstrates intercostal use and sternal retractions. Which of the following most likely represents the patient's condition?

A. flail chest B. ARDS C. upper airway obstruction D. carbon monoxide poisoning C. Intercostal use and retractions is associated with an obstruction in the upper airway.

Which of the following requires a quality control assessment?

A. flexible endotracheal tube B. closed oxygen flowmeter C. point-of-care blood gas analyzer D. air-oxygen blender C. Of the options listed, a point-of-care blood gas analyzer requires routine quality control assessment. This is because a blood gas analyzer is used to make critical decisions regarding the care of the patient and is considered a definitive source of information upon which care decisions are made.

Excessive bubbling is noted in water seal chamber of a chest tube drainage system. Which of the following could be the cause?

A. fluid level in the suction control chamber is too low B. water seal fluid level is too high C. water seal fluid level is too low D. leak in the tubing between the patient and fluid collection chamber D. Excessive bubbling in the water-seal chamber of a chest tubes drainage system is caused from a leak somewhere between the water-seal compartment and the patient's lung. In this case a leak in the tubing between the patient and fluid collection chamber would cause the excessive bubbling.

A patient receiving oxygen at 1 L/min via transtracheal oxygen catheter complains of air hunger and expresses it feels like he is not getting enough oxygen. The therapist should FIRST

A. flush the catheter with saline B. place the patient on a nasal cannula at 2 L/min C. remove the catheter D. increase the flow to the catheter B. When a patient complains of not receiving enough oxygen, before troubleshooting the situation, the patient should be placed on a different oxygen modality to ensure delivery of oxygen. Then, the trouble may be examined and repaired. In this case, a patient is receiving oxygen by transtracheal oxygen catheter at 1 L/min. When switching to a nasal cannula the appropriate amount of oxygen is 2 L/min.

A respiratory supervisor is calculating appropriate staffing levels for a shift. Which of the following should be included in calculation?

A. frequency of refused therapy B. patient areas and locations C. patient medical history D. staff member preferences B. To plan for patient care, staffing must be appropriate. Conditions that affect the amount of staff needed include location of patients (how far staff must travel to get to patients), frequency of ordered therapy (how frequently staff must visit the patient), and the type of therapy (how long the therapy will take). The patient's medical history will not likely affect staffing.

Which of the following will attribute most to an increase in nosocomial infection?

A. frequent suctioning with an inline suction catheter B. changing a small volume nebulizer every 24 hours C. use of an HME D. frequent disconnection of a ventilator circuit D. A nosocomial infection is an infection that spread from patient to patient in a hospital setting. For a patient on a ventilator, a significant cause of nosocomial infection comes from disconnection of the ventilator circuit. Although this must be done from time to time, it should be minimized. Use of an HME does not increase the incidence of infection.

A respiratory therapist is called to stand by at the delivery of a newborn that has experienced cardiac decelerations during labor. After the baby is delivered and presented to the warmer, the respiratory therapist should assess

A. gestational age B. age of the mother C. APGAR D. radiograph C. APGAR (appearance, pulse, grimace, activity, and respiratory effort) is done on every newborn after 1-minute and after 5-minutes of life.

An oxygen-dependent patient ordered for 2 L/min is planning a trip that will last 24 hours. The plane will allow up to 4 extra E-cylinders per passenger. What should the respiratory therapist recommend?

A. go on the trip with 4 extra E-cylinders B. cancel the trip C. pulse-dose unit D. reduce flow to 1.5 L/min C. At 2 L per minute a 24-hour trip will require at least 5 E cylinders to complete. Of the options offered use of a pulse-dose unit is the only option that will conserve oxygen enough to allow the patient to take only four tanks. Reducing the flow will also accomplish this but is considered a deviation from physician orders and is not clinically appropriate for the patient.

Which of the following equations could be used to determine cardiac output?

A. heart rate x ejection fraction B. QT/BSA C. CaO2 - CvO2 D. stroke volume x heart rate D. Cardiac output may be calculated by multiplying the heart rate by the stroke volume. It may also be determined through an equation called the Fick equation.

Several hours after oral endotracheal extubation of an 8.0-mm tube, an adult patient begins to demonstrate inspiratory stridor. Which of the following would be most helpful to the patient?

A. heated aerosol by mask B. Atropine sulfate C. racemic epinephrine D. Albuterol C. When a patient is extubated and has swelling of the upper airway tissues as demonstrated by stridor, racemic epinephrine is an appropriate medication to reduce swelling quickly. Another, less effective option, is to provide a cool mist. In extreme cases, such as marked stridor, reintubation or an emergency tracheotomy may be indicated.

Following a difficult intubation and 24 hours of mechanical ventilation, an oxygen- dependent female patient who is recovering from a drug overdose is unable to speak. Bronchoscopy reveals no movement of her vocal cords. In order to treat the patient's hypoxemia, the respiratory therapist should conside

A. heliox therapy. B. tracheostomy. C. heated aerosol. D. venturi mask. A. Helium is a light gas that has the ability to bypass upper-airway obstructions, and therefore when mixed with oxygen helps deliver oxygen through narrow upper airways resulting from paralysis of the vocal cords.

Which of the following conditions would NOT result in a tracheal shift from midline?

A. hemothorax B. pneumothorax C. severe unilateral atelectasis D. asthma D. Asthma is a condition that affects both lungs equally causing bilateral wheezing associated with bronchoconstriction. This bilateral condition would not result in any shifting of the trachea to one side or the other.

Six hours after a motor vehicle accident, a patient's chest radiograph shows vascular engorgement in the right lung. This finding is most likely evidence of which of the following?

A. hemothorax B. pulmonary embolism C. pleural effusion D. flail chest B.

Which of the following ventilator controls will be primarily used when employing an alveolar recruitment strategy?

A. high pressure limit B. PEEP C. inspiratory flow D. I:E ratio B. Most alveolar recruitment strategies include use of elevated PEEP levels for small periods of time. PEEP provides a baseline pressure that may be used to prevent alveolar collapse and recruit already collapsed alveoli.

Which of the following sounds assessed by a respiratory therapist is potentially the most serious if heard on a pediatric patient in the emergency room?

A. high-pitched continuous on exhalation over the larynx B. high-pitched continuous during inhalation over the larynx C. rhonchi clearing with cough D. barking cough, otherwise clear B. High-pitched continuous sounds in the upper airway, also known as stridor, may be life-threatening.

While performing endotracheal suctioning with a closed-system suction catheter on a patient who is intubated with a 7.5-mm ET tube, the respiratory therapist notices the suction at the tip of the catheter suddenly stops. Which of the following should the therapist check?

A. hospital vacuum system B. collection canister C. patency of the ET tube D. integrity of the suction catheter baggy B. When the collection canister of a suction system becomes full, a valve actuates and interrupts suction. To remedy this, the disposable suction bag or container must be replaced.

In order to properly disinfect a home-use nebulizer, the respiratory therapist should recommend

A. hydrochloric acid B. rinsing with tap water after each use followed by air dry C. disposing of the nebulizers after 48 hours D. acetic acid D. Acetic acid, or common household vinegar, is a very effective and appropriate cleaning agent for home use. Hydrochloric acid would be unsafe. Disposing of nebulizers every 48 hours would be costly for the patient. Simple tapwater will increase the likelihood of infection for the patient.

After a 30-minute soak in alkaline gluteraldehyde (Cidex), viable microorganisms are found on the equipment. Which of the following is a possible reason?

A. inappropriate sterilization solution type B. device requires autoclaving C. high resistance of organism or insufficient soak time D. insufficient aeratime C. Alkaline glutaraldehyde, or Cidex, is a liquid that can disinfect and sterilize equipment. There are several factors that dictate its effectiveness. They are: strength of solution, soak time, and resilience or resistance of the organism. Aeration time does not decrease the effectiveness of the solution.

A patient is receiving non-invasive positive pressure ventilation by mask with an IPAP of 12 cmH2O and an EPAP of 4 cmH2O. Corresponding blood gas data is as follows: pH 7.49 PaCO2 30 mmHg PaO2 87 mmHg HCO3- 24 mEq/L BE -0 mEq/L Which adjustment is most appropriate?

A. increase EPAP B. increase both IPAP and EPAP C. decrease IPAP D. decrease IPAP and EPAP C. In this blood gas the patient is over ventilating. The primary BiPAP control that affects CO2 is IPAP. A reduction in IPAP will decrease the distance between the IPAP and EPAP settings, which will result in less ventilation and will raise PaCO2

A patient with asthma receiving volume-controlled ventilation has the following arterial blood results on the settings below: Mode Assist/control Mandatory rate 16 VT 400 mL FIO2 40% PEEP 5 cm H2O I:E 1:3 pH 7.31 PaCO2 49 torr PaO2 78 torr HCO3- 24 mEq/L BE 0 mEq/L Which of the following changes is most appropriate?

A. increase FIO2 to 0.5 B. increase PEEP to 8 cmH2O C. remove 150 mL of deadspace D. increase inspiratory flow rate C. This patient is under ventilating as manifested by high CO2. Mild hypoxemia is also present. Of these two problems, ventilation should be addressed first. Of the options offered, removing deadspace is the only option that will decrease CO2. Changing flowrate will not have a significant effect and changing FIO2 or PEEP are related to oxygenation, which does not take priority.

A patient is receiving non-invasive positive pressure ventilation with an IPAP of 28 and an EPAP of 12 cmH2O. The following blood gas data on these settings is available: pH 7.37 PaCO2 42 mm Hg PaO2 72 mm Hg HCO3- 24 mEq/L BE 0 mEq/L Which of the following change is most appropriate?

A. increase IPAP B. decrease IPAP and EPAP C. increase IPAP and EPAP D. decrease EPAP C. This patient shows adequate ventilation but mild hypoxemia. To correct hypoxemia, expiratory pressure should be increased. However, increasing expiratory pressure alone will decrease the distance between the inspiratory and expiratory pressures and will inadvertently decrease ventilation. This is not desirable because CO2 is already appropriate. To prevent a change in ventilation IPAP must be increased by the same degree that EPAP is increased.

A 28-year-old male is receiving VC A/C ventilation on the following settings: FIO2 0.50 (f) 18/min VT 500 mL PEEP 5 cm H2O Flow 35 L/min I:E 1:2.9 Arterial blood gas analysis reveals: pH 7.34 PaCO2 45 torr PaO2 68 torr HCO3- 22 mEq/L The respiratory therapist should

A. increase tidal volume. B. remove deadspace. C. increase PEEP. D. increase peak flow. C. ABGs show hypoxemia with acceptable ventilation. Oxygenation should be corrected first. Although FIO2 could be increased up to 60%, prior to a PEEP increase, that option is not available. Thus, the next best answer is to increase PEEP.

A patient is transferred to the post-surgical recovery unit after undergoing a partial lobectomy in the left lung. Three days after surgery, the patient is unable to reach their pre-operative incentive spirometry values, in spite of a report of feeling well with no significant pain. Which of the following could explain this observation?

A. increased FRC B. poor V/Q matching C. diminished Hb level D. reduced total lung capacity D.

Which of the following is most indicative of right heart failure?

A. increased PAP, decreased cardiac output B. decreased CVP, decreased PAP C. increased PAP, decreased PCWP D. increased CVP, decreased PAP D. In order to assess the function of the right heart hemodynamically, one must look to those hemodynamic values immediately preceding and following the right heart. In this case, CVP and PAP. If CVP is increased and PAP is decreased or is normal, blood is having difficulty transitioning through the right heart. Right heart failure and core pulmonale are associated with this condition.

A patient who weighs 68 kg (150 lb) has a minute ventilation requirement of 14 L/min to maintain a PaCO2 of 38 torr. Which of the following can explain the ventilatory requirements?

A. increased dead space ventilation B. excessive caloric intake C. obstructive apnea D. CNS depression A. This patient is requiring a very high minute ventilation to achieve a normal PaCO2. The most likely cause of this is increased dead space ventilation. Dead space ventilation is likely due to collapsed alveoli which results in lower lung space. This would cause the patient to breathe quickly and deeply in order to move significant amounts of air to achieve adequate ventilation.

A patient has been receiving mechanical ventilatory support for one week, following a pulmonary contusion. The physician would like to consider weaning. Current ventilator settings and other data reveal: Mode SIMV f 4/min VT(set) 500 mL PEEP 5 cm H2O FIO2 0.40 VC 1.8 L VT (spont) 525 mL MIP -38 cm H2O The patient follows commands intermittently and, at times, seems somewhat disoriented to time, person, and place. The respiratory therapist should recommend

A. discontinuing ventilatory support. B. continuing mechanical ventilatory support. C. extubation. D. adding pressure support of 5 cm H2O. B. Although data suggests that the patient is ready to wean from ventilatory support and to be extubated, the patient's inability to following commands consistently and the presence of general confusion contraindicate extubation at this time.

A patient in distress has a chest radiograph that shows diffuse alveolar infiltrates while breath sounds reveal moist rales. The following data is available: CVP 13 mm Hg Right-side preload 14 mm Hg mPAP 28 mm Hg PCWP 22 mm Hg C.I. 1.7 L/min/m2 The respiratory therapist should administer

A. diuretics B. inhaled nitric oxide C. Xopenex D. Priscoline A. Examination of this hemodynamic data reveals pressures that are all high. Elevated hemodynamic pressures across the board indicate fluid overload. This can be treated through the administration of diuretic medication.

With the addition of a spacer, the patient self-administering MDIs

A. does not have to closely correlate actuation and inhalation. B. does not have to hold their breath after each maneuver. C. may take a smaller dose than ordered. D. should take a slower, deeper breath. A. An aerosol holding chamber, commonly known as a spacer, not only improves medication delivery when added to a metered-dose inhaler, but also reduces the significant amount of coordination required to properly self-administer an MDI.

What is the formula for finding a patient's mean arterial pressure?

A. double the systolic value, add the diastolic, divide the sum by 3 B. double the diastolic value, add the systolic, divide the sum by 2 C. calculate the sum of all positive pressure during mechanical ventilation D. double the diastolic value, add the systolic value, divide the sum by 3 D. MAP can be calcuated by (1 x systolic) + (2 x diastolic) then dividing the entire sum by 3.

A 14-year old patient with asthma is in the emergency department (ED) with shortness of breath. Two treatments with Albuterol have been given, but the patient is still wheezing and peak flow measurements have not changed. The respiratory therapist should recommend?

A. drawing arterial blood for gas analysis B. administering cromolyn sodium (Intal) C. ordering a chest radiograph D. administering ipratropium bromide (Atrovent) D. When a patient is not responsive to a particular bronchodilator medication, it is appropriate to attempt bronchodilation with a different medication. In this case, Atrovent is a good alternative medication because it is also a short term rescue medication, like albuterol, but with a different mode of action.

An 80-year-old female patient with a 7.5 mm endotracheal tube requires endotracheal suctioning about every hour. She has experienced bradycardia with several suction attempts. Suction pressure is -100 mm Hg and suction duration is 15 seconds. To remedy the problem, the therapist should reduce suction

A. duration. B. depth. C. frequency. D. pressure. A. Suction should be applied for no longer than 15 seconds, but it is acceptable to reduce the suction duration below 15 seconds for a patient experiencing cardiac complications, especially if the suction pressure is already at the lowest level within the range.

Which of the following clinical data can be obtained from a patient who is comatose?

A. dyspnea level B. subjective information C. oxygenation status D. vital capacity C. A patient who is comatose is unable to comply with verbal commands. Of the options offered, the only data that requires compliance to a verbal command is "vital capacity".

Use of overnight oximetry at home is intended to determine the need for which of the following?

A. echocardiography B. electroencephalography C. continuous cardiac rhythm monitoring D. polysomnography D. Of the options listed, the use of overnight oximetry is used to detect periodic oxygen desaturation, which may be connected to obstructive or central sleep apnea. Although the exam is not conclusive, it is enough to indicate further testing with regard to sleep. This kind of testing is called polysomnography.

Which of the following is NOT normally part of a smoking cessation program?

A. education on the hazards of smoking B. source of emotional support C. information on the cost of cigarettes and tobacco products D. nicotine replacement therapy C. In preparation to help the patient stop smoking, the healthcare giver may assess the emotional support required, evaluate the appropriateness of nicotine replacement therapy, and educate the patient on the hazards of smoking. Discussing the cost of cigarettes and other tobacco products will not be helpful.

What is the objective for using a pressure/volume loop to determine optimal PEEP on a mechanically ventilated patient?

A. eliminate PEEP when the FIO2 is 1.0 B. determine how high one can adjust the PEEP C. ensure increase in pressure with any rise in volume D. ensure increase in volume with any rise in pressure D. Pressure/volume loops are used to determine optimal PEEP for mechanically ventilated patients to make sure that increases in peak pressure result in increases in volume while in the pressure control mode.

A patient is receiving volume-controlled ventilation in the assist/control mode. The following data are available: Plateau Pressure Peak pressure Exhaled VT 6 PM 16 cm H2O 22 cm H2O 561 mL 9 PM 22 cm H2O 28 cm H2O 548 mL 1 PM 27 cm H2O 33 cm H2O 550 mL The respiratory therapist would categorize the most significant problem noted from this data as

A. increasing airway resistance B. increasing pulmonary compliance C. decreasing airway resistance D. decreasing pulmonary compliance D. This data shows an increase in peak airway pressures. However, plateau pressures are increasing to the same degree, indicating that the source of the increasing peak airway pressures is due to a decrease in pulmonary compliance, or a stiffening of the lung.

Which of the following is an important strategy when caring for a patient with adult respiratory distress syndrome?

A. keep FIO2 below 0.60 when possible B. ventilate in Control mode with paralytics C. promote AutoPEEP D. keep FIO2 higher than 0.60 A. The primary strategy for a patients with adult respiratory distress syndrome who are on a ventilator is to keep the FIO2 as low as possible. Generally below 0.6 is most appropriate. This, however is a tough battle because hypoxemia is the cardinal attribute of adult respiratory distress syndrome. In order to compensate and keep hypoxemia at a minimum, PEEP is also used.

Which of the following is an appropriate strategy when caring for a patient with ARDS who is receiving mechanical ventilation?

A. keep PEEP as high as possible B. keep the patient in reverse trendelenburg position C. keep FIO2 at 60% or below if possible D. strive for C.I. values less than 2.0 L/min/m2 C. There are two primary strategies for patients who have adult respiratory distress syndrome and are receiving mechanical ventilatory support - keep mean and peak airway pressures as low as possible and keep FIO2 below 60% as much as possible.

While monitoring a patient receiving mechanical ventilation, the respiratory therapist notices the low-pressure and low exhaled volume alarms are sounding. Which of the following could be the cause?

A. kinked endotracheal tube B. herniated endotracheal tube cuff C. excess water in the inspiratory limb of the circuit D. disconnected thermistor line D. If you have a low pressure or low exhaled volume ventilator alarm, you must look for the problem that could cause that specific type of alarm. If you look at the answers, a kink in the endotracheal tube would cause a high-pressure alarm so that cannot be the cause. Excess water in the limbs of the circuit would do the same. A high-pressure alarm would also develope if there is a herniated endotracheal tube cuff. This leaves the only possible answer which is a disconnected thermistor line.

A chest radiograph on a neonate indicates a ground glass appearance. This finding is most closely associated with which of the following?

A. meconium aspiration B. perihilar pneumonia C. IRDS D. tetralogy of fallot C. The radiological description of a neonate's chest x-ray that indicates a "ground glass appearance" is associated with IRDS, or infant respiratory distress syndrome. Another similar description is "reticulogranular pattern". The question is asking for the poper treatment. Because we know this is IRDS the proper treatment is surfactant therapy.

A pulmonary rehabilitation patient is being monitored for compliance to a smoking cessation program. Which of the following values would be most helpful to evaluate?

A. metHb B. SPO2 C. PaO2 D. multiple wave-length spectrophotometry D. A patient on a smoking cessation program may be monitored for compliance by periodically assessing the Multiple wave-length spectrophotometry. This allows the measurement of CO in the blood by finger probe. If COHb remains elevated or increases, the patient is likely not compliant with the program.

A 36-week gestational age infant is receiving high-frequency oscillatory ventilation for the management of pulmonary interstitial emphysema (PIE). The respiratory therapist increases the oscillatory amplitude on the ventilator by physician order. The therapist is most likely addressing which of the following?

A. metabolic acidosis B. alkalosis C. excessive fetal hemoglobin D. hypercapnia D.

Arterial blood gases on a patient in the emergency department are as follows: pH 7.19 PaCO2 46 mm Hg PaO2 65 mm Hg HCO3- 19 mEq/L BE -5 mEq/L Which of the following represents an accurate interpretation of these results?

A. metabolic acidosis B. compensated respiratory acidosis C. compensated metabolic acidosis D. uncompensated respiratory alkalosis A. When we look in the carbon dioxide on this patient we note that it is normal, or very close to the normal range. However, the pH shows significant acidosis. Because the CO2 is nearly normal this rules out the respiratory system as the source of the acidosis. A further look at the reduced HCO3- proves that the source of the acidosis is metabolic. Therefore, the correct interpretation is metabolic acidosis.

A recovering motor vehicle accident patient with multiple trauma suddenly becomes short of breath and develops cyanosis from the waist up. Which of the following conditions should the respiratory therapist suspect?

A. myocardial infarction B. pulmonary embolism C. pulmonary edema D. acute ventilatory failure B. The sudden development of shortness of breath and the development of cyanosis from the waist up is associated with pulmonary embolism. This is considered an emergency and the patient should be given 100% oxygen. Only after 100% oxygen is in place should efforts be made to confirm the diangosis (VQ scan, etc).

A patient who requires supplemental oxygen at 40% has a variable respiratory pattern. Which of the following devices would most reliably provide a consistent oxygen concentration?

A. nasal cannula B. air-entrainment mask C. face tent D. nonrebreather mask B. For a patient who has a variable inspiratory requirement, and therefore breathes with a variable respiratory pattern, oxygen delivery through an air-entrainment device is more appropriate. Air-entrainment systems provide high flow and are more capable of meeting and/or exceeding the patient's inspiratory demand and spite of variability in the respiratory pattern.

A 65-year-old male patient has been ventilator dependent for several weeks. Weaning attempts have failed. The patient will be transferring to a long-term care facility where daily weaning trials will commence. Which of the following artificial airways would be most appropriate for the patient?

A. nasal tracheal tube B. nasal pharyngeal airway C. cuffed tracheostomy tube D. oral endotracheal tube C. A patient who is ventilator-dependent, and is having difficulty weaning from the ventilator, may benefit most from a tracheostomy tube. Compared to an oral or nasal endotracheal tube, a tracheostomy tube is shorter and therefore has less airway resistance

In which of the following situations should an HME be replaced with heated humidity?

A. patient has asthma B. afebrile conditions C. presence of thick secretions D. patient is hyperthermic C. An HME should be changed if the patient has thick secretions or is hypothermic. If the patient is hypothermic he or she cannot breathe out enough heat into the exchanger to capture and rebreathe. In that case the device should not be used.

McKenzie Lee Weber State University The following ECG tracing is recorded for a 65-year-old male who is being monitored in the cardiac intensive care unit. Based on this rhythm, the respiratory therapist will note which of the following in the medical record?

A. patient has sinus tachycardia B. patient has sustained cardiac tissue damage from a previous myocardial infarction C. patient is currently experiencing myocardia injury D. cardiac ischemia is present B. This ECG tracing is generally normal but shows pronounced Q waves. Q waves that are either excessively wide or excessively deep are associated with cardiac tissue death, likely from a previous myocardial infarction. This is especially true in the absence of current signs or symptoms and distress.

Prior to performing an arterial puncture, a modified Allen's test is performed on the patient's right radial artery. When the ulnar artery occlusion is released, a pink color returns in 35 seconds. Based on this result the respiratory therapist should

A. perform a right brachial artery puncture B. perform an Allen's test on the left radial artery C. proceed with the puncture of the right radial artery D. perform a femoral artery puncture B. A modified Allen's test is performed to ensure there is collateral circulation prior to performing an arterial puncture. The presence of collateral circulation, blood flow through the radial and ulnar arteries, helps to lower the risk of the puncture. This is done by including both the ulnar and radial arteries simultaneously. Once the hand becomes blanched (white and seemingly devoid of blood) the ulnar artery is released and the hand is observed to see if color returns in a timely manner. This should occur in a few seconds. In this case, color fails to return in a timely manner. This indicates that the right radial artery should NOT be used because the ulnar artery does not have good back-up circulation. The left ulnar artery should be checked for collateral circulation to determine if a puncture of the left radial artery is safe.

A respiratory therapist is providing chest physiotherapy and postural drainage on a cystic fibrosis patient with pneumonia in the right lateral segment. The patient was in high fowlers prior to therapy. The patient experiences arrhythmias and dyspnea after 5 minutes of therapy. What should the therapist recommend?

A. place bed flat, continue therapy, monitor B. stop therapy, place the patint in semi-fowlers, monitor C. stop percussion, provide 100% oxygen D. stop therapy, return to high-fowlers, provide oxygen, call the physician D Signs or symptoms such as arrhythmias or dyspnea associated with chest physiotherapy or postural drainage should result in the complete cessation of therapy, documentation in the medical record, and notification of the physician. Providing oxygen in this case is appropriate due to the arrhythmias.

Equipment is delivered from the central supply department to the respiratory therapy department and the packaging indicator shows that it is not sterile. What should the respiratory therapist recommend?

A. place in clean equipment bin B. soak in Cidex for 10 hours C. replace the indicator and re-evaluate D. avoid using the equipment D. f the packaging indicator shows that the central supply procedure was ineffective, the simple solution is to return the device or equipment to the central supply department for a repeat sterilization procedure.

A patient on the general floor becomes a primary mouth-breather while asleep, resulting in a drop in SpO2 from 95% to 87% while on 3 L/min nasal cannula. The best remedy while asleep is to

A. place the cannula in the patient's mouth. B. apply CPAP with oxygen bleed-in. C. apply a chin strap. D. change to a 30% air-entrainment mask. D. The FIO2 of a nasal cannula is approximated to be 3 to 4% per liter. Therefore, a 30% Venturi mask most closely approximates the FIO2 of a nasal cannula running at 3 L/min.

A patient is in the emergency room (ER) with paradoxical chest movement on the right side. The trachea is deviated from mid-line. Which of the following should the respiratory therapist suspect?

A. pleural effusion B. pulmonary embolism C. pulmonary edema D. fractured ribs and a pneumothorax D. Paradoxical chest movement on the right side is an indication of the presence of broken ribs. Deviation of the trachea indicates the presence of a pneumothorax.

A chest radiograph of an abdominal post-operative patient shows abnormal elevation of the left hemidiaphragm. Which of the following conditions explains the observation?

A. pneumothorax on the left B. hemothorax C. atelectasis in the left lower lobe D. herniation of the left hemidiaphragm C. Abnormal elevation of the left hemidiaphragm is an indication that the lung on that side is smaller for some reason. This could be due to a partial pneumothorax or profound atelectasis. Oftentimes, atelectasis can develop as a result of surgery. Therefore, the raised hemidiaphragm, combined with the postoperative status of the patient, indicate the most likely problem is atelectasis in the left lower lobe.

A patient admitted to the emergency room for chest pain is diaphoretic with cold extremities. These data are most consistent with which of the following

A. pneumothroax B. pulmonary embolism C. myocardial infarction D. pulmonary tuberculosis C. Chest pain, diaphoresis, and cold extremities is associated with myocardial infarction. Other terminology that would describe this includes cold, wet, clammy skin. The next most appropriate action, although not asked in this question, would be to administer oxygen and to obtain an ECG.

A patient is receiving mechanical ventilation in the pressure-control mode. End-tidal CO2 is being monitored. The following tracing is available on the monitor in real-time. Which of the following interpretations is most consistent with this tracing?

A. poor alveolar recruitment B. normal ventilation C. hyperventilation D. hypoventilation B. An end-tidal CO2 of 40 mmHg correlates with an arterial CO2 level of about 40 mmHg, which is normal.

A patient with acute respiratory distress syndrome is receiving PC ventilation with an FIO2 of 0.6 and PEEP of 22 cm H2O. Arterial blood gas analysis reveals: pH 7.27 PaCO2 62 torr PaO2 56 torr HCO3- 18 mEq/L What additional therapy may be helpful at increasing alveolar ventilation?

A. prone positioning B. a decrease in PEEP C. increased expiratory time D. increased inspiratory flow rate A.

Prior to initiating postural draining and percussion on a patient, the therapist notes a dull percussion note over the lower posterior thorax. Auscultation of the area reveals bronchial breath sounds. The therapist should position the patient in which of the following positions?

A. prone with the head of bed down B. supine with patient in reverse Trendelenburg C. sitting on the side of the bed with elbows propped up on a table D. on the right side with the bed flat A. The proper position to drain secretions from the lower posterior thorax is prone with the head of bed down.

Results of a quality control maneuver for a spirometer using a 3.0 L calibration syringe are as follows: Volume 1 Volume 2 Volume 3 2.65 L 2.68 L 2.66 L According to ATS Standards, the spirometer is

A. proof that the syringe requires calibration B. inaccurate C. operating correctly D. lacking in precision B. When checking a spirometer to ensure its accuracy prior to testing, 3.0 L calibration syringe is used. The spirometer is considered accurate if the results are between 2.85 L and 3.15 L. In this case, the results are considered inaccurate because they are outside this range.

During endotracheal suctioning, a patient becomes agitated, markedly hypertensive, and demonstrates PVCs on the cardiac monitor. The therapist should prepare to

A. provide maximum supplemental oxygen B. discontinue suctioning, report to the physician C. administer a bolus of intravenous fluids D. administer aerosolized lidocaine 4% A. The presence of PVCs during any procedure, especially suctioning, would suggest that the patient has become hypoxic during the procedure and could benefit from maximum supplemental oxygen. This should be provided both before, during, and after the procedure to ensure arterial oxygenation and avoid hypoxemia.

A patient is in the cardiac intensive care unit. Hemodynamic monitoring has been instituted with a Swan-Ganz catheter. From where should the respiratory therapist obtain a measurement of central venous pressure (CVP)?

A. proximal lumen of the PA catheter B. jugular vein C. subclavian vein D. distal lumen of the PA catheter A. The proximal lumen of a pulmonary artery catheter is situated inside or right before the right atrium. The value measured here is called CVP.

A patient presents in the emergency department (ED) with shallow, irregular respirations. Which of the following diagnostic procedures would best help rule out a pleural effusion?

A. pulmonary angiogram B. PA chest radiograph C. ventilation/perfusion (V/Q scan) D. series of oblique chest radiographs D. Pleural effusions may be diagnosed through obtaining a lateral decubitus x-ray or by obtaining a series of x-rays from oblique angles and different positions.

Which of the following hemodynamic values is associated with the function of the left heart?

A. pulmonary capillary wedge pressure B. systemic vascular resistance C. central venous pressure D. mean pulmonary artery pressure A. Of the four hemodynamic pressures, pulmonary capillary wedge pressure is most related to the function of the left heart. Central venous pressure is most related to the right heart and the general fluid status of the patient.

A 19-year-old female receiving volume-controlled ventilation due to status asthmaticus has an I:E ratio of 1:2. Arterial blood gas analysis shows: pH 7.25, PaCO2 55 mmHg, PaO2 81 mmHg. Which of the following should the respiratory therapist increase?

A. pulmonary compliance B. minute ventilation C. inspiratory time D. FIO2 B. The arterial blood gas of this patient indicates acute respiratory acidosis. This means the patient is hyperventilating as manifested by a high arterial CO2. Increasing minute ventilation should help to remedy this problem.

To determine potential post-operative risk of a patient preparing for surgery, the respiratory therapist should recommend which of the following tests?

A. pulmonary diffusion studies B. basic spirometry C. pulmonary stress test D. cardiac stress test B. Potential postoperative risk may be assessed preoperatively through basic spirometry.

The medical record of a patient indicates a chest radiogram shows a concave superior interface border. This description is most consistent with which of the following?

A. pulmonary edema B. ARDS C. COPD and chronic air-trapping D. pleural effusion D. A chest x-ray that shows a "concave superior interface border" is associated with pleural effusion.

Which of the following conditions would be a contraindication for IPPB?

A. pulmonary edema B. fractured ribs C. a patient requiring bronchodilation AND lung expansion D. untreated pneumothorax Pneumothorax is a contraindication for IPPB due to positive pressure which will exacerbate barotrauma of the lung.

A radiological report is placed in the medical record that describes the results of a chest radiograph. The report indicates a batwing pattern. This finding is consistent wi

A. pulmonary edema B. pneumonia C. atelectasis D. pleural effusion A. The radiological term "batwing pattern" is associated with pulmonary edema.

A patient receiving volume-controlled ventilation has the following data on the ventilator flow sheet 10 pm 11 pm Plateau pressure (Pplat) 20 cm H2O 30 cm H2O Peak pressure (Ppeak) 28 cm H2O 38 cm H2O PEEP 5 cm H2O 5 cm H2O Set VT 600 mL 600 mL Return VT 596 mL 599 mL Which of the following can best explain these data?

A. pulmonary edema B. wheezing C. secretions in the airways D. kinked patient circuit (tubing) A. Of the options given, only pulmonary edema will result in a decrease in pulmonary compliance, which is shown by an increase in plateau pressures. Secretions in the airway, wheezing, or a kinked ventilator circuit would result only in an increase in peak pressures - not an increase in plateau pressures.

The therapist is ordered to evaluate evenness of distribution in a patient with chronic obstructive pulmonary disease. Which of the following tests would be helpful for the requested evaluation?

A. nitrogen elimination (SBN2) B. single-breath DLCO C. pulmonary angiogram D. multiple breath DCO A. A single-breath nitrogen elimination test (SBN2) is useful in determining the evenness of gas distribution in the lungs. The result comes in four phases as the patient exhales a single breath. Phase I is the exhalation of pure deadspace gas. Phase II consists of some deadspace and some alveolar gas. Phase III consists of pure alveolar gas and is the phase that indicates the evenness of distribution. Phase IV is called "closing volume".

A 28-year old male reports to the emergency department with dizziness, nausea, and some degree of vertigo. The patient's record indicates the patient is on a skiing vacation but normally lives on the coast. The physician orders sildenafil. For which of the following is the patient mostly likely being treated?

A. nitrogen narcosis B. altitude sickness C. decompression sickness D. avian flu B. The medication sildenafil, also known as Viagra, can be used to treat altitude sickness.

A patient with suspected vocal cord dysfunction and paralysis is undergoing pulmonary function testing to further investigate the problem. Which of the following PFT exams will yield most information about the issue?

A. nitrogen washout B. SBN2 C. DLCO D. FVL D. A pulmonary function exam can reveal the presence of vocal cord dysfunction and/or paralysis by a flow volume loop (FVL). The condition will manifest itself by showing what is called a round loop, or a loop whose inspiratory phase looks similar to the expiratory phase. Normally, these two phases should look drastically different.

A 22-year-old male patient with status asthmaticus is receiving VC A/C ventilation and is orally intubated with a 7.5-mm ET tube. The therapist notices a marked increase in peak airway pressures. BS are clear bilaterally but decreased on the right. The therapist should recommend

A. obtaining a portable chest radiograph B. performing a bronchoscopy C. inserting a chest tube in the right side, mid-clavicular line D. scheduling a ventilation/perfusion scan A. Decreased breath sounds on the right is most suggestive of a possible pneumothorax. A chest radiograph is most helpful in assessing for this possibility.

A patient is receiving volume-controlled ventilation. Which of the following blood gas results is a clear indication for an increase in minute ventilation?

A. pH 7.55, PaCO2 26 torr, Pao2 88 torr B. pH 7.35, PaCO2 60 torr, PaO2 65 torr C. pH 7.50, PaCO2 30 torr, PaO2 82 torr D. pH 7.25, PaCO2 55 torr, PaO2 51 torr D. The need to increase minute ventilation can be determined by examining arterial CO2. In this question, two options have an elevated PaCO2, indicating a need to increase ventilation. However one of these options shows a corrected pH, suggesting that the elevated CO2 is normal for that patient. Therefore, the correct answer is the blood gas results that show elevated CO2 with an uncompensated pH. This is called uncompensated respiratory acidosis.

A patient receiving volume-controlled ventilation has an oxygen saturation of 87% within moments after the low pressure ventilator alarm begins to sound. After providing manual ventilation with a bag valve, the respiratory therapist observes the bag is easier than normal to squeeze and that oxygen saturation continues to fall. Which of the following could be the cause of this observation?

A. partial extubation B. pneumothorax C. ET tube cuff herniation D. excess secretions in the ET tube A. The apparent increase in dynamic compliance (the bag is easier than normal to squeeze) indicates the patient may be partially extubated.

A patient with COPD has blood gases that show a normal acid/base with hypercapnia and moderate hypoxemia. This is most consistent with which of the following blood gas interpretations?

A. partially compensated metabolic acidosis B. respiratory compensated metabolic alkalosis C. metabolically compensated respiratory acidosis D. uncompensated respiratory acidosis C.

A high-pressure alarm is sounding on a ventilator that is set on PC mode with a PEEP of 8 cm H2O. Which of the following could explain this?

A. patient biting on the ET tube B. improper difference between IP setting and PEEP C. inspiratory flow rate is reaching its peak with each breath D. total PEEP and IP pressures exceed the flow demand A.

Which of the following would be most associated with the development of AutoPEEP?

A. patient exhales completely before inhalation begins B. inspiratory flow rate is excessively high C. expiratory time is excessively prolonged D. expiratory flow does not reach zero before inhalation begins D. The cause of autoPEEP is the lack of time a patient has to exhale gas before the inspiratory phase starts on a ventilator. This causes air trapping and increases intrinsic PEEP (autoPEEP). Therefore, an expiratory flow that does not reach zero before inhalation begins will cause autoPEEP. Although not asked in this question, the solution for this is to increase inspiratory flowrate, which will decrease inspiratory time and prolong expiratory time.

A 38-year old male is receiving ventilatory support by a high frequency jet ventilator (HFJV). Heart rate is 120/min and blood pressure is within normal limits. Arterial blood gas results on high frequency ventilation are as follows: pH 7.26 PaCO2 64 torr PaO2 82 torr HCO3- 26 mEq/L BE 0 mEq/L The respiratory therapist should recommend

A. switching to pressure control ventilation B. switching to volume-controlled ventilation C. increasing frequency D. increasing amplitude C. This patient, who is on a high-frequency ventilator, shows evidence of hypoventilation as manifested by high PaCO2. Oxygenation appears adequate. To correct the hyperventilation, the most appropriate action is to increase frequency. This is equivalent to increasing the mandatory rate.

The following data is observed on a patient who is 10 minutes post coronary artery bypass graft (CABG): 5 min after 15 min after 25 min after PaO2 (mm Hg) 92 90 93 PvO2 (mm Hg) 65 60 52 Which of the following can the respiratory therapist conclude?

A. tissues are becoming oxygen deprived B. A-aDO2 is decreasing C. CvO2 is increasing D. cardiac index is decreasing D. This data shows a steady arterial oxygenation level with a decreasing venous oxygenation level. When the gap between arterial and venous oxygen levels start to increase, this is consistent with an increase in tissue oxygen consumption and an increase in the C(a-v)O2. This is related to a decrease in cardiac output. Of the options given, a decrease in cardiac index is another way of indicating a decrease in cardiac output.

Which of the following devices may be used to create aqueous materials of know gas values which are helpful at ensuring the quality control of a blood gas analyzer?

A. tonometer B. multiple wave-length blood testing C. capnograph D. Sanz electrode A. A tonometer is a device used to create aqueous materials for blood gas machines to ensure the precision and accuracy of blood gas analyzers.

After assisting the repositioning of a tracheostomy patient, the respiratory therapist palpates a crackling sensation about the neck and clavicle area. The therapist should conclude the tracheostomy tube is

A. too small B. in the trachea C. too large D. incorrectly positioned D. "Crackling sensations" or "popping sounds" palpated about the neck and clavicle area are associated with subcutaneous emphysema. Subcutaneous emphysema occurs when air enters the dermal spaces (under and inside the skin). This is most likely related to an incorrectly positioned tracheostomy tube.

A respiratory therapist is measuring the gas volume from a patient who exhales maximally after inhaling to inspiratory reserve volume. Which of the following volumes is the respiratory therapist attempting to observe?

A. total lung capacity B. inspiratory reserve volume C. expiratory reserve volume D. vital capacity D. The volume exhaled maximally after a maximum inhalation is called vital capacity.

A respiratory therapist is monitoring the volume of gas a patient can inhale after a passive exhalation of tidal volume. The therapist is attempting to observe which of the following?

A. total lung capacity B. inspiratory reserve volume C. vital capacity D. inspiratory capacity D. The volume achieved when the patient inhales maximally after a passive exhalation of tidal volume is called inspiratory capacity. Comparatively, the maximum volume inhaled after complete active exhalation is called vital capacity.

After performing minimum seal technique on an 80-kg (175-lb) patient who is orally intubated with an 8.0-mm ET tube, the cuff pressure is measured by manometer and found to be 36 cmH2O. Which of the following can best explain this?

A. tracheomalacia B. tracheal stenosis C. ruptured cuff D. transesophageal fistula D. Because the minimum seal technique was used (also called minimum occluding volume), we know that the cuff is touching the tracheal tissues. However, the pressure required to do so is far above the normal pressure. This would indicate that the space inside the trachea is larger than normal. The cause of this is most likely a transesophageal fistula.

A patient is to be intubated in the emergency room (ER) due to a markedly diminished respiratory drive following a motor vehicle accident from which the patient sustained a head injury. Prior to intubation, it is determined the patient has a Mallampati classification score of 4. Which of the following represents the most appropriate method of intubation?

A. tracheostomy B. blind nasal intubation C. direct laryngoscopy D. oral intubation with a video assist device (VAD) D.

A 32-year-old female has a large cancerous mass in the larynx and is having difficulty breathing. A flow-volume loop is rounded in appearance. To prevent ventilatory fatigue, the patient could benefit most from which immediate action?

A. tracheostomy B. BiPAP C. laryngectomy D. heliox therapy D. A cancerous mass in the upper airway will cause a fixed upper airway obstruction and will lead to increased airway resistance. Because a mass, unlike a temporary condition (like bronchoconstriction), is unable to be immediately resolved, the patient may benefit greatly from Heliox therapy. Helium is less viscous than nitrogen and therefore easier to breathe.

A neonatal patient exhibits a heart murmur upon auscultation one hour after birth. The respiratory therapist is having difficulty maintaining adequate oxygenation saturation in spite of increasing FIO2. Which of the following should the respiratory therapist recommend?

A. transillumination B. cardiac angioplasty C. diagnostic chest percussion D. echocardiography D. Although it is common for a neonate to exhibit a cardiac murmur right after birth, when considered in connection with other signs, such as poor oxygenation in spite of increasing oxygen delivery, the respiratory therapist should suspect congenital cardiac abnormalities. The best way to broadly assess for this kind of congenital issue is to perform echocardiography.

A patient with ARDS is receiving PC, A/C ventilation. During suctioning, the patient becomes tachypneic, diaphoretic, mottled, hypertensive, and anxious. Issues continue after the cessation of the suction procedure. Breath sounds are distant on the right compared to the left. Tracheal deviation is noted to the left. The therapist should prepare to respond to

A. pulmonary emboli B. a pneumothorax C. pulmonary edema D. atrial tachycardia B. One of the possible outcomes associated with endotracheal suctioning, particularly on a patient who has low pulmonary compliance (such as patients with ARDS) is a pneumothorax. Clinical evidence, including a diaphoretic mottled look, hypertension, and anxiety, in conjunction with distal or absent breath sounds supports the suspicion that the patient has obtained a pneumothorax.

The following data is available on a 61-kg (135-lb) male patient receiving VC SIMV ventilation: Ventilator settings: FIO2 0.45 VT 450 mL PEEP 4 cm H2O Mandatory rate 12/min ABGs pH 7.41 PaCO2 39 mm Hg PaO2 85 mm Hg HCO3- 24 mEq/L BE 0 mEq/L If PAO2 is determined to be 204 mm Hg, which of the following best represents the patient's condition?

A. pulmonary embolism B. venous admixture (shunting) C. V/Q mismatching D. pulmonary hypertension C. The A-aDO2 (PAO2 - PaO2) is 119 mm Hg. A-aDO2 values greater than 65 mm Hg but less than 300 mm Hg are consistent with V/Q mismatching.

A patient involved in a bus accident has sustained multiple injuries, including a tibia/fibia fracture and a significant strike to the right thoracic cage, indicated by a bruised area over the right lung, which measures 8 inches by 6 inches. Six hours after the accident, the patient is beginning to demonstrate ventilatory distress and is complaining of sharp pleuritic pain on the right. Which of the following is most likely?

A. pulmonary embolus B. diffuse alveolar hemorrhage C. right-side pneumothorax (greater than 60%) D. pleural effusion D.

A galvanic type oxygen analyzer reads zero when exposed to 100% oxygen and ambient air. The respiratory therapist should

A. recharge the unit B. change the alkaline batteries C. replace the electrolyte solution D. change the probe D. With a galvanic fuel cell type oxygen analyzer, when the analyzer fails to read at all, it is because it requires a new power source. Because there are no batteries in the system, the fuel cell, or probe, acts as the source of energy. Therefore, in this case it appears the probe is depleted of energy and requires replacement.

A patient is receiving oxygen by an air-entrainment device combined with cool aerosol. Arterial blood is extracted and analyzed. FIO2 0.80 pH 7.36 PaCO2 44 torr PaO2 485 torr HCO3- 23 mEq/L BE -1 mEq/L SAT 0.99 The respiratory therapist should

A. reject the results. B. repeat the ABG. C. accept and report the results. D. recalibrate the ABG analyzer. C. Although a PaO2 of 485 seems too high and suspicious of error, the result is indeed possible considering the patient is receiving 80% oxygen. The arterial PO2 achievable can be determined by calculating the PAO2 with a given FIO2. On 80%, the maximum PAO2 is about 506 torr. Thus a PaO2 of 485 is possible. The results should be accepted and reported.

A respiratory therapist is adjusting the ET tube cuff pressure on a patient receiving positive pressure ventilatory support. The therapist is using the minimum leak technique to adjust the volume in the cuff. After insertion of air into the cuff, the therapist auscultates the neck and notices a slight passage of air around the cuff at the top of each mechanical inspiration. The therapist should

A. remove air from the cuff, reassess. B. maintain current cuff pressure. C. switch to a larger ET tube size. D. add air to the cuff. B. The minimal leak technique of adjusting cuff pressure is a method that purposefully allows air to escape around the cuff at the top of each inspiration. This method exposes the patient to less risk of damage to the trachea.

While demonstrating the use of a volume-oriented incentive spirometer, the patient asks the respiratory therapist to explain the purpose of the therapy. The therapist will suggest the therapy is intended to

A. remove secretions from the lower lobes B. monitor inspiratory capacity C. prevent parts of the lungs from collapsing D. stretch the alveoli C. The purpose of a volume-oriented incentive spirometer is to keep all aspects of the lungs inflated and to prevent the collapsing of alveol

A patient with acute respiratory distress syndrome is receiving PC, A/C ventilation and is showing signs of distress. The ET tube is 20 cm at the teeth. A chest radiograph shows the tip of the ET tube is 7 cm above the carina and gurgling can be heard coming from the patient's mouth. The respiratory therapist should

A. remove the ET tube and reintubate. B. add 5 mL of air to the ET tube cuff. C. advance the ET tube 3 cm. D. withdraw the ET tube 2 cm. C. The ET tube should be 2-5 cm above the carina, or positioned with the ET tube markings at 21 to 25 cm at the teeth.

If the respiratory therapist notices the reservoir bag on a nonrebreather mask is failing to partially collapse with each breath, the therapist should

A. remove the one-way valve disk B. tighten the elastic straps on the mask C. obtain a new nonrebreather mask D. increase oxygen flow to the reservoir B. For a patient who is breathing oxygen through a non-rebreather mask, failing to cause a partial collapse of the reservoir with each breath is an indication that either the mask is not tight enough or that the flow to the mask is excessive. It is most likely that the mask requires tightening, which can be done by pulling on the elastic straps.

A premature neonate with respiratory distress receives a dose of exogenous surfactant. Three hours following the administration, distress returns. FIO2 has required titration from 0.40 to 0.70. The respiratory therapist should recommend which of the following FIRST?

A. repeat surfactant administration B. chest radiography C. ECMO D. mechanical ventilation with NAVA B. Because improvement from exogenous surfactant does not persist, the infant may have a more significant pulmonary problem. This should be investigated with chest radiography.

The following arterial blood gas values are reported for a patient who is weaning from mechanical ventilation. There is no notable change in the patient's condition. 0800 hrs 1000 hrs pH 7.42 7.38 PaCO2 37 torr 32 torr PaO2 80 torr 41 torr HCO3- 26 mEq/L 26 mEq/L FIO2 0.40 0.40 Based upon this data, the respiratory therapist should

A. repeat the arterial draw. B. increase the FIO2 to 0.50 C. increase the FIO2 to 1.0 D. extubate the patient. A. The respiratory therapist should question all laboratory results to assure that they match the clinical scenario prior to reporting them. In this example, the patient's condition has not changed with the PaO2 of 41 torr. The sample might be a venous sample, and therefore should be redrawn.

Levy-Jennings charts associated with the quality control history of a blood gas analyzer shows one point outside the 4 SD range. Subsequent data points are all within range. The respiratory therapist should

A. replace the blood gas analyzer B. classify the data point as an outlier and continue analysis on blood samples C. call the medical director D. do not use the blood gas analyzer B. Blood gas machines must occasionally have quality control material run through them to prove they are accurate and precise. The data that is produced is plotted on a chart and monitored to ensure the results stay inside a predetermined range. The range is a statistical number that is equivalent to two standard deviations above the mean and two standard deviations below the mean for a total range of a 4-SD. When points are outside this range the machine is said to be "out-of-control" and may not be used to report patient samples.

A respiratory therapist is evaluating a patient who is receiving volume-controlled ventilation through a tracheostomy tube. After inserting 15 mL of air into the tracheostomy tube cuff, the therapist measures the cuff pressure and notes it is at 5 cm H2O. The therapist should

A. replace the tracheostomy tube B. add 15 mL additional air to the cuff C. record the pressure in the patient's medical record D. cut the pilot tube A. After inserting air into the tracheostomy tube cuff and noticing that the cuff pressure does not rise, the therapist can conclude that the tube is defective and should be replaced. Although there are other methods to temporarily keep the cuff inflated, the best answer is always to replace the tube because it is considered damaged. Damaged equipment should not be used.

A patient with asthma monitors their peak flow in the morning and documents that peak flow is 40% of his usual baseline. Based on the NAEP and the patient's asthma action plan, the patient should

A. report to the emergency room B. contact the patient's physician C. take a corticosteroid MDI D. take a short-acting bronchodilator, check peak flow in 1 hour B. According to the national asthma guidelines, a self monitored peak flow of 40% of baseline is an indication for the patient to contact their physician

In response to an Asthma action plan, the patient has attempted to contact their physician after determining peak flow measurement is less than 50% of the patient's usual baseline value. The physician is not responding to the call. According to NAEP guidelines, the patient should NEXT

A. report to the hospital or call an ambulance B. take a corticosteroid inhaler and check again in 20 minutes C. take a short-term bronchodilator, check peak flow in 1 hour D. take a short-term bronchodilator and contact a different physician A. According the national asthma guidelines, a self monitored peak flow of 50% of baseline is an indication to contact one's physician. However, if one's physician is not available, the patient should report to the hospital or to the emergency room or call an ambulance.

A respiratory therapist is performing a routine patient/ventilator check on a mechanically ventilated patient. Returned tidal volume is 150 mL less than delivered. A high-pitched audible leak is auscultated through the neck. What is the appropriate response?

A. reposition the patient B. advance the endotracheal tube 1 cm C. add air to the cuff D. increase delivered tidal volume C. The high-pitched audible leak auscultating over the neck in conjunction with a low returned tidal volume is most likely an indication of insufficient pressure in the endotracheal tube cuff. Adding air to the cuff is most appropriate.

Arterial blood gases on a patient in the cardiac intensive care unit are as follows: pH 7.35 PaCO2 45 mmHg PaO2 90 mmHg HCO3- 24 mEq/L BE 0 mEq/L Which of the following represents an accurate interpretation of these results?

A. respiratory compensated metabolic alkalosis B. compensated respiratory acidosis C. normal acid base relationship D. respiratory acidosis C. To determine the acid-base relationship from an arterial blood gas, one must first look at the CO2. In this case it's normal. Next, look at pH. Because it is also normal, there is a normal acid-base relationship.

Which of the following laboratory examinations would be helpful in further assessing a patient with diabetic ketoacidosis?

A. respiratory quotient B. creatinine C. urine specific gravity D. glucose level D. patient with diabetic ketoacidosis primarily has a problem with the blood glucose level. Further assessment therefore can be done by examining the glucose level.

A COPD patient is brought to the emergency department (ED) due to moderate respiratory distress on 2 L/min nasal cannula at home. Oxygen is being delivered by paramedics with a non-rebreathing mask. The reservoir bag on the mask does not collapse at all on inspiration. SpO2 is 98% and the patient no longer appears in distress. The therapist will change to which oxygen delivery device?

A. transtracheal catheter B. same mask, decrease the flow C. partial rebreathing mask D. air-entrainment mask D. A COPD patient should not have more than 28% oxygen. This cannot be achieved by a non-rebreather mask. The most appropriate device to use would be a Venturi mask, otherwise called an air-entrainment mask.

Which of the following chest radiograph observations is most closely associated with the lungs of a COPD patient?

A. tree-in-winter pattern B. flattened diaphragm C. concave interface border D. batwing pattern B. A chest x-ray of a COPD patient will show increased vascular markings and a flattened diaphragm.

Fine crackles are auscultated at the bases of both lung fields. This is most consistent with the presence of

A. tuberculosis B. atelectasis C. pulmonary secretions D. pleural friction rub B. Fine crackles heard about the bases of the pulmonary tree are most closely associated with atelectasis.

Pulmozyme is best used for

A. tuberculosis patients with blood-tinged secretions B. COPD patients with airway inflammation C. cystic fibrosis patients with a FVC of at least 40% predicted D. asthma patients refractory to sympathomimetic bronchodilators C. Pulmozyme is helpful at helping patients with cystic fibrosis mobilize and expectorate secretions. However, to be effective the patient must be able to at least produce an FVC of 40% of predicted.

Which of the following would be a sufficient reason to discontinue a spontaneous breathing trials

A. respiratory rate increases from 18 to 26 B. heart rate increases from 78 to 92 bpm C. blood pressure increases from 110/80 to 118/88 mmHg D. development of confusion or disorientation D. In addition to monitoring rate, blood pressure, oxygen saturation, and heart rate, the patient's mental status should also be monitored when doing T-piece trials or performing any kind of ventilator weaning. In this case, the patient has developed confusion and has become disoriented to time, person, or place. Any confusion or disorientation noted during any weaning process is an indication the weaning has failed and should be halted.

The respiratory therapist notes the following results of an arterial blood gas: pH 7.40 PaCO2 41 torr PaO2 46 torr HCO3 -24 mEq/L BE 0 mEq/L The patient reports no symptoms. Pulse rate, blood pressure and color are within normal limits. Which of the following can the therapist appropriately conclude?

A. results are consistent with a venous blood sample B. the analysis did not correct for temperature C. hemoglobin level is low D. the patient has sickle cell anemia A. An asymptomatic patient with a PaO2 of 46 mmHg is not normal. The arterial blood gas data should be questioned for its validity. Of the answers offered, the most likely cause of these results is an accidental analysis of venous blood rather than arterial blood. These values are consistent with a venous blood gas sample.

A respiratory-compromised patient who is being discharged from a long-term care facility requires frequent oropharyngeal suctioning with a tonsil-tip suction device. Which of the following infection control cleaning procedures are recommended?

A. soak in sterile water B. soak in Cidex daily C. submerge the device in boiling water, reuse until integrity is lost D. wipe with alcohol swabs C. While perhaps not so appropriate in a hospital setting, guidelines for infection control procedures in the home do include the use of distilled water, boiling, and cleaning with vinegar (acetic acid). Oralpharyngeal suctioning with a tonsil tip device is not considered a sterile procedure.

A mal-positioned tracheostomy tube is detected by chest radiograph for a patient who is still receiving partial ventilation through the tube. Subcutaneous emphysema is present in the upper chest. Palpation of the affected area would produce

A. soft tissue hyperlucency. B. popping or crackling sensations. C. dry crackles. D. air accumulation below the skin. B. A patient with an inappropriately positioned tracheostomy tube may experience air in the dermal and subdermal spaces of the skin around the neck and upper chest. Upon palpation air bubbles below this may move around or may burst, causing crackling or popping sensations during palpation.

A patient in the emergency room has been vomiting profusely for over an hour. Which of the following is mostly likely to develop?

A. spiked T waves B. increased P-R intervals C. hypokalemia D. PVCs C. When a patient is losing fluid rapidly, such as through vomiting, the result will be hypokalemia, or a loss of potassium. Thus, when someone is vomiting profusely, electrolyte replacement may be a primary concern.

Which of the following is considered objective information?

A. spontaneous tidal volume B. dyspnea C. chest pain D. dysphagia A. Subjective information are those data that must be reported by the patient. This kind of data is also known as symptoms. Data that can be observed independent of the patient's input is known as objective information, otherwise called signs. Of the options offered, only spontaneous tidal volume can be measured directly by the practitioner. Chest pain, dyspnea, and dysphagia are all examples of subjective information because they must be reported by the patient.

Immediately after insertion of a nasopharyngeal airway the patient begins to cough violently and does not stop. The respiratory therapist should

A. spray Lidocaine spray in the oropharynx B. obtain and insert a shorter airway C. remove and lubricate the airway, then reinsert through the other nare D. switch to an oral pharyngeal airway B. When a patient begins to cough uncontrollably after the insertion of a nasopharyngeal airway, also called a nasal trumpet, the airway is likely too long. A shorter airway should be used.

A patient has idiopathic pneumonia with consolidation in the right lower lobe. The physician suspects a bacterial infection. Which of the following will provide conclusive data to rule out the physician's suspicions?

A. sputum acid-fast stain B. oral temperature C. color of sputum D. WBC D. A bacterial infection is diagnosed primarily by examining the white blood cell count, also called the leukocyte count. An elevated temperature and yellow sputum indicate the possibility of an infection but are not confirming in nature.

While coaching a patient with cystic fibrosis on the use of a PEP therapy device, the patient suddenly becomes short of breath and cyanotic above the waist. The most appropriate action is to

A. switch to IPV B. discontinue the treatment C. perform NT suctioning D. schedule a V/Q scan after completion of the treatment B. This patient is exhibiting an adverse reaction to the therapy. The first reaction of the therapist should be to stop the therapy and ensure the patient's vital stability. Subsequently, alternative therapies may be considered to accomplish the same objective without causing a similar adverse reaction.

A patient is receiving heliox therapy with a mixture of 80% / 20% by nonrebreathing mask to reduce airway resistance during an asthmatic episode. The respiratory therapist notes the patient seems to be experiencing moderate respiratory distress. The therapist also observes the reservoir bag completely collapses with each inspiration. The therapist should

A. switch to a Venturi mask set at the highest possible FIO2 setting B. switch to a 60%/40% heliox mixture C. increase total flow of gas mixture to the mask D. discontinue heliox therapy C. When the reservoir bag of a non-rebreathing mask collapses completely, which each breath, and this problem persists beyond a few minutes, the flow to the reservoir bag should be increased.

While instructing a patient on the use of a volume-type incentive spirometer, the therapist observes the patient exhaling forcefully in the spirometer, but no volume is registering on the spirometer. The therapist should

A. switch to a flow-type incentive spirometer B. instruct the patient to inhale through the device C. encourage the patient to exhale more forcefully D. obtain a new volume-type spirometer B. Incentive spirometry (volume-type) is a device that measures the inspiratory capacity. The therapy involves the patient to inspire, raising a puck to a desired volume. However, patients often confuse this maneuver and exhale rather than inhale. When this is observation, proper instruction should be provided to the patient. Instruction should be given in non-medical lingo - language that is easily understood by the patient.

A patient who underwent bariatric surgery has an 8.0-mm standard tracheostomy tube in place, secured with cotton tie. Within the last few hours the airway has become displaced several times. Which of the following is most appropriate?

A. switch to a longer version of an 8.0-mm trach tube B. change to a 10.0-mm endotracheal tube C. suture the trach tube in place D. change to a 10.0-mm trach tube A. A patient undergoing bariatric surgery likely has a large neck with a greater distance from the front of the neck to the trachea then an average person due to adipose tissue. Therefore, a longer tracheostomy tube is sometimes indicated.

A COPD patient is receiving oxygen at 1 L/min when he was admitted to the hospital for suspected pneumonia. The physician orders a target SpO2 of 90%. The following clinical and laboratory data is obtained: RR 17 HR 101 BP 142/88 mm Hg SpO2 86% pH 7.35 PaCO2 68 torr PaO2 51 torr HCO3- 34 mEq/L BE +7 mEq/L The respiratory therapist should recommend which of the following FIRST?

A. switch to an air-entrainment mask at 0.55 B. implement NIPPV C. titrate the oxygen flow rate D. place a heated aerosol mask at FIO2 1.0 C. An SpO2 of 86% does not meet the physician's requirement to maintain oxygen saturation at 90%. Therefore, the titration of oxygen is required. Titration refers to a process of increasing or decreasing FIO2 in response to data to achieve a desired SpO2 or PaO2 value.

A patient is receiving oxygen by nasal cannula at 4 L/min. After 24 hours of use with a properly functioning bubble humidifier, the patient complains of nosebleeds. The therapist would do which of the following to modify therapy?

A. switch to an air-entrainment mask at FIO2 35% B. switch to a simple mask at 4 L/min C. provide a heated bubble humidifier D. decrease flow to 2 L/min A. The FIO2 of a nasal cannula is approximated to be 3 to 4% per liter. Therefore, 35% Venturi mask most closely approximates the FIO2 of a nasal cannula running at 4 L/min.

A child receiving 40% heated aerosol by large volume nebulizer begins to demonstrate mild stridor in the upper airway. Which of the following would be an appropriate response?

A. switch to cool aerosol B. increase FIO2 to 1.0 C. administer diuretic medication D. discontinue oxygen therapy A. Especially in pediatric patients, the use of a heated aerosol can sometimes swell upper airway tissue. When stridor develops, use of a cool mist or cool aerosol is appropriate. If the stridor becomes moderate in nature, use of racemic epinephrine is indicated.

A patient with cystic fibrosis develops rhonchi after 5 minutes of PEP therapy at 20 cm H2O. The respiratory therapist should

A. switching to flutter therapy B. begin cool bland aerosol therapy C. continue the therapy D. discontinue treatment, report to findings to the physician C. The development of rhonchi in response to PEP therapy is considered to be a good outcome for the therapy. The purpose of the therapy is to mobilize and promote expectoration of secretions. The development of rhonchi is an indication that secretions are moving from small and middle-sized airways to the larger size airways where they may be naturally expectorated or suctioned. The therapy should be continued because it is effective.

A respiratory therapist is disinfecting reusable mouthpieces in Sonacide. For how long should the equipment soak to achieve disinfection?

A. 2 minutes B. 6 minutes C. 8 minutes D. 10 minutes D. Acid glutaraldehyde, like its counterpart glutaraldehyde, has the ability to disinfect and sterilize. However, soak times are different. The acid version can disinfect in 10 min. but can sterilize in only one hour. In this question, we are asked to merely disinfect the reusable mouthpieces.

A patient is receiving 70%/30% helium-oxygen mixture by nonrebreather mask through a compensated oxygen flow meter. The indicated flow on the meter is 11 L/min. What is the estimated total gas flow to the patient?

A. 20 L/min B. 18 L/min C. 22 L/min D. 15 L/min B.

A physician orders an increase in the I:E ratio for a patient diagnosed with ARDS. Which of the following is consistent with this change? Decreased flow rate Increased expiratory time Increase in inspiratory time I:E of 1:2 to 1:4 1. Yes No No Yes 2. Yes Yes No No 3. Yes No Yes No 4. No No Yes Yes

A. 4 B. 2 C. 3 D. 1 C.

A physician orders an increase in the I:E ratio for a patient diagnosed with ARDS. Which of the following is consistent with this change? Decreased flow rate Increased expiratory time Increase in inspiratory time I:E of 1:2 to 1:4 1. Yes No No Yes 2. Yes Yes No No 3. Yes No Yes No 4. No No Yes Yes

A. 4 B. 3 C. 2 D. 1 B.

The following data is obtained on an infant 5 minutes after birth: Color blue extremities, pink body Heart rate: 94/min Respiratory effort: weak cry Muscle tone some flexion of the extremities Grimace slight grimace noted Based on this information, what represents an appropriate APGAR score?

A. 5 B. 6 C. 4 D. 7 A.

A respiratory therapist notes the word cachectic used in the medical documentation to describe a patient. This is most consistent with

A. a wasted apperance B. obtunded C. obesity D. non-attentiveness A. The word "cachectic" refers to a wasted appearance, skinny, and poor skin turgor. A person with AIDS or pulmonary tuberculosis may demonstrate this.

Which of the following chest radiograph observations is most associated with ARDS?

A. hyper radiolucency B. obliterated costophrenic angles C. concave interface border D. scattered patchy infiltrates D. Scattered patchy infiltrates is associated with adult respiratory distress syndrome. Hyper radiolucency is associated with chronic air trapping, such as seen in COPD. A concave interface border is associated with the pleural effusion.

A 68-year-old, male patient with COPD is participating in a pulmonary rehabilitation program. During a walking activity, the patient's SpO2 falls to 88% while receiving 1 L/min by nasal cannula. Which of the following would be an appropriate reaction to the decline in oxygenation during exercise?

A. Continue the activity, monitor the patient's subjective condition. B. Avoid the walking activity for one month then reassess. C. Ask the patient how he feels. D. Increase oxygen to 3 L/min during the activity. D.

The purpose in using PEEP in cases involving acute respiratory distress syndrome is to

A. accomplish a lower A-a gradient. B. prevent lung injury. C. decrease pulmonary vascular resistance. D. increase FIO2. A. PEEP works by expanding the alveolar gas-exchanging surface area, which allows more oxygen to permeate across the alveolar capillary membrane, resulting in a decreased A-aDO2 (gradient). PEEP usually results in higher PVR and can increase the chance of lung injury due to exposure to excessive pressures.

What is the relative humidity if there is only 11 mg of water/L of gas in the ambient air?

A. 100% B. 75% C. 25% D. 50% C. When relative humidity is 100% the ambient air is holding 44 mg/H2O/L of gas. If only 11 mg/H2O/L of gas exists, the relative humidity can be determined by dividing the 11 mg present by the 44 mg total. In this case, the relative humidity is 25%.

Which of the following should the respiratory therapist evaluate to determine if the patient's perfusion is adequate?

A. blood urea nitrogen level (BUN) B. urine output C. arterial blood gas analysis D. pulse oximetery B. Urine output is the most significant indicator of perfusion and is therefore the best assessment of perfusion adequacy.

A Bronchogram would be most helpful in evaluating and diagnosing which of the following?

A. chronic bronchitis B. bronchiectasis C. ARDS D. mycoplasma pneumonia B. A bronchogram is a procedure used to diagnose bronchiectasis - not helpful for Mycoplasma pneumonia, chronic bronchitis, or adult respiratory distress syndrome.

Which of the following radiographic findings would be consistent with a pleural effusion?

A. engorgement of vessels in the mid lung field B. concave superior interface C. increased vascular markings throughout D. hilar densities B.

Which of the following would result in an increase in CVP?

A. increased SVR B. hypervolemia C. hypokalemia D. decreased PVR B. When the body retains fluid all hemodynamic values are increased especially CVP. By itself a high CVP most likely indicates fluid retention and hypervolemia.

While taking an IPPB treatment, the patient begins to complain of dizziness and tingling in the extremities. The respiratory therapist notices the patient is breathing by actively inhaling with each breath followed by a forceful exhalation. The therapist should

A. instruct the patient to inhale slowly and exhale passively B. decrease expiratory time C. increase inspiratory pressure D. decrease flow A. The proper breathing pattern for an IPPB treatment is active inhalation followed by a passive exhalation.

A low-pressure ventilator alarm is sounding on a patient receiving VC, A/C ventilation. The respiratory therapist should suspect which of the following?

A. loose circuit connection B. excessive peak flow rate C. herniated ET tube cuff D. patient biting on the ET tube A. All of the options listed would more likely result in a high-pressure alarm. Only the presence of a loose circuit connection, in this situation, would result in an alarm related to low-pressure.

A patient is in the emergency room with a fever. Blood pressure is 90/55 mm Hg. The ER physician is concerned about septic shock and orders the infusion of fluid. What exams would be most helpful in assessing the infection status to the patient?

A. nasal swabbing B. chest radiograph C. CBC D. oral temperature C.

A patient receiving oxygen by nasal cannula set at 3 L/min is having difficulty maintaining an adequate oxygen saturation. Minute ventilation is 15 L/min. The therapist should do which of the following to ensure adequate oxygenation of the patient?

A. non-invasive positive pressure ventilation B. Increase flow to 5 L/min C. Place patient on Venturi mask at 32% D. Change to a partial rebreathing mask C. 3 L/min by nasal cannula approximates 30 to 34% oxygen percentage. With a nasal cannula, 1 L approximates a 3-4% change in oxygen.

What is the normal range for RAW?

A. 3.0-6.0 cm H2O/L/sec B. 0.6-2.4 cm H2O/L/sec C. 10.0-15.0 cm H2O/L/sec D. 2.5-3.5 cm H2O/L/sec B.

A patient with ARDS and asthma could benefit from which of the following medications?

A. exogenous surfactant B. Tobramycin and albuterol C. Spiriva and decadron D. cromolyn sodium B. A patient with adult respiratory distress syndrome could benefit from bronchodilators to open up the airway and surfactant therapy to decrease the surface tension of the alveoli.

A patient on the medical floor is receiving supplemental oxygen at 4 L/min by nasal cannula. A pulse oximeter is showing an SPO2 of 74% with a heart rate of 68 bpm. The respiratory therapist determines the pulse by palpation is 98 bpm. The therapist should? A. replace the electrode with a new one B. switch the patient to 50% Venturi mask C. try a different location with the pulse ox probe D. increase flow rate to 5 L/min

try a different location with the pulse ox probe

Which of the following will provide a continuous elevation of baseline pressure during inspiration and expiration on a patient who is breathing spontaneously?

A. NAVA B. APRV C. IPPB D. CPAP D. The description provided in this question is most consistent with CPAP, also called continuous positive airway pressure.

Which of the following units of measure are associated with systemic vascular resistance?

A. dynes-min/cm2 B. dm2 /min C. L/min/m3 D. dynes/sec/cm5 D. SVR = (MAP-CVP)/C.O. X 80 The unit of measure is dynes, or more specifically, dynes/sec/cm5.

A patient has had a large growth removed from his vocal cords. Which of the following test would be most helpful in assessing the impact on airway resistance?

A. flow volume loop B. mPAP C. FRC D. Fev1/FVC% A

Which of the following is most likely true for a patient whose hemodynamic values (mPAP, CVP, PCWP, and C.O.) are all low?

A. Congestive heart failure is present. B. The patient is in need of fluids. C. The patient is in need of diuresis. D. Pulmonary hypertension is present. B. Low hemodynamic values (all low simultaneously) suggest hypovolemia. As such, the patient is in need of fluid administration.

Which of the following would be increased in a patient with advanced COPD?

A. FEF200-1200 B. Fev1 C. Fev1/FVC% D. FRC D.

A physician notes a round mass in the right lower lung field when observing the results of an AP chest radiograph. Which of the following would be most helpful to further evaluate the shape of the mass?

A. spiral CT scan B. lateral decubitus radiography C. PA chest radiography D. lateral neck X-ray A. To further evaluate what appears to be a round mass in the lung field, a radiological procedure is appropriate. The most appropriate radiological procedure would be that which shows three dimensionality. In this case a spiral CT scan will provide that perspective. All other options offered would only show a two dimensional version of the concern

Two hours after abdominal surgery, an orally intubated patient with no history of pulmonary disease is alert and agitated and is repeatedly attempting to pull on on the ET tube. Current ventilator settings are: SIMV, rate 10/min, VT 450 mL, FIO2 0.4, PEEP 5 cm H2O, PS 5 cm H2O. The respiratory therapist should consider

A. spontaneous breathing trials. B. extubation. C. sedation of the patient. D. application of a soft hand restraint system. B.

Which of the following Carboxyhemoglobin levels is consistent with that of a regular, frequent smoker?

A. 2.9% B. 1.5% C. 20% D. 6.5% D. Regular frequent smokers will commonly have carboxyhemoglobin levels between 5 and 10%.

A patient with the following blood gas results has an end-tidal CO2 of 30 mm Hg. pH 7.38 PaCO2 40 mm Hg PaO2 80 mm Hg HCO3- 25 mEq/L What is the VD/VT?

A. 30% B. 50% C. 25% D. 62% C. VD/VT = (PaCO2 - PetCO2) / PaCO2. In this case, VD/VT = (40-30)/40 = 10/40 or 25%.

Which of the following can be used to determine the trend in cardiac output?

A. A-aDO2 B. Charles' Law C. C(a-v)O2 D. Bohr's C.

Which of the following would be most helpful at preventing alveolar collapse on a patient who is receiving VC, A/C ventilation?

A. VT at 10 mL/kg B. APRV C. appropriate PEEP level D. periodic discontinuance of sedation C.

Which of the following tests would be most helpful at diagnosing a patient with cystic fibrosis?

A. calorimetry B. sweat chloride test C. pulmonary function studies D. metabolic studies B. Cystic fibrosis is primarily diagnosed with a simple test called a sweat chloride test.

A respiratory therapist is given the charge to sterilize a bronchoscope. Which of the following should the respiratory therapist use to accomplish sterilization?

A. Alkaline gluteraldehyde (Cidex) B. Pasteurization C. Irradiation followed by ethylene oxide D. Steam autoclave A. The best agent to use when sterilizing a bronchoscope is alkaline glutaraldehyde (Cidex).

A patient is suspected of having a fixed upper airway obstruction. Which of the following would be helpful in assessing for this suspicion?

A. DLCO B. SBN2 C. TLC D. FVL D

A patient with asthma who is currently wheezing would benefit most from which of the following medications?

A. Fluticasone B. Atrovent (Ipratropium Bromide) C. Atropine sulfate D. Tilade (Nedocromil) B. A patient with asthma would benefit most from a bronchodilator medication. The most appropriate short-term bronchodilator medication that is appropriate in this case is Atrovent. Atropine is also a bronchodilator but is rarely used.

A 22-year-old male with CF has procured a job that requires traveling over 50% of the time. What bronchial hygiene technique would be most appropriate during the patient's travel?

A. PEP therapy B. Quad cough technique C. VEST therapy D. IPV A.

During the administration of 2.5 mg nebulized albuterol, a patient complains of nausea and palpations. Heart rate has increased from 98 to 128/min. After terminating the treatment and reporting to the appropriate health care team members, which of the following should the respiratory therapist recommend for the next treatment?

A. Switch to 0.63 mg Xopenex B. Change to 2 puff of beclomethasone (Vanceril) C. Switch to ipratropium bromide (Atrovent) q2 hrs D. Change to fluticasone by MDI A. When the patient experiences an adverse reaction to any particular medication, rather than decrease the dosage of that medication to a low or minimal therapeutic dosage, it is better to switch to a different medication in the same class. In this case, it is appropriate to switch from albuterol to Xopenex.

Immediately after the insertion of a nasal pharyngeal airway, a patient begins to cough uncontrollably. The most likely reason is that the

A. diameter of the nares is less than the outer diameter of the airway. B. insertion has damaged the mucosal tissue. C. patient is allergic to the rubber/plastic material. D. airway is too long. D.

A 35-weeks of gestation infant has a one-minute APGAR score of 4. What therapy is most appropriate?

A. supplemental oxygen B. cardiopulmonary resuscitation C. dry, warm, and monitor the infant D. institute time-cycled ventilation A.

Which of the following patient instructions is appropriate for a 12-year-old male patient who is about to have an appendectomy?

A. "After surgery, you will need to take several deep breaths every hour." B. "Once your anesthesia wears off we will start you on incentive spirometry." C. "When you cough you must splint your incision." D. "During surgery, we will be monitoring your end tidal CO2." A. Lung expansion following abdominal surgery is important in order to avoid pulmonary complications such as atelectasis. Also, in this example, several answers contain the use of terminology that would not be understood by a 12-year old patient.

Which of the following instructions should be given to a patient in preparation for incentive spirometry with a volume-type device?

A. "exhale to expiratory reserve volume" B. "inhale as deeply as possible" C. "breathe in to total lung capacity" D. "breathe in complete inspiratory capacity" B. When instructing a patient on a procedure, the respiratory therapist must use non-medical lingo. For instance, the patient is unlikely to understand total lung capacity, or inspiratory capacity, or expiratory reserve volume. Thus, the best instruction is to "inhale as deeply as possible".

A patient is ordered to be placed on a high frequency jet ventilator in response to a bronchopleural fistula. Which of the following ventilator parameters can the respiratory therapist expect to manipulate in place of the tidal volume control, which can be found on a volume-controlled ventilator?

A. % expiratory time B. Drive pressure C. Rate D. Injector line B. High-frequency jet ventilators do not have a control to set tidal volume directly. Delivered volume is set indirectly through a control called drive pressure, sometimes called oscillatory amplitude.

A respiratory therapist is calibrating the Wheatstone Bridge helium analyzer. What value would the respiratory therapist expect to observe when calibrating the analyzer to air?

A. 0.8% B. 0.2% C. 0% D. 21% C. Because there is virtually no helium in ambient air, calibrating a Wheatstone-Bridge helium analyzer with air should result in a value of 0%.

What side effects are associated with inhaled mucomyst? Tachycardia atelectasis rhinorrhea bronchospasm 1. YES NO NO YES 2. YES YES NO YES 3. NO YES YES NO 4. NO NO YES YES

A. 1 B. 4 C. 2 D. 3 B.

Which of the following is required to calculate minute alveolar ventilation for a patient receiving PC, A/C ventilation 1. weight (lbs) 2. tidal volume 3. inspiratory pressure setting 4. minute ventilation

A. 1 and 2 only B. 1, 3, and 4 only C. 1, 2, and 4 only D. 2 and 3 only C.

The respiratory therapist notes an order in a patient's record for 2 mg of a drug. The normal stock concentration of the drug is 0.05%. How many mL will be required to meet the ordered dosage?

A. 1.0 mL B. 4.0 mL C. 2.5 mL D. 10.0 mL B. To determine the number of mL needed, one must first determine how many mg per mL with that specific concentration of mediction. This can be done by taking the concentration (strength) and multiplying it by 10. ie, 0.05% x 10 = 0.5 mg/mL. In this case, 2 mg is needed. Thus 2 mg/0.5 mg/mL = 4.0 mL.

When selecting a suction catheter to be used in an oral endotracheal tube, the respiratory therapist should select a catheter whose diameter should not exceed what fraction of the internal diameter of the endotracheal tube?

A. 1/4 B. 2/3 C. 1/2 D. 3/4 C. The outer diameter of a suction catheter should not exceed one half of the inner diameter of an endotracheal tube.

Which of the following would NOT be needed in preparation for a bronchoscopy?

A. 10 cc syringe B. normal saline C. anesthetic D. Magill forceps D. There is no purpose for bringing Magill forcepts to assist with a bronchoscopy procedure. Magill forcepts are used primarily for nasal intubation.

How many hours will an H cylinder with 1400 psi last for a patient receiving oxygen at 7 lpm?

A. 10 hours B. 4 hours C. less than 1 hour D. 6 hours A. And H cylinder has a tank factor of 3.14. 3.14x1400 PSI = 4396 L. 4396 L / 7 L per minute = 628 min. 628 min./ 60 min.= 10.4 hours, or about 10 hours. When you get an answer that is not exact, pick the closest number possible. It is common on the exam not see the exact answer from your calculation. This is because the NBRC knows that you're estimating.

An orally intubated patient is breathing spontaneously through an 8.0 mm ET tube. In the ambient air, humidity indicates only 22 mg per liter of air. Based on this, what humidity deficit will the respiratory therapist have to make up to completely meet the humidity needs of the patient?

A. 100 mg/L of gas B. 44 mg/L of gas C. 12 mg/L of gas D. 22 mg/L of gas D. When relative humidity is 100%, the ambient air is holding 44 mg/H2O/L of gas. If the ambient air is only holding 22 mg/H2O/L, there is another 22 mg/H2O/L that is missing. This is known as the humidity deficit. Humidity deficit is always expressed in milligrams whereas relative humidity is expressed as a percentage.

A post-operative patient was able to achieve an inspiratory capacity of 2000 mL prior to surgery. After repeated attempts, the patient is only able to achieve 400 mL. Which of the following represents an appropriate patient goal?

A. 1000 mL B. 1600 mL C. 400 mL D. 500 mL D. The proper goal for incentive spirometry postoperatively is one half of what was achieved preoperatively. However, if the patient is unable to come close to that number, modification of the goal is appropriate. Since the incentive spirometer is supposed to incentivize or encourage its continued use, the most appropriate goal would be a number just slightly higher than the number the patient is able to achieve.

Which of the following pressures is most consistent with a normal pulmonary artery pressure (PAP)?

A. 12/8 mmHg B. 25/4 mmHg C. 25/8 mmHg D. 120/80 mmHg C. To answer this question you must memorize normal mPAP. Normally we look at PAP as a mean value and therefore has a single number. But, it can also be expressed as a systolic over a diastolic number. In this case 25/8 mmHg.

A patient is receiving volume-controlled ventilation in the emergency department (ED). The following data is available: Mode Assist/control VT 550 mL Mandatory rate 10 FIO2 0.5 PEEP 5 cm H2O PetCO2 10 torr pH 7.39 PaCO2 40 torr PaO2 100 torr HCO3- 25 mEq/L BE +1 mEq/L The respiratory therapist should report which of the following as an accurate VD/VT ratio?

A. 15% B. 50% C. 25% D. 75% D. To determine the deadspace-tidal volume ratio (VD/VT), we take arterial CO2 and subtract end-tidal CO2 and divide that by the arterial CO2. - (PaCO2-PetCO2)/PaCO2 - In this case, end-tidal CO2 subtracted from arterial CO2 is 30. 30 divided by 40 is equal to 75%.

The following data is available for a patient receiving volume-controlled mechanical ventilation: Mode Assist/control FIO2 0.5 VT 400 mL Mandatory rate 16 PEEP 5 cm H2O PIP 30 cm H2O PetCO2 30 torr pH 7.40 PaCO2 40 torr PaO2 80 torr PvO2 60 torr HCO3- 22 mEq/L BE -2 mEq/L SaO2 99% SvO2 75% Hb 15 g/dL The respiratory therapist should record which of the following values as an accurate CvO2?

A. 15.3 vol% B. 17.0 vol% C. 12.1 vol% D. 2.9 vol% A. To determine the venous oxygen content, the hemoglobin must be multiplied by the saturation which is then also multiplied by a factor of 1.34. In this case hemoglobin x 1.34 is approximately 20. 20 multiplied by 0.75 is approximately 15 vol%. Because we're using shortcuts the exact value in the answers may not be seen. However, it is suitable to pick the closest answer, which is 15.3 vol%_x000D_

A 65-kg (143-lb) 49-year-old patient in the cardiac intensive care unit is receiving mechanical ventilation on the following settings with the following data: Mode Assist/control Tidal volume 500 mL Mandatory rate 16 FIO2 0.8 PEEP 15 cm H2O Inspiratory flow 50 L/min CVP 8 mm Hg PAP 36/8 mm Hg C.O. 3.9 L The therapist should report which of the following values mPAP?

A. 17 mm Hg B. 36 mm Hg C. 8 mm Hg D. 22 mm Hg A. Mean PAP, as with MAP, is determined by adding the systolic pressure to twice the diastolic pressure and dividing the sum by three. In this case, 36 + 8 + 8 = 52. 52/3 = 17 mm Hg.

A patient is receiving 60% aerosol with the oxygen flow meter running at 14 L/min. What is the total flow of gas to the patient?

A. 20 L/min B. 56 L/min C. 14 L/min D. 28 L/min D. To determine the answer to this question, one must have memorized the air-oxygen ratios. For 60%, the ratio is 1:1. This means that the Venturi mechanism will incorporate 1 part air for every 1 part oxygen. One way to solve this problem is to add the ratio numbers together. (1+1= 2). Then, multiply that total by the indicated flow on the oxygen flow meter. In this case, we multiply 14 L per minute by 2. We get a total flow of 28 L/min.

A patient in the intensive care unit has the following clinical and laboratory data: PB 747 mm Hg PH2O 47 mm Hg pH 7.28 PaCO2 52 mm Hg PaO2 70 mm Hg HCO3- 27 mEq/L BE +2 mEq/L FIO2 0.6 SaO2 1.0 CvO2 12 vol% CaO2 17 vol% Hb 15 gm/dL Which of the following represents the C(a-v)O2?

A. 200 vol% B. 2 vol% C. 5 vol% D. 25 vol% C. To answer this question, one must complete two calculations. First, CaO2 and CvO2 must be determined. Once CvO2 is subtracted from CaO2, 5 vol% is the closest answer.

A patient is receiving helium-oxygen (80%/20%) by compensated flow meter at 12 L/min. What is the total liter flow of gas to the patient?

A. 24 B. 18 C. 20 D. 22 D. When helium oxygen gas flows through an oxygen flow meter, the total flow of gas is greater than that indicated on the meter. For 80%/20% mixture, the total flow conversion factor is 1.8. Thus, 12 L/min X 1.8 = 21.6. The closest answer is 22.

A patient is receiving 28% aerosol with the oxygen flow meter running at 6 L/min. What is the total flow of gas to the patient?

A. 24 L/min B. 66 L/min C. 18 L/min D. 44 L/min B. To determine the answer to this question, one must have memorized the air-oxygen ratios. For 28%, the ratio is 10:1. This means that the Venturi mechanism will incorporate 10 parts air for every 1 part oxygen. One way to solve this problem is to add the ratio numbers together. (10+1= 11). Then, multiply that total by the indicated flow on the oxygen flow meter. In this case, we multiply 6 L per minute by 11. We get a total flow of 66 L/min.

A patient receiving mechanical ventilation has the following pressure-volume loop during a machine delivered breath. Which of the following represents the patient's dynamic compliance?

A. 25 mL/cmH2O B. 32 mL/cmH2O C. 60 mL/cmH2O D. 40 mL/cmH2O D. Most of the time we are given numbers to calculate the patient's dynamic compliance. In this case, however, we are given a graph. Examination of the graph reveals a peak pressure of 25 cmH2O. Closer examination shows that the patient is receiving a PEEP of 5 cmH2O. This means that the peak pressure is 20 cmH2O. When we take the volume, 800 mL, and divide it by 20 cmH2O, we get a dynamic compliance of 40 mL/cmH2O.

A patient who has a 20-year history of smoking cigarettes reports smoking 1.5 packs a day on average over the course of 10 years and 1 pack a day for the remaining ten years. The respiratory therapist should estimate the pack years of the patient to be

A. 25 pack years B. 40 pack years C. 30 pack years D. 45 pack years A. When determining a patient's smoking history pack years are calculated by multiplying the number of years the patient smoked by the average number of packs per day. In this case, the patient has smoked 1.5 packs per day over the course of 10 years. When multiplied together, the smoking history is 15 pack-years. Additionally, the patient smoked one pack a day for an additional 10 years which would add an additional 10 pack years for a total of 25 pack years.

A patient in the intensive care unit has the following clinical and laboratory data: PB 747 mm Hg PH2O 47 mm Hg pH 7.28 PaCO2 52 mm Hg PaO2 70 mm Hg HCO3- 27 mEq/L BE +2 mEq/L FIO2 0.6 SaO2 1.0 CvO2 12 vol% CaO2 17 vol% Hb 15 gm/dL Which of the following represents the C(a-v)O2?

A. 25 vol% B. 2 vol% C. 200 vol% D. 5 vol% D. Explanation : To answer this question, one must complete two calculations. First, CaO2 and CvO2 must be determined. Once CvO2 is subtracted from CaO2, 5 vol% is the closest answer.

A patient is suspected of having a right-side pneumothorax. What additional clinical evidence would be present if the patient is found positive for this suspicion? Tachycardia Distant BS Right tracheal deviation Tachypnea 1. Yes No No Yes 2. Yes Yes No Yes 3. No Yes Yes Yes 4. No No Yes No

A. 3 B. 4 C. 2 D. 1 C. A pneumothorax would likely cause other clinical signs such as tachycardia, tachypnea, distant breath sounds, and tracheal deviation AWAY from the affected side (to the left, not the right).

Which of the following is the tank factor for an H-cylinder?

A. 3.41 B. 3.14 C. 2.41 D. 0.30 B. 3.14 is the tank factor associated with an H cylinder.

In monitoring a newborn for apnea with an apnea monitor, how many seconds of apnea is enough to be considered an apneic event?

A. 30 sec B. 10 sec C. 5 sec D. 20 sec D. In newborns, apnea is somewhat common for periods below 20 seconds. Apnea lasting greater than 20 seconds is considered to be a notable event and would be reordered as an official apneic episode.

A patient in the intensive care unit has the following clinical and laboratory data: PB 747 mm Hg PH2O 47 mm Hg pH 7.28 PaCO2 52 mm Hg PaO2 70 mm Hg HCO3- 27 mEq/L BE +2 mEq/L FIO2 0.6 SaO2 100% CaO2 17 vol% CvO2 12 vol% Hb 15 gm/dL Which of the following represents the oxygen tension in the alveoli?

A. 370 mmHg B. 420 mmHg C. 540 mmHg D. 220 mmHg A. The alveolar oxygen tension may be determined by taking the oxygen percentage and multiplying it by 7 and then subtracting the CO2 +10. In this case is 60% x 7 = 420 mmHg. When we subtract the CO2 +10 we get 370 mmHg.

What are side effects associated with albuterol? Tachycardia nausea tremors heavy palpitations 1. YES NO NO YES 2. YES YES NO YES 3. NO YES YES NO 4. YES NO YES NO

A. 4 B. 2 C. 1 D. 3 A.

A respiratory therapist working in a pulmonary rehabilitation clinic is asked to pursue a smoking cessation program with a patient who has a 90 pack-year history of smoking. The therapist will expect to include which of the following approaches with the patient? Behavior counseling Weekly COHb assessment Nicotine replacement Withdrawal symptom management 1 Yes No Yes Yes 2 No No Yes Yes 3 Yes Yes Yes Yes 4 No No Yes Yes

A. 4 B. 2 C. 1 D. 3 C.

A child has been extubated and is demonstrating mild stridor. Which of the following agents would be most helpful to administer by aerosol?

A. Aminophylline B. racemic epinephrine C. Atropine D. Albuterol B. A child who has mild stridor can be treated by one of two ways - administration of racemic epinephrine or administration of a cool aerosol. Moderate stridor requires the use of racemic epinephrine and severe or marked stridor would require intubation.

Which of the following would be the best medication to use with a patient who will require oral intubation in an emergent setting?

A. Anectine B. Versed C. Flurazepam D. Valium A. The drugs most commonly used to facilitate intubation are Anectine (succinylcholine), Pavulon (pancuronuim bromide), Curare (d-tubocurarine) and Vecuronium. Valium and Versed are anxiety/pain relievers. Flurazepam is used to treat insomnia.

A 42-year-old female presents in the emergency department (ED) complaining of body aches and general malaise. The patient reports sickness over the last two days with excessive vomiting and diarrhea. Which of the following laboratory tests should the respiratory therapist suggest?

A. Arterial blood gas B. Theophylline level C. Serum electrolytes D. Blood urea nitrogen (BUN) C. Body aches and general malaise is most often a clear indicator of electrolyte imbalance. This suggestion is further indicated by the fact that the patient has been expectorating fluid in various forms and has likely become dehydrated. Loss of fluid often results in electrolyte imbalance.

A patient's radiological image is reported to have hyperlucency with diffuse dry crackles auscultated throughout the chest. These findings are most closely associated with

A. Atelectasis B. pulmonary edema C. subcutaneous emphysema D. Pneumonia C. Auscultated dry crackles is the main sign of subcutaneous emphysema in the scenario. Increased radiolucency in the lung is also an indication that extra air exists in the chest, even though it is on the outside of the lung. Pulmonary edema would show fluffy infiltrates. Pneumonia is more localized and is associated with a reduced radiolucency.

BID is a common frequency order for which of the following respiratory medications?

A. Ativan B. Flovent C. Albuterol D. Xopenex B. Flovent (fluticasone) is a corticosteroid used generally to prevent and relieve inflammation of airway walls. Inflammation is one of the components of asthma so corticosteroids are key in treatment. The frequency is twice per day or b.i.d. Flovent may cause Candidiasis, (oral yeast infection), which may be prevented by judiciously rinsing the mouth after inhaler use.

Which of the following medications should the respiratory therapist recommend to provide a neurmuscular blockade and facilitate intubation?

A. Ativan B. Xanax C. Morphine D. Pavulon D. The drugs most commonly used to provide a neurmuscular blockade (paralyze) to the patient and facilitate intubation are Anectine (succinylcholine), Pavulon (pancuronuim bromide), Curare (d-tubocurarine) and Vecuronium. The others listed here are merely anxiety controlling medication and will not provide paralysis.

Auscultation of a child's breath sounds reveals unilateral wheezing in the right side. Which of the following should the respiratory therapist recommend to be most helpful to the patient?

A. Atropine B. Salmeterol C. Bronchoscopy D. Albuterol C. Bilateral wheezing is associated with bronchoconstriction. When the bronchials constrict, they are not usually capable of constricting on one side of the lung only. Therefore, unilateral wheezing indicates the problem must be something other than bronchoconstriction and is most likely associated with foreign body aspiration. To remove the object a bronchoscopy is most appropriate.

The respiratory therapist observes an ECG wave form on a patient that is consistent with atrial tachycardia. The patient is complaining of chest pain, dizziness, and nausea. The respiratory therapist should recommend

A. Atropine sulfate B. cardioversion C. unsynchronized defibrillation D. epinephrine B. Non-deadly arrhythmias, such as this one, may be addressed through cardioversion. Cardioversion is a form of defibrillation with low wattage and with the synchronization set to "active". This allows the shock to be sychronized to the R wave.

The following arterial blood gas results are obtained on a patient who has been on mechanical ventilation for 4 hours. pH 7.30 PaCO2 49 torr PaO2 95 torr HCO3- 18 mEq/L BE -8 mEq/L FIO2 0.55 SaO2 98% The respiratory therapist should suspect an error in recording which of the following values?

A. BE B. pH C. HCO3- D. PaCO2 C. Because PaCO2 is 49 mmHg, the expected pH is about 7.30 or 7.31. The pH, in this problem, falls in that range and therefore the problem is respiratory acidosis, or acidosis caused purely by the respiratory system. Closer observation of the blood gas, however, reveals a HCO3 that is low. This would cause pH to fall below the reported value and therefore suggests an error in the HCO3- result.

A 3-year-old patient is demonstrating ventilatory difficulty with use of intercostal accessory muscles. The patient is febrile and is drooling. A softened inspiratory stridor is audible. Which of the following mostly likely represents the patient's condition?

A. Bacterial pneumonia B. Asthma C. Laryngotracheal bronchitis (croup) D. Acute epiglottitis D. Softened inspiratory stridor in conjunction with drooling is associated with acute epiglottitis. Acute epiglottitis is a bacterial infection, which can cause febrile conditions.

A patient with bronchiectasis is ordered to receive postural drainage and percussion. A bronchogram reveals a concentration of secretions in the anterior basal and lateral basal segments. In which of the following positions should the therapist place the patient to drain these areas?

A. Bed flat, lying on one side B. Supine, bed flat C. Trendelenburg - 15 degrees D. Trendelenburg - 30 degrees D. The basal segments of the lungs are drained by placing the patient's head of bed down 30° or 18 inches.

A radiographic image shows an upper lobe cavitation. Which of the following conditions is most closely associated with this finding?

A. Bronchiectasis B. Tuberculosis C. Asbestosis D. Pneumonia B. Cavitations in the upper lobes are generally associated with tuberculosis. Bronchiectasis is a condition affecting the bronchioles or airways and is seen throughout the lung. Pneumonia is not a cavitation and can also be seen throughout the lung on X-ray. Asbestosis is seen throughout the lung with reduced lucency.

Which of the following is needed to calculate minute alveolar ventilation?

A. C(a-v)O2 and VT B. PAO2 and tidal volume C. VD/VT and PAO2 D. tidal volume, weight, respiratory rate D. There are three items you need to calculate a patient's minute alveolar ventilation- tidal volume, respiratory rate, and the patient's weight in pounds. If the tidal volume is not given you may need to calculate the tidal volume by looking at minute ventilation and dividing it by the rate first.

A patient scheduled for abdominal surgery can be assessed for potential post-operative risk for complications by assessing which of the following?

A. C(a-vDO2) B. alveolar oxygen tension on room air (FIO2 0.21) C. basic spirometry D. incentive spirometry with flow-type spirometer C. Potential postoperative risk may be assessed preoperatively through basic spirometry.

A chest radiograph indicates the presence of plate-like infiltrates. This finding is most closely associated with which of the following?

A. CHF B. mesothelioma C. mycoplasma pneumonia D. atelectasis D. The radiological description of platelike infiltrates is most closely associated with atelectasis.

Which of the following patients or conditions would benefit most from pressure control ventilation?

A. COPD B. pneumonia C. myasthenia gravis D. ARDS D. Pressure control ventilation is primarily helpful for patients with adult respiratory distress syndrome. They require significant pressures to deliver volumes. When pressures get over 50 cmH2O, switching to pressure control ventilation will help to reduce barotrauma and increase alveolar ventilation.

A patient with a history of +3 pitting edema is in severe respiratory distress while receiving oxygen by nonrebreather mask. BS reveal crackles in the bases. The patient is expectorating a slight amount of bloody, frothy secretions. What additional therapy should the respiratory therapist consider?

A. CPAP 5 cm H2O, FIO2 1.0 B. percussion by Vest C. nitric oxide (NO) inhalation D. helium-oxygen, 80/20% A. Clinical evidence indicates the presence of congestive heart failure, which is precipitating cardiogenic pulmonary edema. Application of CPAP will help suppress the pulmonary edema and improve oxygenation.

The physician has asked the respiratory therapist to help determine oxygen consumption by the tissues. The therapist should suggest evaluation of which of the following?

A. PaO2 B. CaO2 C. CvO2 D. C(a-v)O2 D. Oxygen available at the tissue level is best determined by examining the arterial oxygen content or CaO2. However, to determine how much oxygen the tissue is consuming we must look at both the CaO2 and the CVO2. This allows us to determine how much oxygen existed before the tissues and how much oxygen exists after the tissues. When we subtract one from the other we get the C(a-v)O2. This value is most closely related with oxygen consumption at the tissue level.

Which of the following will help a patent to speak while intubated with a tracheostomy tube?

A. Passy-Muir valve B. double lumen endotracheal tube C. inner cannula D. obturator A. In this question one must recognize that a Passy-Muir valve is appropriate for a patient who is intubated with the tracheostomy tube. This allows the patient to speak. This valve allows inspiration to go through into the lungs but closes during expiration to redirect air over the vocal folds, allow speech to occur.

After analyzing arterial blood on a patient with a history of arterial blood gas analysis, the respiratory therapist concludes the results are not possible. They are very different than the patient's historical blood gas results. Quality control records show no evidence of any trends, shifts, or out-of-control situations in the last month. The therapist should

A. Perform a two-point calibration on the analyzers B. Repeat the analysis with the same blood sample and same analyzer C. Ask the medical director for direction in this situation D. Inform the physician of the suspect blood gas B. Suspicious blood gas results is cause to repeat the analysis utilizing the same blood.

2 PM 5 PM Peak airway pressure 45 cm H2O 56 cm H2O Static airway pressure 35 cm H2O 46 cm H2O PEEP 5 cm H2O 5 cm H2O Delivered tidal volume 750 mL 750 mL Inspiratory flow 55 L/min 55 L/min The following data is available for a patient receiving mechanical ventilation: What is the next action the respiratory therapist should take?

A. Perform bronchial hygiene B. Decrease tidal volume C. Increase inspiratory flow D. Obtain a chest radiograph D. This data shows an increase in static airway pressure which indicates the lungs are becoming stiffer or less compliant. There is no specific, immediate treatment for this but additional data may be gathered to determine the possible cause of the change in compliance. Of the options offered, obtaining a chest radiograph is most appropriate.

Which phase of the nitrogen elimination (SBN2) test should the therapist primarily look at to find a mixture of deadspace and alveolar gas?

A. Phase III B. Phase I C. Phase II D. Phase IV C. A single breath nitrogen elimination test (SBN2) I is useful in determining the evenness of gas distribution in the lungs. The results come in four phases as the patient exhales a single breath. Phase I is the exhalation of pure deadspace gas. Phase II consists of some deadspace and some alveolar gas. Phase III consists of pure alveolar gas and is the phase that indicates the evenness of distribution. Phase IV is called "closing volume".

An 80-kg (176-lb) male patient was placed on mechanical ventilation 3 days ago for respiratory failure secondary to septicemia. After a one-hour trial on a T-tube Briggs adapter, the patient is having periods of 30 seconds of apnea. What should the respiratory therapist do FIRST?

A. Place the patient on pressure control ventilation B. Place the patient on pressure support C. Provide verbal stimulation to the patient to encourage spontaneous breathing D. Place the patient on CPAP 5 cm H2O A. Due to the apneic episodes, the primary focus of the respiratory therapist should be to re-establish effective ventilation. In this example the only choice that provides ventilation is placing the patient on pressure control. The respiratory therapist may feel uneasy selecting this option, but it is the only ventilatory mode, and therefore must be selected.

During a routine check of a patient on mechanical ventilation, the respiratory therapist palpates less chest rise on the left compared to the right during inspiration. The therapist should first do which of the following?

A. Prepare for needle decompression in the left chest B. Auscultate breath sounds bilaterally for equal air movement C. Check the integrity of the ET tube cuff D. Advance the endotracheal tube and recheck chest rise B. Asymmetrical chest movement, or chest rise during mechanical ventilation, is an indication that the endotracheal tube may be improperly placed. The first, more immediate action that should be taken is to further assess the problem by auscultating the chest. If breath sounds are absent on one side, it is appropriate to withdraw the endotracheal tube until breath sounds can be heard bilaterally. After doing so, a chest x-ray should be ordered to confirm proper placement of the endotracheal tube.

A chest radiograph on a 38-week gestational age infant shows bubble-like structures radiating outward, confirming pulmonary interstitial emphysema. The respiratory therapist should recommend which of the following in response to an order to provide ventilatory support?

A. Pressure-cycled, time-limited ventilation B. Bi-level ventilation C. Time-cycled, pressure-limited ventilation D. High frequency oscillatory ventilation D. PIE in an infant is a very serious pulmonary condition that benefits most of a high-frequency ventilation strategy.

A 6-year-old patient has been intubated in the emergency room for respiratory failure at a small rural hospital. The hospital has a volume-cycled ventilator with pediatric mode, but has no pediatric ventilator circuits. What should the respiratory therapist do?

A. Provide IPPB with normal saline QID B. Use an uncuffed endotracheal tube C. Use an adult circuit following a pre-operational test D. Use an infant designed time-cycled, pressure-limited ventilator C. A pre-operational test on most microprocessor ventilators will result in a correction to the delivered tidal volume after measuring tubing compliance of the circuit. Even without this correction, the compliance factor differences between adult and pediatric circuits is not considered significant enough to matter. Therefore, adult circuits may be used with pediatric patients if necessary.

A ventilator-dependent patient is scheduled for an MRI, which will require a transport of 90 minutes. The patient is receiving humidification with a heated- humidification system. A transport ventilator is available. How should a respiratory therapist assure that humidification will be provided to the patient during the MRI procedure?

A. Provide an HME B. Provide a pass-over humidifier C. Provide ventilation with a manual resuscitator D. Use the same humidification system in use on the primary ventilator A. A heat moisture exchanger (HME) is appropriate for a transport of only 90 min. Furthermore, the scenario does not suggest that the patient has thick, retained secretions, therefore an HME may be applied.

Six minutes after delivery, a newborn has APGAR scores of 2 and 6. What is the next best course of action?

A. Provide oxygen and reassess B. Perform cardiopulmonary resuscitation. C. Warm the heel for a capillary draw. D. Intubate and provide mechanical ventilation. A. APGAR (appearance, pulse, grimace, activity, and respiratory effort) is done on every newborn after 1-minute and after 5-minutes of life. In this case, the second score, after 5 minutes is "6". 0-3 points means that CPR should be initiated. 4-6 points means one should provide support with oxygen, warmth, and general stimulation. 7-10 points means one should provide routine care. The APGAR should be repeated every 5 minutes until it is above 6.

A 38-year-old female patient is admitted to the ICU with acute ventilatory distress and pneumonia. Arterial blood gas results show: pH 7.22 PaCO2 59 torr PaO2 62 torr HCO3- 24 mEq/L BE -2 mEq/L FIO2 0.40 The physician orders the institution of mechanical ventilation. The patient is (5 ft 1 in) 155 cm and weighs 95 kg (210 lb) The respiratory therapist will recommend which of the following initial settings?

A. Rate 12, VT 350 mL, FIO2 0.40, PEEP 10 cm H2O B. Rate 14, Vt 650 mL, FIO2 0.40, PEEP 5 cm H2O C. Rate 16, Vt 400 mL, FIO2 0.40, PEEP 5 cm H2O D. Rate 16, Vt 550 mL, FIO2 0.65, no PEEP C. This patient is obese. Therefore, we must calculate the patient's predicted body weight before calculating appropriate tidal volume range. The patient should weigh about 48 kg. Based on the patient's height and weight, appropriate tidal volume is between 288 and 480 mL. Initial rate should be 10-20. FIO2 should be 30-60% (better to be 40-55%). PEEP should be 4 to 6 cm H2O

A patient receiving volume-controlled ventilation weighs 75 kg (165 lb) and is 5 ft, 7 in (170 cm) tall and within ideal body weight. The following clinical data is available: Mode SIMV Mandatory rate 12 Total rate 32 FIO2 0.40 VT 500 mL Exhaled minute volume 9.6 L pH 7.44 PaCO2 32 torr PaO2 97 torr HCO3- 22 mEq/L BE +1 mEq/L The respiratory therapist should recommend which of the following changes?

A. Reduce mandatory rate to 6 B. Institute pressure support of 5 cm H2O C. Administer sedative medication D. Add inspiratory pause B. This patient has reduced spontaneous tidal volumes. This may be determined by taking minute ventilation and dividing it by rate. This calculation reveals a tidal volume of 300 mL, which is far below the minimum 5 mL/kg tidal volume needed to sustain life. The elevated respiratory rate, in conjunction with a low tidal volume, is an indication that the patient is in need of pressure support. Pressure support will help by increasing the size of spontaneous tidal volumes, which will help naturally lower rate and decrease work of breathing.

A patient with cystic fibrosis complains of shortness of breath during a postural drainage and percussion treatment after the head of bed is down for 10 minutes. A bronchogram has revealed consolidation of secretions in the lateral basal segment of the left lower lobe. The respiratory therapist should modify the therapy in which way?

A. Reduce the time in Trendelenburg position B. Utilize reverse Trendelenburg position C. Use a pneumatic precursor D. Switch to IPPB therapy A. A patient who experiences difficulty tolerating the head of bed down during postural drainage and percussion may benefit from decreasing the duration of the treatment.

Quality control material has published values for pH of 7.50, PO2 of 110 mmHg. Documentation indicates the range of pH is 0.5% and is 3% for PO2. Machine results are as follows: pH PO2 (mmHg) Run 1 7.39 101 Run 2 7.48 107 Run 3 7.52 112 Which quality control run(s) indicate the machine is out of control?

A. Run 1 B. Run 3 C. Runs 2 and 3 D. Run 2 A. This question is just a matter of math. You must check each pH and add and subtract 0.5% and do the same for the PO2 except add and subtract 3%. The question is which Runs for which data are out of control. Analysis shows that only Run one is out of control. Be on guard for this question. This question is a time consumer, so practice doing this kind of math before the test.

A patient with COPD who has been receiving mechanical ventilation for 3 weeks has been weaned down to the following settings: FIO2 0.30 Mandatory rate 8 Mode SIMV VT(set) 400 mL VT(spont) 320 mL PEEP 3 cm H2O PS 4 cm H2O On these settings, the following laboratory data is observed: pH 7.36 PaO2 65 torr PaCO2 70 torr HCO3- 30 mEq/L BE -1 mEq/L The respiratory therapist should recommend

A. SBT B. PRVC C. APRV D. NAVA A. This data shows a blood gas for a COPD patient. Although CO2 is elevated, pH is compensated. All other data suggests the patient is ready to begin ventilator liberation. Because it can be difficult to wean a COPD patient from a ventilator, the use of spontaneous breathing trials is helpful. Spontaneous breathing trials are a way to build respiratory muscle strength slowly and facilitate ventilator liberation for patients who are prone to becoming ventilator dependent.

A patient with suspected laryngotracheobronchitis is demonstrating moderate stridor with a barking cough. Which of the following should the respiratory therapist consider?

A. nebulized epinephrine B. heated bland aerosol by mask C. administration of an aerosolized decongestant D. aerosolized Lidocaine by mask C. Moderate stridor may be treated with racemic epinephrine. This drug is classified as a decongestant. This alpha-1 medication can constrict vessels and tissues and reduce inflammation. If stridor is considered to be severe or marked, racemic epi not the best choice. An airway should be established immediately. This action is preferred over racemic epinephrine because of the time it takes for the medication to have effect. Therefore, racemic epinephrine should be used in cases involving mild and moderate stridor.

A respiratory therapist observes the following data on a post-abdominal surgery patient who has been receiving VC SIMV ventilation for 18 hours: Spontaneous VT 480 mL MIP -38 cm H2O RR (spont) 18/min SpO2 99% FIO2 0.40 Mandatory rate 12/min VT (set) 550 mL The patient is alert and appropriately responsive to questions. Which of the following would be the BEST weaning method for this patient?

A. SIMV, rate 10, reduce rate by 2 every 4 hours until at 4/min B. CPAP of 8 cm H2O, FIO2 0.60 C. spontaneous breathing trial D. PS ventilation at 15 cm H2O C. The evaluation of this data shows a patient who appears to be strong in terms of independent ventilation. Thus, weaning is appropriate and a spontaneous breathing trial would be suitable for this patient. Other methods listed here are generally appropriate, but the levels associated with them are excessive.

A patient diagnosed with a pulmonary infection is febrile with a temperature of 39 deg C (102 deg F). A recent arterial blood gas shows the following: pH 7.50 PaCO2 30 torr PaO2 65 torr HCO3- 22 mEq/L BE -2 mEq/L Which of these reported values would be lower than the actual when not corrected for temperature?

A. SaO2 B. HCO3- C. pH D. PaCO2 D. PaCO2 and PaO2 would be lower than actual if the blood gas is not corrected for temperature.

A patient who has been receiving pulmonary rehabilitation for one year is being evaluated. Upon which criteria should the progression of the patient be primarily based?

A. Serial arterial blood gases B. Patient's view of progress C. Successive P/F ratio determinations D. Period pulmonary function testing B.

A physician suspects a patient has a restrictive pulmonary defect. Which of the follow tests would be most helpful at determining this?

A. Slow vital capacity B. Forced vital capacity C. Fev1.0/FVC% D. Fev1.0 A. To determine if the patient has a restrictive pulmonary defect, pulmonary function volumes must be obtained. Specifically, slow vital capacity (SVC) or forced vital capacity (FVC) are used for this evaluation. An SVC is more accurate than a FVC. SVC should be chosen first if given the option.

A respiratory therapist is determining the proper sterilization technique to clean a pneumatically driven IPPB machine. The therapist should choose which of the following methods?

A. Sonacide (acid gluteraldehyde) B. Cidex (alkaline gluteraldehyde) C. Irradiation D. Ethylene oxide (ETO) D. Because an IPPB machine cannot be placed in a liquid for sterilization, gas sterilization is most appropriate. This can be accomplished with ethylene oxide or ETO.

Which of the following is required to determine RAW in a spontaneously breathing ambulatory patient?

A. nitrogen analyzer B. plethysmograph C. galvanic oxygen analyzer D. helium analyzer B. To determine airway resistance, otherwise known as RAW, a body box is needed. Another name for a body box is a plethysmogaph.

Which of the following weaning parameters would NOT be useful in determining whether the patient is ready for cessation of mechanical ventilation for a 75-kg (165-lb) male patient?

A. Spontaneous tidal volume of 375 mL B. MVV of 75 L in 15 seconds C. Vital capacity of 650 mL D. PaCO2 increase of 8 torr during weaning trial B. MVV, or maximum voluntary ventilation is a test completed during a complete pulmonary function evaluation. It is not a part of weaning parameters assessed on mechanically ventilated patients. It is also a test that should be completed on patients who are otherwise healthy and in their best condition, not a critically ill patient receiving mechanical ventilation. The other tests are appropriate to assess when deciding to terminate mechanical ventilation.

An adult male patient has reacted unfavorably to a small-volume nebulizer given with 0.63 mg of Xopenex. Increased dyspnea and excessive coughing have resulted. What should the respiratory therapist recommend?

A. Stop therapy, call the doctor, and document the findings. B. Reduce the dosage to 0.31 mg. C. Stop therapy and document the findings. D. Place the patient on a NRB mask at 100% oxygen. A. In this example, the patient did not tolerate the therapy. Therefore, the respiratory therapist should discontinue therapy & notify the ordering physician. A note about the patient's tolerance should also be made in the medical record.

A patient with cystic fibrosis is unable to tolerate postural drainage and percussion with the head of the bed down. A recent bronchogram reveals consolidation of the secretions in the basal segments of the lower lobes. The respiratory therapist should do which of the following to continue to promote mobilization and removal of pulmonary secretions?

A. Suction the patient q2 hours B. PEP therapy C. Perform postural drainage without placing the head-of-bed down D. Encourage the patient to endure the therapy B. When a patient is unable to tolerate postural drainage with the head of bed down, the respiratory therapist must modify the therapy to accomplish the same objective without dysfunctionalizing the therapy. Of the options listed, PEP therapy is most appropriate. Performing postural drainage and percussion without placing the head of bed down is not appropriate for that therapy (it dysfunctionalizes that therapy). Suctioning is not yet appropriate because the objective of postural drainage is to move secretions from the lower airways to the larger airways, where they may be expectorated and/or suctioned.

A respiratory therapist is providing homecare education to family members who will be acting caregivers to a patient on a home-use ventilator at least 50% of the time and who requires tracheal suctioning from time to time. To encourage aseptic technique, the therapist will include which of the following in the teaching?

A. Suction the patient with at least one sterile hand. B. Rinse the suction catheter after each use with sterile water. C. Use a closed system suction catheter. D. Utilize disposable, sterile suction catheter kits each time. C.

A patient with suspected vocal cord dysfunction and paralysis is undergoing pulmonary function testing to further investigate the problem. Which of the following PFT exams will yield most information about the issue?

A. nitrogen washout B. DLCO C. FVL D. SBN2 C. A pulmonary function exam can reveal the presence of vocal cord dysfunction and/or paralysis by a flow volume loop (FVL). The condition will manifest itself by showing what is called a round loop, or a loop whose inspiratory phase looks similar to the expiratory phase. Normally, these two phases should look drastically different.

A 14-year-old patient, with a history of asthma, is brought to the emergency room (ER) by the paramedics. The reports indicates the patient is wheezing bilaterally and is only able to speak in one-word-sentences. The patient is receiving oxygen by nasal cannula at 2 L/min. The initial action of the respiratory therapist should be to

A. Switch to a nonrebreathing mask B. Increase oxygen flow rate to 4 L/min C. Administer a breathing treatment with Abluterol and Ipratropium Bromide (Atrovent) D. Adminsiter Solu-Mederol, IV A. When a patient is only able to speak in one word sentences, the degree of ventilatory compromise is severe. This is considered an emergency situation, according to national asthma guidelines. In such a case, the situation must be treated as an emergency and the patient should receive 100% oxygen. A non-rebreathing mask is most appropriate

A patient in the cardiac care unit is found to have the following cardiac rhythm. A pulse is not palpable. Which of the following is the best initial action

A. Synchronized defibrillation at 360 joules B. Cardioversion C. Defibrillate at 50 joules D. Begin BLS D. The absence of a pulse indicates the patient's heart is not contracting. Chest compressions are required. The best option is to begin basic life support.

Which of the following radiographic findings would most likely be noted if a patient was improperly rotated during the exposure?

A. The clavicles lean to one side. B. Apexes are level bilaterally. C. The radio-opaque line on the ET tube cannot be seen. D. The tracheal tube is not in alignment with the spinal process. D. If a patient is improperly rotated during an x-ray, any artificial airway (ET tube or trach tube) would appear displaced to one side of the spinal process (vertebrae). Clavicles that lean to one side indicate the patient was leaning and would not have the same affect as improper rotation.

Which of the following should be set when preparing to perform cardiac defibrillation to a patient with pulseless ventricular tachycardia?

A. The defibrillator should be synchronized to the 'R' wave. B. Energy setting should be kept at 50 joules or less, monophasic. C. Synchronization should be off. D. Monophasic is more effective than biphasic. C.

A patient is breathing spontaneously through an aerosol mask connected to a single large volume air-entrainment nebulizer set at 100%. The oxygen flow meter is set to 15 L/min. The patient's tidal volume is 600 mL and the respiratory rate is 34/min. Which of the following can the respiratory therapist conclude?

A. The flow rate at the oxygen flow meter is set too high B. The patient should be sedated C. The inspiratory demand of the patient is not being met D. The patient should be instructed to breathe slower C. To answer this question one must understand how to calculate the total inspiratory demand of the patient and compare it to the total flow being received. This patient is on a large-volume nebulizer set at 100% and therefore is not entraining any room air. This means whatever the flow meter is set at is the total flow of gas to the patient. Therefore, the patient is receiving 15 L per minute. However if we take the tidal volume and multiply it by the rate we get a minute ventilation of 20.4 L. This means the patient requires 5.4 L more than is being delivered by the nebulizer. The additional 5 L is being entrained through the aerosol mask, which means the patient is not receiving 100% oxygen and the inspiratory demand of the patient is not being met.

A patient who has an 80-pack-year history of smoking and is chronically obstructive has the following arterial blood gas results on room air at standard temperature and pressure (STP): pH 7.35 PaCO2 69 torr PaO2 148 torr HCO3- 31 mEq/L BE + 5 mEq/L COHb 20% The respiratory therapist can accurately assume which of the following?

A. The patient is a significant V/Q mismatch B. The patient has a shunt in excess of 20% C. The blood gas analysis is erroroneous D. The patient is no longer a smoker C. This blood gas reveals a PaO2 of 148 mmHg. A patient who is breathing room air has a maximum oxygen tension of 100 mmHg in the alveoli, making a PaO2 over 100 mmHg and impossibility. This blood gas mostly likely reveals an analyzer error.

What determination can be made by observing the following ventilator graphic?

A. The patient triggered the breath. B. The machine delivered a controlled breath. C. The sensitivity is set appropriately. D. Peak flow is inadequate. A. The presence of a negative deflection in the pressure-volume loop (fishtail) is evidence of a patient-trigged breath. The degree of deflection would suggest that sensitivity is inadequate. A controlled breath would not show a fishtail pattern. The inadequacy of flow cannot be determined using this graphic alone.

A patient is receiving heliox therapy at 70% / 30% mixture by nonrebreathing mask. The oxygen flow meter is set to 10 L/min. The patient has a minute ventilation of 18 L/min. What can the respiratory therapist conclude?

A. The total flow of the gas mixture is not meeting the demands of the patient B. The set flow of gas at the oxygen flow meter should be reduced to 8 L/min C. An air flow meter should be used in place of the oxygen flow meter D. The patient should be switched to 100% oxygen A. A Heliox mixture of 70%/30% that flows through an oxygen flow meter at 10 L per minute is actually producing a total flow of 16 L per minute. The patient has a minute ventilation of 18 L per minute and therefore the total flow of gas mixture to the patient is not meeting the patient's inspiratory demand.

A 32-week-gestional age infant is in respiratory distress following a cesarean section birth. A chest x-ray shows a reticulogranular pattern. Which of the following would be most helpful to the patient?

A. Theophylline B. Atropine sulfate C. Exosurf D. Subcutaneous epinephrine 1:1000 strength C. Reticulgranular pattern on a chest x-ray is an indication of lung immaturity and the onset of IRDS. Infant lung immaturity can be treated by administering surfactant. Exosurf is the only surfactant agent offered among these answers.

A post-operative patient who received a tracheotomy during surgery remains in the intensive care unit receiving volume-controlled ventilation two weeks after surgery. During a position change for the purpose of bathing, the ventilator high pressure alarm begins to sound with each breath. A suction catheter can be passed only 9 cm into the tracheostomy tube. The respiratory therapist should do which of the following?

A. Turn patient to original position prior to bathing B. Reposition the tracheostomy tube C. Provide bag and mask manual ventilation D. Remove the inner cannula B. Difficulty passing the suction catheter down the tracheostomy tube, in addition to the ventilator high-pressure alarm, indicates a problem with the tracheostomy tube. Because the patient is postoperative and has only recently received the tracheostomy tube, the positional changes required by bathing most likely indicate that the tracheostomy tube has become dislodged or Mal positioned. Repositioning of the tube is most appropriate.

Which of the instructions are appropriate for a patient receiving both Proventil and Flovent?

A. Use Flovent for acute bronchoconstriction. B. Wait one hour after Proventil before taking Flovent. C. Do not use a spacer with Flovent. D. Use the Proventil first when taking the medications together. D. Provintil (albuterol sulfate) is a bronchodilator that should be given first when taken with Flovent (fluticasone), a corticosteroid used to prevent inflammation of the airway walls.

A respiratory therapist is providing homecare education to family members who will be acting caregivers to a patient on a home-use ventilator at least 50% of the time and who requires tracheal suctioning from time to time. To encourage aseptic technique, the therapist will include which of the following in the teaching?

A. Use a closed system suction catheter. B. Rinse the suction catheter after each use with sterile water. C. Utilize disposable, sterile suction catheter kits each time. D. Suction the patient with at least one sterile hand. A.

A patient involved in a mass transit accident involving a train is brought to the emergency department (ED). An initial assessment reveals adequate ventilation, a history of CHF, and long-bone fractures in both legs. Eight hours later, the patient begins to demonstrate ventilatory distress and decreased chest excursion on the right. The patient is becoming febrile and complains of pleuritic pain on the right during deep inhalations. The respiratory therapist should recommend

A. V/Q scan. B. PET scan. C. lateral decubitus radiograph. D. PA chest radiograph. C. The patient is demonstrating signs of a pleural effusion. A lateral decubitus x-ray would be most helpful in exploring this suspicion. A P-A x-ray is not appropriate because the patient is not likely ambulatory with two broken legs. A V/Q scan would be appropriate if a clot was suspected, but sings and symptoms are not consistent with this finding. A Pet scan is used to assess the function of a specific organ and is not relevant in this case.

A spontaneously breathing patient has the following arterial blood gases: pH 7.25 PaCO2 55 torr PaO2 54 torr HCO3- 25 mEq/L BE -1 mEq/L Which of the following should the respiratory therapist recommend?

A. VC AC ventilation B. Bi-level therapy, IPAP of 15 cm H2O, EPAP of 5 cm H2O C. supplemental oxygen by partial rebreathing mask D. supplemental oxygen by nonrebreathing mask A. This patient is in acute ventilatory failure, as shown by an increased CO2 and a pH of 7.25 or less. When acute ventilatory failure is present, the patient would benefit most from positive pressure ventilation, such as mechanical ventilation.

Which of the following would suggest that ventilation liberation should not be attempted?

A. VC of 10 mL/kg B. VD/VT ratio > 60% C. spontaneous Vt of 6 mL/kg D. MIP -31 cmH2O B. be a candidate for ventilator weaning, deadspace-tidal volume ratio must be less than 60% while on the ventilator.

An 80 kg (176 lb) male patient is admitted to the cardiac care unit after sustained complications following cardiac catheterization and angioplasty of several coronary arteries. The patient is receiving oxygen at 100% via bag-valve mask. There are no spontaneous respirations. SpO2 is 97%. Which of the following would be appropriate initial ventilator settings?

A. VC, A/C, rate 14, FIO2 1.0, VT 550 mL, PEEP 5 cm H2O B. VC, SIMV, rate 12, FIO2 0.6, VT 650 mL, PEEP 5 cmH2O C. VC, SIMV, rate 10, FIO2 1.0, VT 400 mL, PEEP 5 cm H2O D. PC, rate 10, IP 40 cm H2O, FIO2 0.5, I-time 1.0 secL, no PEEP A. Appropriate tidal volume for this patient is between 6-10 mL/kg of ideal body weight. Rate should be set between 10-20. Because the patient is receiving 100% oxygen, the initial ventilator oxygen setting should also be 100%. Finally, PEEP of 4- 6 cm H2O is appropriate.

A patient is four hours post operative for a partial lobectomy of the lingular lobe. The patient complains of shortness of breath and has rhonchi in the middle right lobe. The respiratory therapist should recommend which of the following?

A. VEST therapy B. postural drainage and percussion C. quad cough D. vibratory PEP D. Of the options given, vibratory pet therapy would be most appropriate at promoting movement and expectoration of secretions and general bronchial hygiene. Postural drainage and percussion would also provide this but because the patient is recovering from a lobectomy and thoracic surgery in general, percussing over the thorax is contraindicated.

Which of the following is most indicative of a patient's ability to ventilate?uti

A. VT B. TLC C. MIP D. IC C.

The respiratory therapist notes a fall in pulse from 80/min to 60/min with a proportional drop in blood pressure during a suction procedure. Which of the following would be the most likely cause of these observations?

A. Vagal reflex B. Transient hypoxemia C. Copious secretions D. Suction pressure is too high A. During suctioning, a patient's heart rate may fall. This is a normal physiological response associated with the vagal reflex, or the vagal nerve response. This response will also lower blood pressure.

Which of the following would be the best medication to use with a patient who will require oral intubation in an emergent setting?

A. Versed B. Anectine C. Flurazepam D. Valium B. The drugs most commonly used to facilitate intubation are Anectine (succinylcholine), Pavulon (pancuronuim bromide), Curare (d-tubocurarine) and Vecuronium. Valium and Versed are anxiety/pain relievers. Flurazepam is used to treat insomnia.

A respiratory therapist is delivering a small-volume nebulizer treatment to a patient with asthma. In addition to the aerosolized Abluterol, what other medication may the therapist mix in the treatment to enhance bronchodilation?

A. Xopenex B. Metaproterenol (Alupent) C. Ipratroprium Bromide (Atrovent) D. Terbutaline C. Albuterol is considered to be a front door bronchodilator. To maximize bronchodilation a front door bronchodilator may be combined with a back door bronchodilator, such as Atrovent.

The following Levey-Jennings quality control chart is noted for an ABG analyzer. The therapist should recognize the control data as

A. a demonstration of a shift. B. out-of-control. C. in control with an outlier. D. trending downward. C. The graphic is a plot of quality control samples logged every 8 hours. All points are within 2 SD of the mean (middle line) except for a single point. When this happens, the point be deemed as an outlier and ignored if the subsequent points are within the control area.

A patient with COPD has had three hospitalizations in the last 12 months. Medications have been changed over the course of the year. The patient complains of declining health in general. Which of the following could benefit the patient?

A. a disease management program B. corticosteroid therapy C. a change in the patient's family doctor D. IV antimicrobial therapy A. Disease management is an integrated approach to care that looks at the whole approach. The goal of COPD disease management is to keep care from becoming fragmented and therefore ineffective. A change in the family doctor would only fragment care more. There is no clear indication for corticosteroid therapy, given the current information, nor is there an indication for an antibiotic regimen

A 48-year-old female completes the several FVL maneuvers. Both the expiratory and inspiratory side of the loops demonstrate a similar inverse pattern that is round in appearance. Which of the following could the respiratory therapist potentially conclude about this pattern?

A. a restrictive pulmonary defect is likely present B. the shape of the loop demonstrates normal pulmonary function C. a fixed upper airway obstruction is present D. an obstructive pulmonary defect is likely present C, In a normal flow volume loop, the inspiratory side of the loop (the topside), is vastly different than the bottom side of the loop. However, when upper loop resembles the bottom part of the loop in shape (this is often called a round loop) it indicates that the patient is having difficulty getting air in AND out. This is most commonly associated with a fixed upper airway obstruction or vocal cord paralysis or cancer.

While observing the PetCO2 value on a patient who is receiving mechanical ventilation, the tracing shows a sharp rise in the CO2 on the capnograph. This rise represents

A. a soiled infrared CO2 sensor B. the flushing out of dead space C. massive atelectasis D. pulmonary embolism B. Initial end-tidal CO2 readings are often low but demonstrate a sharp rise as carbon dioxide builds up in the dead space is exhaled past the infrared CO2 detector.

A patient is undergoing a maximal exercise tolerance test. During the exam, as workload is increased, an increase in heart rate from 90 to 120 bpm while blood pressure remained steady at 110/88 mmHg is noted. Which of the following can be correctly stated about the exam results?

A. abnormal cardiac response, normal blood pressure response B. abnormal cardiac response, abnormal blood pressure response C. normal cardiac response, abnormal blood pressure response D. normal cardiac response, normal blood pressure response C. During an exercise tolerance test, also known as a stress test, blood pressure and heart rate should rise as workload is increased. If this does not occur, the patient is said to have an abnormal blood pressure and/or abnormal cardiac response. In this case the patient's heart rate did increase but the blood pressure did not. The best interpretation is "normal cardiac response with an abnormal response in blood pressure".

The respiratory therapist evaluates a patient in the emergency department (ED). Physical assessment reveals an ashen or pallor appearance. This is most consistent with which clinical condition?

A. abuse of barbiturates B. anemia or shock C. excessive bilirubin D. infection or inflammation B. The terms "ashen" and "pallor" are related to anemia or shock.

A patient is receiving oxygen by an air-entrainment device combined with cool aerosol. Arterial blood is extracted and analyzed. FIO2 0.80 pH 7.36 PaCO2 44 torr PaO2 485 torr HCO3- 23 mEq/L BE -1 mEq/L SAT 0.99 The respiratory therapist should

A. accept and report the results. B. recalibrate the ABG analyzer. C. repeat the ABG. D. reject the results. A. Although a PaO2 of 485 seems too high and suspicious of error, the result is indeed possible considering the patient is receiving 80% oxygen. The arterial PO2 achievable can be determined by calculating the PAO2 with a given FIO2. On 80%, the maximum PAO2 is about 506 torr. Thus a PaO2 of 485 is possible. The results should be accepted and reported.

After completing four flow-volume loops on a bedside pulmonary function screening device, the therapist notices that the volume reported with all FVC maneuvers are less than all of the SVC maneuvers. The therapist should

A. accept the results B. repeat the entire flow-volume loop maneuver C. repeat the FVC maneuvers D. repeat the SVC maneuvers A. Essentially, this question is demonstrating normal results relative to FVC and SVC volumes. In other words, SVC volumes should always be higher than FVC volumes. Therefore the respiratory therapist should accept the results.

A patient who has an indwelling arterial catheter is being monitored for wide fluctuations in blood pressure. The ART line is zeroed while the pressure transducer is level with the lower abdomen. Ten minutes later, a blood pressure alarm sounds and the therapist discovers the patient has raised his arm above his head, causing the transducer to now be level with the head. Which of following would the therapist expect to observe?

A. accurate reported blood pressure B. no reported blood pressure due to an error C. low indicated blood pressure D. high indicated blood pressure C. When the transducer of an ART line is raised above the level at which it was zeroed and calibrated, the blood that pumps against the transducer will exert less pressure as it attempts to climb up to the transducer. This, therefore, will cause the indicated blood pressure to appear lower than actual. The opposite is also true. If the transducer is placed below the point at which it was zeroed, gravity will cause the blood to exert additional pressure against the transducer diaphragm, which will produce an indicated blood pressure that is higher than actual.

While preparing for a beside pulmonary function test on a 23-year-old patient with asthma, the respiratory therapist notices the patient seems listless and is not responding well to questions or commands. SpO2 is 84% on a 40% high-flow oxygen mask, RR 32/min, breathing pattern is shallow. HR 118/min. The respiratory therapist should

A. activate the emergency medical response team. B. administer Narcan (naloxone). C. activate a full cardiopulmonary resuscitation code. D. obtain an arterial blood sample for gas analysis (ABG). A.

A 74-kg (166-lb) 5-ft 8-in (173-cm) female is receiving mechanical ventilation by a volume-controlled ventilator in the assist/control mode on the following settings and has the corresponding ABG results: Mandatory rate 14/min VT 600 mL FIO2 0.5 PEEP 5 cmH2O pH 7.48 PaCO2 32 mmHg PO2 90 mmHg HCO3- 24 mEq/L BE 0 mEq/L Which of the following represents the most appropriate action?

A. add 150 mL deadspace B. decrease rate to 10 C. increase inspiratory flow D. decrease VT to 500 mL A. In this problem the CO2 is slightly low. When we need to raise CO2 there are three choices - decreasing tidal volume, decreasing rate, or adding dead space. In this case, decreasing the rate from 14 to 10 to change CO2 by only a few points is too drastic. A tidal volume of 500 mL is less than the patient's range based on their ideal body weight. The best option is to add 150 mL of dead space.

Which of the following actions would most likely resolve a high-pressure alarm on a volume-cycled adult ventilator?

A. add air to the ET tube cuff B. decrease heater temperature setting C. suction the patient D. tighten the patient-ventilator interface connection C. To determine the answer to this question one must first look at the cause of the high pressure alarm on the ventilator. The need to add air to the ET tube cuff would result in a low-pressure alarm. The need to tighten the patient ventilator interface connection would presumably be caused by a leak and would result in a low-pressure alarm. The heater temperature should have no effect. Therefore, suctioning the patient is the only solution that is an appropriate response to the high-pressure ventilator alarm

A 55-year old male is receiving VC, SIMV ventilation. The following parameters are observed: f 4/min Total rate 32/min VT (set) 500 mL VT (spont) 180 mL FIO2 0.45 Which of the following would be an appropriate change?

A. add pressure support B. administer Narcan (naloxone) C. begin a spontaneous breathing trial (SBT) D. increase rate to 12/min A.

A 34-year old male is receiving mechanical volume-ventilation. The following machine-triggered breath is shown in the following pressure-volume graph. Based on this data, the respiratory therapist should

A. add pressure support B. increase PEEP C. decrease pressure limit D. decrease tidal volume D. This pressure volume ventilator graphic has a pronounced beak. This means at the end of the breath there is a significant increase in pressure to achieve a small amount of volume. Because mean airway pressure is of primary concern, sacrificing a small amount of tidal volume to achieve significantly lower mean airway pressure is helpful.

Stridor is noted immediately after endotracheal tube extubation. The respiratory therapist should

A. administer 2 puffs of Serevent by inhaler. B. reintubate the patient. C. administer racemic epinephrine. D. apply a benzocaine spray into the oropharynx. C.

The respiratory therapist is administering ultrasonic aerosol therapy to a patient with inspissated secretions. The therapist should monitor for the development of transient tachypnea because

A. aerosol particles cause bronchoconstriction B. additional hydration can cause fluid overload of the pulmonary system C. secretions may dry as a result of the aerosol D. secretions may increase in volume as they become hydrated D. When hydrating secretions with ultrasonic aerosol therapy, one central concern is the swelling of the secretions as they become hydrated. As secretions swell, airways become temporarily occluded could cause tachypnea.

Which of the following airway clearance therapies should be administered first in the order of therapy?

A. aerosolized Acetylcysteine (Mucomyst) B. nasal tracheal suctioning C. postural drainage and percussion D. verbal coaching for coughing A. Of the options offered, the first most logical step would be to thin secretions through use of the medication Mucomyst. The next step, would be postural drainage and percussion followed by verbal coaching for coughing and possibly nasal tracheal suctioning.

A patient with pneumococcal pneumonia is having difficulty expectorating sputum that is thick and tenacious. Which of the following would most effectively help thin secretions?

A. aerosolized acetylcysteine, once daily B. instillation of saline into the airway C. IV fluids D. aerosolized albuterol treatments C. The most effective way to thin secretions is by increasing the overall fluid status of the patient. This is most effectively done through IV fluid administration and by oral consumption of water (and other fluids).

A patient with increased airway resistance from a fixed upper airway obstruction would benefit most from which of the following?

A. aerosolized atropine B. epinephrine C. heliox therapy D. bronchodilator therapy C. A fixed upper airway obstruction means air is having difficulty moving both in and out. It is usually caused by cancer or other non-changeable conditions. In this case the best thing we can do is decrease airway resistance by using helium or Heliox therapy.

A reduction in urinary output for a patient receiving any positive chest-pressure intervention is a result of

A. altered electrolytes B. reduced cardiac output C. kidney failure D. dehydration B. Reduced urinary output on a patient receiving positive chest pressure intervention is most likely associated with a reduced cardiac output.

For a patient receiving volume-controlled ventilation in the assist/control mode, the starting of the inhalation cycle before the expiratory flow rate returns to zero would result in which of the following?

A. alveolar collapse B. atelectasis C. decreased FRC D. autoPEEP D. When a patient on a ventilator is unable to exhale completely before the inhalation cycle starts, the result will be air-trapping, which will result in autoPEEP.

If inspiratory flow is increased on a time-cycled ventilator when pressure is not limited, the respiratory therapist would expect which of the following to also increase?

A. alveolar recruitment B. V/Q mismatching C. tidal volume and inspiratory pressure D. total rate and expiratory time C. Assuming the ventilator does not have a pressure limit set, an increase in inspiratory time on a time cycled ventilator will result in an increase in inspiratory pressure and an increase in tidal volume.

What parameter change would be required on a volume-controlled ventilator in the A/C mode to change from an I:E ratio of 1:2.6 to 2:1.0?

A. an decrease in inspiratory flow B. a reduction of inspiratory hold time C. a decrease in rate D. an increase in expiratory time A. A decrease in inspiratory flow will lengthen inspiratory time, which will help increase the I:E ratio (move from 1:2.6 to 2:1). A reduction in inspiratory hold and an increase in expiratory time would have the opposite effect. A decrease in rate would also result in increased expiratory time, which opposes the desired change.

A patient in the emergency room has been diagnosed with status asthmaticus. She has received IV steroids, MDI steroids, bronchodilators and antibiotics. Which medication should be discontinued first once the patient becomes stable?

A. antibiotics B. bronchodilators C. IV steroids D. MDI steroids C. A patient with status asthmaticus would benefit from IV steroids such as methylprednisolone. Of the medications given IV steroids should be discontinued first.

A patient is brought by paramedics to the emergency room after being found unconscious in a garage with a car running. A note was found on the scene. Which of the following devices will be most helpful in immediately assessing the patient?

A. arterial blood gas analyzer B. blood gas analyzer with CO-OX ability C. obtain a reading from a multiple wavelength spectrophotometer D. single wavelength pulse oximetry C. This patient is likely to have an increased carbon monoxide level. To assess the degree of carbon monoxide poisoning, COHb would be an appropriate exam. Although this is an option, there is a better option. Unlike a normal pulse oximeter, a multiple wavelength spectrophotometer can assess carbon monoxide in a similar fashion as a pulse oximeter - by finger probe. This is a superior option because it is quicker, less invasive, less painful, and probably less expensive.

A patient in the cardiac intensive care unit has pitting peripheral edema rated at +2. What other laboratory data would be helpful to assess to further understand the nature of the problem?

A. arterial blood gases B. PD50 C. creatinine and BUN D. serum electrolytes C. The presence of pitting edema is a fluid shifting problem that is related to the kidney and sometimes the heart. Any kind of fluid shifting problem can be evaluated further by looking at the creatinine and BUN. Of these tests, creatinine is considered more accurate. But both will supply the same kind of information.

For which of the following reasons is the use of an oral pharyngeal airway on a patient who is orally intubated and receiving mechanical ventilator support indicated?

A. as a bite block to prevent tube occlusion B. to prevent nosocomial infection C. to improve spontaneous ventilation D. to facilitate suctioning A. A oral pharyngeal airway is normally used to prevent soft tissue obstruction (to keep the tongue from falling back and occluding the airway). However, when a patient is intubated, there is no need to prevent soft tissue obstruction. In this case the oral pharyngeal airway takes on a different purpose -to serve as a bite block to keep the patient from biting on and occluding the endotracheal tube.

A nine-year old child with asthma is being transported by a fixed-wing, non-pressurized aircraft. The child was breathing well on the ground, but once the aircraft reaches 11,000 feet, a change in condition is noted. BS are suddenly diminished on the left, BP has dropped to 90/50 mm Hg. The respiratory therapist should recommend

A. asking the pilot to decrease altitude to 2,000 feet. B. administering albuterol. C. performing needle decompression of the left chest. D. having the pilot land at the next available site. C. Because asthma includes a certain amount of air-trapping, a change in altitude can result in gas expansion in the lungs and a consequent hemothorax. This can be immediately addressed by inserting a large-bore needle into the affected side.

A patient is transferred to the intensive care unit after abdominal surgery. Vital signs are observed within 20 minutes of arrival. BP 90/58 mm Hg and SpO2 97% on a 40% air-entrainment mask. The patient's color is dusky. The respiratory therapist should

A. assess INR B. check Hb level C. administer anticoagulant therapy D. institute volume ventilation B. The low BP and dusky color after surgery is suggestive of anemia and possible internal hemorrhaging, likely a problem that developed as a result of the surgery. An Hb check will definitively determine if internal blood loss is occurring.

The following flow-volume loop is obtained from a 45-year-old male with vocal cord cancer. Which of the following interpretations is most accurate?

A. asthma B. COPD C. fixed airway obstruction D. foreign body aspiration C. This flow volume loop is round in appearance, indicating a problem with getting air in and out. This is associated with a fixed upper airway obstruction.

A patient has a RAW value of 2.8 cm H2O/L/sec as determined by a body box. Laboratory values indicates an elevated eosinophil count. What is the most likely diagnosis of the patient?

A. asthma B. chronic bronchitis C. emphysema D. cystic fibrosis A.

A patient reports to a pulmonary function laboratory for testing associated with a worker's compensation claim. The respiratory therapist gathers the following pulmonary function data: Observed Predicted DLco mL/min/mmHg 20 25 FVC (L) 2.83 4.66 Fev1/FVC(%) 85 75 Which of the following is consistent with these data?

A. asthma B. pulmonary embolism C. pulmonary fibrosis D. emphysema C. Flows are with normal range, ruling out an obstructive pulmonary defect. Because volume is decreased in FVC (less than 80% of predicted), a restrictive defect is confirmed. Pulmonary fibrosis is the only disease listed associated with a restrictive disorder.

In which of the following situations would a point-of-care arterial blood gas analysis be most appropriate?

A. at college for a cystic fibrosis patient B. in surgery C. at a health fair for the public D. at home for a COPD patient B. Point-of-care analytical devices are most appropriate in temporary situations or in places where quick information is required. Surgery is an example of this type of requirement. If a physician in surgery requires arterial blood gas data, it would perhaps take too long for the sample to leave the sterile area to be analyzed. Consequently an analyzer within the sterile area would provide data more quickly and allow for better care of the patient.

The radiological report of a chest radiogram indicates a batwing pattern. This description is most consistent with which of the following?

A. atelectasis B. pulmonary edema C. ARDS D. pleural effusion B. The radiological report and interpretation of "batwing pattern" is associated with pulmonary edema.

A bronchoscopy is NOT used to diagnose

A. atelectasis. B. malignant processes. C. pathogenic involvement. D. foreign body obstruction. A. Bronchoscopy is used to diagnose foreign body obstructions, active bleeding causes, pathogenic involvement (to identify organisms) and cancerous or malignant processes or areas. It may help decrease atelectasis if immobilized mucous plugs are blocking a portion of the lung from expanding, but it does not serve in the diagnosis of atelectasis.

A 40-year-old female presents in the emergency room after being rescued from a building fire where she was found unconscious. Which of the following assessments would be most helpful in the evaluation of the patient?

A. beside pulmonary function testing B. single-wavelength pulse oximetery C. measurement of PaO2 D. hemoximetry D. The primary concern for a patient who has had significant exposure to smoke is smoke inhalation and carbon monoxide poisoning. Of the options offered, hemoximetry will be most helpful. Pulse oximetry and measurement of PaO2 will only assess oxygenation and pulmonary function testing is not helpful.

A patient with fulminating pulmonary edema has the following data obtained from a Swan-Ganz pulmonary artery catheter: CVP 10 mmHg PAP 25 mmHg PCWP 15 mmHg C.O. 3.9 L/min The patient would benefit most from which of the following?

A. blood expanding agents B. Penicillin-class antibiotics C. loop diuretics D. administration of whole blood C. Hemodynamic values are all increased. This is most closely associated with over hydration of the patient, or hypervolemia. The solution to this is to administer a diuretic medication. The most appropriate medication is Lasix, which is a loop diuretic.

An infant remains cyanotic after the administration of high level supplemental oxygen. The physician is concerned about the presence of a condition, patent ductus arteriosis, and asks the respiratory therapist to gather data to confirm or rule out that condition. Which of the following data will the therapist require?

A. blood gases from the umbilical and radial arteries B. blood glucose C. CT scan with contrast D. chest radiography A. Patent ductus arteriosus (PDA) is a congenital abnormality that may be found in some infants. If found, the correction is surgery. To diagnose PDA, the PO2 is analyzed from the right brachial artery and the umbilical artery. If PO2 differs by more than 15 mmHg then PDA is present.

For which of the following vagal reflex response should the respiratory therapist monitor in a patient that is receiving nasotracheal suctioning:

A. bradycardia B. increased systemic vascular resistance C. hypertension D. third degree heart block A. Suctioning can cause bradycardia as well as hypotension. This natural physiological response of the body is associated with stimulation of the vagal nerve during suctioning. This is also known as the vagal reflex.

Which of the following would be most helpful in the presence of elusive pulmonary organisms, such as Legionella, that cannot be isolated and detected through tradition suctioning and specimen collection methods?

A. bronchoalveolar lavage (BAL) B. thoracentesis C. percutaneous lung biopsy D. use of coude' tipped catheters A.

An adult male has had a unilateral wheeze, when auscultating the chest, for several months. Which of the following is the most likely cause?

A. bronchoconstriction B. a cancerous mass in the pulmonary tree C. ARDS D. idiopathic pulmonary fibrosis B. The presence of a unilateral wheeze that has persisted for several months is not likely related to bronchoconstriction. When bronchoconstriction is present, wheezing is usually noted bilaterally. The unilateral wheeze, in conjunction with its persistence for several months, is most likely caused by a mass in the pulmonary tree. This mass may be cancerous or benign.

A patient diagnosed with bronchiectasis, and who has a tracheostomy, has reported an increase in secretions and secretion thickness. A chest radiograph shows a cavitary disease. The patient is admitted. The respiratory therapist should recommend

A. bronchoscopy to clear secretions B. routine bronchodilator therapy C. therapy focusing on airway clearance D. placing the patient in respiratory isolation C. A key treatment strategy associated with bronchiectasis is significant, aggressive bronchial hygiene.

A respiratory therapist is educating a patient who will be receiving supplemental oxygen at home with an oxygen concentrator. Which of the following should the respiratory therapist recommend as a backup in the event electricity supply to the home is interrupted?

A. bulk liquid oxygen supply B. manifold and bank of H tanks C. multiple E cylinders D. a second oxygen concentrator C. A an appropriate backup for an oxygen concentrator is an E cylinder.

The patient results from a Co-oximeter are questionable and do not fit the clinical scenario presented. In order to assess whether the machine is accurate, the respiratory therapist should

A. calibrate the instrument and repeat the patient sample B. compare the results with that of a pulse oximeter C. run quality controls and evaluate results D. perform proficiency testing C. The respiratory therapist should always compare laboratory results with the clinical picture. In the case of a mismatch, or questionable result, one should utilize quality control measures to assess accuracy and calibration methods to make adjustments when instruments are inaccurate.

After attempting to pass a catheter down an oral ET tube, the respiratory therapist feels strongly that the endotracheal tube cuff is herniated over the end of the ET tube. The bag-valve remains difficult to squeeze and oxygen saturation is 86% and falling. The therapist should do which of the following?

A. call the physician and report the finding B. perform nasal intubation with the oral tube left in place C. remove the ET tube and insert a new one D. switch to a high pressure demand valve C. A herniated endotracheal tube cuff indicates th ET tube is defective and therefore should be replaced.

The high pressure alarm is sounding on a patient receiving volume controlled ventilation. The respiratory therapist should FIRST

A. cancel the alarm B. provide manual ventilation C. suction the patient D. observe the alarm panel to determine source B. The NBRC knows that in real life, when you hear a ventilator alarm, you're tempted to cancel the alarm or attempt to immediately solve the problem. However, the correct answer is always to first provide manual ventilation to the patient. This is true of any ventilator alarm. Your first action should be to stabilize the ventilatory status of the patient, and then troubleshoot the problem.

Which of the following are associated with a fenestrated tracheostomy tube?

A. cap B. foam cuff C. button D. silver A. A fenestrated tracheostomy tube has an inner cannula with a hole to facilitate talking and weaning. It also has an inner cannula for resuscitation, or mechanical ventilation. The cuff is like that of an endotracheal tube, low-pressure, high-volume. And finally, a cap is used to close the tube for speech therapy. When the cap is used, the inner cannula should be removed and the cuff should be deflated.

From which location is the most accurate mixed venous sample taken?

A. capillaries B. pulmonary artery C. vena cava D. right atrium B. The most accurate mixed venous sample is found in the pulmonary artery.

Which of the following gases replaces nitrogen when administering heliox therapy?

A. carbon dioxide B. oxygen C. helium D. inert gases C. Because helium is a light gas it has the ability to bypass upper airway obstructions, and therefore helps deliver oxygen more easily than nitrogen, a heavier gas that exists naturally in room air. Heliox therapy should be delivered within a NRB in order to avoid entrainment of nitrogen-enriched room air.

A patient's tissue oxygen consumption has increased and cardiac output has decreased. Which of the following is also likely true?

A. cardiac index has increased B. A-aDO2 has decreased C. C(a-v)O2 has increased D. SaO2-SVO2 has decreased C. When cardiac output decreases, the speed of blood flow across the tissue bed is also decreased. As blood flows slower over tissues, the tissues have less ability and time to absorb oxygen molecules and therefore will demonstrate increased oxygen consumption. As oxygen consumption increases, the difference in arterial oxygenation and venous oxygenation also increases. Thus, an increase in C(a-v)O2 is observed.

Which of the following is most likely associated with hypervolemia?

A. cardiac index of 2.0 B. PAP of 15 mmHg C. PCWP of 7 mmHg D. CVP of 10 mmHg D. Of all the hemodynamic values, loss of fluid (hypovolemia) or (hypokalemia), the first and most significant hemodynamic manifestation will be found in the elevation of CVP. Normal CVP is about 5 mmHg. So, a CVP of 10 mmHg is high. Hypovolemia, or under hydration, is manifested by a low CVP. CVP is known by many other names. Be sure to know all of the names that are associated with CVP.

During the performance of cardioversion, what setting on the defibrillator is required?

A. cardiac pacing turned on B. joules (power) set to 300 or higher C. bi-phasic mode selected D. synchronization set to active D. Cardioversion is performed by delivering the shock in synchronization with the 'R' wave of the ECG tracing. Therefore, synchronization should be set to 'active' or 'on'.

To determine potential post-operative risk of a patient preparing for surgery, the respiratory therapist should recommend which of the following?

A. cardiac stress test B. measurement of inspiratory capacity C. pulmonary stress test D. pulmonary diffusion studies B. Potential postoperative risk may be assessed preoperatively through measuring inspiratory capacity

Which of the following indicates a properly exposed chest radiograph?

A. cardiac vessels are visible B. spaces between the vertebrae are clear and distinct C. no heart shadow is visible D. apecies are level B. There are several ways to determine proper exposure of a chest radiograph. Of the options listed, spaces between the vertebrae that are clear and distinct is one way for confirming the quality of the x-ray exposure. A heart shadow should be visible and cardiac vessels are rarely visible. Level apexes helps to know if the patient was rotated properly but does not provide information regarding exposure.

A respiratory therapist should increase suction pressure on an intubated patient in which of the following situations?

A. catheter is too long, endotracheal tube is untrimmed B. effective suction duration is greater than 5 seconds C. catheter is too small, but suction is urgently needed D. secretion clearance is ineffective, within the acceptable pressure range D. If suctioning is not adequate, correct the problem in this order: 1. check connections, change collection bottle if full 2. ensure suction pressure is in the right range 3. increase to the maximum size catheter within range 4. increase pressure within range 5. increase suction time

A respiratory therapist is having difficulty suctioning secretions from a 6-year old cystic fibrosis patient. The patient is being suctioned with an 8 Fr suction catheter through a 5.5 ET tube at a pressure of 85 mmHg for 10 seconds. To increase the effectiveness of suctioning, the therapist should increase which of the following?

A. catheter length B. catheter size to 10 Fr. C. suction pressure to 100 mmHg D. suction time to 20 seconds C. There are several ways to increase suctioning effectiveness. The best way is to increase the suction catheter diameter size. The next best way is to increase suction pressure. Finally, increasing suction time up to 15 seconds is also appropriate. Since the suction catheter diameter size is already as big as is permitted the next step would be to increase suction pressure. A pediatric patient should have a suction pressure range of 80 to 100 mmHg. Thus, the best option would be to increase the suction pressure to 100 mmHg, which is at the top of the range.

Which of the following is used to evaluate a patient for fluid imbalance?

A. cerebral perfusion pressure B. pitting edema C. minute ventilation D. P50 B. Fluid imbalance may result in pitting edema, changes in sensorium, and altered capillary refill time. Minute ventilation, P50, and CPP is not helpful.

A patient in the emergency room has a blood pressure of 90/60 mmHg and a palpated heart rate of 110 bpm. Pulse oximetry shows a saturation of 70% and a heart rate of 60 bpm. The respiratory therapist should

A. change the pulse oximeter sensor B. record the pulse ox saturation in the medical record C. move the pulse ox electrode to different location D. recalibrate the pulse oximeter C. When a pulse oximeter is reading a low oxygen saturation but has a different heart rate compared to that which can be palpated, the oxygen saturation reading is in error and should not be reported. The cause for this is likely poor peripheral perfusion. An appropriate response is to first move the pulse ox probe to a different location on the body. A secondary option is to replace the probe.

An adult female experiences nausea, vomiting, tachycardia, and PVCs following a routine aerosolized bronchodilator treatment with 0.63 mg Xopenex. The therapist should recommend which of the following for the next treatment?

A. change to 0.5 mL albuterol B. reduce Xopenex does to 0.31 mg C. administer Spiriva (tiotropium) in place of the Xopenex D. administer IV lidocaine twice daily A. Because this patient is demonstrating an adverse reaction to Xopenex, a change to a different bronchodilator medication is appropriate. Albuterol is a similar front-door bronchodilator that could be exchanged for Xopenex or levalbuterol.

An adult patient receiving VC, A/C ventilation is demonstrating respiratory distress. A chest radiograph shows the tip of the ET tube is level with the clavicles and breath sounds are diminished bilaterally. Ventilator settings are: FIO2 0.45 VT 400 mL f 16/min PEEP 5 cm H2O The therapist should

A. check cuff pressure. B. increase tidal volume. C. increase minute alveolar ventilation. D. advance the ET tube. D. The tip of the ET tube is considered too high if level with the clavicles. This could cause respiratory distress if there is an inadequate seal due to improper placement of the ET tube.

Blood pressure obtained by a sphygmomanometer reads higher than the indicated blood pressure from an arterial line on the same patient. To correct problem, the respiratory therapist should FIRST

A. check for air bubbles in the transducer dome B. flush the art line with sodium heparin C. advance the arterial line catheter D. discontinue use of the sphygmomanometer A. When blood pressure by sphygmomanometer (cuff) differs from the blood pressure obtained from an indwelling arterial line, the blood pressure taken by cuff is considered more accurate and reliable. Problems that may occur in the ART line include clots in the line and bubbles in the transducer dome.

A female patient is intubated with a 7.0 mm endotracheal tube. The tube is noted to be positioned at 26 cm at the teeth. Breath sounds are not bilateral. Which of the following will best confirm placement of the artificial airway?

A. chest radiograph B. chest rise C. breath sounds D. capnography A. There are several ways to assess proper endotracheal tube placement, such as auscultation of breathsounds, observance of symetric chest rise, or end-tidal CO2 evaluation. However, this scenario asks for confirmation of proper artificial airway placement, which is accomplished with a chest X-ray.

A pulmonary function study reveals a forced vital capacity (FVC) that is greater than a slow vital capacity (SVC). To which of the following can this be attributed?

A. chronic air-trapping B. poor patient effort on the SVC maneuver C. partial diaphragmatic paralysis D. excessive effort on the FVC maneuver B. When the patient exhales slowly they are able to breathe out more air than if they were to exhale quickly. For this reason, and FVC should always be less than an SVC. If FVC is noted to be higher than the SVC, this is clear evidence that the patient's effort during the SVC maneuver was poor or insufficient.

The respiratory therapist is educating the family of a patient who will be receiving mechanical ventilation at home. On which of the following devices should the therapist educate the family to prepare them to deal with a power failure?

A. circuit breaker box B. manual resuscitator (bag-valve) C. ventilator reset switch D. telephone and the appropriate numbers to call B. In the case of a power failure a patient who is ventilator-dependent at home should have a manual resuscitator bag and mask at the ready.

A respiratory therapist is treating all patients in the intensive care unit as if they have HIV and hepatitis. This approach is called

A. co-horting. B. standard precautions. C. enteric isolation. D. patient discrimination. B. he standard precautions (also called universal precautions) approach includes treating all patient as if they are infectious.

Which of the following classes of medication is most helpful in treating an adult who has moderate stridor following endotracheal extubation?

A. corticosteroid B. vasoactive C. aminoglycocide D. decongestant D. Mild stridor can be treated with a cool mist or racemic epinephrine. Moderate stridor should be treated with racemic epinephrine. Severe or marked stridor should be managed as an emergency and treated through intubation or an emergency tracheotomy

Which of the following data represents subjective information?

A. cyanosis in the lips B. scattered infiltrates on a chest radiography C. dyspnea D. decreased tidal volume C. Subjective information is that clinical data that can be only reported by the patient - it cannot be independently observed by the caregiver. Of the options listed, dyspnea is in this category.

A patient receiving volume-controlled ventilation is developing auto PEEP on the following settings Mode A/C VT 600 mL PEEP 10 cm H2O Mandatory rate 18/min Flow 50 L/min Which of the following adjustments on the ventilator would help to decrease auto PEEP?

A. decrease in mandatory rate B. increase in PEEP C. switching to P/C mode D. decrease in flow rate A. AutoPEEP is caused by insufficient time to exhale gases before the next mechanical breath begins. This may be reduced by decreasing tidal volume, decreasing respiratory rate, and increasing inspiratory flow. All of these factors cause a decrease in inspiratory time, allowing more time for expiration, which will decrease air-trapping and will lower AutoPEEP.

For a patient receiving volume-controlled ventilation, which of the following will not results in increased mean airway pressure?

A. decrease in mandatory rate B. use of inspiratory pause C. decrease in inspiratory flow rate D. use of an expiratory retard A. Decreasing inspiratory flowrate, use of an expiratory retard, and use of inspiratory pause will all cause an increase in mean airway pressure. Decreasing rate will lower mean airway pressure.

A patient receiving volume-controlled ventilation is experiencing a steady increase in both peak inspiratory and plateau pressures over the last 2 days. The following current data is available: VT(set) 600 mL mandatory rate 14 Flow rate 60 L/min FIO2 1.0 PEEP 15 cm H2O Pressure (peak) 58 cm H2O Pressure (plateau) 39 cm H2O Which of the following may be helpful to the patient?

A. decreased inspiratory time B. high flow rates C. inspiratory hold D. Inverse I:E ratio D. This patient has significantly decreasing pulmonary compliance. Methods of ventilation that may assist this patient include the use of high-frequency jet ventilation, inverse I:E ratio ventilation, pressure-control ventilation, and high-frequency oscillation ventilation. All of these options are correct and may be helpful.

An 80-kg (165-lb), 180-cm (6-ft 1-in) patient who was intubated for respiratory failure as a result of barbiturate overdose is receiving PC, A/C ventilation. The following is observed: FIO2 0.6 IP 18 cm H2O f 16/min PEEP 18 cm H2O ABGs pH 7.30 PaCO2 49 torr PaO2 80 torr Exhaled VT 410 mL I:E 1:2.4 The respiratory therapist should recommend

A. decreasing the rate. B. decreasing inspiratory time. C. increasing inspiratory pressure. D. decreasing inspiratory flow. C.

A patient with a history of elevated pulmonary pressure, caused by chronic failure of the mitral valve, has a butterfly pattern on a chest radiograph. The trachea is in alignment with the spinal process. Breath sounds are bilateral but distant with moist rales. Which of the following conditions is most likely?

A. dehydration B. pulmonary hypertension C. pulmonary emboli D. acute respiratory distress syndrome B.

A ventilator is sounding two alarms - low temperature and low volume. This could be caused by which of the following?

A. depleted humidification reservoir B. ruptured ET tube cuff C. disconnect in the patient-ventilator interface D. disconnected temperature probe in the ventilator circuit D. The combination of low temperature and low volume alarms is commonly caused by a problem related to the temperature probe. As the temperature probe is inserted inside the inspiratory limb of the ventilator circuit, it can sometimes become dislodged creating a leak in the circuit. If the probe becomes dislodged, it is no longer exposed to the heated gases and can therefore, trigger a low-temperature alarm. A low volume alarm will result from the leak through the port where the probe is normally nestled.

A radiographic image of the chest is the best way to

A. determine whether a mechanically ventilated patient is ready for weaning. B. perform a quick assessment of endotracheal tube mal-positioning. C. assess for respiratory failure. D. confirm proper endotracheal tube placement. D. There are several ways to check for proper endotracheal tube placement, such as auscultation of breathsounds, observance of symetric chest rise, or end-tidal CO2 evaluation. However, this scenario asks for confirmation of proper tube placement, which is accomplished with a chest X-ray.

A 58-year old adult complains of a non-productive cough for the past 2 months. Auscultation reveals a slight inspiratory and expiratory wheeze over the left lobe. Which of the following most likely represents the patient's condition?

A. development of a mass in the bronchials of the left lung B. left-sided bronchoconstriction C. foreign body in the left mainstem bronchus D. ALS (acute lung injury) A. When a patient is wheezing bilaterally the most likely cause is bronchoconstriction. However, when the patient is wheezing unilaterally, the cause is either foreign body aspiration, or a cancerous mass in the upper airway. In this case, the nonproductive cough has persisted for two months, indicating a greater likelihood of it being a mass in the lungs. If a foreign body is apirated the affect is acute the patient usually seeks medical treatment right away.

Immediately following oral endotracheal extubation the patient should first be

A. directed to cough B. receive hydration C. suctioning D. trained on PEP therapy A. Immediately following extubation the patient should be first instructed to cough. If effective this is the quickest method to remove secretions immediately from the upper airway. Suctioning is appropriate if the patient is unable to cough. Hydration is time-consuming and while important will not help to immediately remove secretions in the upper airway. PEP therapy is intended to mobilize secretions from the lower airways which is not the most immediate concern following extubation.

A patient receiving oxygen therapy at 60% has the following arterial blood gas results: pH 7.41 PaCO2 39 torr PaO2 579 torr HCO3- 24 mEq/L The respiratory therapist should

A. discontinue oxygen therapy B. change the Sanz electrode C. decrease FIO2 to 0.40 D. reject the results D. This blood gas shows a PaO2 of 279 mmHg. However, the patient is only on 60% oxygen. When one calculates the alveolar area equation, the maximum oxygen tension found in the alveoli is about 370 mmHg. Therefore, a PaO2 of 579 mmHg is impossible and suggests the analyzer is in error. The results should be rejected.

A 43-year-old male is receiving VC ventilation in the SIMV mode with FIO2 0.4, rate 6/min, VT 400 mL, PS 30 cm H2O. Spontaneous VT is 380 mL. MIP is - 22 cm H2O. The respiratory therapist should recommend

A. discontinuing mechanical ventilatory support. B. increasing minute alveolar ventilation. C. decreasing pressure support. D. increasing ventilatory support. C. Generally, the patient appears to be ventilating fairly well with an adequate MIP. Weaning is appropriate. However, a PS of 30 cm H2O is high and should be decreased before additional ventilator weaning is attempted.

Which of the following is an important element of instruction to a home care patient who will be receiving continuous oxygen therapy at 2 L/min by nasal cannula?

A. do not change flow rate without a physician order B. increase flow rate during exercise C. run a flow calibration check once monthly D. check oxygen concentrator flow rate once a week A. When instructing a home care patient on the use of their oxygen concentrator, and using oxygen in general, the respiratory therapist must include those things that are appropriate for the patient. In this question no open flames, use of grounded plugs only, and a restriction on changing the flow rate without a physician order are appropriate instructions. It is inappropriate to ask the patient to check or calibrate the concentrator's flow rate once a week. This sort of maintenance should be done by a respiratory therapist.

The respiratory therapist notices that the blood pressure reading from an indwelling arterial line differs from that of the cuff and sphygmomanometer. The therapist should

A. document the arterial line blood pressure. B. document BP ascertained by the cuff. C. have the patient change arm positions and obtain another reading from the arterial line. D. replace the transducer on the arterial line. B. The cuff pressure should be most trusted. If there is a difference in the two sources, the arterial line is most likely the problem. This can be solved by 'zeroing' the arterial line. Replacing the transducer or having the patient arbitrarily change arm position is not helpful.

What unit of measure is associated with SVR?

A. dynes B. newton's C. cm H2O D. mm Hg A.

A 45-year old male with a BMI of 43 is undergoing pulmonary function testing with the following results: Observed Predicted DLco mL/min/mmHg 20 25 SVC (L) 4.16 4.66 Fev1/FVC(%) 88 75 Which of the following is consistent with these data?

A. emphysema B. asthma C. pulmonary embolism D. restrictive impairment D.

The respiratory therapist measures the volume of a single exhalation on a patient who is breathing at normal rate and rhythm. Which of the following volumes has the therapist observed?

A. expiratory reserve B. tidal C. inspiratory reserve D. functional residual B. The volume that is being observed when a patient is breathing normally is called tidal volume.

A patient receiving mechanical ventilation is experiencing an increase in autoPEEP. Which of the following should the respiratory therapist increase to lower autoPEEP?

A. flow rate B. mandatory rate C. pressure support D. PEEP A. AutoPEEP is caused when a mechanical ventilator delivers the next breath before the patient has exhaled completely. This air-trapping causes over distention of the alveoli and results in intrinsic PEEP. To minimize this, greater time for expiration must be allowed. This can be done by increasing flow rate, which will decrease inspiratory time and prolonged expiratory time.

A respiratory therapist notes during a ventilator/patient check that the RAW is lower than in previous assessments. One explanation for this change is a reduced

A. flowrate. B. fever. C. I:E. D. waveform. A. As flow rate increases airway resistance also increases due to the turbulence of air movement. Therefore, reductions in flow rate will likewise reduce airway resistance.

A tandem aerosol device is used

A. for patients requiring FIO2 of 0.50 or greater. B. to deliver sufficient flowrates through large-volume nebulizers. C. on patients with croup D. whenever the flowrate is set to 15 L/min or above. B. A tandem aerosol device is used when only one device is incapable of producing enough total gas flow to meet or exceed the patient's inspiratory demand.

Which of the following is LEAST likely to contribute to spreading of nosocomial infections in a hospital?

A. frequent disconnection of a ventilator circuit B. poor hand washing among staff C. use of same equipment for multiple patients D. use of a closed in-line suction catheter D. Poor handwashing, disconnection of the ventilator circuit, and use of the same equipment for multiple patients will all increase the incidence of nosocomial infection. Use of a closed in-line suction catheter is the only option that will decrease the chance of nosocomial infection.

Which of the following is LEAST likely to promote the spread of nosocomial infection in a hospital?

A. frequent disconnection of a ventilator circuit B. use of unit dose medication C. changing small volume nebulizer equipment every 48 hours D. poor hand washing among staff B. The best option is to use unit dose medication. The use of multiple-dose bottles of medication used to be more prelavent. When doing so the dropper stem of the medication bottle could touch the internal surface of the nebulizer, possibly picking up and depositing pathogenic organisms. Changing the nebulizer every 48 hours may also be helpful but not as effective at preventing nosocomial infections. The other options in this question are likely to increase nosocomial infection rates in hospitals.

A neonate delivered 3 minutes ago has an APGAR score of 5. The therapist should do which of the following?

A. get an arterial blood gas B. perform CPR C. place the infant on supplemental oxygen D. obtain umbilical cord blood gas values C. Apgar scores of 4 to 6 indicate the administration of supplemental oxygen and general stimulation of the infant. Apgar scores higher than six require only routine care of the infant. Apgar scores of 0-3 indicate the need for CPR.

A patient is receiving IPPB treatments following surgical hernia repair. Several hours after surgery, the patient is alert and following commands. Incentive spirometry is started. Which of the following best describes the direct purpose of the therapy?

A. guard against barotrauma B. prevent arterial hypocapnia C. prevention of post-operative pneumonia D. increase arterial oxygenation C. The central, direct purpose of incentive spirometry is to prevent post-operative complication such as alveolar collapse and/or consolidation of secretions (pneumonia).

A patient receiving oxygen at home via a molecular sieve device complains that he is not feeling any oxygen coming from the cannula. The respiratory therapist should

A. have the patient utilize portable oxygen until the oxygen concentrator is examined B. inform the patient that this finding is normal C. instruct the patient to call the manufacturer of the concentrator D. instruct the patient to do a flow check/calibration of the concentrator A. When the patient complains of not receiving enough oxygen or when the function of oxygen delivery device is a question, the first best option is to ensure adequate oxygenation of the patient by switching to another modality that is known to be functioning properly.

The preferred method of delivery for humidification for a pediatric patient during a transport lasting twelve hours is a (an)

A. heated humidifier B. cascade humidifier C. bubble humidifier D. HME D. A heat moisture exchanger (HME), is inexpensive, convenient, and practical for a relatively short duration transport. The other options would be difficult or impossible to manage for a patient transport.

Which of the following methods should the respiratory therapist recommend for a patient receiving mechanical ventilation who requires chest physiotherapy?

A. high-frequency chest wall compression B. pressure control ventilation C. autogenic drainage D. huff coughing A. High-frequency chest wall compression, also called oscillation or VEST therapy, is a suitable airway clearance method in place of chest percussion for a patient who is on a ventilator.

A family of four is admitted to the emergency room after being found sleeping in their car on the side of the road. Paramedics placed all four patients on nonrebreathing masks. All four patients are lethargic and generally unresponsive. A blood gas shows COHb to be 60%. The respiratory therapist should recommend

A. hyperbaric oxygen therapy B. provision of mechanical ventilatory support C. a reduction in FIO2 D. administration of Doxapram A. A COHb of 60% is indicative of carbon monoxide poisoning. One effective treatment for carbon monoxide poisoning is hyperbaric oxygen therapy. Hyperbaric therapy drastically increases the oxygen tension in the alveoli and lowers the half-life of carbon monoxide that is tied to the hemoglobin.

The respiratory therapist should respond to the following graph by checking for which of the following?

A. hypercapnia B. hypercarbia C. disconnection in the patient-ventilator interface D. poor alveolar perfusion C. The capnograph wave form shows a sudden fall to Zero end-tidal CO2. This is most likely caused from a disconnection of the ventilator circuit.

A patient is in the intensive care unit (ICU) with severe mitral valve dysfunction. The patient is being continuously monitored for ECG, SVO2, and SAO2. The following hemodynamic data is available: mPAP 30 mm Hg PCWP 18 mm Hg CVP 10 mm Hg C.O. 3.5 L/min SVO2 68% SAO2 97% Which of the following would provide most benefit to the patient?

A. implement Extracorporeal membrane oxygenation (ECMO) B. administer prostacyclin C. give IV epinephrine D. schedule the patient for pulmonary angioplasty B. Hemodynamic evidence is consistent with severe pulmonary hypertension secondary to left cardiac pump failure. Action should be taken to lower pulmonary pressures. Prostacyclin is an appropriate drug for that purpose.

Prior to a pulmonary function test, a respiratory therapist conducts spirometry calibration with a 3.0 Liter super-syringe obtaining the following results: Trail 1 (L) Trial 2 (L) Trial 3 (L) 2.74 2.68 2.72 The spirometer is

A. imprecise. B. inaccurate. C. accurate. D. precise. B. The spirometer is considered accurate if the results are no more than 3% above or 3% below the 3 L calibration syringe. This means the accuracy range is 2.85 L -3.15 L. In this case all values are outside this range and therefore the spirometer is considered to be inaccurate.

A pressure-volume ventilator graphic fails to return to the point of origin. Which of the following could be the cause?

A. inadequate PEEP B. leak in the ventilator circuit C. pulmonary distension D. excessive cuff pressure B. A pressure volume loop on a ventilator should end in the same place it starts. If a pressure volume loop fails to return to zero, or its point of origin, there is likely a leak in the circuit, reducing return volume.

To prevent post-operative complications, a patient is receiving intermittent positive pressure breaths by inflatable mask through a pressure-cycled ventilator. During a breath, the respiratory therapist notices that the pressure needle fails to rise to the set limit and the machine fails to cycle into exhalation. Which of the following could be the cause?

A. inadequate inspiratory flow rate B. unseated sensitivity valve C. occlusion of the mouthpiece with the tongue D. inadequate lip seal around the mouthpiece D.

Pursed-lip breathing with extended expiratory time will have what effect on the I:E ratio?

A. increase B. no change C. decrease D. inverse C. Pursed-lips breathing, coupled with an increase in expiratory time will result in a reduced I:E ratio, where inspiratory time is decreased, and expiratory time is increased.

A patient is receiving non-invasive positive pressure ventilation with an IPAP of 20 and an EPAP of 10 cm H2O. Which of the following changes would remedy the patient's snoring while asleep?

A. increase EPAP B. decrease EPAP C. increase IPAP D. decrease IPAP A. A patient who has signs of obstructive sleep apnea, such as snoring, would benefit most from increased expiratory pressure. This continuous pressure combats obstruction caused by relaxation of upper airway tissues.

An infant is receiving CPAP to help resolve hypoxemia. Radiological examination reveals a perforation in the right lung. Cyanosis persists with increasing respiratory acidosis. The respiratory therapist should suggest

A. increase FIO2 B. increase CPAP C. high frequency jet ventilation D. decrease CPAP C. There are two ways to deal with a perforation in one of the lungs. When the patient is receiving mechanical ventilation, the respiratory therapist must be careful not to expose that lung to too much pressure. This can be accomplished by either employing independent lung ventilation with a double lumen endotracheal tube, or by employing high-frequency jet ventilation, which will keep mean airway pressure low and avoid further damage to the lung.

The respiratory therapist obtains the following blood gas data on a patient receiving mechanical ventilation in the assist/control mode at FIO2 0.6, PEEP 5 cmH2O and tidal volume of 450 mL. Corresponding arterial blood gas results are as follows pH 7.28 PaCO2 52 torr PaO2 45 torr HCO3- 24 mEq/L BE +1 mEq/L The therapist should recommend which of the following changes?

A. increase FIO2 B. switch to pressure-control mode C. increase PEEP D. increase alveolar ventilation D. Blood gases reveal poor ventilation as well as poor oxygenation. Ventilation should be addressed before oxygenation. To correct this, of the options given, alveolar ventilation should be increased. Other options either relate to oxygenation or the management of airway pressures.

An 75-kg (165-lb), 5-ft 11-in. female patient with pneumonia is receiving mechanical ventilator support by a Servo adult ventilator on the following settings with corresponding arterial blood gas values: Mode A/C Mandatory rate 16 VT 400 mL FIO2 0.5 pH 7.32 PaCO2 48 mmHg PaO2 72 mm Hg HCO3- 24 mEq/L BE 0 mEq/L Which of the following actions is most appropriate?

A. increase FIO2 B. increase tidal volume C. increase PEEP D. increase mandatory rate B. The blood gas reveals the patient is not ventilating enough. An increase in mandatory rate seems to be appropriate, but a closer look at the ventilator settings shows a different problem. The patient's ideal body weight is about 75 kg. This means tidal volume must be at least 450 mL. The ventilator is set to only 400 mL. So, the first step would be to adjust the tidal volume to the appropriate range. This could, by itself, correct the hypoventilation.

A patient with COPD, receiving mechanical ventilation, is undergoing early extubation to non-invasive ventilation. Immediately following extubation, the patient is placed on bi-level ventilation with settings IPAP 20 cm H2O, EPAP 5 cm H2O. Thirty minutes later, the patient begins to exhibit moderate inspiratory stridor that advances to a marked condition. The respiratory therapist should

A. increase IPAP pressure to 25 cm H2O. B. administer nebulized Albuterol. C. intubate the patient. D. administer racemic epinephrine by small volume nebulizer. C. Because of the marked status of the stridor (life-threatening level), aerosolized racemic epinephrine is insufficient. The required action is to reestablish an airway immediately.

A patient is engaged in a smoking cessation program. During a routine visit to the counseling clinic, the patient describes shortness of breath (dyspnea) after every meal. Which of the following instruction should the therapist provide?

A. increase complex carbohydrate consumption between meals B. eat small meals more frequently, avoid too many carbohydrates C. consume only low-fat meats and pure grains D. drink 10-16 8oz glasses of water daily, even when not thirsty B. The general nutritional advice for a patient with chronic obstructive pulmonary disease is to eat smaller meals more frequently and to avoid excessive consumption of carbohydrates, which produce carbon dioxide and place a greater ventilatory load on the patient.

The following data is obtained on a patient with asthma who is receiving continuous aerosolized Albuterol at 10 mg/hr for the last 45 minutes. 8:00 8:30 8:45 Peak flow (% of pred) 30 40 60 Sat (%) 86 88 90 Heart rate (bpm) 120 109 101 Breath sounds have changed from diminished to audible wheezing. The therapist should recommend

A. increase dosage B. continue therapy C. discontinue therapy and report finding to the physician D. switch to continuous Xopenex B. A patient is receiving continuous albuterol treatments has peak flows that show an improvement over time. However, peak flow is not yet normal. Therefore, continuous bronchodilator therapy should be continued. Since the current drug is considered effective, it should not be changed.

The following data is obtained on a patient with asthma who is receiving continuous aerosolized Albuterol at 10 mg/hr for the last 45 minutes. 8:00 8:30 8:45 Peak flow (% of pred) 30 40 60 Sat (%) 86 88 90 Heart rate (bpm) 120 109 101 Breath sounds have changed from diminished to audible wheezing. The therapist should recommend

A. increase dosage B. switch to continuous Xopenex C. continue therapy D. discontinue therapy and report finding to the physician C. A patient is receiving continuous albuterol treatments has peak flows that show an improvement over time. However, peak flow is not yet normal. Therefore, continuous bronchodilator therapy should be continued. Since the current drug is considered effective, it should not be changed.

A patient receiving volume-controlled ventilation has an infrared CO2 detector attached at the end of the ET tube. The capnograph shows a PetCO2 of 47 torr. The respiratory therapist should

A. increase expiratory time B. continue current therapy C. increase minute ventilation D. increase inspiratory flow rate C. An end-tidal CO2 of 47 mmHg is consistent hypoventilation.

A burn trauma patient who is also suspect for carbon monoxide poisoning is receiving oxygen therapy by a nonrebreathing mask. The respiratory therapist observes that the reservoir bag collapses completely with each inspiration. The therapist should

A. increase flow to the reservoir bag B. continue current therapy as this is normal C. obtain a larger reservoir D. remove the one-way valve in the reservoir A. When using a non-rebreathing mask, the reservoir bag should collapse partially with each breath. If the bag does not collapse at all, there is likely a problem in the seal between the mask and the patient's face. If the reservoir collapses completely, the problem is most likely inadequate flow to the reservoir bag. The solution is to increase the flow.

While administering an IPPB treatment with 3.0 cc normal saline, the therapist notices the needle on the manometer is NOT showing a smooth rise to pressure. The therapist should

A. increase flow. B. adjust sensitivity. C. check the air-mix knob. D. decrease pressure. A. When a needle on the pressure manometer of an IPPB machine does not rise smoothly with inspiration is likely due to the fact that the patient's inspiratory flow is exceeding the flow of the machine. Therefore, the most appropriate adjustment is to increase the flowrate of the IPPB machine.

The following vetilator settings and data is available: Rate 12 VT 800 mL I:E 1:2 FIO2 0.40 PEEP 5 cm H2O Flow 20 L/min The ventilator is alarming. What should the respiratory therapist recommend?

A. increase flowrate B. increase I:E C. obtain an arterial blood gas D. remove PEEP A. A quick glance at these ventilator settings shows everything to appear normal except for the flowrate. The minimum flow rate required to sustain an I:E ratio of 1:2 and a minute ventilation of 9.6 L is 28.8 L. The flowrate should be increased.

A patient receiving volume-controlled ventilation is developing auto PEEP on the following settings Mode A/C VT 600 mL PEEP 10 cm H2O Mandatory rate 18/min Flow 50 L/min Which of the following adjustments on the ventilator would help to decrease auto PEEP?

A. increase in PEEP B. decrease in flow rate C. switching to P/C mode D. decrease in mandatory rate D. AutoPEEP is caused by insufficient time to exhale gases before the next mechanical breath begins. This may be reduced by decreasing tidal volume, decreasing respiratory rate, and increasing inspiratory flow. All of these factors cause a decrease in inspiratory time, allowing more time for expiration, which will decrease air-trapping and will lower AutoPEEP.

The C(a-v)O2 on a patient in the cardiac intensive care unit has decreased from 9.1 vol% to 4.3 vol%. What other change has also likely occurred?

A. increase in PaO2 B. increase in cardiac output C. increase in tissue oxygen consumption D. decrease in cardiac stroke volume B. When the C(a-v)O2 is changing in one direction, cardiac output is changing in the opposite direction. In this case the C(a-v)O2 has fallen from 9.1 vol% to 4.3 vol%. This decrease indicates cardiac output must be increasing. It also indicates tissue oxygen consumption is decreasing. This is true because when cardiac output decreases the blood flowing past the tissue perfusion beds slows, allowing more time for the tissues to extract and consume more oxygen. However, when cardiac output is high the opportunity to consume more oxygen from the blood is decreased because the blood is moving more quickly.

A 62-kg (136-lb), 165-cm (5-ft 5-in) male patient is receiving PC, A/C ventilation at the following parameters: FIO2 0.60 IP 38 cm H2O (f) 10 VT (exhaled) 690 mL PEEP 5 cm H2O I-time 1:2.8 ABGs pH 7.55 PaCO2 28 torr PaO2 112 torr HCO3- 24 mEq/L A proper change in the ventilator parameters should include a(n)

A. increase in flow rate. B. decrease in inspiratory pressure. C. reduction in the mandatory rate (f). D. reduction in FIO2. B.

Which of the following may occur as a result of stimulation of the vagal reflex during a suction procedure?

A. increase in heart rate B. PVCs C. decrease in blood pressure D. first degree heart block C. Suctioning can cause a decrease in heart rate and a decrease in blood pressure. This natural, physiological response is associated with stimulation of the vagal nerve during suctioning. This is also called the vagal reflex.

What would occur on a time-cycled ventilator with a fixed rate if the inspiratory flow rate were increased?

A. increase in inspiratory time B. increase in tidal volume C. decrease in inspiratory time D. decrease in tidal volume B. On a time-cycled ventilator inspiratory time is predetermined. So, if the flow rate is increased while the mandatory rate is unchanged, the result would be an increase in tidal volume.

A patient is returned from open-heart surgery, where the mitral valve was replaced. Within thirty minutes, a low PvO2 is noted. This would indicate a(n)

A. increase in oxygen delivery to the tissues. B. decrease in cardiac output. C. reduction in oxygen consumption at the tissue level. D. decrease in C(a-v)O2. B. A decrease in PvO2 indicates a widening of the arterial-venous gap in PO2 (oxygenation). This means oxygen consumption is increasing and therefore, cardiac output must be decreasing. Oxygen delivery to the tissues would not be increased. The C(a-v)O2 would increase if PvO2 is decreasing.

A respiratory therapist changes from a normal adult ventilator circuit to a heated-wire circuit. Arterial blood gases are as follows: pH 7.31 PaCO2 48 torr PaO2 81 torr HCO3- 24 mEq/L BE 0 mEq/L Which of the following changes is most indicated?

A. increase inspiratory flow rate B. add 100 mL of deadspace at the patient-ventilator interface C. revert to the normal non-heated wire circuit D. remove 50-100 mL of deadspace between the wye and patient D. When arterial carbon dioxide is high, there are three options. Respiratory rate may be increased, tidal volume may be increased, or deadspace may be removed. Of the options offered removing deadspace is the best option and is the only option that will reduce carbon dioxide. What also makes this appropriate is the fact that CO2 is off target by a very small amount, making a change in deadspace appropriate.

Which of the following is most effective at reducing arterial carbon dioxide levels in a patient receiving mechanical ventilatory support in the pressure control ventilation (PCV) mode?

A. increase inspiratory time B. increase inspiratory flow rate C. decrease peak inspiratory pressure D. increase set tidal volume A. When the patient is receiving mechanical ventilatory support in the pressure control mode, reducing CO2 is not accomplished by changing tidal volume but rather, increasing inspiratory time. Tidal volume is not a control that is set directly in pressure control ventilation mode. But, increasing inspiratory time has the same result.

A 7-year-old patient is receiving mechanical ventilation with a PB 840 volume ventilator with an adult circuit. The end-tidal CO2 monitor is indicating a PetCO2 of 56 mmHg. Which of the following is most appropriate?

A. increase mandatory rate B. add 50 mL of deadspace C. remove 50 mL of deadspace D. switch to a pediatric circuit A. An end-tidal CO2 of 56 mmHg approximates an arterial CO2 of about 66 mm Hg. This is a definite indication of hypoventilation and would best be remedied by increasing minute ventilation. This may be done by increasing tidal volume or increasing rate. Adding dead space would increase end-tidal and arterial CO2 even further. Removing dead space, while a step in the right direction, isn't a sufficient response. Changing to a pediatric circuit is not helpful.

A patient in the intensive care unit is showing signs of air-hunger while receiving mechanical ventilation on the following settings and parameters: Mode SIMV VT (set) 600 mL VT (spont) 220 mL Mandatory rate 4 Total rate 24 FIO2 0.5 PEEP 5 cm H2O SpO2 88% Which of the following will most likely correct the problems?

A. increase mandatory rate B. add pressure support C. increase FIO2 to 0.7 D. increase PEEP to 10 cmH2O B. Examination of the ventilator settings on this patient show that the patient is weaning. This is manifested by the SIMV mode and the set mandatory rate of 4. When we look at what the patient can achieve, we see their spontaneous tidal volume is only 220 mL. That is not enough to sustain life. We also see their total rate is 24. That is too high. The patient is compensating for low spontaneous tidal volumes by increasing their rate. Together, both of these things are manifestations of high work of breathing and the patient will be unable to wean in this condition. To assist the patient, we must help them increase their spontaneous tidal volume. This is done by adding pressure support.

A 62-kg (136 lb) male is receiving volume-controlled ventilation on the following settings: Mode SIMV Mandatory rate 10 Total rate 16 Tidal volume 400 mL Pressure support 6 cm H2O VT (spontaneous) 400 mL PEEP 10 cm H2O Arterial blood gas analysis reveals pH 7.29 PaCO2 53 torr PaO2 83 torr HCO3- 25 mEq/L BE -1 mEq/L Based on this data, the first recommendation of the respiratory therapist should be to

A. increase mandatory rate to 14 B. increase pressure support to10 cm H2O C. increase tidal volume to 550 mL D. remove deadspace A. Arterial blood gas analysis reveals hypoventilation but normal oxygenation. This may be resolved by increasing minute ventilation, which can be done by either increasing tidal volume or mandatory rate. Because CO2 is out of range by more than 4 mmHg the best option would be to increase mandatory rate to 14.

The following arterial blood gas results are recorded for a patient during cardiopulmonary resuscitation: pH 7.10 PaCO2 46 torr PaO2 208 torr HCO3- 12 mEq/L FIO2 1.0 On the basis of these values and the following ECG waveform, the respiratory therapist should recommend

A. increase manual ventilation rate B. administer sodium nitroprusside C. sodium bicarbonate administration D. decrease FIO2 C. This patient shows acidosis, as manifested by the pH, but has a nearly normal CO2. A further examination of the HCO3 shows that the source of the acidosis is metabolic. Administration of sodium bicarb (HCO3-) is appropriate.

A patient with COPD is receiving volume-controlled ventilation with an I:E ratio of 1:2. A chest radiograph shows increased air-trapping. Which of the following will decrease air-trapping?

A. increase minute ventilation B. decrease inspiratory flow C. decrease expiratory time D. decrease rate D. A patient receiving mechanical ventilation who does not have sufficient expiratory time may begin to trap gases in the lungs causing development of autoPEEP. The cause of this is either an inspiratory flow rate that is too slow or a rate that is too high. The primary solution is to increase flowrate. That option is not offered here. So, the next best solution would be to decrease mandatory rate.

A patient with ARDS is receiving mechanical ventilatory support. An end-tidal CO2 monitor shows a PetCO2 of 59 torr. The therapist should

A. increase minute ventilation B. obtain an arterial blood gas C. continue current therapy and ventilator settings D. recalibrate the capnography A. Remember that the end-tidal CO2 data from a capnometer is usually 10 mmHg less than the corresponding arterial CO2 level. Thus, an end-tidal CO2 of 59 mmHg is consistent with an arterial CO2 of about 69 mmHg. The best solution to correct this problem is to increase ventilation.

A COPD patient who receives 2 lpm continuous oxygen therapy by nasal cannula is exercising in conjunction with a monitored pulmonary rehabilitation program. The patient has begun breathing quickly and deeply. To ensure consistent arterial oxygenation, the respiratory therapist should

A. increase oxygen flow rate B. use a partial rebreathing mask C. use a nonrebreathing mask D. decrease oxygen flow to 1 L/min A. Although COPD patients should rarely have more than 2 L/min. continuous oxygen, when they exercise and breathe more deeply, they inadvertently lower their FIO2, especially when on a nasal cannula. In such a case, it is appropriate to increase the flow rate temporarily for the duration of the exercise.

A patient is being mechanically ventilated by volume controlled ventilation. A chest tube drainage system is in place and 1200 mL of fluid has been collected from the left chest. A low volume alarm on the ventilator begins to sound. The respiratory therapist notices vigorous bubbling in the water seal chamber. The following data is available: pH 7.44 PaCO2 38 torr PaO2 88 torr HCO3- 24 mEq/L BE -1 mEq/L Mode Assist/control VT 600 mL Exhaled VT 396 mL Mandatory rate 12/min Total rate 12/min FIO2 0.60 The respiratory therapist's first action should be to

A. increase rate to 14 B. look for a leak between the water seal chamber and the patient C. decrease suction pressure to the system D. discontinue chest tubes B. Vigorous bubbling in the water seal chamber is abnormal and indicates the presence of a leak in the chest tube drainage system. The leak may exist somewhere between the water seal compartment and the patient. However, the leak may be caused from a perforated lung.

A patient has the following pressure volume graphic while receiving mechanical ventilatory support in the assist/control mode. The respiratory therapist should

A. increase sensitivity B. increase PEEP C. increase inspiratory flow rate D. decrease inspiratory flow rate A. The pressure volume loop has a pronounced "fishtail". This portion of the graph shows the negative deflection of pressure that is produced by the patient before the machine is triggered to administer a mechanical breath. The more significant the negative deflection, the more work and effort the patient must produce to receive a machine triggered breath. Ultimately, this will result in increased work of breathing and will tire the patient. To reduce this effort the therapist should increase sensitivity of the ventilator. Although not offered here, flow triggering is another method for increasing sensitivity and reducing work of breathing.

A spontaneously breathing patient has the following arterial blood gas results: pH 7.38 PaCO2 42 mmHg PaO2 76 mmHg HCO3- 24 mEq/L BE 0 mEq/L Which of the following supplemental oxygen levels is most appropriate?

A. non-rebreathing mask B. 5 L/min nasal cannula C. 2 L/min nasal cannula D. Venturi mask at 30% B. A patient who is showing signs of hypoxemia should receive supplemental oxygen. If the patient is not a COPD patient and the situation is not an emergency, then the proper supplemental oxygen is an adult therapeutic dose, which is 40% to 55%. Of the options available only 5 L/min nasal cannula will approach this. Other options are either insufficient or too much.

A respiratory therapist is asked to administer 70%/30% helium oxygen to a patient with a fixed upper airway obstruction. The therapist will expect to employ which of the following?

A. nonrebreather mask B. non-compensated flow meter C. high-flow nasal cannula D. blender A. Helium-oxygen gas mixtures are delivered to a spontaneously breathing patient by nonrebreather mask

Arterial blood gases on a patient in the cardiac intensive care unit are as follows: pH 7.35 PaCO2 45 mmHg PaO2 90 mmHg HCO3- 24 mEq/L BE 0 mEq/L Which of the following represents an accurate interpretation of these results?

A. normal acid base relationship B. respiratory acidosis C. compensated respiratory acidosis D. respiratory compensated metabolic alkalosis A. To determine the acid-base relationship from an arterial blood gas, one must first look at the CO2. In this case it's normal. Next, look at pH. Because it is also normal, there is a normal acid-base relationship.

A respiratory therapist is observing a chest radiograph of a 44-year-old patient with congestive heart failure. The therapist notes that the vertebrae are clearly visible through the heart shadow. It can be concluded that the film is

A. normal. B. overexposed. C. underexposed. D. improper for a patient with CHF. A.

A respiratory therapist would like to review the physician's recent thoughts regarding the prognosis and anticipated care plan for the patient. In which section of the medical record should the therapist plan on reviewing?

A. nursing notes B. discharge plans C. progress notes D. physician orders C. Technically, documentation in the progress notes does not constitute an official change in care. Notes in this section represent thoughts of healthcare workers regarding the potential problems, diagnoses, and treatment plans of the patient. To be made official, orders must be written in the chart as physician orders.

A bronchogram on a 14-year-old patient with cystic fibrosis shows consolidation in the left lung. To facilitate appropriate ventilation and draining of the affected area, the respiratory therapist should recommend placing the patient in which position?

A. on the side with the right side down B. on the side with the left side down C. supine D. reverse-Trendelenburg A. When significant consolidation is found in one lung, the patient should be positioned with the unaffected lung up. This allows for better total ventilation.

A patient with cystic fibrosis has had a single-bore tracheostomy tube in place for 6 months. While being examined in the E.R. with signs and symptoms consistent with pneumonia, the patient suddenly demonstrates marked ventilatory distress. The respiratory therapist inserts a 14-Fr. suction catheter and encounters resistance. With some effort, the therapist is able to finally pass a catheter and the patient's vital signs stabilize within a few moments. The therapist should suggest

A. oral intubation during hospitalization. B. administration of 20% acetylcysteine nebulizer, q4 hrs. C. changing to a fenestrated tracheostomy tube. D. hydration therapy with heated aerosol. C.

A respiratory therapist is alerted by a low-volume ventilator alarm on a patient who has a chest tube drainage system in place. Set tidal volume is 600 mL while return tidal volume is 190 mL. The therapist should FIRST

A. increase tidal volume to compensate B. clamp the chest tube near the patient C. disconnect the chest tube from wall suction pressure D. insert an additional chest tube B. A patient who is receiving mechanical ventilation, and has a chest tube drainage system in place, must be monitored carefully for lost volume through the lung through the chest tube drainage system. The clinical evidence of this is found in the return tidal volumes. If the volume administered is far greater than the return volume, volume must be being lost in the system somewhere. A loss of volume may be noticed by the excessive bubbling in the water seal compartment. To determine how it is being lost, the first step would be to clamp the chest tube near the patient. If bubbling in the water seal compartment stops, the volume being lost is most likely coming from a perforation in the lung. The patient requires surgery to repair this.

A pulse oximeter reading is significantly less than oxygen saturation by blood gas. Which of the following could be the cause?

A. increased SVR B. decreased QsQt C. marked hypotension D. decreased CVP C. Pulse oximeters are also known as single wavelength spectrophotometers. There are multiple reasons why they may read inaccurately. Those reasons include: high ambient light, the use of fingernail polish, poor peripheral perfusion, and marked hypotension. All are correct.

A physician asks the respiratory therapist to evaluate the effectiveness of PEP therapy on a patient with cystic fibrosis. The therapist can conclude the therapy is effective if

A. increased inspiratory capacity is observed B. improved arterial blood gas values are observed C. the patient develops rhonchi that clears with coughing D. increased oxygen saturation during treatment is noted C. Of the options listed the development of rhonchi, which means secretions in the large upper airways, is the best evidence that secretions are being mobilized by the PEP therapy. The other options offered are either too indirect or are not related.

The measurement of a patient's FRC is 50% higher when measured by body box compared to the results when measured by helium dilution method. Which of the following is the likely reason for the difference in this patient?

A. increased non-ventilated lung space B. poor patient effort during the helium dilution C. presence or choanale atresia D. small leak during the helium dilution maneuver A. A patient with COPD has significant non-ventilated lung space. The measurement of total lung capacity, FRC, and RV done by helium dilution and nitrogen washout will likely be less accurate and show a smaller FRC compared to pulmonary function testing done by a body box. A body box can indirectly measure non-ventilated alveolar space.

A respiratory therapist is reviewing the status of a 65-year-old patient who has been treated for obstructive sleep apnea for 3 years. The patient is recently complaining of daytime sleepiness and takes a nap in his car during his lunch break. He is currently receiving nasal CPAP at 10 cm H2O. Recently ordered polysomnography reveals multiple periods of apnea with no chest or abdominal movement. The therapist should recommend

A. increasing CPAP to 14 cm H2O B. switching to full face mask C. administering aminophylline by mouth nightly D. bi-level positive airway pressure D. Clinical evidence suggests that this patient is continuing to have signs and symptoms associated with obstructive sleep apnea, even though they're using CPAP at night. When CPAP is insufficient, bilevel therapy becomes the next step. This type of therapy provides additional pressure during the inspiratory phase in addition to the positive airway pressure provided continuously.

The following data has been collected over the last six hours: 1 pm 4 pm 7 pm Plateau pressure (cm H2O) 16 17 16 Peak pressure (cm H2O) 24 28 35 The respiratory therapist should respond to this information by

A. increasing inspiratory time B. suctioning the patient C. ordering a chest radiograph D. administering pulmonary surfactant B. Clinical data shows an increase in peak pressures. Peak pressures increase for two different reasons. One reason may be due to lungs that are becoming less compliant. The other reason may be due to increased airway resistance caused by such things as secretions in the airways or bronchoconstriction. This data shows a plateau pressure that is not increasing significantly. Therefore, increasing peak pressures are most likely caused by secretions in the airway and are best remedied by suctioning.

Nitric oxide is the appropriate therapeutic treatment for

A. infant pulmonary hypertension. B. wound therapy. C. significant increase in oxygen content. D. surgical anesthesia. A. Nitric oxide therapy is used to treat pulmonary hypertension. It works by dilating pulmonary vessels, especially when poor perfusion is present. The dosage is expressed in parts per million (ppm). It is also used in the treatment of a ARDS, pulmonary embolism, and pulmonary fibrosis.

A COPD patient with bacterial pneumonia and a long-standing, single-bore tracheostomy tube in place has experienced a sudden increase in ventilatory difficulty. The respiratory therapist is unable to pass a 12 Fr. suction catheter through the airway and into the trachea. The therapist should immediately

A. inject normal saline down the trach tube. B. instill 20% mucomyst (acetylcysteine) directly down the tube. C. utilize a 14 Fr. catheter. D. replace the airway with a fenestrated tracheostomy tube. D. The most likely cause of the non-patent airway is thick secretions or a mucus plug. The best, most timely course of action, would be to replace the tube. Because the patient may continue to have thick secretions, which can continue to obstruct the airway, use of a fenestrated tube would be a good decision. A fenestrated tube has an inner cannula that can be replaced without extubating the patient, should it become obstructed.

During tracheostomy care for a homebound patient who has had her stoma for two weeks, the fenestrated trach tube becomes blocked with a mucous plug. The respiratory therapist cannot pass a suction catheter. The next best action is to remove the

A. inner cannula. B. cotton tie. C. entire trach tube. D. subcutaneously trapped air. A. A fenestrated tracheostomy tube is a special tube used to allow patients to speak. It functions by having an inner cannula that rests inside an outer tracheostomy tube. When this cannula becomes clogged, the most immediate remedy would be to remove the inner cannula, and thus the plug, to allow the patient to breathe through the outer cannula of the tracheostomy tube.

During routine tracheostomy care, for a patient who is ventilator dependent and has had his trach in place for 2 weeks, the trach tube becomes dislodge during an acute coughing episode. The therapist should

A. insert a temporary laryngectomy tube, call the physician B. place gauze over the stoma, monitor carefully C. reinsert the trach tube D. perform oral intubation, call the anesthesiologist C. An expectorated tracheostomy tube may be reinserted immediately in an acute situation.

A patient in the cardiac intensive care unit is receiving dopamine HCl. Hypotension persists in spite of the administration of 2 units of blood and a quick infusion of additional blood expanding fluid in the past 20 minutes. The therapist should recommend

A. inserting an indwelling arterial catheter B. placing the patient in Trendelenburg position C. placing the patient in reverse Trendelenburg position D. administering additional fluid A. A patient who is having drastic changes in blood pressure due to depleting blood volume would do well to have an indwelling arterial catheter. This line will help to administer blood more quickly and will allow a live monitoring of blood pressure.

A patient undergoes a tracheostomy in an attempt to remedy severe sleep apnea due to obesity and a large neck size. Within a few hours after insertion of an 8.0-mm standard trach tube, the trach tube has become dislodged three times. The therapist should recommend

A. insertion of a longer trach tube B. insertion of a 7.0-mm standard trach tube C. switch to nasal CPAP D. insertion of an 8.0-mm fenestrated trach tube

Evaluating MIP (maximum inspiratory pressure) is most useful in evaluating which of the following?

A. inspiratory muscle strength B. presence of a restrictive pulmonary defect C. presence of obstructive pulmonary defect D. effectiveness of cough A. Maximum inspiratory pressure is most useful in evaluating for inspiratory muscle strength, or the ability to sustain ventilation. To determine if a patient has a restricted pulmonary defect one would need to do a pulmonary function test. The effectiveness of a cough would be assessed through doing a maximum expiratory pressure maneuver, or MEP.

Which of the following should be increased first for a patient on a mechanical ventilator who is showing signs of refractory hypoxemia?

A. inspiratory time B. end-expiratory pressure C. flow D. FIO2 B. The main treatment for refractory hypoxemia, which is seen in patients with ARDS, is PEEP, also called end-expiratory pressure.

A COPD patient with bacterial pneumonia and a long-standing, single-bore tracheostomy tube in place has experienced a sudden increase in ventilatory difficulty. The respiratory therapist is unable to pass a 12 Fr. suction catheter through the airway and into the trachea. The therapist should immediately

A. instill 20% mucomyst (acetylcysteine) directly down the tube. B. replace the airway with a fenestrated tracheostomy tube. C. inject normal saline down the trach tube. D. utilize a 14 Fr. catheter. B. The most likely cause of the non-patent airway is thick secretions or a mucus plug. The best, most timely course of action, would be to replace the tube. Because the patient may continue to have thick secretions, which can continue to obstruct the airway, use of a fenestrated tube would be a good decision. A fenestrated tube has an inner cannula that can be replaced without extubating the patient, should it become obstructed.

A patient receiving oxygen via a pulse-dose oxygen system complains of difficulty breathing and indicates she believes the cannula is not working properly. The respiratory therapist should

A. instruct the patient call the manufacturer B. instruct the patient to switch to her oxygen concentrator with a regular nasal cannula C. troubleshoot the pulse-dose regulator D. replace the oxygen tank B. When the function of an oxygen delivery device is in question, the first best option is to switch to a different mode of oxygen delivery to ensure that the patient is receiving oxygen. After the patient's oxygenation is secure, troubleshooting of the problem may begin.

The following laboratory data is observed in a patient who has been experiencing massive vomiting for 12 hours: BUN 18 mg/dL K+ 2.8 mEq/L Cl- 82 mEq/L Na+ 101 mEq/L RBC 6.0 g/dL Hb 12 g/dL Which of the following may also be observed?

A. involuntary muscle contraction in the extremities B. flattened T waves on ECG C. elevated pulmonary artery pressure D. loose bowel B. Close examination of this data reveals that the patient has hypokalemia, as shown by a potassium level that is well below normal. Normal potassium is about 4.0 mEq/L. Low potassium will cause the ECG tracing to demonstrate what is called flattened or dull T waves.

Which of the following is the correct postural position for drainage of the right lateral segment?

A. left side, HOB down 18 inches B. supine and slightly rotated to the left, HOB down 12 inches C. right side, HOB down 12 inches D. flat on stomach, HOB down 18 inches The right lateral segment is drained with the head of bed down 18 inches with the patient lying on the left side. A. The right lateral segment is drained with the head of bed down 18 inches with the patient lying on the left side.

A patient has the following pressure volume graphic while receiving mechanical ventilatory support in the assist/control mode. This graph indicates which of the following?

A. machine-triggered breath B. patient-triggered breath C. pulmonary distension D. inadequate PEEP A. A pressure volume graphic that has no "fishtail", or no negative deflection, is indicative of a machine-triggered breath. This means the patient did not initiate the breath.

Which of the following modalities is appropriate for a patient with cystic fibrosis who has a history of chronic hypoxemia and air-trapping?

. aerosol mask B. nasal cannula C. nonrebreathing mask D. partial rebreathing mask B. A patient with cystic fibrosis is a COPD patient. COPD patients should receive oxygen by Venturi mask running at 28% or less, or by nasal cannula running at 1 to 2 L/min.

A 65-year-old female patient experiencing progressive shortness of breath is diagnosed with COPD. Her pulmonary function test shows a slow vital capacity that is 95% of predicted. Which of the following would most likely be her residual volume compared to predicted?

A. 115% B. 80% C. 100% D. 95% A. COPD, chronic obstructive pulmonary disease, is a condition which results in air-trapping, which increases one's residual volume. Therefore the actual residual volume would be higher than the predicted residual volume.

Which pressure should not be exceeded when adding air to an oral endotracheal tube cuff?

A. 25 cmH2O B. 30 cmH2O C. 35 cmH2O D. 15 cmH2O A. When inflating an oral endotracheal tube cuff the pressure not to be exceeded is 25 cmH2O. More commonly, the exam uses the units of mmHg or torr. In those units, the pressure should not exceed 20 mmHg or torr. The reason for this limitation is that the capillary bed in the trachea has a mean pressure of about 20 mmHg. If the cuff pressure exceeds that, there is a risk of restricting blood flow and causing damage to the tissues of the trachea.

A 150-lb (68-kg), 5-ft 6-in (168 cm) female patient is receiving mechanical ventilation as follows: Mode Assist/control Alveolar minute ventilation 7.5 L/min Total Rate 15 PB 734 mm Hg What is the patient's set tidal volume?

A. 500 mL B. 550 mL C. 350 mL D. 700 mL A. Tidal volume is calculated by dividing the minute ventilation by the respiratory rate. By knowing any two of these three data, one can calculate the third. For example, to calculate minute ventilation, multiply the tidal volume by the respiratory rate. To calculate respiratory rate, divide minute ventilation by tidal volume.

Which of the following arterial carbon dioxide tension values is consistent with alveolar hyperventilation?

A. 55 torr B. 65 torr C. 45 torr D. 32 torr D. Alveolar hyperventilation will cause a reduction in both arterial and exhaled CO2. Since normal CO2 is around 40 torr, a CO2 of 32 would be consistent with alveolar hyperventilation.

A homebound oxygen-dependent patient receives 2 L/min oxygen by molecular sieve device. An electrical storm has resulted in the loss of electricity. She has 2 full E-cylinders. How many hours will her oxygen last?

A. 6 B. 12 C. 10 D. 8 C. A. 6 B. 12 C. 10 D. 8

In the presence of a normal pH, a PaO2 of 75 mm Hg would correlate to an SpO2 of approximately

A. 85% B. 90% C. 95% D. 80% B. According to a normal oxygen dissociation curve, at a normal pH, a PaO2 of 75 mmHg correlates to an SpO2 of approximately 90%.

Which of the following patients or conditions would benefit most from PEP therapy?

A. ARDS B. chronic bronchitis C. myasthenia gravis D. Guillain-Barre' syndrome B. The purpose of PEP therapy is to help mobilize secretions. Of the diseases presented only chronic bronchitis has a significant problem with excess secretions and is therefore more suitable for PEP therapy.

A patient in the emergency room has the following arterial blood gas results: pH 7.18 PaCO2 30 mmHg PaO2 80 mmHg HCO3- 18 mEq/L BE -7 mEq/L SAT 94% This data is most typical of which of the following patient diagnoses?

A. ARDS B. diabetic ketoacidosis C. Flail chest D. COPD B. This patient has a profound state acidosis as manifested by a low pH of 7.18. CO2 is low, which is associated with alkalosis. In this case the alkalosis is being caused by a profound decrease in HCO3-. This combination is associated with diabetic ketoacidosis.

A respiratory therapist notices an increasing high-pitched noise coming from the upper airway of a child receiving 30% oxygen by heated aerosol. Which of the following would be most appropriate for further evaluation?

A. CT scan of the upper airway B. bedside pulmonary function test with flow volume loop C. MRI of the upper airway D. lateral neck radiograph D. To evaluate the source of upper airway inflammation, a neck x-ray is most appropriate. This allows direct visualization of the tissue which can be helpful in determining the source and location of the inflammation.

Which of the following would be most helpful to a COPD patient to prevent airway inflammation?

A. Cromolyn sodium (Intal) B. Pulmicort (Budesonide) C. Albuterol D. Atrovent (Ipratropium Bromide) B. Pulmicort is considered to be the best anti-inflammatory medication for people with COPD.

During the administration of an IPPB treatment, the patient becomes unresponsive and develops the ECG rhythm as shown below Which of the following is the best initial action?

A. Defibrillate at 50 joules B. Arterial blood gas analysis C. Connect a pacemaker D. Being basic life support D. The rhythm shown indicates the absence of cardiac contraction and is considered an emergency. Chest compressions is the best initial action.

A respiratory therapist is determining a VD/VT ratio. Which of the following is needed to complete this analysis?

A. FIO2 and PaO2 B. PECO2 and PaCO2 C. PECO2 and PaO2 D. VT and VA B. There are two things required to determine a VD/VT ratio - end tidal CO2 and PaCO2.

A patient with a history of myasthenia gravis receives Edrophonium for the purpose of diagnosis of a suspected myasthenic crisis. Moments after administration, the patient's ventilatory drive becomes markedly diminished. What medication would be most helpful to the patient at this time?

A. Halcion (Triazolam) B. Propranolol C. Atropine sulfate D. Neostigmine C. When patients have a history of myasthenia gravis and as a result of receiving Tensilon, the respiratory therapist must be on guard for an adverse reaction from the Tensilon. The medication that can reverse the effects of Tensilon is atropine sulfate.

A patient is suspected to have methemoglobinemia. Which of the following would be an appropriate method of evaluation?

A. Hemoximetry B. reflexive oximetry C. single-wave length spectrophotometry D. SVO2 monitoring A. Hemoximetry can detect MetHb

A 28-week gestational age infant is requiring high ventilatory pressures on the ventilator. Transillumination shows a halo-effect bilaterally. A recent chest radiogram shows a honeycomb pattern. Which of the following mostly likely represents the patient's condition?

A. IRDS B. Diaphragmatic hernia C. Patent ductus arteriosis D. Transposition of the great vessels A. The honeycomb pattern seen on an infant's chest x-ray, in addition to the fact that the infant is premature, is an indication of underdeveloped lungs. The term for this condition is called infant respiratory distress syndrome or IRDS. Transillumination that shows a halo effect bilaterally is an indication that a pneumothorax is not present.

Which of the following may be done to increase the effectiveness of gas distribution for a patient with ARDS who is receiving volume-controlled ventilation?

A. Implement an inverse I:E ratio B. Decrease inspiratory time C. Lengthen expiratory time D. Increase inspiratory flow rate A. For a patient with ARDS, a key strategy is to prolong inspiratory time allowing the gases to permeate distal parts of the lung more effectively. To accomplish this, inspiratory flow rate is drastically decreased, sometimes to the point that inspiratory time is greater than expiratory time. When this happens, this is known as an inverse I:E ratio.

A respiratory therapist is attempting to sterilize a non-disposable mouthpiece used on a patient with confirmed active tuberculosis. Which of the following methods should the therapist use?

A. Incinerate the mouthpieces B. Soak in alkaline gluteraldehyde for 20 minutes C. Rinse with acid gluteraldehyde D. Wash in warm water, follow with an acetic acid rinse, then air dry B. The tubercle bacilli is sensitive to alkaline glutaraldehyde (Cidex). The equipment used by patients who have active tuberculosis should be soaked in Cidex for a minimum of 20 min.

The respiratory therapist is caring for a 28-week-old gestational age infant currently receiving mechanical ventilatory support on the following settings: PIP 30 cm H2O FIO2 0.70 PEEP 5 cm H2O Mandatory rate 40 A chest radiograph shows bilateral haziness. Arterial blood gas analysis shows pH 7.22 PaCO2 67 torr PaO2 44 torr HCO3- 27 mEq/L BE +1 mEq/L The most appropriate recommendation would be to

A. Increase FIO2 B. Increase PIP C. Decrease mandatory rate D. Administer Exosurf® D. Bilateral haziness observed on the chest x-ray, in conjunction with the fact that the infant is premature, is consistent pulmonary prematurity. Pulmonary prematurity may be treated by administering pulmonary surfactant. Surfactant decreases the surface tension of the alveoli, which allows and promotes explansion and causes improved gas exchange.

A 65-kg (143-lb), 170-cm (5-ft, 7-in) patient who is nasally intubated and receiving mechanical ventilatory support has a sudden drop in SpO2 from 97% to 82%. Breath sounds are clear on the right, absent on the left. The endotracheal tube is at the 28 cm mark at the right nare. Which of the following action should the therapist take first?

A. Increase FIO2 to 1.0 B. Increase the pressure limit of the ventilator C. Manually ventilate the patient with a bag-valve D. Insert chest tubes in the right chest A. The sudden drop in oxygen saturation and breath sounds that are absent on the left but present on the right, Is associated with a sudden pneumothorax. This is considered an emergency and requires the administration of 100% oxygen.

A patient receiving volume-controlled ventilation remains hypoxic in spite of high levels of FIO2 and positive end-expiratory pressure. Peak pressure has been rising over the last week and is currently 53 cm H2O. Plateau pressure is also rising over the same period and is currently 28 cmH2O. The respiratory therapist should do which of the following to improve ventilation?

A. Increase PEEP B. Insert a pulmonary artery catheter and assess hemodynamic values C. Switch to pressure-controlled ventilation D. Perform an optimal PEEP study C. The point at which a patient should be switched to pressure-controlled ventilation is when the peak pressures rise above 50 cmH2O. However, elevated peak pressures must be the result of decreasing pulmonary compliance. If pressures are elevated due to temporary conditions, such as secretions in the airway, a kink ET tube, or excess water and circuit, for example, those problems should be resolved first. In other words, if the source of high peak pressures can be resolved immediately, pressure-control ventilation is not indicated.

A patient is receiving oxygen by non-rebreathing mask at a flow of 10 L/min. Spontaneous tidal volume is 500 mL and spontaneous respiratory rate is 20 /min. What change should the therapist recommend?

A. Increase flow to 14 L/min B. Increase flow until reservoir bag collapses completely with each breath C. Ensure an adequate mask seal on the patient's face D. Order arterial blood gas analysis A. A patient who is breathing a tidal volume of 600 mL at a rate of 20/min has a minute ventilation of 12 L. If the non-rebreathing mask is set at only 10 L/min, the total flow to the patient is insufficient. The flowrate should meet or exceed the inspiratory demand of the patient. Therefore, increasing to 14 L/min is most appropriate.

The following data is available for a patient receiving mechanical ventilation: 6 PM 9 PM PEEP 5 cm H2O 5 cm H2O Static airway pressure 35 cm H2O 36 cm H2O Peak airway pressure 42 cm H2O 52 cm H2O Delivered tidal volume 550 mL 550 mL Inspiratory flow 50 L/min 50 L/min What is the next action the respiratory therapist should take?

A. Increase inspiratory flow B. Decrease tidal volume C. Obtain a chest radiograph D. Perform bronchial hygiene D. This data shows that pulmonary compliance is remaining steady with static airway pressures that are not significantly changing. However, peak airway pressures are increasing. Increasing peak airway pressures while static pressures remain constant is an indication of either secretions in the airway or another artificial blockage of the airway. Of the options offered, performing bronchial hygiene is the only one that would address the decrease in dynamic compliance.

A 45-year old male is receiving mechanical ventilation on the following settings: Mode Assist/control Mandatory rate 16 Tidal volume 550 mL FIO2 0.60 Pressure limit 60 cm H2O PEEP 8 cm H2O I:E 1:1.5 The patient is developing autoPEEP. The respiratory therapist should do which of the following?

A. Increase inspiratory flow rate B. Discontinue PEEP C. Set PEEP to match the Auto PEEP value D. Decrease PEEP to 5 cm H2O A. When a patient is developing auto PEEP, the solution is to either decrease mandatory rate or increase expiratory time. Because decreasing mandatory rate will likely negatively affect ventilation, the most appropriate action is to manipulate the I:E ratio. To lengthen expiratory time, inspiratory flow rate should be increased.

Blood gas results for a patient receiving mechanical ventilation with an FIO2 0.60 and PEEP of 5 cm H2O are as follows: pH 7.41 PaCO2 32 torr PaO2 55 torr HCO3- 22 mEq/L BE -2 mEq/L Which of the following may help correct the blood gas abnormality?

A. Increase inspiratory flow rate B. decrease FIO2 C. Sedate the patient D. Increase PEEP D. Blood gases reveal hyperventilation and hypoxemia. Of these two problems, hypoxemia should be corrected first. Of the options listed, the two methods that will be helpful in resolving hypoxemia include increasing PEEP and increasing FIO2.

A 35 weeks-of-gestation infant is exhibiting an expiratory grunt with each breath while receiving 0.75 L oxygen by nasal cannula. SpO2 is 90%. What represents an appropriate recommendation?

A. Increase oxygen to 1.5 L/min by nasal cannula. B. Switch to an oxygen hood at FIO2 0.28. C. Administer CPAP at 4 cm H2O. D. Implement oxygen through an infant nonrebreather mask C.

While receiving nasal CPAP delivered by an infant ventilator, the patient is retracting and creating a fluctuating baseline pressure on the monometer. What should the respiratory therapist do?

A. Increase the flowrate B. Adjust the threshold resistor C. Suction the patient D. Increase the size of the fixed orifice resistor A. The patient is demonstrating signs of air hunger and is not in sync with the ventilator. Most likely the patient requires more flow to satisfy inspiratory demand.

A patient with diabetes is demonstrating a Kussmaul breathing pattern. The patient is receiving supplemental oxygen at FIO2 1.0 by a single large volume nebulizer device. The inspiratory flow produced by the patient is measured and found to 20 L/min. Pulse oximetry reveals a saturation of 89%. Which of the following would be most helpful in this situation?

A. Increase the oxygen flow rate at the flowmeter to 22 L/min B. Administer a mild sedative C. Replace the large volume nebulizer with an ultrasonic nebulizer device D. Install a tandem large volume nebulizer in the current system D. When a large-volume nebulizer device is set to 100% oxygen, the maximum achievable flow rate is about 15 L/min. Because this patient has a minute ventilation of 20 L/min, his or her inspiratory demand is not being met. This can be overcome by installing a second, or tandem, large-volume nebulizer. Together, the nebulizers may produce up to 30 L/min, which will exceed the patient's inspiratory demand.

A 6 year-old child is requiring frequent suctioning through a 5.0 mm endotracheal tube. Secretions are thick and copious and difficult to suction in spite of repeated instillation of normal saline prior to suction attempts. The suction catheter size is 6 Fr and suction pressure is set to -65 mm Hg. The respiratory therapist should do which of the following to improve the effectiveness of secretion removal?

A. Instill atropine down the endotracheal tube B. Switch to a shorter catheter C. Use a 10 Fr suction catheter D. Increase suction pressure D. The appropriate suction pressure for an infant is between 60-80 mmHg. The patient is only receiving 65 mmHg of suction pressure and therefore has room for an increase in suction pressure. This will increase the effectiveness of secretion removal.

For a patient who requires mechanical ventilatory support only at night, which of the following would be the most appropriate tracheostomy tube type

A. Jackson tube B. Kamen-Wilkensen C. fenestrated D. double-cuffed C. For a patient who must maintain a tracheostomy tube, but only requires ventilatory support or positive pressure breathing at night, a fenestrated tracheostomy tube would be most appropriate. This will allow the patient to speak during the day with the tube in.

Which of the following patient conditions would CONTRAINDICATE the use of postural drainage and percussion?

A. Kyphoscoliosis B. Sarcoidosis C. Untreated pulmonary tuberculosis D. Cystic fibrosis C. A patient with untreated pulmonary tuberculosis experiences degradation in the lung tissue. Postural drainage and percussion, specifically percussion, is harmful to the patient because it could accidentally dislodge tissues of the lung and the patient may ultimately expectorate that tissue.

A patient who suffers from head trauma has an ICP of 25 mm Hg and is receiving mechanical ventilatory support in control mode. The patient is sedated and paralyzed. To best manage intracranial pressure, which of the following medications would be most helpful?

A. Lasix (furosemide) B. Diamox (Acetazolamide) C. Exosurf D. Neostigmine B. The medication Diamox is a cerebral diuretic. This will lower the ICP. The other medications on this list do not relate. Be sure to research them and know how they're used.

Under which of the following circumstances should a respiratory therapist consider stopping a PEP therapy treatment of a child?

A. Lingular lobe pneumonia B. Cold C. Bronchitis D. Middle ear infection D. PEP therapy is contraindicated by sinusitis, epistaxis, and a middle ear infection.

A patient with cystic fibrosis is receiving oxygen via a nasal cannula set at 5 L/min become somewhat unresponsive and confused. SpO2 is 99%. The therapist should recommend which of the following as a first action?

A. Lower the oxygen flow rate B. Increase the oxygen flow C. Begin mechanical ventilation D. Switch to a non-rebreathing mask A. Cystic fibrosis is a disease associated with chronic obstruction and is therefore considered to be COPD. Patients with COPD should not receive more than 1-2 liters per minute by nasal cannula or more than 28% oxygen by any device. Excessive oxygen delivery can lead to suppression of the ventilatory drive and result in unresponsiveness and confusion. An oxygen saturation of 94% or less should be maintained.

A patient has a balloon-tipped, flow-directed catheter in place in the pulmonary artery. Which pressure can the respiratory therapist expect to obtain from the distal lumen?

A. MAP B. CVP C. PAP D. SVR C. Because the distal lumen of a pulmonary artery catheter is in the pulmonary artery, the pressure monitored through that lumen is called the pulmonary artery pressure or PAP.

A patient has a balloon-tipped, flow-directed catheter in place in the pulmonary artery. Which pressure can the respiratory therapist expect to obtain from the proximal lumen?

A. MAP B. CVP C. PCWP D. PAP B. The proximal lumen of a pulmonary artery catheter is situated inside or right before the right atrium. The value measured here is called CVP.

Left heart failure would be manifested in which of the following values?

A. MAP and SVR B. mPAP and wedge pressure C. CVP and mPAP D. cardiac output and wedge pressure D. The function of the left heart, specifically the left ventricle, is best assessed hemodynamically by looking at those values that precede and come after the left heart. In this case pulmonary capillary wedge pressure and cardiac output (or cardiac index) are the values found before and after the left heart.

A respiratory therapist is performing pulmonary function testing on a patient with a 45 pack-year history of smoking. The therapist desires to asses flow rates to determine the presence of a possible obstructive defect. Which of the following maneuvers will the patient most likely be asked to complete?

A. MVV B. FVC C. SVC D. SBN2 B. To determine flow rates in a patient with suspected obstructive pulmonary defect, a forced vital capacity or FVC maneuver would be done. Although this procedure actually produces a volume (L), the data can also be compared to time and thus the maneuver can produce flow rate data.

Which of the following diagnostic procedures should the respiratory therapist recommend to more clearly identify what looks like a mass in the upper right lung field?

A. Magnetic resonance imaging (MRI) B. AP chest radiograph C. PA chest radiograph D. Transesophagus echocardiogram (TE) A. A mass in the lung field can be further evaluated by examinations that show the three dimensionality of the suspicious area. Because a mass is three-dimensional in nature, magnetic resonance imaging, or MRI, would be the most appropriate exam, of the options given.

A patient with a chest tube drainage system in place, with the chest tube inserted into the 5th interspace, mid-axillary line, has had no significant accumulation of drainage for 24 hrs. A chest radiograph shows vascular markings on the right. The physician is considering discontinuing and removing the drainage tube. Which of the following should the respiratory therapist suggest?

A. Massage the chest tube proximal to the patient. B. Replace the chest tube. C. Examine the system for a leak. D. Clamp the chest tube proximal to the patient. D. The presence of vascular markings indicates that the lung tissue is expanded. Once drainage appears to have virtually stopped, the next step is to clamp the chest tube and monitor the patient for ventilatory difficulty. If no difficulty is observed, chest tubes can probably be discontinued and removed.

A 31-year-old female exhibits a stuffed-up nose, a periodic cough, and bouts of dyspnea. She indicates the symptoms return every spring. Which of the following drug classifications should the respiratory therapist recommend to help control the patient's symptoms?

A. Mucolytic B. beta I agonist C. IgE immunoglobulin agonist D. leukotriene inhibitor D. Leukotriene medication is helpful in long-term control of inflammatory processes in the upper and lower airways.

A 31-year-old female exhibits a stuffed-up nose, a periodic cough, and bouts of dyspnea. She indicates the symptoms return every spring. Which of the following drug classifications should the respiratory therapist recommend to help control the patient's symptoms?

A. Mucolytic B. leukotriene inhibitor C. IgE immunoglobulin agonist D. beta I agonist B Leukotriene medication is helpful in long-term control of inflammatory processes in the upper and lower airways.

A 38-year-old male is breathing spontaneously at a respiratory rate of 30/min with moderate accessory muscle use. Arterial blood gas results and other clinical data are listed : pH 7.32 PaCO2 48 torr PaO2 74 torr HCO3- 25 mEq/L BE -1 mEq/L The respiratory therapist should recommend

A. NIV B. CPAP C. expiratory flow resistor therapy D. high frequency chest wall oscillation (HFCWO) A. This patient is demonstrating respiratory acidosis. Although the patient is under ventilating, acute respiratory failure is not yet shown. When pH falls below 7.25, acute respiratory failure is present and the patient is in need of full mechanical ventilatory support. In this case, the patient is showing beginning signs of under ventilation, as shown by an increasing CO2. Additionally, the respiratory rate is climbing, which indicates a significant increase in work of breathing. The most appropriate way to combat this is to use NIV therapy (BiPAP, NIPPV).

A COPD patient on 5 L/min nasal cannula in the emergency room is complaining of continuing shortness of breath and has the following arterial blood gas results: pH 7.37 PaCO2 48 mm Hg PaO2 39 mm Hg HCO3- 29 mEq/L BE +2 mEq/L What is the appropriate intervention?

A. NRB mask B. CPAP 5 cmH20 C. 8 L/min nasal cannula D. intubation A. The patient is experiencing severe hypoxemia to a level that is not consistent with maintaining life function. While we are often uncomfortable increasing oxygen flows to COPD patients, this should not stop us from treating the problem. 8 L /min is not appropriate because it exceeds the limits of a nasal cannula.

A 38-year-old male patient presents to the emergency room with difficulty swallowing and double vision. He reports a history of Myasthenia Gravis at a prior time, but was never ventilator-dependent. He now has the following clinical data: Vital Capacity Tidal volume MIP 0.9 L 350 mL -22 cm H2O What should the respiratory therapist recommend?

A. NRB mask B. Tensilon challenge C. manual ventilation D. Atropine B. A Tensilon challenge will help diagnose this patient. If positive, intubation and mechanical ventilation is indicated immediately because VC is already below 1.0 L.

A respiratory therapist is ordered to suction a patient in the emergency room with a strong, productive cough in order to acquire a sputum sample for the lab. Which of the following is the most appropriate method?

A. Nasal-pharyngeal suctioning B. In-line sputum trap C. Oral-pharyngeal suctioning D. It is not appropriate to suction this patient C. A patient with a strong, productive cough should not require a catheter to be inserted into the trachea. However, it may be appropriate to use an oral tip suction catheter to clear secretions from the mouth. The general rule is to apply the least amount of invasiveness necessary to clear the airway.

A respiratory therapist has been ordered to change the I:E ratio on a patient receiving VC A/C ventilation to allow for a longer expiratory time. Which of the following can be manipulated to accomplish this order?

A. O2 saturation by pulse oximetry B. multiple wavelength spectrophotometry C. PaO2 by arterial blood gas D. COHb by arterial blood analysis B. A COHb is the gold standard for determining if a patient has carbon monoxide poisoning. When this is suspected normal pulse oximetry should not be trusted. However, multiple wavelength spectrophotometry is a method that works like pulse oximetry but is capable of reading more than just oxygen. It also has the ability and to determine carbon monoxide in the blood. A normal pulse oximeter is also called a single wavelength spectrophotometer whereas an oximeter capable of reading other gases, including CO, is called a multiple wavelength spectrophotometer.

After insertion of the chest tube for treatment of a hemothorax, a chest radiograph is ordered. When observing the radiograph results, the respiratory therapist would expect to see the tip of the chest tube positioned

A. over the lower left lung field. B. in the pleural space. C. over the hilar area. D. in the apical region. B. The tip of a chest tube should be observed in the pleural space at the lung periphery.

Polysomnography reveals a patient has chest movement with no nasal airflow and severe oxygen desaturation. Which of the following therapies should the respiratory therapist recommend for the patient?

A. oxygen via nasal cannula when sleeping B. Doxapram (dopram) C. nocturnal mechanical ventilatory support D. nasal CPAP D. During a sleep study, the presence of chest movement indicates the patient is attempting to breathe. The absence of nasal airflow indicates obstruction, or the absence of air movement. Together, these two data support a diagnosis of obstructive sleep apnea. The gold standard for treatment, in this case, is nocturnal nasal CPAP or BIPAP.

The following ABG results were obtained after increasing the minute ventilation by 2.0 L /min on a critically ill patient with refractory hypoxemia. pH 7.37 PaCO2 45 torr PaO2 60 torr HCO3- 25 mEq/L What would the respiratory therapist expect the ABG results to have been prior to the increase in VE?

A. pH 7.25, PaCO2: 59 torr, PaO2 62 torr B. pH 7.34, PaCO2: 46 torr, PaO2 62 torr C. pH 7.29, PaCO2: 54 torr, PaO2 57 torr D. ph 7.49, PaCO2: 32 torr, PaO2 54 torr C. Increasing minute ventilation should reduce PaCO2, and increase pH. Because CO2 is an acid, the higher the level of the CO2 in the blood, the lower the pH. You will notice also that the PaO2 has increased. It is important to solve the problem of insufficient ventilation prior to insufficient oxygenation because as ventilation improves, oxygenation should also improve to some degree.

Which of the following runs should be closely reviewed?

A. pH 7.44 PCO2 38 mmHg PO2 82 mmHg B. pH 7.45 PCO2 37 mmHg PO2 81 mmHg C. pH 7.40 PCO2 40 mmHg PO2 78 mmHg D. pH 7.36 PCO2 42 mmHg PO2 79 mmHg B. The answer that shows a pH of 7.45 and a CO2 of 37 should be examined closely as these values appear to be outside the range of tolerance. If these results are accurate the arterial blood gas machine requires calibration and should not be used to report patient results until maintenance has occurred.

Which of the following is the most appropriate delivery modality for a patient receiving 80%/20% helium-oxygen gas mixture?

A. partial rebreather mask B. high-flow, high-humidity nasal cannula C. non-rebreather mask D. high-flow air-entrainment mask C. Because the Helium-oxygen is already mixed, a non-rebreather should be used for delivery to avoid air-entrainment as much as possible.

Which of the following supplemental humidity devices is most appropriate for a patient who has a tracheostomy and is receiving long-term mechanical ventilatory support?

A. passover humidifier B. bubble humidifier C. Wick humidifier D. heat-moisture exchanger (HME) C.

Which of the following infection control methods is the most effective for eradicating harmful microorganisms on patient care equipment?

A. pasteurization B. iodine wipe C. autoclave D. isopropyl alcohol wipe C. Of the options given here, autoclave would be the most effective sterilization procedure.

A wick humidifier would be an optimal device for delivery of home-based heat and humidification for a

A. patient with chronic nasal congestion on CPAP. B. newborn with central sleep apnea. C. pediatric patient with a tracheostomy. D. COPD patient requiring supplemental oxygen. C. A wick humidifier is electronically powered and can provide sufficient heat and humidification for any patient including pediatric patients with a tracheostomy at home.

A respiratory therapist is doing pulmonary function studies on a patient. The upper inflection point of a flow-volume loop is considered which of the following pulmonary function values?

A. peak expiratory flow rate B. functional residual capacity C. inspiratory capacity D. expiratory reserve volume A. The very top (inflection point) of a flow volume loop reveals peak expiratory flow rate.

Following a full cardiopulmonary arrest and successful resuscitation, the patient has received dopamine to raise blood pressure. The respiratory therapist his having difficulty obtaining arterial blood from the radial artery due to hypotension. From which of following sites should the therapist attempt to perform an arterial puncture?

A. pedal artery B. carotid artery C. femoral artery D. umbilical artery C. During cardiopulmonary resuscitation blood pressure exists because of cardiac compressions and is usually low. Palpation of pulse, therefore, is difficult in the usual locations such as the radial or brachial arteries. When blood gases are needed during a code with a patient who has very low blood pressure, the femoral artery is the location of choice.

A patient with a history of +3 pitting edema is in severe respiratory distress while receiving oxygen by nonrebreather mask. BS reveal crackles in the bases. The patient is expectorating a slight amount of bloody, frothy secretions. What additional therapy should the respiratory therapist consider?

A. percussion by Vest B. nitric oxide (NO) inhalation C. helium-oxygen, 80/20% D. CPAP 5 cm H2O, FIO2 1.0 D. Clinical evidence indicates the presence of congestive heart failure, which is precipitating cardiogenic pulmonary edema. Application of CPAP will help suppress the pulmonary edema and improve oxygenation.

A respiratory therapist is preparing a patient for chest tube removal. What instruction will the therapist provide to facilitate extraction of the tube

A. perform a Valsalva maneuver during withdraw B. exhale completely just prior to withdraw C. cough vigorously during removal D. pause at the top of a maximal inhalation prior to removal A. A Valsalva maneuver consists of a patient attempting to exhale against a closed glottis. People commonly perform this maneuver when they wish to equalize their ears after a change in altitude. The maneuver allows the lungs to stay inflated during the actual removal of the chest tube from the pleural space

A patient has been ordered to receive Beclamethasone (Beclovent) by metered dose inhaler (MDI) once a day. Which of the following should be included in the instruction of the patient?

A. perform a breath hold for 10 seconds with each treatment B. rinse mouth after each treatment C. avoid bronchodilator medication for 8 hours D. follow treatment with Nystatin via MDI B. Beclomethasone is considered to be a corticosteroid. Inhaled corticosteroids can cause an infection in the mouth known as candidiasis or thrush mouth. To prevent this, the patient should rinse his or her mouth out after taking inhaled corticosteroids of any type. If an infection does develope, the problem may be treated with Nystatin or Amphotericin

In preparation for an arterial puncture in the right radial artery, the respiratory therapist performs a modified Allen's test. When the ulnar artery is released, color returns to the hand in 35 seconds. The therapist should

A. perform a femoral artery puncture B. perform a brachial artery puncture C. perform the puncture in the right radial artery D. perform an Allen's test on the left ulnar and radial arteries D. An Allen's test is an assessment for collateral blood flow in to the hand in preparation for a radial arterial puncture. The presence of bilateral blood flow into the hand reduces risk of the arterial puncture because there is another source of blood if the radial artery is damaged. The procedure occurs by occluding both the radial and ulnar artery and allowing the hand to blanche or the blood to leave the hand. When the ulnar artery is released the amount of time required to not color in the hands is observed. Blood must return in under seconds. A duration of 35 seconds is too long in indicates poor blood flow through the ulnar artery. This would suggest an alternative site should be considered.

A female patient, who receives oxygen at home by concentrator, calls in to the providing home care company. When the call is routed to the respiratory therapist, the patient indicates the oxygen concentrator is not working even though the switch is "on" position and the machine is plugged in. The respiratory therapist should first ask the patient to

A. perform a flow calibration on the concentrator. B. check the circuit breaker. C. remain at rest until a therapist arrives. D. switch to her 'E' cylinder for oxygen. D.

While moving a patient from the radiology procedure table to a transport gurney, the respiratory therapist suspects the oral endotracheal tube position has been inadvertently changed. To quickly assess airway position, the therapist should FIRST

A. perform bronchoscopy B. auscultate breath sounds C. order a chest radiograph (x-ray) D. inspect symmetry of chest rise D. There are many ways to assess the position of an ET tube. This question is asking what should be done first to quickly assess the tube position. Of the options offered the most immediate data would come from a visual inspection of the symmetry of chest rise. The next best option would be auscultation of breath sounds, although not as quick as visual data. Finally, the last best option would be to get a chest x-ray. This option is time-consuming and not considered quick but is considered more conclusive than the other two. But keep in mind, the question is asking for the first evidence, not conclusive evidence.

Immediately after the insertion and securing of an indwelling arterial line, the respiratory therapist should do which of the following to discourage clot formation in the catheter?

A. periodically allow free-flow of blood for 2-4 seconds B. flush the catheter with 10.0 mL NS C. keep a cap securely on the end when not actively withdrawing blood D. attach a pressurized bag of heparinized saline D. A pressurized bag of heparinized saline (or normal saline) should be used to apply back pressure to the artery and prevent free flow and loss of blood.

Which of the following is most helpful in preventing mucosal damage and epistaxis associated with use of a nasal pharyngeal airway?

A. petroleum jelly B. Lidocaine C. epinephrine D. water soluble lubricant D. When suctioning the nasal passage through a nasal pharyngeal airway (nasal trumpet), mucosal damage and bleeding is primarily prevented through the use of water-soluble lubricant and by employing a gentle technique.

A patient vomits and aspirates during postural drainage and percussion while in the appropriate position to drain the left lower lobe. The respiratory therapist's immediate reaction should be to

A. place the patient in the upper right lobe drainage position B. suction the patient's oropharynx C. implement incentive spirometry D. place the patient in the right lower lobe drainage position D. When somebody vomits during postural drainage and percussion, when draining a specific side, the immediate response should be to drain the opposite side.

A COPD patient who is orally intubated and is receiving VC, SIMV ventilation is having difficulty weaning following a 10-day care stent for the treatment of pneumonia. Radiographic analysis shows that lungs are clear of infiltrates but the diaphragm is elevated and the patient has persistent increased vascular markings. Daily spontaneous breathing trials are not progressing and rarely last more than 1.5 hours. Which of the following would be most helpful in decreasing the patient's risk for ventilator associated pneumonia?

A. placement of a fenestrated tracheostomy tube B. extubation to non-invasive ventilation C. frequent suctioning D. continuous antimicrobial therapy B.

After completing arterial puncture and blood extraction into a disposable 3.0 mL syringe, the respiratory therapist should handle the sample by

A. placing the sample in an ice bath B. adding heparin to the syringe C. agitating the sample vigorously D. purging all air from the syringe D. A significant step in the performance of arterial blood extraction is to purge all air from the syringe after the blood has been extracted. Removing air bubbles prevents excessive oxygen from dissolving in the blood and producing erroneous results.

A pleural effusion would show which of the following on a radiologic exam?

A. plate-like infiltrates B. blunt costophrenic angles C. flattened diaphragm D. ground glass appearance B. A chest X-ray reveals a pleural effusion if we see descriptors such as "blunt or obliterated costophrenic angles", or "concave superior interface".

A patient following abdominal surgery has severe bilateral atelectasis in both bases. What would one expect to hear upon auscultation?

A. pleural rub B. end-inspiratory crackles C. diffuse inspiratory crackles D. bronchial breath sounds B. Late inspiratory crackles, or end-inspiratory crackles are commonly auscultated with a patient experiencing atelectasis. Crackles are also known as rales.

A patient is experiencing shortness of breath. The radiological AP view of the chest shows a blunt costophrenic angle in the left lower lobe. This would indicate

A. pneumonia. B. pleural effusion. C. atelectasis. D. pulmonary embolus. B. A chest X-ray reveals a pleural effusion if we see descriptors such as "blunt or obliterated costophrenic angles", or "concave superior interface".

A pulse oximeter is showing an SPO2 of 58% and a pulse of 60 bpm. Palpated pulse rate is 102 bpm. Which of the following could best explain the difference in pulse rates?

A. poorly calibrated pulse oximeter B. hypoxemia C. hypertension D. poor peripheral perfusion D. When a pulse oximeter is reading a low oxygen saturation but has a different heart rate compared to that which can be palpated, the oxygen saturation reading is in error and should not be reported. The cause for this is likely poor peripheral perfusion, which may be caused from a low body temperature, low blood pressure, or shock among other things.

The sudden sounding of a high-pressure ventilator alarm is noted on a patient receiving volume controlled mechanical ventilation. The respiratory therapist notes the endotracheal tube is at 27 cm at the teeth and that breath sounds are absent when auscultating the left hemithorax. The therapist should immediately

A. prepare for insertion of chest tubes in the left thorax B. advance the endotracheal tube until breath sounds can be auscultated over the left chest C. check the air pressure of the tube cuff D. withdraw the endotracheal tube until breath sounds are bilateral The proper placement of an oral endotracheal tube is when the teeth or lip line matches with markings on the tube somewhere in the low 20s. This endotracheal tube is 27 cm at the teeth, indicating the tube is advanced too far. In this case, it is likely the tube has advanced into the right mainstem bronchus, which would cause a high-pressure alarm to occur. The appropriate action is to withdraw the endotracheal tube until breath sounds are bilateral. D.

The following flow-time ventilator graphic data is observed. The therapist should suggest

A. pressure control ventilation B. inverse I:E ratio ventilation C. increasing I:E ratio D. increasing inspiratory flow rate D. The development of autoPEEP, which is shown in this graphic, is treated by allowing more time for expiration. This is most commonly is done by increasing the flow, thereby decreasing inspiratory time and lengthening expiratory time. Although not offered here, a decrease in mandatory rate is another method for decreasing autoPEEP.

A patient receiving volume-controlled ventilation in the assist/control mode has a peak inspiratory pressure of 55 cm H2O. Both dynamic and static compliance have decreased over the last few days. The patient will benefit most from which of the following?

A. pressure support B. pressure-cycled ventilation C. Bi-level therapy D. pressure control ventilation (PCV) D. A peak pressure over 50 cmH2O is an indication for pressure control ventilation. This is true only if the cause of the peak pressures are not a result of temporary conditions such as secretions in the airway or a kinked ET tube. The problem states that the patient's dynamic and static compliance has been decreasing over the last few days, indicating pulmonary changes. This data suggests that the source of the high airway pressure is not caused by a temporary condition.

A disadvantages of using a self-inflating resuscitator on an infant is that:

A. pressure-release valves make over-inflation of the lungs less likely. B. maximum FIO2 of 1.0 is difficult to maintain. C. use more commonly results in gastric insufflation. D. it is difficult to assess the seal between the mask and face. D. It is customary to use flow-inflating resuscitator bags on infants. There are advantages and disadvantages to both types of resuscitators. According to NRP guidelines, assessment of the seal and delivery of oxygen are more difficult with a self-inflating resuscitator. A required gas source and a pressure-release valve are part of a flow-inflating device.

A patient is receiving Anectine (succinylcholine chloride) to facilitate oral intubation. Two minutes after administering the medication, the respiratory therapist notices involuntary minor muscle twitching about the neck and upper thorax region. The therapist should

A. proceed with the intubation B. administer rocuronium (Zemuron) C. administer antagonist medication immediately D. allow time for the muscle twitching to cease A. Upon administration of the drug Anectine, muscle twitching about the face, neck, and upper thorax is an indication that the drug has had sufficient time to impose paralysis. Therefore, it is a sign to proceed with the intubation procedure.

A respiratory therapist is asked to assess breath sounds on a patient in the emergency room who presents with a high fever and dyspnea. Scattered rhonchi are auscultated throughout both lungs. The respiratory therapist should

A. provide nasal tracheal suctioning. B. place the patient on oxygen at 2 L/min nasal cannula. C. check a sputum gram stain. D. administer a small volume nebulizer with Albuterol. A. Rhonchi (coarse rales) indicate there are secretions in the large airways, which are often remedied by suctioning. While one may feel more comfortable encouraging a directed cough, it is acceptable to suction these patients for exam purposes.

Which of the following will result in a decrease in mean airway pressure for a patient on a mechanical ventilator in the assist/control mode?

A. pulmonary hypertension B. pleural effusion C. pulmonary tuberculosis D. pneumonia B. A physical examination that reveals a dull sound to percussion and decreased tactile fremitus, in conjunction with shortness of breath and a nonproductive cough, is most suggestive of a pleural effusion.

Which of the following provides the most conclusive evidence of acute epiglottitis in a child?

A. radiological report of supraglottic swelling B. drooling C. inspiratory stridor D. red arytenoids A. The presence of acute epiglottitis is diagnosed primarily from the radiological finding of supraglottic swelling. This radiological finding may also be described as a thumb sign

Rinsing one's mouth thoroughly after therapy is an appropriate instruction point to make for a patient who is

A. receiving MDI therapy at home through a ventilator circuit. B. experiencing nausea after a sympathomimetic. C. experiencing bleeding gums after Proventil use. D. self-administering Fluticasone. D. Flovent (fluticasone) is a corticosteroid used generally to prevent and relieve inflammation of airway walls. Inflammation is one of the components of asthma so corticosteroids are key in treatment. The frequency is twice per day or b.i.d. Flovent may cause Candidiasis, (oral yeast infection), which may be prevented by judiciously rinsing the mouth after inhaler use. Treat with Nystatin.

A motorcycle accident victim is in the emergency room with multiple wounds including a contusion to the head and torn deep muscle tissue in the right leg. The patient is bleeding massively. The respiratory therapist should first

A. recommend blood type and crossmatch evaluation in preparation for blood administration B. place the patient on oxygen at FIO2 1.0 C. monitor respiratory rate q 10 minutes D. recommend a CT scan of the head B. A massive bleed is considered an emergency because the loss of hemoglobin can quickly affect the oxygen transport to tissues, which can result in death. This emergency should be treated by administering 100% oxygen to the patient.

While measuring the cuff pressure of a patient on a ventilator, the respiratory therapist notes a pressure of 43 mmHg. Upon auscultation of the neck, significant air can be heard passing around the cuff at peak airway pressure. The therapist should respond by

A. recommending replacement of the endotracheal tube B. adding air to the cuff C. decreasing cuff pressure to less than 20 mmHg D. adjusting the patient's position in bed A. Normally, an endotracheal tube cuff can seal in the trachea at around 16 to 20 mmHg pressure. In this case it is taking 43 mmHg of pressure while still showing evidence of air leaking around the cuff. The cuff is not touching the tracheal wall. Together, these two data suggests an endotracheal tube cuff that is too small. Replacing the endotracheal tube with a larger size is most appropriate.

A 98-kg (216-lb), 165-cm (5-ft, 5-in) male is in the intensive care unit receiving VC, A/C ventilation on the following settings: FIO2 0.55 f 14 VT 700 mL PEEP 4 cm H2O ABGs pH 7.49 PaCO2 30 mm Hg PaO2 102 mm Hg HCO3- 24 mEq/L BE +4 mEq/L SAT 0.99 The respiratory therapist should

A. reduce FIO2. B. reduce tidal volume. C. reduce the rate. D. reduce inspiratory flow rate. B.

Which of the following could result from the application of positive end-expiratory pressure?

A. reduced alveolar ventilation B. reduced intra-thoracic pressure C. reduced cardiac index D. increased A-aDO2 C. Positive end expiratory pressure, like all positive pressure ventilation techniques, leads to decreased venous return, decreased cardiac output, and decreased cardiac index.

A patient receiving oxygen therapy at 60% has the following arterial blood gas results: pH 7.41 PaCO2 39 torr PaO2 579 torr HCO3- 24 mEq/L The respiratory therapist should

A. reject the results B. discontinue oxygen therapy C. change the Sanz electrode D. decrease FIO2 to 0.40 A. This blood gas shows a PaO2 of 279 mmHg. However, the patient is only on 60% oxygen. When one calculates the alveolar area equation, the maximum oxygen tension found in the alveoli is about 370 mmHg. Therefore, a PaO2 of 579 mmHg is impossible and suggests the analyzer is in error. The results should be rejected.

What will occur if a gas that is saturated with 100% relative humidity with no humidity deficit at 33.4 deg C (92.1 deg F) cools to 25.1 deg C (77.2 deg F)?

A. relative humidity will drop by about 25% B. relative humidity will remain 100%, rainout will occur C. evaporation will decrease the humidity deficit D. a humidity deficit will develop B. As the temperature of any gas decreases, its ability to hold water also decreases. Therefore, a gas that has a relative humidity of 100% that then undergoes a temperature decrease must eject the excessive water. This is called condensate or rainout. This phenomenon can be seen commonly in the morning when there is dew found on plants and objects.

A homecare patient indicates that she is not getting enough air from her transtracheal oxygen catheter. The therapist should instruct the patient to

A. remove the catheter. B. use a nasal cannula. C. flush the catheter with saline. D. increase the flow of oxygen to the catheter. B. When a patient reports difficulty getting air or oxygen through one oxygen delivery device, before troubleshooting, the most appropriate first action is to ensure adequate ventilation by changing to a different, reliable device. In this case, switching to a nasal cannula is most appropriate.

A patient with a fenestrated tracheostomy tube in the speaking configuration has experienced a complete cardiopulmonary collapse. In preparation for cardiopulmonary resuscitation, the respiratory therapist should FIRST do which of the following?

A. remove the tracheostomy tube B. remove the cap C. inflate the cuff D. insert the inner cannula B. A fenestrated tracheostomy tube is used to both provide positive pressure ventilation and to allow the patient to speak when temporarily off ventilatory support. The fenestrated tube comes with an inner cannula and has a cap or a button. These parts are used to configure the tube for speech or positive pressure ventilation. The first step to transition to a positive pressure ventilation configuration is to remove the button or cap. Although this question does not ask, the next steps would be to insert the inner cannula and inflate the cuff.

A patient in the emergency room (ER) has multiple fractured ribs bilaterally after a motor vehicle accident. ABGs show pH 7.51 PaCO2 29 torr PaO2 102 torr HCO3- 24 mEq/L BE +3 mEq/L FIO2 0.21 The respiratory therapist should

A. repeat the ABG. B. determine the level of pain. C. assess the patient for diabetes. D. perform a bedside pulmonary function test. B. The blood gas demonstrates respiratory alkalosis, which suggest the patient is simply breathing too much and/or too quickly. Having been in a motor vehicle accident, the most likely source of the hyperventilation is pain from fractured ribs.

A parent of a 7-year-old child with asthma reports having difficulty controlling the asthmatic episodes. The patient has an albuterol inhaler and is taking beclovent by MDI twice daily. The mother indicates that episodes have been increasing and she is confused regarding when to continue treatment and when to call the doctor. The respiratory therapist should

A. replace Beclovent with Azmacort B. recommend switching from albuterol to Xopenex C. create an asthma action plan with the parent and patient D. set an appointment for allergen testing C. The use of an asthma action plan can help the patient and/or caregiver manage the asthma, especially during episodic attacks. The plan helps direct when and how to treat oneself, as well as when to seek medical attention. The plan consists of personal monitored data translated to a green yellow red light status.

An infant is receiving 50% oxygen by hood with a heated aerosol. Oxygen is analyzed in the hood with a galvanic-type oxygen analyzer and is consistently found to be 35%. The therapist should

A. replace the batteries B. remove the heated aerosol C. replace the electrolyte solution D. perform high and low calibration D. When a galvanic type oxygen analyzer fails to produce a reading (the needle fails to move), the solution is to change the electrode which serves as the battery to the analyzer. If, however, the analyzer is reading incorrectly, the analyzer must be calibrated. Calibrations should be done at low and high oxygen percentage points -100% and 21%.

Which of the following would be a good goal to be included in a pulmonary rehabilitation program for a COPD patient?

A. restore normal ABG values B. reduce infections C. omit ALDs D. return to normal life B. The purpose of a pulmonary rehabilitation program for a patient with COPD relates to improving their life from their point of view. Legitimate goals include recognizing signs of infection, reducing hospitalizations, increasing their ability to perform activities of daily living, and generally increasing exercise tolerance. Because lung disease cannot be reversed it is unreasonable to think that we can return the patient to normal life or reduce their need for oxygen. It is also unreasonable to acheive normal blood gas or pulmonary function values.

A patient undergoing pulmonary function testing in a body box has an airway resistance (Raw) of 2.7 cm H2O/L/sec. The respiratory therapist should provide which of the following interpretations?

A. restrictive pulmonary disease B. asthma C. Guillain-Barre Syndrome D. normal results B. An airway resistance of 2.7 cmH2O/L/sec is considered high. Elevated airway resistance is associated with asthma.

A patient who is undergoing dialysis is experiencing moments of hypoxemia as shown by a decreasing SpO2. Arterial blood gas analysis is ordered. A shunt is in place on the right arm and ulnar circulation on the left arm is markedly reduced. From what location should the respiratory therapist obtain an arterial sample?

A. right brachial B. right femoral C. left brachial D. left radial C. When there is a shunt in place on one arm, the other are must be used for any puncture. Because ulnar circulation is reduced on the left arm, obtaining a blood gas through radial penetration is contraindicated. Therefore, use of the left brachial artery is most appropriate.

A patient with right lower lobe pneumonia has "sweet" smelling sputum. This is most likely related to what organism?

A. staphylococcus B. pseudomonas C. tubercle bacilli D. e. coli B.

A therapist palpates a popping sensation in the upper chest and neck. Which of the following is the most likely cause?

A. subcutaneous emphysema B. crepitus rales C. discontinuous rhonchi in the upper airways D. alveolar distension A. Popping sensations, also referred to as crackling sensations, during palpation of the tissues of the upper chest and neck are related to subcutaneous emphysema, which is a condition of air entering the dermal and subdermal spaces. It is usually caused by a mal-positioned tracheostomy tube.

While performing postural drainage and percussion to the basal segments with the head of the bed down 30 degrees, the respiratory therapist notes frequent PVCs on the monitor. The therapist should do which of the following?

A. switch to aerosolized Albuterol treatments B. administer Lidocaine C. switch to PEP therapy D. elevate the head of bed and continue percussion over the basal segments C. When a patient has an adverse reaction to a particular therapy the first step is to stop the therapy and the next step is to modify the therapy to another method that has the same objective. In this case, the patient has PVCs because the head of bed is down 30°. Because we are draining the basal segments, bringing the head of bed up and continuing therapy is not appropriate because you are no longer draining the basal segments. Thus, a change in modality that is more significant is required. In this case PEP therapy is most appropriate.

A patient has just undergone oral endotracheal intubation. Which of the following should be examined FIRST to determine proper positioning of the ET tube?

A. symmetry of diaphragmatic excursion B. chest radiograph C. tracheal deviation D. breath sounds D. To determine proper positioning of the endotracheal tube the respiratory therapist should first assess that which is quickest. Of the options given, the quickest method would be to auscultate breath sounds. This method is not conclusive but is quick. If asked to provide conclusive evidence, a chest radiograph is most appropriate.

Which of the following clinical information can be obtained from a neonatal assessment?

A. symptoms B. dyspnea C. signs D. subjective information C. Neonatal patients have no ability to report their own condition and therefore subjective information is not attainable. Of the options listed only "signs" and "color" may be determined objectively.

After receiving 1 hour of continuous bronchodilator therapy at 10 mg/hr, a patient's breath sounds have changed from absent to bilateral wheezing. The respiratory therapist would conclude:

A. the patient's condition is improving B. the patient's condition is worsening C. the bronchodilator should be changed to Atrovent D. the patient is being overdosed with Albuterol A. When a patient's breath sounds change from absent or diminished to audible wheezing, the patient is considered to have improved. This is because audible wheezing requires more airflow and is evidence that the airways are opening up.

In preparation for pulmonary function testing, a therapist performs calibration of the PFT machine. After injecting gas from a certified 3.0-L super syringe, the following results are obtained: First Second Third 2.85 L 3.14 L 2.99 L Which of the following can be accurately stated?

A. the super syringe requires calibration B. the PFT machine is not accurate and requires calibration C. the PFT machine lacks accuracy but maintains precision D. the PFT machine is accurate D. When using a 3.0 L syringe to ensure the quality control of pulmonary function testing equipment, it must be remembered that the machine does not have to report exactly 3 L to be considered accurate. It is acceptable for the machine to report +/-5%. In other words, 2.85 L - 3.15 L is considered accurate and acceptable. Therefore, these results are appropriate.

The respiratory therapist records a patient's blood pressure to be 120/80 mmHg as measured by a sphygmomanometer. Simultaneously, the therapist notes the blood pressure measured by an indwelling radial artery catheter and transducer is reading 145/90 mm Hg. Which of the following could explain the difference in these results?

A. the transducer is lower than the level to which it was originally zeroed and calibrated B. the transducer is 8 inches or more above the level of the heart C. erratic movement of the arm with the arterial catheter D. the sphygmomanometer is likely in error A. The transducer associated with an indwelling arterial catheter should be level with the heart in order to produce an accurate blood pressure reading. If the transducer is lower than the heart, increased blood pressure will be demonstrated on the monitor and the reported pressure will be erroneously high. If the transducer is above the level of the heart, the reported blood pressure on the monitor will be erroneously low, or less than the actual blood pressure of the patient

A 5-ft, 2-inch (157-cm), 208-lb (95-kg) male patient has just been diagnosed with obstructive and central sleep apnea. Which of the following will provide most help to the patient?

A. weight loss B. bi-level therapy with a back up rate C. nasal CPAP D. tracheostomy B. Because the patient has central sleep apnea, supportive ventilation with a backup rate is required.

Prior to inline bronchodilator therapy, which of the following devices should be removed at the risk of increasing the chance of nosocomial infections?

A. wick humidifier B. heated concha C. heat/moisture exchanger D. holding chamber C. While breaking the circuit to remove an HME increases the risk for nosocomial infections, it must be removed in order to properly deliver the medication. The device is made of spongelike material that filter out the medication and therefore must be removed.

A radiology report of a chest x-ray indicates the oral endotracheal tube is 1 cm beyond the carina, in the right mainstem bronchus. The respiratory therapist should

A. withdraw the ET tube by 4 cm B. withdraw the ET tube by 1 cm C. advance the ET tube by 1 cm D. withdraw the ET tube by 2 cm A. The endotracheal tube is properly positioned when the end is 2 to 5 cm above the carina. In this case, the tube is only 1 cm above the carina. Withdrawing the by 2 to 4 cm is appropriate.

Immediately after the insertion of a nasopharyngeal airway (nasal trumpet), the patient begins to cough uncontrollably. The therapist should consider

A. withdrawing the airway by 1 cm B. reinserting the airway with a viscous lidocaine lubricant C. replacing the airway with one that is shorter D. spaying the oropharynx with benzocaine C. The patient's response to cough vigorously immediately after insertion of the nasal trumpet indicates the airway is likely too long. It should be replaced with a shorter one.

Close monitoring of VC is indicated in which of the following situations?

A. within 24 hours after cardiopulmonary resuscitation B. neuromuscular disorder C. post abdominal surgery D. after the administration of a neuromuscular blockade B. Patients with suspected neuromuscular disorders, like Guillain-Barré and myasthenia gravis, should be monitored for basic ventilation parameters, including vital capacity.

A 10-year-old child admitted to the emergency room with an acute asthmatic episode has received 3 aerosolized bronchodilator treatments and a dose of solu-mederol, IV. Auscultation prior to any treatment revealed inaudible breath sounds. Current auscultation reveals diminished breath sounds with expiratory wheezing. The therapist should conclude the bronchospasm is

A. worsening B. unchanged C. no longer present D. improving D. The presence of wheezing, after initially auscultating absence of breath sounds, is considered an improvement. This is because at least some air movement is required to emit a wheezing sound.

A 52-year-old patient is receiving VC SIMV ventilation due to myasthenia gravis. The patient has no history of pulmonary disease and is alert and oriented. The following data is observed: Mode SIMV Set Rate 6 Total rate 12 VT(spont) 425 mL PEEP 5 cm H2O FIO2 0.40 pH 7.42 PaCO2 38 torr PaO2 122 torr HCO3- 24 mEq/L BE 0 mEq/L MIP -42 cm H2O The respiratory therapist should recommend

A. CPAP 5 cm H2O, PS 15 cm H2O B. daily SBT for several days, then reevaluate C. decreasing the set rate to 2/min, ABGs in 20 minutes D. extubation D. The clinical and laboratory data for this patient indicates that the patient is able to ventilate independently and should continue the ventilator liberation effort. Since the patient has no history of pulmonary disease, extubating the patient from a rate of six is perfectly appropriate.

It is suspected that a patient is bleeding from a site within the pulmonary tree. Which of the following would be most appropriate to locate the potential bleeding site?

A. CT scan of the chest B. bronchoscopy C. V/Q scan D. percutaneous biopsy B. A bronchoscopy is the best procedure to use when a specific bleeding site needs to be located within the pulmonary tree. Although the bronchoscope is unable to penetrate the distal portions of the tree, most bleeding will occur higher up and can be visualized through the bronchoscope. Additionally, once the bleeding site is observed, measures can be taken with the bronchoscope to stop or minimize bleeding. Some of these measures include instillation of epinephrine, compression of the site with the scope, or placement of a Fogarty catheter, which is a long-term tamponade of the bleeding site.`

The physician has asked the respiratory therapist to help determine oxygenation at the tissue level. The therapist should suggest obervation of which of the following?

A. CaO2 B. QS/QT C. CvO2 D. PAO2 A. To determine the amount of oxygen available to the tissues at the tissue level, we must look at the oxygen that is bound with hemoglobin, in addition to the oxygen that is dissolved the plasma. Because the oxygen bound with hemoglobin represents the most significant portion of oxygen in the blood, determing the arterial oxygen content is most helpful in determining the amount of oxygen available at the tissue level.

A 60-year-old patient with COPD is brought to the emergency department in ventilatory distress. The patient is current receiving supplemental oxygen at 50% via Venturi mask. The respiratory therapist notes the patient is difficult to arouse and has short, shallow respirations. Which of the following should the therapist do?

A. Calculate C(a-v)DO2 B. Manually ventilate the patient C. Obtain a A-P chest radiograph D. Obtain arterial blood for a blood gas analysis D. A COPD patient should not receive more than 1-2 L/min oxygen by nasal cannula or no more than 28% oxygen by any other device. Over oxygenating a COPD patient may result in suppression of the hypoxic drive and will be noticed by a decrease in the depth and rate of respiration. The most appropriate action would be to lower the oxygen percentage. However, this option is not offered. The best option, therefore, would be to obtain an arterial blood gas, which would prove that the patient's PaO2 is too high.

A patient's parent has been following the guidelines on their child's asthma action plan. For more than 24 hours their child has been requiring rescue medication at a frequency greater than the prescribed frequency. The response has been temporary and minimal at times. Currently the patient is having difficulty speaking in between breaths. The parent should be advised to

A. Call 911, activate the emergency response system. B. continue treatment for 24 more hours, then call the doctor. C. report to a quick-care medical facility immediately. D. contact the patient's doctor or healthcare provider. A. According to the National Asthma Education and Prevention Program (NAEPP), breathing difficulty that affects the ability of the patient to speak should be met with swift, emergent medical attention. Calling 911 would be most appropriate.

A patient in the cardiac care unit is found to have the following cardiac rhythm. A pulse is not palpable. Which of the following is the best initial action?

A. Cardioversion B. Begin BLS C. Synchronized defibrillation at 360 joules D. Defibrillate at 50 joules B. The absence of a pulse indicates the patient's heart is not contracting. Chest compressions are required. The best option is to begin basic life support.

The galvanic-type oxygen analyzer is reading 5% when the respiratory therapist is attempting to calibrate it with 100% oxygen. To remedy the problem, the therapist should do which of the following?

A. Change the batteries B. Change the electrolyte solution C. Discard the analyzer and get a new one D. Change the fuel cell D. When a galvanic-type oxygen analyzer is reading extremely low or zero, the problem is most likely related to a depleted power source. Because the analyzer does not have batteries but receives power from the probe (the fuel cell), the most appropriate action is to change the fuel cell. If this analyzer, however, is only slightly off the actual FIO2, , it is more likely the analyzer requires a high and low calibration.

The respiratory therapist is having difficulty getting the Polorgraphic (Ohio) oxygen analyzer to calibrate to 100% oxygen. The analyzer consistently reads 70%. Which of the following may be a helpful remedy to the problem?

A. Change the fuel cell B. Send the analyzer to the manufacture for repair C. Tape a note to the analyzer indicating a 30% offset D. Change the batteries D. A polargraphic (Ohio) oxygen analyzer requires batteries, computer, and an electrolyte solution in the probe to function properly. When the analyzer fails to calibrate, the most likely cause is depleted batteries. Because this is something the respiratory therapist can remedy, sending the analyzer in for repair is not indicated. A fuel cell change is not indicated because only a galvanic-type oxygen analyzer has a fuel cell.

An adult female patient has severe airway resistance for which she is being treated with an 80/20 mixture of heliox via a NRM. The problem is improving but not yet resolved. ABGs are: pH 7.36 PaCO2 45 torr PaO2 67 torr HCO3- 6.2 mEq/L What should the respiratory therapist recommend?

A. Change to 70/30 heliox mixture. B. Replace heliox with nasal cannula at 5 L/min. C. Change to 60/40 heliox mixture. D. Replace heliox with CPAP, FIO2 0.40. A. This is an oxygenation problem as seen by the PaO2 of 67 torr. Therefore more oxygen is required. A 70/30 heliox mixture would provide approximately 10% higher FI02.

A female patient has the following ABG results: pH 7.29 PaCO2 49 mm Hg PaO2 62 mm Hg HCO3- 28 mEq/L SaO2 96% COHb 1.5% (by Co-oximeter) COHb 6.5% (by multiple wavelength spectrophotometer) What should the respiratory therapist recommend?

A. Check oxygen saturation by single wavelength spectrophotometer B. Run quality control and evaluate the accuracy of the instrument C. Place on NRB mask D. Remove fingernail polish and reevaluate B. In this example there are several inconsistencies within the numeric values of the results. For example, a PaO2 of 62 mm Hg with an oxygen saturation of 96% does not make sense. Because we have no clinical information about the patient's condition the best answer is to check the accuracy of all of the instrumentation before going any further.

A chest radiograph reveals excess fluid in the lower left chest, causing a partial hemothorax and considerable dyspnea for the patient. The respiratory therapist should recommend?

A. Chest tubes in the left chest between 2nd and 3rd intercostal space B. Palpate the trachea to check for deviation C. Thoracentesis of the left chest cavity D. Administration of Lasix (furosemide) C. Excess fluid in the lower left chest may be removed by performing a thoracentesis of the left chest cavity.

A respiratory therapist is asked to suggest a care plan for a COPD patient with bronchiectasis and a pseudomonas aeruginosa infection. What therapy in which order is best?

A. Combivent, heated mist, suction, CPT B. Atrovent, USN, PD&P, cough C. USN, cough, Albuterol, PD & P D. Alupent, Atrovent, cough, PD & P B. The proper order of therapies for mobilization and removal of secretions is to first provide bronchodilation followed by therapies that move secretions from lower airways to upper, larger airways. In this case, Atrovent should be administered first, followed by hydration through an ultrasonic nebulizer. Finally, postural drainage and percussion can be administered to help secretions in lower airways mobilize to larger airways where the patient may cough or may be suctioned.

Multiple wavelength spectrophotometry oxygen saturation is 97% on a patient with a respiratory rate of 14/min, heart rate 80/min, and clear breath sounds. Arterial blood gas results are as follows: pH 7.42 PaCO2 37 torr PaO2 125 torr HCO3- 25 mEq/L FIO2 0.21 What should the respiratory therapist recommend?

A. Conduct proficiency testing on the Clark electrode. B. Check the most recent quality control values. C. Discontinue supplemental oxygen. D. Run a two-point calibration on the ABG analyzer, then repeat the sample. D. In this example, the PaO2 is not possible while breathing on room air, which can be seen more easily after calculating the alveolar air equation. A multiple wavelength spectophotometer (oximeter) is a non-invasive way to measure oxygen saturation, metHb, COHb, etc. The ABG analyzer should be recalibrated and the sample repeated.

The following data is observed on a patient who is 10 minutes post coronary artery bypass graft (CABG): 5 min after 15 min after 25 min after PaO2 (mm Hg) 92 90 93 PvO2 (mm Hg) 65 60 52 Which of the following can the respiratory therapist conclude?

A. CvO2 is increasing B. tissues are becoming oxygen deprived C. cardiac index is decreasing D. A-aDO2 is decreasing C. This data shows a steady arterial oxygenation level with a decreasing venous oxygenation level. When the gap between arterial and venous oxygen levels start to increase, this is consistent with an increase in tissue oxygen consumption and an increase in the C(a-v)O2. This is related to a decrease in cardiac output. Of the options given, a decrease in cardiac index is another way of indicating a decrease in cardiac output.

Which of the following calculations/equations can be used to determine cardiac output in a patient with pulmonary hypertension?

A. Dalton's Law B. Bohr's C. Qs/Qt D. Fick D. The Fick equation for cardiac output is the only equation listed that can be used to calculate cardiac output.

A newborn with IRDS is being mechanically ventilated in the time-cycled, pressure-limited SIMV mode. The physician would like to increase minute ventilation. In order to accomplish this request, the respiratory therapist would make what modification?

A. Decrease flowrate B. Change to a volume-cycled mode C. Increase tidal volume D. Increase I-time D. Unlike volume ventilation, time-cycled pressure-limited mechanical ventilation will deliver more volume per breath when inspiratory time is increased. Because the breath will be delivered for a longer period of time at a constant flow rate, ventilation will increase. It is not appropriate to change to a volume-cycled mode on a newborn. Tidal volume is not a direct control in this mode and therefore is an incorrect answer. Decreasing the flow rate would result in a reduction in ventilation.

A mechanically ventilated patient in the PCV mode has the following data with a PIP of 35 cm H2O: 0800 hrs 1200 hrs 1600 hrs Tidal volume (mL) 500 450 400 The low-volume alarm is sounding. How should the respiratory therapist remedy this problem?

A. Decrease the peak-pressure alarm. B. Increase the tidal volume. C. Reset the low-volume alarm. D. Initiate bronchodilator therapy C. In the pressure-control ventilation mode, tidal volume is a result of PIP, I-time, and lung compliance. As lung compliance dereases, the tidal volume will also decrease and the respiratory therapist should adjust the low-volume alarm accordingly.

A patient recovering from congestive heart failure is intubated and receiving mechanical ventilation via a PB 840 ventilator in the spontaneous mode. Pressure support is set at 12 cm H2O and CPAP is set at 5 cm H2O. Which of the following alarms is the respiratory therapist's highest priority for monitoring effective ventilation?

A. High respiratory rate B. Low minute ventilation C. Low PEEP D. Low peak pressure B. In the spontaneous mode all breaths are determined by the patient's respiratory drive. Therefore, it is necessary to set alarms so that the respiratory therapist is alerted if the patient stops breathing or reduces ventilation to an unsafe level.

Which of the following calculations can be used to determine TLC?

A. IC + FRC B. RV + ERV + VT + IC C. FRC + VT + IC D. RV + ERV + VT A. One must be familiar with all of the components of total lung capacity in order to answer this question. This kind of question can be time consuming because the calculation possibilities are limitless. To answer this type of question, one must simply calculate each answer until the correct calculation is found.

A patient with COPD requires NIPPV. Which of the following would be suitable for initial settings?

A. IPAP 20 cm H2O, EPAP 10 cm H2O B. IPAP 5 cm H2O, EPAP 5 cm H2O C. IPAP 15 cm H2O, EPAP 3 cm H2O D. IPAP 10 cm H2O, EPAP 5 cm H2O D. Appropriate initial settings are: IPAP 10-12 cm H2O, EPAP 4-6 cm H2O.

A mechanically ventilated patient on an FIO2 of 1.0 has an A-a gradient of 275 mmHg. Which of the following is most closely associated with this finding?

A. IPPB is indicated B. Hyperbaric therapy is indicated C. PEEP is indicated D. The patient has a shunt C. PEEP is the most appropriate response and should help the patient in this scenario. A shunt is defined as A-a gradient of 300 mm Hg or more. Hyperbaric therapy is excessive. IPPB is not practical considering the presence of mechanical ventilation.

A 44-year old female with a history of myasthenia gravis is receiving full ventilatory support with a volume-controlled ventilator at the following settings: Mode Assist/control FIO2 0.60 Rate 14/min VT 500 mL PEEP 10 cm H2O Arterial blood gas analysis reveals the following: pH 7.24 PaCO2 56 torr PaO2 110 torr HCO3- 22 mEq/L BE - 4 mEq/L Which of the following actions should the respiratory therapist take?

A. Decrease the tidal volume B. Switch to SIMV and add pressure support C. Decrease PEEP to 5 cmH2O D. Increase the mandatory rate The arterial blood gas results show that the patient is under ventilating and over oxygenating. Because under ventilation should be corrected before over oxygenation minute volume should be increased. Because CO2 is out of range by more than 4 mmHg, the most appropriate method for correction is to increase the mandatory rate.

A patient is found to have a rhythm on the ECG monitor consistent with asystole. Which of the following is the best initial action of the respiratory therapist?

A. Defibrillate at 360 joules, unsynchronized B. Confirm in a second lead C. Begin chest compressions D. Defibrillate at 50 joules, synchronized B. This rhythm is called asystole. When observed, prior to starting chest compressions, the rhythm should be confirmed in his second lead.

A female patient who was intubated for airway protection is breathing room air spontaneously through a 7.5 mm endotracheal tube. Her temperature is 97.4 deg F and uncorrected arterial blood gases are: pH 7.36 PaCO2 39 torr PaO2 68 torr HCO3- 24 mEq/L BE -0.2 mEq/L What should the respiratory therapist do?

A. Extubate and place on a nasal cannula, 5 L/min B. Place on a mechanical ventilator; Mode CPAP, FIO2 of 0.30 C. Place an HME on the endotracheal tube D. Provide supplemental oxygen via heated aerosol D. In this example the respiratory therapist is dealing primarily with an oxygenation problem. Therefore, the two viable options are to place the patient on a heated aerosol or a nasal cannula. Because the patient's temperature is low, the heated aerosol is the best choice.

A patient with myasthenia gravis who is on mechanical ventilatory support is being monitored every 2 hours. Recent findings show that the maximum inspiratory pressure (MIP) has changed from -30 cm H2O to -18 cm H2O in the last 2 hours. The respiratory therapist should recommend

A. FEV1.0 sec B. FEV1.0/FVC ratio C. measure tidal volume and vital capacity D. SpO2 C. A patient with myasthenia gravis will suffer a slow degradation of ventilatory muscles as paralysis spreads down the body. The point at which the patient is in need of mechanical ventilatory support is identified primarily by observation of three measurements. These include MIP, VC, and VT. In this question, MIP has fallen significantly. Vital capacity should be monitored closely as well. As soon as VC falls below 1.0 L, the patient is in need of mechanical ventilatory support.

The best test determination for a forced vital capacity maneuver is done by which of the following calculations?

A. FEV1/FVC B. TLC - FVC C. FEV1 + FVC D. SVC + FVC C. To determine the best trial or best patient effort for a forced vital capacity maneuver, the trial with the highest sum of the FEV1 and FVC is considered to be the patient's best effort

Which of the following would be helpful in determining if a patient is restrictive?

A. FEV1/FVC% B. FEF25-75% C. Vital capacity (VC) D. FEV1 C. Restrictive lung disease is primarily indicated by a reduction in volume-related pulmonary function parameters, such as vital capacity (slow or forced). Flows help to determine the presence of an obstructive defect.

The respiratory therapist reviews the results of a patient diagnosed with mixed restrictive and obstructive pulmonary defects. Which of the following data is NOT associated with this diagnosis?

A. FEV1/FVC% of 78% B. FEV1 of 75% of predicted C. SVC of 68% of predicted D. FEF 200-1200 of 74% of predicted A. To be considered restrictive, the patient must have a slow file capacity or forced vital capacity of less than 80% of predicted. To be considered obstructive a patient must have an FEV1 of less than 80% of predicted or an FEV1/FVC% of less than 75%.

A patient with significant pulmonary shunting is receiving volume-controlled ventilation on the following settings and has the following arterial blood gas results: Mode Assist/control Mandatory rate 18 VT 500 mL FIO2 0.70 PEEP 25 cm H2O C.O. 3.2 L/min Heart rate 118 pH 7.36 PaCO2 45 torr PaO2 54 torr HCO3- 26 mEq/L BE +2 mEq/L The therapist should decrease which of the following?

A. FIO2 B. PEEP C. flow D. rate B. This patient is hypoxic. To correct this either PEEP or FIO2 should be increased. However, closer examination of hemodynamic data, namely cardiac output, shows instability. Normal cardiac output is 4-8 L/min. This patient has low cardiac output most likely brought about by excessive levels of PEEP. The most appropriate action, in spite of hypoxemia, is to decrease PEEP.

A patient with asthma receiving volume-controlled mechanical ventilation has the following arterial blood results on the settings below: Mode Assist/control ABGs Mandatory rate 18 pH 7.33 VT 400 mL PaCO2 47 torr FIO2 0.40 PaO2 78 torr Mech PEEP 5 cm H2O HCO3- 26 mEq/L Total PEEP 8 cm H2O BE +2 mEq/L I:E 1:2 Which of the following should be increased?

A. FIO2 B. flowrate C. tidal volume D. PEEP B. This patient is developing autoPEEP as manifested by a total PEEP that is higher than the set mechanical PEEP. This is caused from insufficient expiratory time and is remedied by shortening inspiratory time, which may be done by increasing flowrate.

Which of the following is NOT a control found on a high frequency jet ventilator?

A. FIO2 B. tidal volume C. injector line D. PEEP B. A high-frequency jet ventilator does not have traditional controls compared to a volume-controlled ventilator. It's typical controls include injector line, PEEP, FIO2, oscillatory amplitude, and driving pressure. Tidal volume is not a setting found on a high-frequency jet ventilator.

A patient with COPD is receiving SIMV, A/C ventilation by oral endotracheal tube. Through written communication, the patient expresses that she feels she is not getting enough air. The respiratory therapist should include which of the following assessments to investigate the patient's complaint?

A. FIO2 setting B. PetCO2 C. cuff pressure D. chest radiograph C. Low cuff pressure may allow air to escape around the cuff with each ventilator breath, resulting in a reduction in delivered tidal volume and the development of "air-hunger". The FIO2 setting would not likely result in such a feeling. A chest radiograph is time-consuming and is not likely helpful.

While performing pulmonary function testing, the respiratory therapist asks the patient to inhale and exhale maximally at a maximum rate for 12 seconds. Which of the following is being measured?

A. FVC B. minute alveolar ventilation C. FEV1 D. MVV D. The pulmonary function maneuver expressed, when the patient is asked to exhale maximally at a maximum rate for 12 seconds, is called maximum voluntary ventilation or MVV. Since the patient is most likely unable to breathe in and out maximally for a whole minute, the patient may be asked to do so for 12 seconds and then the result is multiplied by 5. 15 seconds is also another interval that may be used and it would be multiplied by 4 to calculate the MVV.

A patient who experienced an acute asthmatic episode has received three aerosolized bronchodilator treatments. Which of the following could be measured to determine the effectiveness of the therapy?

A. Fev1 B. VC C. FLV D. IC A. FEV1 is a flow measurement, compared to IC and VC, which are volume measurements. As bronchodilation occurs, an increase in expiratory flow rate would most likely be observed. Therefore this is the most effective measurement of bronchodilator efficacy.

Which of the following exams provides the best diagnostic information for an obstructive defect in a COPD patient?

A. Fev1/FVC% B. SVC C. FEF200-1200 D. DLCO A. To determine if the patient has an obstructive defect, as would be seen with a COPD patient, the best pulmonary function test would be the Fev1/FVC%. The second best test would be the Fev1.0 by itself. In order to determine if a patient is obstructive you must examine the flows. Slow vital capacity is a volume and therefore would be helpful in determining if the patient is restrictive. DLCO helps determine the diffusion capacity of the alveoli and will not differentiate between obstruction and restriction.

What is the unit of measure for cardiac output?

A. Fick equation B. L/min C. dm2 /min D. stroke volume X heart rate B. Cardiac output is calculated by multiplying the stroke volume by the heart rate, or by using the Fick equation, and the unit of measure is L/min.

An infant born 24 hours prior is experiencing frequent periods of apnea lasting more than 60 seconds. Which of the following medications would be helpful in stimulating the infant's respiratory drive?

A. Fluticasone B. hormone C. Exosurf D. Aminophylline D. An infant who experiences apnea periods greater than 60 seconds may need respiratory stimulation. This may be accomplished by administering the medication Aminophylline. Aminophlylline is only intended for ventilatory stimulation in infants. The same medication is used for long-term bronchodilation in adults.

Which of the following can be used to determine fetal lung maturity

A. Glasgow B. Blood glucose C. APAGAR D. PC level (phosphatydlchloride) D. Blood glucose is not helpful in assessing infant pulmonary maturity.

The results of a spinal tap on patient complaining of flu-like symptoms and sluggish lower extremities reveals significant presence of protein. Which of the following is associated with these findings?

A. Guillain-Barre Syndrome B. Muscular distrophy C. Myasthenia gravis D. Salmonella poisoning A. Significant proteins found in the spinal fluid indicate a diagnosis of Guillain-Barre' Syndrome.

Peak flow measurements are indicated in which of the following conditions, if present?

A. Guillain-Barre' B. myasthenia gravis C. airway obstruction D. ARDS C. Peak flow measurements are especially helpful in patients with asthma or patients who are suspected to have airway obstruction, mostly due to bronchoconstriction.

Which of the following medications is shown to have sufficient affect when facial muscle twitching is observed?

A. Halcion B. Anectine (succinylcholine chloride) C. Romazicon D. Curare (d-turbo curare) B. Anectine is the medication that manifests muscle twitching about the face and neck when it is working properly.

A respiratory therapist notes endotracheal suctioning is ineffective as the patient's secretions have become thick and tenacious. The patient is intubated with an 8.0 mm endotracheal tube and is being suctioned at a pressure of 100 mm Hg with a 10 Fr closed system suction catheter. Which of the following modifications in therapy would be most beneficial to the patient?

A. Have the patient orally consume water B. Use a 12 Fr suction catheter C. Increase suction pressure to 110 mm Hg D. Diurese the patient B. This patient has an 8.0-mm endotracheal tube but is being suctioned by a 10 French catheter. This catheter is too small for the patient. To determine the appropriate maximum size catheter, simply divide the endotracheal tube size by 2 and then multiply that number by 3 three to obtain the maximum size in French units. In this case, the patient should be using a 12 French catheter.

A patient, who is nasally intubated and is breathing spontaneously, has been developing thick secretions over the last few days. The respiratory therapist can best help the situation by

A. Having the patient increase oral consumption of water B. Attaching an heat moisture exchanger to the end of the endotracheal tube C. Ensuring inspired gas is 35 degrees C D. Providing routine postural drainage and percussion C. Secretions are best hydrated with heated humidity. This is especially true for patients who are orally or nasally intubated. In order to deliver proper humidification, inspired gases must be at temperatures that are close to normal body temperature (37 deg C).

A patient is in the intensive care unit following the repair of a hip fracture. Within 2 hours after surgery, the patient is hypotensive and is increasingly short of breath. Heart rate has risen from 90 to 110 bpm. Which of the following test results would be most helpful at this time?

A. Hb and HCT B. Creatinine C. Arterial blood gases D. COHb A. Immediately following surgery a patient that is hypotensive is most likely hemorrhaging from a leaky vessel from the operation. To confirm the loss of blood an Hb and HCT may be done to examine hemoglobin level. If low, the patient should be sent back to surgery to explore for a leak but not before blood is adminisitered, if the is an option.

A patient with pneumonia has thick secretions and is complaining of difficulty coughing and expectorating sputum. Which of the following should the therapist recommend as the most effective method of hydrating secretions?

A. Heated aerosol treatments B. Cool aerosol treatments C. Ultrasonic nebulizer treatments D. Oral consumptions of water D. For a patient who is breathing spontaneously and independently with no artificial airway in place, the best way to hydrate secretions is to orally consume water.

The respiratory therapist is paged to the intensive care unit (ICU) to assess a patient who has just self-extubated. Currently, the patient is resting comfortably with an SpO2 of 98% on FIO2 0.40 by air-entrainment mask. Respiratory rate is 14/min and breath sounds are normal, for a mild inspiratory stridor which is audible without the stethoscope. The therapist will recommend:

A. Re-intubate the patient B. Switch to cool aerosol at FIO2 0.40 C. Send the patient to surgery for a tracheostomy D. Administer racemic epinephrine - 3 doses 20 minutes apart B. Mild inspiratory stridor may be treated with racemic epinephrine and/or cool aerosol. In this case, since the patient is already receiving 40% by air-entrainment device, switching to a cool aerosol is the best option. Multiple doses of racemic epinephrine are not indicated.

A respiratory therapist has been paged to the neonatal intensive care unit to assess an infant who is 38 weeks of gestation, delivered 3 hours prior. Which of the following would be appropriate to include in the assessment?

A. Record of APGAR, at 1 and 5 minutes B. symptoms C. R/Q value D. oxygen index A. An infant is incapable of communicating problems such as shortness-of-breath and pain. This type of information is considered to be subjective and is also referred to as "symptoms". APGAR scores can be very helpful in directing the further care of the patient. Oxygen index is generally used in complex oxygenation cases and is not really appropriate at this stage in the case.

An adult female weighing 168 lbs is receiving pressure-cycled noninvasive ventilation with a Bird Mark 7. The machine-delivered pressure breaths are not cycling off as expected. What should the respiratory therapist do?

A. Reduce inspiratory time B. Look for a leak in the circuit C. Observe the pressure/volume curve in the ventilator graphics D. Suction the patient B. Pressure-cycled mechanical ventilation cycles into exhalation once the preset pressure is reached. If the breaths will not cycle off, a leak exists. The leak may be coming from the patient's mouth or the circuit itself. Therefore the best answer is to look for a leak in the circuit.

After 20 minutes into a spontaneous breathing trial (SBT) a mechanically ventilated female patient becomes anxious. Her baseline heart rate has increased by 20 /min. What should the respiratory therapist do?

A. Return to full mechanical ventilation B. Calculate the RSBI C. Measure MIP D. Increase the FIO2 and continue to monitor A. An increasing heart rate by at least 20 bpm defines the failure of a weaning trial. Further calculations or measurements are not appropriate as the patient needs mechanical ventilation prior to further deterioration.

A bronchogram reveals increased pulmonary infiltrates and some consolidation in the lateral and posterior basal segments. The respiratory therapist should place the patient in which of the following positions to concentrate on the areas with increased infiltrates?

A. Reverse Trendelenburg B. Supine position C. Trendelenburg with head-of-bed down 30 degrees D. Prone position with a pillow under the hips C. To drain the lateral and posterior basal segments, or any of the basal segments, the appropriate position is Trendelenburg, or the head-of-bed down 30°.

How would the respiratory therapist assure that a bedside spirometer is accurate prior to testing a patient?

A. Review the most recent PM report. B. Test with a 3-liter super syringe. C. Perform a pre-operational test. D. Test the patient on two different spirometers and assure correlation. B. A known, reliable source of gas volume of 3 liters can be used to calibrate a simple spirometer prior to use.

A patient diagnosed with cystic fibrosis has difficulty expectorating secretions, which are thick and tenacious. Which of the following medications are appropriate?

A. Romazicon B. Pulmicort (Budesonide) C. Dornase alpha (Pulmozyme) D. Sublimaze D. Patients with thick secretions require mucolytic medication. For patients with cystic fibrosis the most appropriate mucolytic Dornase Alpha.

A patient has chocolate-colored blood as a result of excessive absorption of benzocaine. The respiratory therapist should recommend

A. Romazicon (flumazenil). B. sodium bicarbonate. C. administration of methylene blue. D. heparin. C

A patient is brought to the emergency room after being found cold and coughing uncontrollably in a stairwell of an abandoned building. First responders indicate the patient was coughing up frank blood. The following chest radiograph is produced. The respiratory therapist should recommend which of the following further testing?

A. Test for VRE B. HIV C. Urine culture D. TB skin test (mantoux) D. Coughing up frank blood is an indication of a significant pulmonary problem. In addition to the circumstantial data indicating that the patient was cold and coughing uncontrollably, suggests the possibility of tuberculosis. A TB skin test is appropriate.

A respiratory therapist takes report on a neonatal patient who is in an incubator, receiving humidity and oxygen therapy. The therapist notices the "red-flag" warning device is in use. Which of the following can the therapist conclude?

A. The incubator is sealed and should not be opened. B. The humidity reservoir is low or out of sterile water C. FIO2 is 1.0 D. The patient is in strict isolation C. The "red-flag" warning system on incubators indicates the use of 100% oxygen. Continual use of high FIO2 can result in retinopathy of prematurity or ROP.

The respiratory therapist should discontinue beta sympathomimetic aerosol therapy in which of the following scenarios?

A. The patient experiences nausea and tingling in their extremities B. A COPD patient is smoking cigarettes between therapy C. A pediatric patient does not cooperate with the therapy D. An asthmatic patient has a peak flow rate that is 75% of predicted or better A. Beta sympathomimetic medications such as albuterol, Alupent, Xopenex, etc., should be discontinued if the patient does not tolerate them well. Tingling in the extremities, nausea, & increased heart rate greater than 20 bpm, are all examples of poor tolerance

Which of the following laboratory examinations is most appropriate to further evaluate a patient with kidney failure and CHF?

A. Theophylline level B. creatinine C. urinalysis D. BUN B. When a patient has congestive heart failure, one of the concerns is fluid shifting in the body. This fluid shifting shows up in the form of pitting edema, among additional physical signs. Not only does this relate to cardiac condition, it has implications regarding the kidney. There are two examinations that are helpful in evaluating this condition. One is creatinine and the other is BUN, or blood urea nitrogen. Both are good tests but creatinine is considered more accurate than BUN.

Oral-pharyngeal suctioning is appropriate for which of the following?

A. To facilitate sputum specimen collection for analysis B. Deep suctioning for a cachectic patient with a weak cough C. clear nasal passages D. Immediately following extubation D. Oropharyngeal describes the location for the tip of the catheter, which is the oral-pharynx, or back of the mouth. This is as far as one would need to apply suction for patients with a strong, productive cough. This is also common following extubation as secretions above the cuff of the endotracheal tube are often accumulated in the oropharynx. It is also common to apply oropharyngeal suctioning prior to intubation. Deep suctioning would be performed in the trachea itself.

How should the respiratory therapist dispose of a disposable ventilator circuit from a patient with active pulmonary tuberculosis?

A. Transport the circuit straight to the incinerator B. Place in Cydex for 60 minutes, then discard in a biohazard bag C. Place in a biohazard waste bag D. Utilize a double-bag waste system C. A ventilator circuit from a patient with active pulmonary tuberculosis is considered to be highly infectious. To dispose of this equipment, a biohazard waste bag should be used. These bags are also referred to as red bags.

In which of the following positions can the respiratory therapist expect to place a patient who will receive chest physiotherapy and postural drainage to address secretions found in the upper lobes, anterior segments of the lungs?

A. Trendenlenburg - 30 degrees B. Trendelenburg - 15 degrees C. Supine position D. Prone position C. Secretions in the upper lobes in the anterior segments of the lungs can best be drained by placing the patient in supine position.

A patient with asbestosis has a PaO2 of 48 torr after being on a nasal cannula at 6 L/min for 1 hour. The respiratory therapist should recommend

A. a nonrebreathing mask B. IPPB therapy C. switch to a Venturi mask at 35% D. intubation and endotracheal suctioning A. The delivery of 6 L/min oxygen by nasal cannula is equivalent to a 40 - 45% oxygen percentage. Yet, under this condition the patient's PaO2 shows moderate hypoxemia. To achieve a normal PaO2, a non-rebreathing mask is the most appropriate option.

A respiratory therapist is assisting the anesthesiologist in the placement of an 8.0-mm tracheostomy tube on a patient who has ARDS and is receiving mechanical ventilation in the PC mode. During a troublesome insertion, the therapist observes the development of tachycardia, a marked reduction in the return volume, and absent breath sounds over the right lung field. The therapist should suspect

A. a pleural effusion is developing on the right B. massive bilateral atelectasis C. the tracheostomy tube is too large D. right-sided pneumothorax D. One of the possible negative outcomes that can occur with the insertion of a tracheostomy tube is a pneumothorax. Data indicating absent breath sounds over the right lung field as well as a reduction in return volume suggests that a pneumothorax may have occurred on the right. A chest radiograph would be required to confirm this.

A patient with asthma receiving volume-controlled ventilation has the following arterial blood results on the settings below: Mode Assist/control Mandatory rate 14 VT 600 mL FIO2 40% PEEP 5 cm H2O I:E 1:2 pH 7.48 PaCO2 32 torr PaO2 81 torr HCO3- 24 mEq/L BE 0 mEq/L Which of the following changes is most appropriate?

A. add 50-100 mL of deadspace B. increase rate to 16 C. decrease rate to 10 D. increase inspiratory flow A. This patient is over ventilating as manifested by low CO2. CO2 may be raised by decreasing mandatory rate, decreasing tidal volume, or adding deadspace. Because CO2 is only barely off of target adding 50-100 mL of dead space is most appropriate. While decreasing rate is a step in the right direction it is likely that decreasing the rate by 4 is too significant

The low exhaled volume alarm is sounding on a patient receiving volume controlled ventilation. Which of the following could correct the problem?

A. add air to the ET tube cuff B. check for a pneumothorax C. suction the patient D. drain excess water in the ventilator circuit A. The solution to this problem is to pick those things that could cause a low volume alarm to sound. In most cases this would be a leak of some sort. When you examine the answers you do not directly see the causes of the alarms but you see the possible solutions. You must examine each solution and determine if the alarm would be a high pressure or a low-pressure alarm or something else. Suctioning the patient would presumably be because of excess secretions and would result in a high-pressure alarm. Adding air to the ET tube cuff, if the cuff was deflated, would result in a low-pressure alarm. So, that option has to be a correct one. Excess water and the ventilator circuit and the possibility of a pneumothorax would both result in a high-pressure alarm, and therefore cannot be the correct answer. Two only is the correct answer.

A patient with a history of status asthmaticus is admitted to the emergency room (ER) for the treatment of acute bronchoconstriction. After two treatments with aerosolized Albuterol, the patient demonstrates no relief. Peak flow measurements are unchanged. The respiratory therapist should recommend which of the following?

A. administer cromolyn sodium (Intal) B. switch to corticosteroid administration C. repeat the Albuterol treatment, then reassess D. administer ipratropium bromide (Atrovent) D. When a patient is not responsive to a particular bronchodilator medication, it is appropriate to attempt bronchodilation with a different medication. In this case, Atrovent is a good alternative medication because it is also a short term rescue medication, like albuterol, but with a different mode of action.

A patient is transferred to the intensive care unit after abdominal surgery. Vital signs are observed within 20 minutes of arrival. BP 90/58 mm Hg and SpO2 97% on a 40% air-entrainment mask. The patient's color is dusky. The respiratory therapist should

A. administer anticoagulant therapy B. institute volume ventilation C. assess INR D. check Hb level D. The low BP and dusky color after surgery is suggestive of anemia and possible internal hemorrhaging, likely a problem that developed as a result of the surgery. An Hb check will definitively determine if internal blood loss is occurring.

When the respiratory therapist notices the reservoir bag on a nonrebreather mask is collapsing with each breath, the therapist should

A. administer lorazepam (Ativan) B. switch to a partial rebreather mask C. increase oxygen flow to the bag D. inspect the valve between the mask and reservoir C. A reservoir bag on a non-rebreather mask that collapses with each breath is an indication that the flow to the mask is likely insufficient and should be increased.

A post-op adult patient achieved an inspiratory capacity of 1800 cc with an incentive spirometer prior to surgery. After the surgery, the patient appears to be confused on how to use the incentive spirometer. The therapist notes the patient is having difficulty getting the balls to rise. The therapist should

A. administer oxygen at 5 L/min nasal cannula B. assess the patient for the cause of the confusion C. switch to a volume-type device D. coach the patient more diligently C. Incentive spirometry comes in two types: a flow type, which requires keeping balls floating, and the volume type, which simply measures inspiratory capacity. The volume type of incentive spirometer is considered to be the easiest. It is appropriate to switch to a volume spirometer when the patient is having difficulty with a flow type spirometer.

The following patient data is observed: Blood gas and CO-oximeter results pH 7.35 PaCO2 27 torr PaO2 40 torr Hemoglobin 14.0 g/dL Oxyhemoglobin 96% HCO3- 13 mEq/L Carboxyhemoglobin 2% Methemoglobin 2% SaO2 76% BE -11 mEq/L

A. administer potassium chloride, IV B. perform a calibration of the instrument and repeat the analysis C. administer sodium bicarbonate D. report SaO2 as 76% B. A close examination of the blood gas reveals some contrasting information. Oxyhemoglobin is 96% while the O2 sat is only 76%. These data indicate a need to recalibrate the instrument and repeat the analysis.

A newborn has a one-minute APGAR score of 6. The respiratory therapist should

A. administer supplemental oxygen B. begin cardiopulmonary resuscitation C. obtain an arterial blood gas D. monitor the infant, reassess in 4 minutes A. An Apgar score can be used to determine the appropriate intervention of an infant without knowing the details of the score. A score from 0-3 indicates a need for cardiopulmonary resuscitation. A score from 4-6 indicates the infant requires supplemental oxygen, warming, and general stimulation. A score of 7-10 is normal in indicates routine care, which includes drying the infant and placing on a radiant warmer.

The respiratory therapist is performing an initial assessment on a new client in the pulmonary clinic. The patient has a 90 pack-year history of smoking, an FEV1 of 42% of predicted, and a DLCO of 15 mL CO/min/mm Hg. The following arterial blood gas data is available pH 7.37 PaCO2 56 torr PaO2 43 torr HCO3- 32 mEq/L BE + 6 mEq/L Which of the following is the most appropriate NEXT step

A. admit the patient a pulmonary rehabilitation program B. initiate supplemental oxygen C. perform a chemical cardiac stress test D. schedule the patient for sleep studies B. The patient's history and pulmonary function result (the FEV1 of 42%), in addition to an excessive smoking history, suggests chronic obstructive pulmonary disease. This suspicion is further corroborated by noting a normalized pH in the presence of hypercapnia (excessive CO2). Additionally, the patient is very hypoxic and thus requires supplemental oxygen.

A radiology report of a chest x-ray indicates the end of the radio-opaque line on an oral endotracheal tube is resting 1-cm above the carina. The respiratory therapist should suggest

A. advance the ET tube 2 cm B. withdraw the ET tube 2 cm C. advance the ET tube 1 cm D. withdraw the ET tube 5 cm B. The endotracheal tube is properly positioned when the end is 2 to 5 cm above the carina. In this case, the tube is only 1 cm above the carina. Withdrawing the by 2 to 4 cm is appropriate.

A patient recovering from an appendectomy is receiving mechanical ventilatory support in the A/C mode at a mandatory rate of 12/min. The respiratory therapist notices a total rate of 24/min and an SpO2 of 88%. A chest radiograph reveals the end of the ET tube is 1 cm below the carina. The therapist should immediately

A. advance the ET tube by 1 cm. B. adjust cuff pressure using minimal leak technique. C. momentarily increase mandatory rate to 18/min. D. withdraw the ET tube by 5 cm. D.

A patient in the cardiac intensive care unit has a Swan-Ganz pulmonary artery catheter in place. The following data is available: CVP 6 torr mPAP 19 torr PCWP 12 torr C.I. 1.7 L/min/m2 Which of the following is the most likely cause of these data?

A. affects of high PEEP B. pulmonary hypertension C. fluid overload D. right side heart failure A. CVP is essentially normal and mPAP is elevated. This does not indicate a problem with the right heart but rather a problem downstream. mPAP is elevated and PCWP is also elevated, indicating no problem within the vasculature of the lungs. PCWP is elevated but cardiac output is low. Remember, cardiac output is determined by doubling cardiac index. That would give a cardiac output of 3.4 L which is low. The problem, therefore, is in the left heart. The answer that can explain this most closely is the affects of high PEEP.

An HME is the optimal method for heat/moisture delivery through an invasive airway when the respiratory therapist is trying to prevent

A. air-trapping. B. a change in the PaCO2. C. retained secretions. D. a nosocomial infection. D. An HME, heat/ moisture exchanger, also known as an artificial nose is the best device to prevent nosocomial infections because there is not a standing water source that is heated within the circuit of the mechanical ventilator. If changed on a regular frequent basis, these devices are effective at preventing nosocomial infections.

An 11-year-old patient is brought to the clinic by his mother. She explains that her son becomes short of breath and emits a wheezing sound while playing soccer. The physician suspects asthma and recommends rescue medication PRN. Which of the following can be administered PRN for the purpose of bronchodilation?

A. albuterol B. Beclovent (beclomethasone) C. Intal (cromolyn sodium) D. Floradil (formoterol) A.

A respiratory therapist is providing instruction to a home-care patient regarding different methods of increasing the effectiveness of airway clearance. The therapist may suggest all of the following techniques EXCEPT

A. autogenic drainage B. pursed-lip breathing C. serial coughing D. huff coughing B. Of the options given, huff coughing, serial coughing, and autogenic drainage are methods associated with mobilization of secretions and airway clearance. Pursed-lip breathing is not specifically designed to clear the airway but is intended for those patients who have chronic air trapping to help exhale gases more effectively and prevent air-trapping. Specifically, it is intended to help COPD patients decrease their FRC.

A patient is transferred from a gurney to a bed. During the movement, the security device on the ET tube becomes detached. To confirm the location of the ET tube, the respiratory therapist should

A. auscultate the lungs bilaterally. B. observe the symmetry of chest rise. C. order a chest radiograph. D. observed the tube markings at the teeth. C. Confirmation of tube position should be accomplished by X-ray. Auscultation of breath sound and observance of symmetry of chest rise are helpful bedside techniques, but do not provide absolute confirmation of ET tube position.

A written teaching plan for asthma patients using peak flow meters should include

A. avoidance of placing one's teeth around the mouthpiece. B. the importance of trending over baseline results. C. the unit of measure for the flow meter. D. the meaning of red, yellow and green zone results. D. The respiratory therapist may determine that there is value in the three options that are incorrect in this scenario, but it is important to consider the best answer, which, in this case, is a thorough understanding of the color zones. Understanding the zones will help an asthma patient effectively manage their condition over time.

A home care patient is ventilator dependent for more than 50% of the time. Which of the following is appropriate to have on hand in the event of a power failure?

A. bag-valve resuscitator B. compressed oxygen C. non-rebreathing mask with an E-cylinder D. back-up ventilator A. A homecare patient who is ventilator-dependent more than 50% of the time should have a backup ventilator. However, they should also have a bag/valve resuscitator in case of power failure.

After performing bedside pulmonary function testing with a computer and spirometer, the respiratory therapist examines the results and compares them to some predicted values for the patient. When doing so the therapist is comparing the patient results to

A. baseline values for this patient B. a national database C. other patients with the same diagnosis D. statistical averages based on other tests in the computer B. Predicted values for a patient comes from the evaluation of patients across the nation and is a statistical derivative of that data.

A 35-year-old female patient presents to the emergency room with droopy eyelids and facial muscles. She complains of frequent fatigue that improves with rest, but then returns. Which of the following diagnostic procedures are not indicated?

A. basic spirometry B. arterial blood gases C. Tensilon challenge D. nitrogen washout D. The patient is showing signs of myasthenia gravis for which basic spirometry, a Tensilon challenge, and an arterial blood gas are indicated. A nitrogen washout would detect lung volume changes associated with chronic obstructive pulmonary disease, but has little to no value for a patient with myasthenia gravis.

A respiratory therapist is instructing the family of a patient who will be completely ventilator-dependent at home. Which of the following are appropriate to include in that instruction?

A. basic ventilator setting changes B. titration of FIO2 C. recognizing signs of infection D. setting ventilator alarms C. For the family of the patient who is ventilator-dependent at home, providing instruction on recognizing signs of infection, providing CPR, and performing bronchial hygiene are all appropriate. The patient will not be expected to make ventilator changes because this should be done by a qualified professional and only at the order of a physician.

For what reason should a transcutaneous PO2 and PCO2 value not be trusted immediately after placing the electrode on the skin?

A. because the electrode is not yet calibrated B. insufficient time for circulation to slow under the electrode C. capillaries are not yet heated to dilate vessels properly D. lack of normalize acid-base balance of the skin C. When obtaining transcutaneous PO2 and PCO2 data, it is important that the electrode be placed on the skin and given enough time to heat the skin surface so that the capillaries are properly dilated. Until that is done readings may be erroneous.

A 62-year-old male, six hours post CABG, experiences a sudden onset of tachypnea 3 hours after extubation. Breath sounds are markedly decreased over the right lung field. Respiratory rate is 24/min, HR 110/min, BP 160/85 mm Hg, and SpO2 is 89% on 35% oxygen via air-entrainment mask. Which of the following should the respiratory therapist recommend NEXT?

A. begin CPAP by mask at 8 cm H2O B. radiological examination of the chest C. deliver oxygen via HHF D. V/Q scan with Xenon gas B. The superficial evidence in this question indicates some sort of respiratory/ventilatory problem. However, the source of the problem is not evident and additional information is required. In this case, radiological examination of the chest may provide the additional information needed to determine the source of the problem.

A patient with a confirmed pulmonary embolism is receiving oxygen by non-rebreathing mask. When oxygen therapy was initiated, the reservoir collapsed completely with each breath. Thirty minutes later the reservoir bag collapses only partially with each breath. Respiratory rate has fallen from 28/min to 10/min. The therapist should respond by

A. beginning mechanical ventilation B. assisting ventilation with a manual resuscitator C. increasing flow to the NRB mask D. continuing current therapy D. When the reservoir on a non-rereading mask collapses completely, one must suspect that the inspiratory flow the patient is producing is exceeding the total flow to the patient from the flow meter. The solution to this is to increase the flow. However, in this case after a few moments, the bag begins to collapse only partially which is normal. This indicates that the patient has become less distressed and that the oxygen therapy has been effective. Continuing current therapy is most appropriate.

Which of the following provides the most significant evidence of improper placement of a nasal endotracheal tube?

A. breath sounds are bilateral B. right hemi diaphragm is higher than the left C. ET tube markings are 21 cm at the right nare D. SpO2 of 90% C. Proper placement of an endotracheal tube is manifest by bilateral breath sounds. Of the options given the only one that indicates an improper placement of the nasal endotracheal tube is the ET tube markings at the right nare. When the patient is nasally intubated markings at the nose should be in the high 20s.

Which of the following conditions would be most helped by therapy intended for secretion mobilization?

A. bronchiolitis B. pulmonary edema C. bronchiectasis D. hemothorax C. Bronchiectasis is a condition which may exist along with several other disease states. While there are multiple types of bronchiectasis, the common outcome is a considerably high amount of sputum produced by the lung. Secretion mobilization therapies should be selected for a patient with bronchiectasis.

A respiratory therapist is assisting a physician with a bronchoscopy when an active bleed is found in the bronchus. After instilling epinephrine down the bronchoscope directly onto the bleeding site, the next step to stop the bleeding is to

A. compress the site with the bronchoscope B. instill 3.0 cc of Heparin down the bronchoscope C. manually ventilate the patient with a bag/valve D. instill 10.0 cc of cold normal saline A. The first step in addressing a bleed that occurs during a bronchoscopy procedure is to instill epinephrine on the bleeding site through the bronchoscope. Following the installation, the site should be compressed with the bronchoscope. If the bleeding continues, a Fogarty catheter should be inserted to tamponade the site.

After 15 seconds following birth, a neonate presents with vigorous crying and a strong cough. Obvious secretions and amniotic fluids are present from the airway and stomach. The baby has peripheral cyanosis. You would FIRST

A. check the heart rate. B. provide blow by oxygen. C. suction the nose. D. suction the mouth. D. Clearing the oropahrynx or mouth of a newborn is a first step to prevent the baby from aspirating the secretions. Following the suctioning of the mouth, one should also gently suction the nose.

Which of the following should be examined to conclusively determine proper placement of an oral endotracheal tube?

A. chest radiograph B. ET tube marking at the teeth C. lateral neck radiograph D. breath sounds A. There are various ways to determine the proper positioning of an endotracheal tube. Examining ET tube markings at the teeth and listening to breath sounds are both quick ways to roughly determine the position of the tube. However, neither of those methods are conclusive. The only conclusive way to determine the proper position of the ET tube is to do a chest x-ray. Keep in mind the question asks for a "conclusive" method. If the question asked which should be done first or immediately, the best answer would be to visualize the ET tube markings at the teeth and the next best answer would be to check for bilateral breath sounds.

Which of the following should be looked at FIRST to assess for proper placement of an oral endotracheal tube?

A. chest radiograph B. breath sounds C. lateral neck radiograph D. left chest symmetry D. When examining for proper placement of an endotracheal tube the first actions that should be taken should be those that are less time-consuming. Three of these options are all suitable for determining proper positioning of the endotracheal tube. However, the quickest way to determine positioning would be to examine the left chest symmetry. Breath sounds is appropriate but takes a little more time. Obtaining a chest radiograph will definitely help to confirm positioning but will take significant time. So, if asked "what is the first step one would take", the answer would be whatever is the quickest. But, if asked what is the best way to determine the position of the endotracheal tube, the best answer would be a chest x-ray.

A patient who is intubated and receiving PS ventilation with a delta pressure of 20 cm H2O and a PEEP of 5 cm H2O, states she feels she is not getting enough air. The respiratory therapist should investigate which of the following to quickly determine the source of the patient's complaint?

A. chest radiograph B. excessive inspiratory pressure setting C. cuff pressure D. arterial blood gas C.

A physician in the emergency department has asked that a patient be evaluated for pulmonary obstruction while in the emergency room. Which of the following tests should the therapist plan on performing?

A. chest radiographic film at full expiration B. beside pulmonary function testing C. maximum inspiratory pressure D. pulmonary assessment by body box B. A patient in emergency room will be unable to perform complex pulmonary function testing. However, to determine if the patient is obstructive or restrictive, a bedside pulmonary function test is adequate. Pulmonary function testing such as DLCO, nitrogen washout, helium dilution, and airway resistance determined by body box would not be appropriate at bedside in the emergency room.

A patient with a 60 pack-year history of smoking complains of a non-productive cough over the last 2 months. A chest radiograph reveals a focal infiltrate in the right upper lung field. The respiratory therapist should recommend which of the following?

A. chest spiral CT scan B. bronchodilator therapy 4 times daily and PRN C. percutaneous lung biopsy D. flexible bronchoscopy D. Of the options given the most appropriate way to visualize and determine the nature of a focal infiltrate in the right upper lung field would be through the use of bronchoscopy. A spiral CT scan could reveal some of the infiltrate's shape but would be very expensive and ultimately inconclusive.

A patient in the emergency room is being treated for an asthmatic episode. After a treatment with Albuterol and Ipratropium Bromide (Atrovent), auscultation of the chest reveals increased wheezing. Peak flow rates have increased. The therapist should

A. administer cromolyn sodium (Intal) B. repeat the bronchodilator treatment C. administer Acetylcysteine (mucomyst) D. wait 4 hours and re-evaluate B. The presence of increased wheezing, after the delivery of a bronchodilator medication, indicates improvement by the patient or responsiveness to the bronchodilator medication. This improvement is manifested by an increase in peak flow rates. This evidence indicates the need to continue bronchodilation therapy.

A patient is found to have uncompensated respiratory acidosis. This would correlate with what kind of change in the oxygen dissociation curve?

A. compressed waveform B. expanded waveform C. shift to the right D. shift to the left C. The oxygen dissociation curve is a graphical depiction of the relationship between arterial PO2 and SpO2. It is based upon a fixed pH. When the pH changes, or the patient becomes alkaline or acidotic, the oxygen dissociation curve can shift to the right or left. In this case, acidosis will cause the curve to shift to the right, which will change the numerical relationship between PaO2 and SpO2. A shift to the right also indicates a decreased affinity between hemoglobin and oxygen molecules.

An MVA victim with severe facial trauma has been trached in the emergency room. The respiratory therapist should now provide

A. cool aerosol at FIO2 1.0 B. heated/humidified aerosol at FIO2 0.50 C. heated/humidified aerosol at FIO2 1.0 D. cool aerosol at FIO2 0.50 C. In this example, the patient's natural upper airway has been by-passed and the respiratory therapist should be concerned about delivery of cold dry gas. Heat and humidification are the therapist's primary concern. 100% oxygen is also necessary due to the emergent nature of the patient's condition having suffered a motor vehicle accident.

Immediately after extubation, a patient demonstrates moderate inspiratory and expiratory stridor. The respiratory therapist should prepare to administer

A. cool bland aerosol B. topical spray Lidocaine C. racemic epinephrine D. oral Benzocaine C. Moderate inspiratory and expiratory stridor may be treated with racemic epinephrine. This alpha-1 medication can constrict vessels and tissues and reduce inflammation. If stridor is considered to be severe or marked, racemic epi not the best choice. An airway should be established immediately. This action is preferred over racemic epinephrine because of the time it takes for the medication to have effect. Therefore, racemic epinephrine should be used in cases involving mild and moderate stridor.

A patient will be receiving short-term mechanical ventilatory support after surgery. Which of the following methods of humidification is appropriate?

A. cool, passover humidification B. HME C. centrifugal nebulizer D. large volume nebulization B. When providing humidity to the patient on a ventilator, because the natural humidification processes of the body are bypassed, 100% humidity must be provided through the ventilator. Only heated humidification can accomplish this. Non-heated passover humidification is inadequate. An HME is meant for short-term use only (a few hours). A centrifugal and large volume nebulizers are not devices used with the mechanical ventilation.

A patient in the intensive care unit is receiving mechanical ventilation on the following settings: Mode Assist/control VT 450 mL Mandatory rate 18 FIO2 0.6 PEEP 8 cm H2O Other clinical data includes: CVP 12 mm Hg PAP 15 mm Hg PCWP 8 mm Hg C.O. 4.8 L/min Which of the following most likely represents the patient's condition?

A. cor pulmonale B. left-sided heart failure C. congestive heart failure D. pulmonary embolus A. CVP is high. PAP is normal. This indicates a problem in the right heart. PCWP is normal and cardiac output is normal. This data suggests that the left heart and the lungs are okay. In the options offered only cor pulmonale is a right heart condition. Congestive heart failure is associated with a left heart problem and pulmonary embolism with a lung problem.

During routine tracheostomy care for a patient with a two-week old trach, the patient coughs, resulting in the complete dislodgement of the tracheostomy tube, which falls on the floor. The respiratory therapist should NEXT

A. cover the stoma with gauze. B. suction the patient through the stoma. C. clean the tube and re-insert. D. assess breathing. D. A patient who is breathing spontaneously through a tracheostomy tube that is suddenly ejected may be at risk for airway occlusion from soft tissues where a tracheostomy tube was placed. The first step following the extubation should be to monitor breathing and ensure adequate ventilation.

A patient being treated for a left-sided hemothorax following multiple injuries sustained in a motor vehicle accident is receiving positive pressure ventilation by a PB 840 volume ventilator. A 3-chamber chest tube drainage system is in place and functioning properly. During a routine ventilator check, the respiratory therapist notes the low-return volume begins to alarm. Which of the following would be appropriate to check to determine the cause of the alarm?

A. cuff pressure for possible excessive inflation B. possible ET tube occlusion C. possible excess condensate in the ventilator circuit D. degree of bubbling in the water-seal compartment D. When the ventilator emits a low return volume alarm and the patient is described as having chest tubes in place, one must be worried about volume being lost through a hole in the lungs. This hole is also called a perforation. One way to quickly determine whether this is the case is to examine the water seal compartment of the chest tube drainage system. Normally, only gentle bubbling should be observed in that compartment. If significant bubbling is observed, then ventilatory volume is likely being lost through the chest tube drainage system to a perforation in the lung tissue.

Under which of the following circumstances should a respiratory therapist consider discontinuing a PEP therapy treatment of a child?

A. cystic fibrosis and asthma B. epistaxis and middle ear infection C. lingular lobe pneumonia and ear infection D. bronchitis and epistaxis B. The use of PEP therapy is contraindicated by epistaxis and a middle ear infection.

A respiratory therapist is reviewing the medical record of a patient receiving PC, A/C ventilation. The ventilator flow sheet shows both a gradual decrease in plateau pressures and a recent increase in peak airway pressures. What conclusions can be made?

A. deceased pulmonary compliance, increased dynamic compliance B. decreased static compliance, decreased airway resistance C. increased static compliance, decreased dynamic compliance D. increased pulmonary compliance, increased airway resistance D.

A patient is receiving supplemental oxygen therapy at FIO2 0.60 with heated humidity by large volume air-entrainment nebulizer. What can the respiratory therapist expect to occur with FIO2 as excessive water develops in the aerosol tubing?

A. decrease B. increase C. will rise to 1.0 D. will remain unchanged B. Anytime an entertainment device encounters back pressure that develops in the device as a result of a kinked tube or water in the tubing, the result will be an increase in FIO2. This is because back pressure on the air-entrainment device will cause the gas passing through the device to slow. The slowing of gas causes the relative pressure to increase. When the pressure increases, less room air is entrained. This results in an increase in FIO2.

A patient is receiving volume-controlled ventilation. The ventilator settings and corresponding clinical data are as follows: Mode Assist/control Mandatory rate 12 Total rate 18 FIO2 0.65 Pulse 102/min Blood pressure 135/76 mm Hg (stable) PAP 15 mm Hg C.O 5.3 L/min pH 7.50 PaCO2 30 torr PaO2 52 torr HCO3- 23 mEq/L BE -1 mEq/L The respiratory therapist should recommend

A. decrease mandatory rate to 10/min B. begin Bi-level therapy C. initiate PEEP at 5 cm H2O D. increase FIO2 0.70 C. Blood gas results show hypoxemia and hyperventilation. Of these two problems, hypoxemia should be addressed first. To correct hypoxemia, one may either increase FIO2 or PEEP. Because oxygen percentage is already at or above 60% the best option is to initiate PEEP.

A 65-year-old patient with increased work of breathing is on bi-level therapy at an IPAP of 16 cmH2O and EPAP of 8 cmH2O, FIO2 0.55. The following blood gases are reported on these settings: pH 7.31 PaCO2 49 torr PaO2 110 torr HCO3- 24 mEq/L BE 0 mEq/L The therapist should

A. decrease EPAP B. increase IPAP C. increase IPAP and EPAP D. increase EPAP and IPAP A. Arterial blood gases reveal the patient is under ventilating as manifested by high CO2. Additionally, the patient is over oxygenating. Normally, to increase ventilation we raise IPAP, thereby increasing the distance between the inspiratory and expiratory pressure. However, in this case, we can simply lower the EPAP which will decrease oxygenation but will also increase ventilation because the distance between the inspiratory and expiratory pressures is increased.

A patient is receiving non-invasive positive pressure ventilation by mask with an IPAP of 16 cmH2O and an EPAP of 8 cmH2O. Corresponding blood gas data is as follows: pH 7.29 PaCO2 51 mmHg PaO2 108 mmHg HCO3- 24 mEq/L BE -0 mEq/L Which adjustment is most appropriate?

A. decrease EPAP and IPAP B. increase IPAP only C. decrease EPAP only D. increase IPAP and EPAP C. In this case there are two problems: poor ventilation, as manifested by high CO2, and over oxygenation, as manifested by a high PaO2. Both problems may be fixed with one change which is to lower the EPAP setting. Lowering the EPAP will decrease oxygenation but will also increase ventilation by increasing the distance between the EPAP and IPAP settings.

A patient with COPD is on non-invasive positive pressure ventilation to decrease work of breathing. Current settings and blood gas results are: IPAP 15 cm H2O EPAP 8 cm H2O FIO2 0.28 pH 7.31 PaCO2 65 torr PaO2 62 torr HCO3- 27 mEq/L BE +3 mEq/L The respiratory therapist should make which of the following changes?

A. decrease EPAP to 6 cmH2O B. increase IPAP to 18 cmH2O C. increase EPAP to 12 cmH2O D. increase both IPAP and EPAP by 4 cmH2O B. Blood gases reveal the patient is hypoventilating and under oxygenating. When we have a problem with both ventilation and oxygenation, we should attempt to correct ventilation first. This is because the hypoxemia may be a result of the hypoventilation. To correct ventilation, in this case, we need only to raise IPAP.

Arterial blood gases on a patient in the cardiac intensive care unit are as follows: pH 7.31 PaCO2 35 mm Hg PaO2 100 mm Hg HCO3- 18 mEq/L BE -6 mEq/L Which of the following is an appropriate action?

A. decrease FIO2 B. increase minute volume C. increase FIO2 D. administer sodium bicarbonate D. When we look at a patient's blood gas we usually start by looking at the ventilation aspect. In this case the PaCO2, which is 35 mmHg, shows normal ventilation and therefore should have normal acid-base relationship. However, when we observe the acid-base relationship we see that the pH is 7.31 and therefore is acidodic. If the source of the acidosis is not respiratory, then it must be metabolic. A quick glance at the rest of the blood gas shows the HCO3- is low. Therefore, the administration of sodium bicarbonate is most appropriate to treat metabolic acidosis.

An 80 kg (176 lb) adult male is receiving volume-controlled ventilation. Current ventilator settings and corresponding arterial blood gas results are: Mode SIMV Mandatory rate 4 Total rate 12 Tidal volume 550 mL VT(spont) 450 mL PEEP 5 cm H2O FIO2 0.40 pH 7.46 PaCO2 38 torr PaO2 90 torr HCO3- 25 mEq/L BE +1 mEq/L The respiratory therapist should recommend

A. decrease mandatory rate to 2/min B. discontinue mechanical ventilation C. add pressure support D. decrease FIO2 to 0.35 B. This patient is obviously weaning from mechanical ventilatory support as evidenced by the SIMV mode and a rate of 4/min. In this condition, the patient appears to be ventilating adequately with the tidal volume that is greater than 5 mL/kg. Blood gas results are good. This data indicates the patient is ready to be weaned from mechanical ventilatory support.

An 84-kg (185-lb) 6-ft 1-in (185-cm) male is receiving mechanical ventilation by a volume-controlled ventilator in the assist/control mode on the following settings: Mandatory rate 16 VT 600 mL FIO2 0.5 PEEP 10 cmH2O ABGs pH 7.46 PaCO2 34 mmHg PO2 100 mmHg HCO3- 24 mEq/L BE 0 mEq/L Which of the following represents the most appropriate action?

A. decrease rate to 10 B. decrease VT to 500 mL C. add 50 mL deadspace D. decrease PEEP C. In this problem the CO2 is slightly low. When we need to raise CO2 there are three choices - decreasing tidal volume, decreasing rate, or adding dead space. In this case, decreasing the rate from 16 to 10 to change CO2 by only a few points is too drastic. A tidal volume of 500 mL is less than the patient's range based on their predicted body weight. The best option is to add 50 mL of dead space

A 75-kg (166-lb), 173-cm (5-ft, 8-in) patient is receiving PC, SIMV ventilation on the following settings: FIO2 0.50 Mandatory rate 16/min Exhaled VT 780 mL IP 32 cm H2O PEEP 5 cm H2O I:E ratio 1:1.9 ABGs are as follows: pH 7.51 PaCO2 30 torr PaO2 109 torr HCO3- 25 mEq/L BE 5 mEq/L The respiratory therapist should

A. decrease set inspiratory pressure. B. decrease FIO2. C. decrease mandatory rate. D. discontinue PEEP. A.

A patient is being suctioned by the respiratory therapist. The therapist suctions the patient by inserting the catheter for 10-15 seconds and applying intermittent suction while withdrawing the catheter. During the last several attempts, the therapist notices multiple PVCs on the ECG monitor during the procedure. Which of the following can the therapist do to correct the problem?

A. decrease suction pressure B. suction for less than 10 seconds C. switch to a larger catheter D. apply continuous suctioning when withdrawing the catheter B. Decreasing the suction duration is an appropriate method to reduce the cardiac stress and prevent PVCs during the suctioning procedure.

To decrease autoPEEP on a patient receiving volume-controlled ventilation, the physician orders a decrease in the I:E ratio. The respiratory therapist will expect to make which of the following changes in ventilator settings?

A. decrease the mandatory rate B. increase the mandatory rate C. increase the flow rate D. decrease the flow rate C. Flow rate must be increased to decrease I:E ratio. Increasing the flowrate will lower inspiratory time and will prolong the I:E ratio. This will allow more time for exhalation and will help to reduce autoPEEP.

The following pressure-volume ventilator graphic is observed on a patient receiving VC A/C ventilation. The therapist should recommend

A. decrease the minute ventilation B. decrease the pressure limit C. decrease the tidal volume D. decrease in rate C. The image shown here is a ventilator graphic that shows the relationship between the volume and the pressure. The presence of the small bird beak indicates that there is some level of pulmonary overdistension. This can be remedied by decreasing the tidal volume.

While performing nasal tracheal suctioning on a patient with mycoplasma pneumonia, the respiratory therapist notices PVCs on the monitor. The first action of the therapist should be to

A. decrease the suction pressure B. discontinue suctioning C. recommend administration of Lidocaine for future suctioning of the patient D. monitor the patient carefully B. When PVCs develop during the suction procedure, this indicates acute hypoxemia that is most likely caused from the procedure. The first action should be to stop the procedure and allow the patient to recover. Future PVCs may be prevented by decreasing suction duration and by hyper oxygenating the patient.

A respiratory therapist is assisting a physician in the performance of bronchoalveolar lavage (BAL). Which of the following will most likely be used for the procedure?

A. isotonic saline B. Xylocaine C. hypertonic saline D. isopropyl alcohol A.

The respiratory therapist is asked to estimate the alveolar minute ventilation on a spontaneously breathing 68 kg (150 lb) female who is receiving oxygen therapy by air-entrainment mask at FIO2 0.50. The following data is available Exhaled VT 450 mL mPAP 15 torr PaCO2 40 torr PaO2 70 torr Respiratory rate 12/min The therapist should report an alveolar minute ventilation of:

A. 4.6 L/min B. 3.6 L/min C. 5.4 L/min D. 12.0 L/min B. To determine alveolar ventilation, dead space should be subtracted from each tidal volume. The amount of dead space per tidal volume is equivalent to 1 mL/lb of ideal body weight. In this case, the patient weighs 200 lbs and therefore has 150 mL of dead space for every inhaled tidal volume. The amount of gas that goes to the alveoli is, therefore, 300 mL per breath or 3.6 L per minute. This is calculated by subtracting 150 mL from 450 mL (the patient's VT) and then multiplying by the respiratory rate.

The following data is available for a patient receiving volume-controlled mechanical ventilation: Mode Assist/control VT 500 mL Mandatory rate 16 FIO2 0.5 PEEP 5 cm H2O PIP 28 cm H2O PetCO2 28 torr pH 7.42 PaCO2 38 torr PaO2 98 torr PvO2 76 torr HCO3- 22 mEq/L BE -2 mEq/L SaO2 98% SvO2 78% Hb 15 g/dL The respiratory therapist should record which of the following values as an accurate CaO2?

A. 5.0 vol% B. 15.2 vol% C. 25.1 vol% D. 19.4 vol% D. To determine the arterial oxygen content, the hemoglobin must be multiplied by the saturation and also multiplied times a factor of 1.34. In this case 15 x .98 x 1.34 is nearly 20 vol%. The closest answer is 19.4 vol%

A 150-lb (68-kg), 5-ft 6-in (168 cm) female patient is receiving mechanical ventilation as follows: Mode Assist/control Alveolar minute ventilation 7.5 L/min Total Rate 15 PB 734 mm Hg What is the patient's set tidal volume?

A. 550 mL B. 350 mL C. 500 mL D. 700 mL C. Tidal volume is calculated by dividing the minute ventilation by the respiratory rate. By knowing any two of these three data, one can calculate the third. For example, to calculate minute ventilation, multiply the tidal volume by the respiratory rate. To calculate respiratory rate, divide minute ventilation by tidal volume.

What is the maximum flow setting for a nasal cannula on an adult receiving oxygen therapy?

A. 6 L/min B. 5 L/min C. 10 L/min D. 8 L/min A. The maximum flow setting for a nasal cannula on adult receiving oxygen therapy is 6 L/min. This is due to a limitation in the size of the reservoir that is found in the nasopharynx. Flows higher than 6 L/min will not benefit the patient and will likely cause additional harm by drying out mucosal tissues.

What is the minute alveolar ventilation (L/min) of a 68-kg (150-lb) IBW patient who has a tidal volume of 500 mL and a respiratory rate of 14?

A. 6.0 B. 4.9 C. 2.1 D. 7.0 B. To determine the minute ventilation of the patient, one should multiply the tidal volume by the rate. In this case, tidal volume is expressed in milliliters. Because minute ventilation should be expressed as L/min, tidal volume must be translated to liters. To subtract deadspace from the tidal volume, one must estimate deadspace by looking at the ideal body weight in lbs. In this case, 150 lbs is about 150 mL of deadspace. Thus alveolar tidal volume (the amount of volume that reaches the alveoli) is 350 mL, or 0.35 L. This number should then be multiplied by the RR (14) to get the minute alveolar ventilation. (0.35 x 14 = 4.9)

What is the minute ventilation (L/min) of a 150-lb (68-kg) male who has a tidal volume 425 mL and a respiratory rate of 12?

A. 6.2 B. 5.1 C. 3.3 D. 7.4 B. To determine the minute ventilation of the patient, one should multiply the tidal volume by the rate. In this case, tidal volume is expressed in milliliters. Because minute ventilation should be expressed as L/min, tidal volume must be translated to liters. Thus, .425 x 12 = 5.1 L.

During surgery, a patient is placed on an infrared absorption device. Capnographic waveforms are normal. The difference between measured PaCO2 by arterial blood gas analysis and expired gas measurement by capnography is within expectations. PaCO2 is 40 mm Hg. What is the patient's estimated VD/VT?

A. 60% B. 40% C. 25% D. 50% C. VD/VT = (PaCO2 - PetCO2)/PaCO2. End tidal CO2 varies from PaCO2 by about 10 mm Hg. (reading lower) Therefore, (40-30)/40 = 0.25 or 25%.

During the evaluation of a post-operative patient who completed surgery 24 hours prior, the respiratory therapist coaches the patient on incentive spirometry and notes the patient is able to achieve an inspiratory capacity of 750 mL. The goal for the patient should be

A. 750 mL B. 1500 mL C. 900 mL D. 1100 mL C. The goal for incentive spirometry should be placed slightly higher than the patient's current inspiratory capacity achievement.

A 150-lb male patient has an alveolar minute volume of 4.2 L. Spontaneous tidal volume is measured at 500 mL. What is the patient's respiratory rate?

A. 8 B. 14 C. 12 D. 10 C. Alveolar ventilation is calculated by subtracting the anatomical dead space from the tidal volume then multiplying by the respiratory rate. Therefore, if alveolar minute volume is already known, one can calculate the respiratory rate.

A patient is on 80%/20% heliox therapy by nonrebreathing mask. If flowing through an oxygen flow meter, what is the actual flow of the mixture if the flow meter indicates 10 L/min?

A. 8 L/min B. 18 L/min C. 14 L/min D. 16 L/min B. When helium-oxygen mixtures are run through flow meters that are intended for oxygen only, the indicated flow on the flow meter is inaccurate. This is because the helium-oxygen mixture is less viscous and does not push the indicator ball up as much as does oxygen. In this case, a correction factor must be used depending upon the mixture percentages. For an 80/20% mixture a correction factor of 1.8 must be used. A 70/30% mixture has a correction factor of 1.6 and a 60/40% mixture has a correction factor of 1.4. In this problem the 80/20% mixture factor of 1.8 must be multiplied by the indicated flow on the oxygen flow meter to calculate actual total gas flow. When we do this we get a total gas flow of 18 L/min.

The respiratory therapist notes in the medical record of a 65-year-old male that the patient is ordered to receive bronchodilator therapy with Albuterol. The therapist also notes the patient is receiving beta-blocker medication. The therapist should recommend

A. Add Xopenex to the bronchodilator regimen B. Replace Albuterol with Beclamethasone (Beclovent) C. Switch from Albuterol to ipratropium bromide (Atrovent) D. Administer Dexamethasone (Decadron) in place of Albuterol C. Because albuterol is a beta-agonist medication, patients who are taking beta-blockers should utilize other bronchodilation medication.

The respiratory therapist is asked to estimate the alveolar minute ventilation on a spontaneously breathing 91 kg (200 lb) male who is receiving oxygen therapy by air-entrainment mask at FIO2 0.50. The following data is available Exhaled VT 500 mL SpO2 92% PetCO2 30 torr PaO2 70 torr Minute ventilation 10 L/min The therapist should report an alveolar minute ventilation of:

A. 8.2 L/min B. 12.0 L/min C. 6.0 L/min D. 4.0 L/min C. To determine alveolar ventilation, dead space should be subtracted from each tidal volume. The amount of dead space per tidal volume is equivalent to 1 mL/lb of ideal body weight. In this case, the patient weighs 200 lbs and therefore has 200 mL of dead space for every inhaled tidal volume. The amount of gas that goes to the alveoli is, therefore, 300 mL per breath or 6.0 L per minute. This is calculated by subtracting 200 mL from 500 mL (the patient's VT) and then multiplying the result by the respiratory rate. The respiratory rate is not givin here but can be determined by dividing the minute ventilation by the exhaled tidal volume.

The respiratory therapist obtains the following blood gas data on a patient breathing spontaneously on room air: pH 7.35 PaCO2 45 torr PaO2 50 torr HCO3- 27 mEq/L BE +2 mEq/L The therapist could accurately estimate the patient's SaO2 to be which of the following?

A. 80% B. 75% C. 85% D. 90% A. A PaO2 of 50 mmHg most closely correlates with an oxygen saturation of about 80% on the oxygen dissociation curve.

A 90-kg (198-lb), 175 cm (5-ft 9-in) male patient receiving mechanical ventilation has the following data: Mode SIMV Total VE 8.8 L/min Mandatory Rate 10 Total Rate 16 pH 7.36 PaCO2 45 torr PaO2 68 torr HCO3- 26 mEq/L BE 1.2 mEq/L FIO2 0.60 PEEP 10 cm H2O PB 727 mm Hg What is the patient's mean tidal volume?

A. 850 mL B. 725 mL C. 880 mL D. 550 mL D. The respiratory therapist may easily be confused because excessive information is provided in this problem. The answer is very simple as only two pieces of data are necessary. One must divide the total minute ventilation by the total respiratory rate to calculate the mean (average) tidal volume. 8.8/16 = 0.55 L, or 550 mL.

Into which interspace should a chest tube be inserted for the purpose of draining serosanguinous fluid from the pleural space and for the re-expansion of lung tissue?

A. 8th interspace B. 5th interspace C. 2nd interspace D. 4th interspace B. If fluid is anticipated, chest tubes should be place in the 5th interspace, mid-axillary line.

A patient complaining of shortness of breath with exercise has the following pre/post bronchodilator results. Which is the most meaningful indicator that a bronchodilator is effective?

A. 9% improvement in FVC B. Patient reports the maneuver is easier after the bronchodilator C. 10% improvement in FEV1/FVC%. D. 15% improvement in FEV1 D. Improvement with bronchodilator therapy is considered significant if flows increase by 12% or more, or if FEV1 increases by at least 200 mL. In this example we see a 15% improvement in FEV1, well beyond the 12% threshold.

The respiratory therapist obtains the following blood gas data on a patient breathing spontaneously on room air: pH 7.28 PaCO2 52 torr PaO2 45 torr HCO3- 24 mEq/L BE +1 mEq/L The therapist could accurately estimate the patient's SaO2 to be which of the following?

A. 90% B. 75% C. 85% D. 94% B. A PaO2 of 45 mmHg correlates to an oxygen saturation of about 75% on the oxygen dissociation curve.

Which of the following equations would help determine cardiac output?

A. A-aDO2 B. oxygen consumption C. ejection fraction D. Fick D. There are many ways to determine cardiac output. One way is called the Fick equation.

A respiratory therapist has gathered the following information for a mechanically ventilated patient in the intensive care unit: 0800 0900 1000 Peak Pressure (cm H2O) 38 34 28 Plateau Pressure (cm H2O) 32 30 26 PEEP (cm H2O) 10 10 8 FIO2 0.60 0.55 0.55 PaO2 (mm Hg) 64 64 64 VT (mL) 600 600 600 Which of the following is NOT accurate about the patient?

A. A-aDO2 is increasing. B. Dynamic compliance is increasing. C. Oxygenation is improving. D. Static compliance is increasing. A. Dynamic compliance is calculated by dividing VT by the peak inspiratory pressure minus the PEEP. Static compliance is calculated by dividing VT by the plateau pressure minus the PEEP. In this case, both are diminishing, which would result in more difficulty with oxygenation. Alveolar oxygen pressure would remain high without diffusing through the lung to reach the arteries. Therefore, A-aDO2 would increase.

Which of the following parameters is NOT consistent with readiness to wean from mechanical ventilation?

A. A-aDO2 of 180 mmHg B. MIP of -30 cmH2O C. QS/QT of 30% D. RSBI of 89 C. To wean from the ventilator pulmonary shunting must be less than 20%. RSBI must be less than 106 and the MIP must be more than -28 cmH2O. The alveoli-arterial gradient must be less than 300 mmHg. In this case a shunt of 30% is the only data indicating the patient is not ready to wean from the ventilator.

While assessing a patient receiving positive pressure ventilation with a Servo adult ventilator, the therapist notes a sudden low return volume alarm begins sounding. Which of the following could be the cause?

A. ARDS B. pulmonary embolism C. bronchopleural fistula D. excess condensate in the ventilator circuit C. The solution to this problem is to determine which answer could lead to a low-pressure alarm. Excess condensate in the ventilator circuit and adult respiratory distress syndrome would more likely lead to high-pressure alarms. A pulmonary embolism is not related to any particular alarm. That leaves a bronchopleral fistula as the only possible cause.

After a 6-hour drive a couple arrives in the emergency room complaining of headaches and nausea. The physician on duty confirms carbon monoxide poisoning. Which of the following should be include the care of the patients?

A. Acetaminophen B. Partial rebreathing mask C. inhaled CO2 D. Hyperbaric oxygen therapy D. Carbon monoxide poisoning is primarily treated by giving the patient the maximum oxygen tension possible. To start, 100% oxygen is indicated. However, the oxygen tension may be vastly improved by placing the patient in a hyperbaric chamber. When the barometric pressure is increased, the oxygen tension of the ambient air inside the hyperbaric chamber may be increased significantly, drastically reducing the half-life of carbon monoxide on the hemoglobin

A respiratory therapist is preparing a smoking cessation program to be used in conjunction with pulmonary rehabilitation. The therapist should include which of the following methods of support in the plan?

A. Acetylzolamide (Diamox) B. Daily telephone reminders not to smoke C. electronic vapor cigarrettes D. Counseling for associated emotional challenges D. A person trying to stop smoking must conquer both physiological and psychological barriers. Appropriate methods of support include emotional counseling and nicotine replacement therapy.

A patient is receiving volume ventilation through a tracheostomy tube. In response to a high pressure alarm, the respiratory therapist attempts suctioning but notices the suction catheter will not pass beyond the end of the tracheostomy tube. Which of the following would be the first action the therapist should take?

A. Deflate the cuff B. Bronchoscopy C. Increase catheter size and suction pressure D. Resume volume ventilation A. Because the catheter will not pass beyond the end of the tracheostomy tube, the most obvious suspicion is a herniated tracheostomy tube cuff. To quickly determine if this is the cause of the high-pressure and the inability to pass the catheter, the cuff should be deflated and the catheter should be inserted again to determine if the blockage remains. If the cather is able to pass, then a herniated cuff is present, and the tube should be replaced.

A long-term care patient who experienced a stroke has left-sided paralysis about the face and mouth. The patient is unable to produce a seal with her lips while performing IPPB treatments. How would a respiratory therapist expect the IPPB machine to respond in this situation?

A. Deliver a higher FIO2 B. Fail to cycle into exhalation C. Fail to cycle into inhalation D. Deliver a higher FIO2 B. The scenario suggests that a leak will occur when administering IPPB. Circuit leaks result in a failure to reach a preset pressure, which then results in failure to cycle into exhalation.

During a series of ventilator checks the respiratory therapist notices secretions from a patient receiving mechanical ventilatory support are becoming increasingly thick and tenacious. The HME has been changed several times in the last 48 hours but is currently clear of secretions and functional properly. Which of the following is the most appropriate action to take in this situation?

A. Discard the HME, install a heated humidifier B. Retain the HME, install a humidifier C. Plan routine cleaning of the HME D. Plan routine changes of the HME A. An HME is intended for short-term use and only when the patient does not have thick is copious amounts of secretions. Thick and tenacious secretions require heated humidification.

Which of the following would be the most important action prior to administering a metered dose inhaler (MDI) treatment on a patient receiving long-term volume controlled ventilation?

A. Discharge the MDI on the expiratory side of the circuit B. Discharge the MDI directly down the ET tube C. Have the patient squeeze the MDI at mid-inspiration D. Remove the HME prior to MDI discharge D. An HME device inside the ventilator circuit of the patient, who is to receive metered dose inhaler treatments, should be either removed, or switched to divert gases and medications so that they do not touch the sponge found in the HME device.

The respiratory therapist is preparing to administer bronchodilation medication by metered dose inhaler (MDI) to a patient with COPD. Prior to administration, the therapist checks the MDI canister to determine if any medication remains. To do this, the therapist should do which of the following?

A. Discharge the canister and visually note the output B. Float the canister in the glass of water C. Tap on the canister, listen for a dull tone D. Ask the patient how many doses have been taken A. The only suitable method to check a metered dose inhaler for remaining medication is to discharge the canister and visually note the output.

A respiratory therapist notes endotracheal suctioning is ineffective as the patient's secretions have become thick and tenacious. The patient is intubated with an 8.0 mm endotracheal tube and is being suctioned at a pressure of 100 mm Hg with a 10 Fr closed system suction catheter. Which of the following modifications in therapy would be most beneficial to the patient?

A. Diurese the patient B. Use a 12 Fr suction catheter C. Have the patient orally consume water D. Increase suction pressure to 110 mm Hg B. This patient has an 8.0-mm endotracheal tube but is being suctioned by a 10 French catheter. This catheter is too small for the patient. To determine the appropriate maximum size catheter, simply divide the endotracheal tube size by 2 and then multiply that number by 3 three to obtain the maximum size in French units. In this case, the patient should be using a 12 French catheter.

A patient receiving oxygen therapy at 40% has the following arterial blood gas results: pH 7.41 PaCO2 39 torr PaO2 349 torr HCO3- 24 mEq/L The respiratory therapist detects an error in the results. Which of the following could be the cause of the error?

A. Dysfunctional membrane on the Sanz electrode B. Oxygen toxicity C. Damaged Severinghaus electrode D. Improperly functioning Clark electrode D. This blood gas shows a PaO2 of 279 mmHg. However, the patient is only on 40% oxygen. When one calculates the alveolar area equation the maximum oxygen tension found in the alveoli is about 230 mmHg. Therefore, a PaO2 of 374 mmHg is impossible and suggests the analyzer is in error. The error would be related to the PO2 electrode, which is also called the Clark electrode.

What therapy would likely be most helpful on a patient whose peak airway pressures are increasing while plateau pressures remain steady during VC, A/C ventilation?

A. ECMO B. helium-oxygen therapy C. endotracheal suctioning D. chest physiotherapy C.

Which of the following can be examined to initially determine the approximate position of the oral endotracheal tube?

A. ET tube markings at the lips B. diaphragmatic excursion C. lateral neck radiograph D. chest radiograph A. When initially examining for proper positioning of the endotracheal tube, the first option should be to perform the quickest examination as possible. In this case the best answer would be to visualize the ET tube markings at the lips. Obtaining an x-ray of any sort would be too time-consuming. Examining diaphragmatic excursion would not be helpful at all.

While coaching a postoperative patient on the use of an incentive spirometer, the respiratory therapist notices the patient is not exhaling completely before starting the inspiratory phase. The therapist should provide which of the following instructions?

A. Exhale completely, then pause before starting inhalation B. Decrease breath size C. Pause at maximum inhalation for 2-3 seconds before exhalation D. Perform the incentive spirometry therapy at tidal volume A. The appropriate instruction for a patient performing sustained maximal inspiration with an incentive spirometer is to exhale completely, pausing before starting installation.

A pulmonary rehabilitation patient has an order to utilize nicotine replacement therapy by transdermal preparation to help stop smoking. The rehabilitation respiratory therapist should examine which of the following to determine patient's compliance to the smoking cessation program?

A. check nicotine level in the blood B. COHb C. pulse oximetry D. single wavelength spectrophotometry B. The best way to determine if the patient has stopped smoking is to evaluate the COHb.

A 44-year old female with a history of myasthenia gravis is receiving full ventilatory support with a volume-controlled ventilator at the following settings: Mode Assist/control FIO2 0.60 Rate 14/min VT 500 mL PEEP 10 cm H2O Arterial blood gas analysis reveals the following: pH 7.24 PaCO2 56 torr PaO2 110 torr HCO3- 22 mEq/L BE - 4 mEq/L Which of the following actions should the respiratory therapist take?

A. Increase the mandatory rate B. Decrease PEEP to 5 cmH2O C. Decrease the tidal volume D. Switch to SIMV and add pressure support A. The arterial blood gas results show that the patient is under ventilating and over oxygenating. Because under ventilation should be corrected before over oxygenation minute volume should be increased. Because CO2 is out of range by more than 4 mmHg, the most appropriate method for correction is to increase the mandatory rate.

A home care patient with a tracheostomy tube uses an electrically-powered ventilator during sleeping hours. Recent nocturnal oximetry results reveal moderate hypoxemia while asleep on the ventilator. What should the respiratory therapist recommend?

A. Increase the set respiratory rate by 2/min. B. Perform complete polysomnography. C. Provide oxygen bleed-in by molecular sieve device. D. Provide a tracheostomy collar with FIO2 0.40 heated aerosol. C. The best device for home oxygen therapy is a molecular sieve device, commonly known as an oxygen concentrator. This device is powered with electricity, which makes it appropriate for home use.

Which of the following patient conditions would CONTRAINDICATE the use of Trendelenburg's position?

A. Increased intracranial pressures B. Independent lung ventilation C. Lobectomy D. Massive blood loss A. Placing the patient's head-of-bed down would be harmful to the patient, who has increased intracranial pressures. This could cause further increase which could result in herniation of the brainstem and permanent neurological damage.

A newborn infant experiences apnea when breast feeding, but recovers quickly when breast feeding is discontinued. Which of the following mostly likely explains the condition?

A. Infant respiratory distress syndrome B. Tetraology of fallot C. Diaphragmatic hernia D. Choanale atresia D. A newborn infant who experiences apnea when breast or bottle feeding, but who recovers quickly once feeding is discontinued, most likely has difficulty breathing through their nose. Choanele Atresia is a congenital defect where one or both nares are anatomically closed.

A patient is receiving NPPV on the general floor to mobilize secretions by portable CPAP connected to medical wall gas. The threshold resistor is not maintaining consistent positive airway pressure when the patient attempts to sigh. What should the respiratory therapist do?

A. Instruct the patient to avoid sighing B. Increase CPAP level C. Replace the CPAP unit D. Increase the flowrate C. In this example we have improperly functioning equipment. If given an option to replace such equipment, the respiratory therapist should always choose to do so. One might be tempted to increase the flow rate in this scenario, however, it will not make a difference with this type of resistor.

A patient with congestive heart failure, who is demonstrating moist crepitant rales upon auscultation, would benefit most from

A. Lasix B. administration of aminoglycocide medication C. PEP therapy D. postural drainage and percussion A. A patient with congestive heart failure who is demonstrating moist crepitant rales is most likely in a state of fluid overload. The administration of Lasix is most appropriate in order to diurese the patient.

After confirming the blood gas analyzer is "in control", the respiratory supervisor introduces arterial blood from a patient breathing room air and obtains the following results: pH 7.39 PaCO2 41 torr PaO2 210 torr HCO3- 24 mEq/L BE -2 mEq/L Which of the following should the respiratory therapist investigate?

A. Membrane integrity of the Severinghaus electrode B. Fluid level of the reference electrode C. Function of the Clark electrode D. Membrane integrity of the Sanz electrode C. Close examination of these blood gas results shows a PaO2 of 210 mmHg. This would be an impossible result for a patient who is breathing room air, or 21% oxygen. Estimation of the alveolar oxygen tension proves this. The maximum oxygen tension that could be found in the alveoli is around 100 mmHg. The arterial oxygen tension can not exceed alveolar oxygen tension. This would suggest a malfunctioning of the oxygen electrode in the blood gas machine. This electrode is sometimes called the Clark electrode.

An 8-year-old hypotensive male patient has been brought in by paramedics to the emergency room following a house fire. He was found unconscious in the garage where smoke was prevalent. Which of the following is a definitive measure to assess carbon monoxide poisoning?

A. Methemoglobin level B. Arterial blood gas C. Carboxyhemoglobin level D. Rapid measurement with a multiple wavelength spectrophotometer C. The clinical background strongly suggests smoke inhalation, which would result in carbon monoxide poisoning. The most definitive way to assess for this condition is by measuring the carboxyhemoglobin level in the blood. One might be tempted to select rapid measurement with a spectrophotometer, but the keyword "definitive" in the question indicates that a diagnostic procedure should be utilized.

A 32-year old, 73-kg (161-lb), 178-cm (5 ft, 10 in) male patient is receiving VC ventilatory support on the following settings with the following ABG results: Mode SIMV VT 400 mL (f) 12/min PEEP 5 cm H2O FIO2 0.45 ABGs pH 7.29 PaCO2 50 mm Hg PaO2 81 torr HCO3- 24 mEq/L BE -5 mEq/L The respiratory therapist should make which of the following observations about the patient's condition?

A. Minute ventilation is too low. B. Refractory hypoxemia is present. C. The patient has developed ARDS. D. Hypoxemia secondary to hypoventilation is present. A. The patient is obviously hypo-ventilating as shown by an elevated PaCO2 level. The source of the problem is that the tidal volume is set too low. It should be at least 6 mL/kg IBW. In this case, the patient weighs 73 kg. As such, the minimum tidal volume should be 438 mL. Although the rate is fine, the less-than-minimum VT is exposing the patient to a minute ventilation that is too low.

A patient has increased systemic arterial blood pressure and increased ventricular preload. The respiratory therapist may recommend which of the following to decrease both of these values?

A. Nipride (sodium nitroprusside) B. dopamine (Intropin) C. Full-strength epinephrine D. Atropine Sulfate A. Systemic arterial blood pressure may be decreased with the administration of Nipride (sodium nitroprusside). This works by causing vasodilation, thereby decreasing blood pressure. This will also decrease ventricular preload.

A family of 4 is suspected to have CO poisoning from exhausted fumes that leaked into the cabin of their vehicle during a trip. Which of the following would be most helpful at quickly assessing the degree of carbon monoxide exposure?

A. O2 saturation by pulse oximetry B. multiple wavelength spectrophotometry C. COHb by arterial blood analysis D. PaO2 by arterial blood gas B. A COHb is the gold standard for determining if a patient has carbon monoxide poisoning. When this is suspected normal pulse oximetry should not be trusted. However, multiple wavelength spectrophotometry is a method that works like pulse oximetry but is capable of reading more than just oxygen. It also has the ability and to determine carbon monoxide in the blood. A normal pulse oximeter is also called a single wavelength spectrophotometer whereas an oximeter capable of reading other gases, including CO, is called a multiple wavelength spectrophotometer.

A patient is post-op for a triple coronary artery bypass graft. In the recovery unit the respiratory therapist notices a steady decrease in SVO2 while SaO2 remains unchanged at 96%. The patient is receiving oxygen by high-flow mask at 40%. Which of the following can be concluded?

A. Oxygen consumption at the tissue level is decreasing. B. FIO2 is excessive and should be decreased. C. Cardiac output is reduced. D. PvO2 is increasing. C.

A patient is receiving VC A/C ventilation at a rate of 18/min, VT 600 mL, FIO2 0.6, PEEP 5 cm H2O, I:E ratio of 1:1.8. A flow-time ventilator graphic shows the exhalation phase fails to return to baseline prior to the initiation of the inspiratory phase, possibly promoting air-trapping. What additional data would be helpful in further assessing this problem?

A. P/F ratio B. AutoPEEP determination C. mPAP and CO comparison D. OI (oxygen index) B. When air-trapping is suspected, AutoPEEP may develop. Therefore, AutoPEEP determination is helpful. The procedure involves utilizing the ventilator's computer to determine the amount of 'PEEP' above the set PEEP level to which the patient is exposed.

A physician suspects a patient has a pleural effusion. Which of the following radiography procedures is helpful in diagnosing the problem?

A. PA chest radiograph B. lateral decubitus radiograph C. apical lordotic radiograph D. AP chest radiograph B. Pleural effusion is diagnosed through direct observation of a chest radiograph, specifically, the lateral decubitus x-ray.

A patient with chronic obstructive pulmonary disease is on a ventilator as a result of ventilatory failure secondary to bacterial pneumonia. After 4 days of antibiotic therapy, the following blood gases are obtained while on SIMV, rate 10/min, VT 400 mL, FIO2 0.35, PEEP 3 cm H2O: The patient would benefit most from

A. PC ventilation. B. a tracheostomy. C. ventilator liberation starting with SBT. D. a reduction in the mandatory rate. C. Although PaO2 appears low and the patient is obviously hypercapneic, pH is normal, suggesting the patient is at his normal baseline level for ventilation and oxygenation. Ventilator liberation efforts are appropriate.

Which of the following data would provide the most information about arterial oxygenation relative to administered supplemental oxygen?

A. PD50 B. C(a-v)O2 C. SvO2/FIO2 D. P/F ratio D. The P/F ratio is a value helpful in determining the relationship between PaO2 and FIO2. This provides information related to the effectiveness of oxygen administration. If the value is below 300, acute lung injury is suspected. If the value is below 200, adult respiratory distress syndrome is likely.

A patient with COPD who has been receiving mechanical ventilation for 3 weeks has been weaned down to the following settings: FIO2 0.30 Mandatory rate 8 Mode SIMV VT(set) 400 mL VT(spont) 320 mL PEEP 3 cm H2O PS 4 cm H2O On these settings, the following laboratory data is observed: pH 7.36 PaO2 65 torr PaCO2 70 torr HCO3- 30 mEq/L BE -1 mEq/L The respiratory therapist should recommend

A. PRVC B. APRV C. NAVA D. SBT D. This data shows a blood gas for a COPD patient. Although CO2 is elevated, pH is compensated. All other data suggests the patient is ready to begin ventilator liberation. Because it can be difficult to wean a COPD patient from a ventilator, the use of spontaneous breathing trials is helpful. Spontaneous breathing trials are a way to build respiratory muscle strength slowly and facilitate ventilator liberation for patients who are prone to becoming ventilator dependent.

A respiratory therapist is caring for a patient who has a sign posted outside their hospital room door that indicates, "Respiratory Isolation". The therapist should observe all of the following EXCEPT:

A. Patient should use a mask when visitors are present B. Use N95 respirator C. Universal precautions D. Do not allow visitors to enter the room D. Respiratory isolation requires use of an N95 respirator, and universal precautions. The patient's use of a mask when visitors are present is also required. Disallowing visitors in the room is not required.

An 80-kg (176-lb) male patient was placed on mechanical ventilation 3 days ago for respiratory failure secondary to septicemia. After a one-hour trial on a T-tube Briggs adapter, the patient is having periods of 30 seconds of apnea. What should the respiratory therapist do FIRST?

A. Provide verbal stimulation to the patient to encourage spontaneous breathing B. Place the patient on CPAP 5 cm H2O C. Place the patient on pressure support D. Place the patient on pressure control ventilation D. Due to the apneic episodes, the primary focus of the respiratory therapist should be to re-establish effective ventilation. In this example the only choice that provides ventilation is placing the patient on pressure control. The respiratory therapist may feel uneasy selecting this option, but it is the only ventilatory mode, and therefore must be selected.

Which of the following can be used to estimate the level of Hemoglobin?

A. RBC B. Theophylline level C. C(a-v)O2 D. PAO2 A. There is a relationship between hematocrit, hemoglobin, and red blood cell count. Each of these items have a multiplier of three in common. For instance, normal RBC is about 5. Hemoglobin can be estimated by taking the RBC and multiplying is by 3. That would give you a hemoglobin of 15. Hematocrit can be estimated by taking the hemoglobin and multiply it by 3, which would give you 45%. Therefore, to estimate hemoglobin, hematocrit, and RBC, one only has to know one of the other values. From there, you can estimate the other two.

The low exhaled volume alarm is sounding on a patient receiving volume controlled ventilation. Which of the following could correct the problem?

A. add air to the ET tube cuff B. suction the patient C. check for a pneumothorax D. drain excess water in the ventilator circuit A. The solution to this problem is to pick those things that could cause a low volume alarm to sound. In most cases this would be a leak of some sort. When you examine the answers you do not directly see the causes of the alarms but you see the possible solutions. You must examine each solution and determine if the alarm would be a high pressure or a low-pressure alarm or something else. Suctioning the patient would presumably be because of excess secretions and would result in a high-pressure alarm. Adding air to the ET tube cuff, if the cuff was deflated, would result in a low-pressure alarm. So, that option has to be a correct one. Excess water and the ventilator circuit and the possibility of a pneumothorax would both result in a high-pressure alarm, and therefore cannot be the correct answer. Two only is the correct answer.

A patient with a history of congestive heart failure has been transferred to the cardiac intensive care unit with fulminating pulmonary edema, crepitations in the left lung field, and +3 pitting peripheral edema. A non-rebreathing mask is in place on the patient and the oxygen flow rate is set to 15 L/min. Which of the following would best resolve the patient's condition?

A. administer Albuterol sulfate B. diurese the patient C. administer aerosolized alcohol D. administer lactated ringers, IV B. Pulmonary edema and crepitant breath sounds, in conjunction with pitting peripheral edema, is a direct indication for diuresis. These clinical data indicate fluid overload.

A patient in the emergency department (ED) becomes unconscious and has a cardiac rhythm consistent with asystole. The respiratory therapist should IMMEDIATELY

A. administer Atropine B. confirm asystole in a second lead C. begin chest compressions D. administer epinephrine B. The primary treatment for a cardiac rhythm asystole is chest compressions. However, prior to administration of chest compressions the asystole rhythm must be confirmed. This should be done by ensuring the leads are properly placed and that the rhythm can be seen in the second lead.

A patient has received three aerosolized bronchodilator treatments over the past 2 hours with no relief noted. Which of the following may benefit the patient?

A. administer Atropine sulfate (SCH 1000) B. administer cromolyn sodium (Intal) C. administer continuous Albuterol at 1 mg/hr D. administer Atrovent (Ipratropium bromide) with Albute D. A patient who continues to wheeze after a few bronchodilator treatments is in need of further bronchodilator medication. Of the options offered, Atrovent with albuterol is the best option. Continuous bronchodilator therapy would be appropriate but the dose offered is subtherapeutic. For a pediatric patient at least 7 mg per hour is appropriate and 10 mg per hour is required for an adult.

An adult female is unable to keep her lips tightly around the mouthpiece when taking an IPPB treatment with 15 cm H2O of pressure. Which of the following is most appropriate?

A. administer IPPB by mask B. increase sensitivity C. increase inspiratory flow rate D. decrease inspiratory pressure A. If the patient is unable to keep their lips tightly around the mouthpiece during a positive pressure treatment, the best response is to utilize an inflatable facemask. Decreasing inspiratory pressure would undermine the intent of the therapy. Increasing inspiratory flow rate would worsen the problem and increasing sensitivity would likely have no effect.

A 7 year-old patient is still wheezing in spite of multiple aerosolized Albuterol treatments at home. The patient is now in the emergency department (ED). Which of the following should the respiratory therapist do?

A. administer Mucomyst B. administer continuous bronchodilator therapy at 7 mg/hr C. administer cromolyn sodium (Intal) D. administer Atropine sulfate, IV B. A patient who continues to wheeze after a few bronchodilator treatments is in need of further bronchodilator therapy. Of the options offered, continuous bronchodilator therapy at 7 mg per hour is most appropriate. The administration of atropine, cromolyn sodium, or Mucomyst will not help to provide immediate bronchodilation.

Following administration of an aerosol treatment with 2.0 mL of 20% Acetylcysteine (Mucomyst), the patient demonstrates expiratory wheezing bilaterally. Which of the following represents the best action?

A. administer Narcan (naloxone) B. decrease the volume of mucolytic administration to 1.0 cc C. administer Albuterol sulfate by small volume nebulizer D. decrease the strength of future treatments to 10% C. Mucomyst is used for thinning secretions. Unfortunately, it can also cause bronchoconstriction. When this happens, administration of a bronchodilator is appropriate. Often, administering a bronchodilator with the Mucomyst is also indicated.

A patient with a history of asthma reports to the emergency department after self-treating bronchoconstriction for the past 3 days. Periodic relief has been achieved but the patient is in current distress and is wheezing bilaterally. Which of the following treatment actions would be most appropriate?

A. administer Sublimaze, PO B. sildenafil, IV C. continuous aerosolized bronchodilator at 7 mg/hr D. full pulmonary function testing with DLCO measurement C. The patient shown in this question obviously demonstrates lack of response to bronchodilator therapy. Of the options offer, IV methylprednisolone and continuous aerosolized bronchodilators are appropriate.

A patient on long-term supplemental oxygen via a transtracheal oxygen catheter is hospitalized on a general observation floor. The patient complains of shortness of breath and indicates the device is not functioning properly. The respiratory therapist should

A. administer aerosolized Ipratropium Bromide (Atrovent) by mask B. suction the catheter, recheck oxygen saturation C. remove the transtracheal catheter, begin mechanical ventilation D. begin supplemental oxygen delivery by nasal cannula D. When a patient complains of shortness of breath or complains that the oxygen delivery equipment is not working properly, the best first option is to ensure oxygenation by changing to another device and then troubleshooting the problem.

During endotracheal suctioning, a patient becomes agitated, markedly hypertensive, and demonstrates PVCs on the cardiac monitor. The therapist should prepare to

A. administer aerosolized lidocaine 4% B. administer a bolus of intravenous fluids C. discontinue suctioning, report to the physician D. provide maximum supplemental oxygen D. The presence of PVCs during any procedure, especially suctioning, would suggest that the patient has become hypoxic during the procedure and could benefit from maximum supplemental oxygen. This should be provided both before, during, and after the procedure to ensure arterial oxygenation and avoid hypoxemia.

A 48-year-old female completes the several FVL maneuvers. Both the expiratory and inspiratory side of the loops demonstrate a similar inverse pattern that is round in appearance. Which of the following could the respiratory therapist potentially conclude about this pattern?

A. an obstructive pulmonary defect is likely present B. a fixed upper airway obstruction is present C. a restrictive pulmonary defect is likely present D. the shape of the loop demonstrates normal pulmonary function B. In a normal flow volume loop, the inspiratory side of the loop (the topside), is vastly different than the bottom side of the loop. However, when upper loop resembles the bottom part of the loop in shape (this is often called a round loop) it indicates that the patient is having difficulty getting air in AND out. This is most commonly associated with a fixed upper airway obstruction or vocal cord paralysis or cancer.

A patient with cystic fibrosis is receiving VC, SIMV ventilation. The respiratory therapist notices secretions have become thick and tenacious over the last 24 hours. Which of the following would be most helpful in addressing the thickness of secretions?

A. an ultrasonic nebulizer B. aerosolized atropine sulfate C. heated humidity D. a heat-moisture exchanger C. Traditional heated humidity is the most effective way of addressing thick secretions. A heat moisture-exchanger could cause secretions to become thicker. An ultrasonic nebulizer does not provide heat and is not normally associated with mechanical ventilation. Although atropine is known to help improve secretion mobilization, it is inadequate to keep secretions hydrated.

A patient complaining of shortness of breath presents with severe hyperglycemia, marked hyperpnea, and normal oxygen levels. What should the respiratory therapist expect when analyzing an ABG?

A. anemic hypoxemia B. respiratory failure C. metabolic acidosis D. compensated respiratory alkalosis C. Metabolic acidosis results in a significantly low pH, which causes an increased respiratory drive in order to attempt to normalize the pH. High glucose levels are often seen with metabolic acidosis. Oxygenation remains normal.

A left-to-right shunt, due to a leaking septum wall in the heart, is best determined by an increase in mixed venous oxygen saturation. From what location would the respiratory therapist recommend blood extraction for analysis to determine mixed venous oxygen saturation?

A. aorta B. pulmonary artery C. vena cava D. pulmonary vein B. Mixed venous blood, for the assessment of oxygen saturation, is taken from the distal port of the pulmonary artery catheter, or the pulmonary artery.

A patient underdoing a cardiopulmonary stress test has the following data: Baseline Increased workload HR 98/min 122/min BP 124/85 mm Hg 122/85 mm Hg SpO2 98% 95% Which interpretation is most accurate?

A. appropriate cardiac and blood responses B. appropriate cardiac response, inappropriate blood pressure response C. inappropriate cardiac and blood pressure responses D. inappropriate cardiac response, appropriate blood pressure response B. During a cardiopulmonary stress test, when workload is increased, both heart rate and blood pressure should also naturally increase. This is known as normal cardiac and blood pressure responses, respectively. In this case, data shows that only the

A patient underdoing a cardiopulmonary stress test has the following data: Baseline Increased workload HR 98/min 122/min BP 124/85 mm Hg 122/85 mm Hg SpO2 98% 95% Which interpretation is most accurate?

A. appropriate cardiac response, inappropriate blood pressure response B. inappropriate cardiac and blood pressure responses C. inappropriate cardiac response, appropriate blood pressure response D. appropriate cardiac and blood responses A. During a cardiopulmonary stress test, when workload is increased, both heart rate and blood pressure should also naturally increase. This is known as normal cardiac and blood pressure responses, respectively. In this case, data shows that only the HR increased (normal cardiac response). The blood pressure, however, failed to increase and is therefore considered to be an abnormal (inappropriate) blood pressure response.

Coarse crackles are auscultated over both lung fields. This is most consistent with the presence of

A. atelectasis B. mucus plug C. pulmonary secretions D. pleural inflammation C. Course crackles are most commonly associated with pulmonary secretions. Atelectasis would be described as fine crackles and pleural inflammation would most often be described as a friction rub

A radiological report is placed in the medical record that describes the results of a chest radiograph. The report indicates plate-like infiltrates bilaterally. This finding is consistent with

A. atelectasis B. pulmonary edema C. pleural effusion D. pulmonary embolus A. The term platelike infiltrates is associated with atelectasis.

A bronchoscopy is used to diagnose

A. atelectasis. B. emphysema C. idiopathic chronic bronchitis D. malignant processes. D. Bronchoscopy is used to diagnose foreign body obstructions, active bleeding causes, pathogenic involvement (to identify organisms) and cancerous or malignant processes or areas. It may help decrease atelectasis if immobilized mucous plugs are blocking a portion of the lung from expanding, but it does not serve in the diagnosis of atelectasis.

A respiratory therapist reviews the medical record of a patient and notes the HbCO is 10%. This data was collected 1 hour prior and the patient is asymptomatic. From these results, the respiratory therapist can conclude the patient most likely

A. attempted to commit suicide B. has pulmonary fibrosis C. has an exhaust leak into the cabin of the vehicle D. is a heavy smoker D. And HbCO of 10% is much higher than normal and indicates the patient likely smokes tobacco.

While transporting at patient from the emergency department (ED) to the intensive care unit (ICU), the respiratory therapist suspects the oral endotracheal tube has changed position. The quickest way to assess the ET tube position would be to

A. auscultate breath sounds B. obtain a chest radiograph C. visualize diaphragmatic excursion D. analyze end-tidal CO2 A. There are many ways to determine the location or position of the ET tube. The question is asking for the quickest way. Of the options offered, auscultation of breath sounds would provide the most immediate pertinent data. If an option such as examination of equal and bilateral chest rise were offered, that option would be even better because it is visual in nature and therefore quicker.

A patient in the emergency room is demonstrating respiratory difficulty after an 18-foot drop from a roof onto a hard surface. BP is falling to 90/60 mm Hg and the patient's color is grey. HR is 132/min. SpO2 is 96% on 40% oxygen. What further evaluation would be most appropriate?

A. complete blood count B. electroencephalography C. bone density scan D. electrocardiography A.

The respiratory therapist observes a gradual increase in peak inspiratory pressure over the last 24 hours on a patient who is receiving mechanical ventilation. Additionally, the patient has become febrile and is producing purulent pulmonary secretions. The therapist should recommend which of the following?

A. bronchodilator therapy B. sputum gram stain C. exploratory bronchoscopy D. lung tissue biopsy B. The presence of fever and purulent pulmonary secretions suggests the possibility of an infection. A CBC would confirm the presence of an infection but a sputum gram stain would actually identify the class of offending pathogenic organism (gram-positive or gram-negative) Although not offered here, a more appropriate exam would be a sputum C&S which would provide specific organism information and suggest the most appropriate antimicrobial therapy. If a sputum gram stain is done and the class of organism is identified, a general class of antibiotics may be started until more specific pathogenic organism identification is done.

A COPD patient that requires intubation and VC SIMV ventilation has a Mallampati classification of 1. Which of the following methods and/or equipment should the respiratory therapist expect to utilize for oral intubation?

A. bronchoscopic intubation B. bougie device C. direct laryngoscopy D. intubation with a video-assist device C. A Mallampati classification is a 4 point scale that relates to intubation difficulty. (Four points being complex and one point being considered non-complex). In this question, Mallampati classification score is a 1 and therefore is considered to be non-complex. In such a case, normal direct laryngoscopy may be used.

It is suspected that a patient is bleeding from a site within the pulmonary tree. Which of the following would be most appropriate to locate the potential bleeding site?

A. bronchoscopy B. CT scan of the chest C. percutaneous biopsy D. V/Q scan A. A bronchoscopy is the best procedure to use when a specific bleeding site needs to be located within the pulmonary tree. Although the bronchoscope is unable to penetrate the distal portions of the tree, most bleeding will occur higher up and can be visualized through the bronchoscope. Additionally, once the bleeding site is observed, measures can be taken with the bronchoscope to stop or minimize bleeding. Some of these measures include instillation of epinephrine, compression of the site with the scope, or placement of a Fogarty catheter, which is a long-term tamponade of the bleeding site.

A high-pitched noise heard on inspiration immediately following extubation is likely the result of

A. bronchospasm B. premature extubation C. upper airway swelling D. improper extubation technique C. High-pitched sounds are generally considered to be stridor, a result of subglotic swelling, which is common following extubation.

Which of the following most effectively makes up the humidity deficit for a patient who is orally intubated and receiving mechanical ventilation?

A. bubble humidifier B. centrifugal nebulizer C. HME D. heated humidity D. A patient who is intubated cannot provide 100% humidity to their lungs because the ET tube bypasses the natural humidification processes of the body. In such case a humidification device is needed to make up the entire humidity deficit. Of the devices listed in the options, only a heated humidifier can accomplish this. A bubble humidifier is used with a nasal cannula and an HME device does not provide sufficient humidity. A centrifugal nebulizer is not used in conjunction with a mechanical ventilator.

A patient, who is on a 3-chamber chest-tube drainage system, is receiving volume controlled ventilation in the assist/control mode. The low return volume alarm begins to sound. Which of the following should the respiratory therapist troubleshoot to determine the cause of the alarm?

A. bubbling in the water seal chamber B. excess condensate in the circuit C. secretions in the airway D. loose circuit connection A. The presence of a low-return-volume alarm for a patient who also has a chest tube drainage system in place, is an indication that volume may be being lost to a perforation in the lung tissue and out through the chest tube drainage system. If this is happening, excessive bubbling in the water seal chamber may be observed. Consequently, to determine the source of the problem examining the ventilator circuit connections and looking for excessive bubbling in the water seal compartment are two methods that will help find the source of the problem.

While analyzing the oxygen in an oxygen hood with a galvanic-type analyzer, the respiratory therapist observes the analyzed percentage is 50% while the air/oxygen blender is set at 30%. Which of the following will most likely correct the problem?

A. calibrate the air-oxygen blender B. replace the fuel cell C. replace the air-oxygen blender D. perform a high-low calibration of the analyzer D. When a galvanic-type oxygen analyzer reads incorrectly, the solution is to recalibrate the analyzer to a high and low point. For instance, calibration should be done at 100% oxygen and at 21% oxygen. If the oxygen analyzer fails to produce any reading at all (the needle fails to move) the problem is the power source. It is not a calibration problem. In such a case the probe, also called the fuel cell, serves as the power to the unit and must be replaced.

A 65-year-old patient with COPD is receiving a routine assessment in a clinic. The respiratory therapist notes the patient is not looking well. SpO2 is 84%, RR 30/min, marked accessory muscle use is noted, and temperature is 38.8 deg C (102 deg F). The therapist should

A. call a code. B. administer oxygen by non-rebreather mask. C. obtain an arterial blood gas. D. activate the medical emergency team. D.

Placement of a pulmonary artery catheter is associated with which of the following most common complications?

A. cardiac arrhythmias B. internal bleeding C. hypotension D. pulmonic valve damage A. Several complications may arise from the placement of a pulmonary artery catheter, otherwise called a Swan-Ganz catheter. The development of cardiac arrhythmias is the most common complication of the options offered. Another serious complication is perforation of a vessel or cardiac muscle during the insertion.

Which of the following clinical outcomes is a side effect of inhaled nitric oxide?

A. cardiac artery hardening B. contraction of smooth muscle tissue C. systemic hypotension D. elevation of leukocyte count C. One of the side effects associated with inhaled nitric oxide is systemic hypotension. The purpose of nitric oxide is to dilate pulmonary vessels and relieve pulmonary hypertension. However, it also decreases systemic blood pressure. Therefore, systemic blood pressure must be monitored closely in patients who are taking inhaled nitric oxide.

A patient is being evaluated for idiopathic dyspnea that occurs unpredictably at rest and during exertion. The following pulmonary function test reveals: Pre-bronchodilator Post-bronchodilator FEV1 2.6 L 2.6 L FVC 3.2 L 3.1 L Chest radiograph and ECG are normal. Which of the following should the therapist recommend NEXT?

A. cardiopulmonary stress testing B. pulmonary angiography C. analysis of CSF D. V/Q scan A. A patient who complains of periodic dyspnea may benefit from pulmonary function testing. However, these pulmonary function results do not provide helpful information or show a possible reason for the dyspnea. Therefore, a greater examination is indicated. Cardiopulmonary stress testing can determine if the heart is the source of the problem. Analysis of cerebral spinal fluid, a VQ scan, and pulmonary angiography are specific tests that confirm diagnoses in other areas and are premature in this case.

A patient with a history of cardiac insufficiency is participating in a cardiac stress test on a treadmill for the purpose of identifying target and maximal workload. The following data is observed: Starting base level After 20% increase in workload HR 90/min 122/min BP 118/90 mm Hg 139/95 mm Hg SpO2 95% 96% The respiratory therapist should suggest

A. cessation of the exam, monitor the patient closely. B. continuation of the test. C. a decrease in workload back to baseline, continue and monitor D. administration of supplemental oxygen and continue B.

The respiratory therapist is having difficulty getting a reading from a galvanic-type oxygen analyzer when attempting to analyze the oxygen percentage on the inspiratory limb of a ventilator circuit. The therapist should do which of the following?

A. change the electrolyte solution B. change the batteries C. analyze oxygen on the expiratory limb of the circuit D. change the fuel cell D. When a galvanic-type oxygen analyzer fails to produce a reading it is likely a problem with the power source. These type of analyzers do not have batteries. The electrode is in essence a battery. So, the solution is to change the electrode. The electrode is also called the fuel cell.

A polargraphic oxygen analyzer shows no reading even when the power switch is in the ON position. The most likely remedy is which of the following?

A. change the electrolyte solution B. change the batteries C. replace the electrode D. change the fuel cell B. If a polargraphic oxygen analyzer is not working, even though the power switch is in the on position, it is likely that the batteries need to be replaced. Polargraphic analyzers have batteries but galvanic type analyzers do not. In a galvanic type analyzer the electrode is the battery (also called the fuel cell).

A 90-kg male is receiving mechanical ventilation through a 7.5 ET tube. Bronchial hygiene has been difficult due to thick secretions. The patient is being suctioned with a 10 Fr. catheter at a pressure of 110 mmHg. Which of the following can be done to increase the effectiveness of suctioning:

A. change to a 12 Fr. catheter B. change to an 8.5 ET tube and 12 Fr. catheter C. administer sodium bicarbonate D. increase suction pressure to 130 mmHg B. When having difficulty with bronchial hygiene, there are three things that one should check. In this order - appropriate suction catheter size, adequate suction pressure, and appropriate suction time. The correct suction catheter size is determined by taking the ET tube size and dividing it by 2. You then take that number and multiply it by three to get the catheter size in French units. In this case the best option is to increase the ET tube size so that you may use a larger suction catheter. Generally speaking a 10 French catheter size is too small for an adult.

A patient receiving heliox therapy at 70%/30% mixture by non-rebreathing mask has the following arterial blood gas results: pH 7.35 PaCO2 45 mmHg PaO2 110 mmHg HCO3- 24 mEq/L BE 0 mEq/L Which of the following actions is appropriate?

A. change to an 80%20% heliox mixture B. switch to a partial rebreathing mask C. discontinue heliox therapy D. change to a heliox mixture of 60%/40% A. This patient is receiving 30% oxygen though a Heliox mixture. One look at the blood gas shows a PaO2 of 110 mmHg. This over oxygenation state indicates a need to lower supplemental oxygen. Changing to an 80-20% Heliox mixture is most appropriate

A 24-year-old patient is brought to the emergency department after being retrieved from a 3-alarm residential fire and is currently receiving oxygen by nasal cannula at 7 L/min. SpO2 is 100%. The patient was found to be alert but confused and lost when discovered. Arterial blood gas results are as follows: pH 7.31 PaCO2 29 torr PaO2 210 torr HCO3- 15 mEq/L BE -10 mEq/L SaO2 99% O2Hb 77% COHb 22% Hb 12.8 g/dL The respiratory therapist should recommend

A. changing to a nonrebreather mask at 15 L/min B. changing to an air-entrainment mask, FIO2 0.50 C. switching to a humidified high-flow nasal cannula D. decreasing oxygen flow rate to 5 L/min NC A. Although the SpO2 is 100%, the environment from which the patient was retrieved was likely full of smoke and carbon monoxide. This makes pulse oximetry data unreliable. Oxygenation should be addressed by administering 100% which will also address potential carbon monoxide poisoning.

A 24-year-old patient is brought to the emergency department after being retrieved from a 3-alarm residential fire and is currently receiving oxygen by nasal cannula at 7 L/min. SpO2 is 100%. The patient was found to be alert but confused and lost when discovered. Arterial blood gas results are as follows: pH 7.31 PaCO2 29 torr PaO2 210 torr HCO3- 15 mEq/L BE -10 mEq/L SaO2 99% O2Hb 77% COHb 22% Hb 12.8 g/dL The respiratory therapist should recommend

A. changing to an air-entrainment mask, FIO2 0.50 B. changing to a nonrebreather mask at 15 L/min C. switching to a humidified high-flow nasal cannula D. decreasing oxygen flow rate to 5 L/min NC B. Although the SpO2 is 100%, the environment from which the patient was retrieved was likely full of smoke and carbon monoxide. This makes pulse oximetry data unreliable. Oxygenation should be addressed by administering 100% which will also address potential carbon monoxide poisoning.

A 32-year-old male who weighs 60 kg (143 lb) is receiving VC SIMV ventilation following the surgical repair of a hernia with slow emergence from sedation. The following data is observed: pH 7.48 PaCO2 32 torr PaO2 110 torr HCO3- 23 mEq/L BE -1 mEq/L FIO2 0.45 Mandatory rate 12 Total rate 12 Tidal volume 650 mL The respiratory therapist should

A. check autoPEEP B. decrease tidal volume to 450 mL C. decrease FIO2 to 0.21 D. increase mandatory rate to 14 B. The blood gas demonstrates a small amount of hyperventilation. Additionally, based upon the patient's predicted body weight, a tidal volume of 650 mL is too much. Thus, lowering the tidal volume to something more within range will help raise arterial CO2 to the proper level.

A 68-year-old male patient has the following pulmonary function values: Fev1/FVC% 79% FVC 62% of predicted FEF25-75 81% of predicted FEF200-1200 84% of predicted Which of the following could represent the patient's diagnosis?

A. chronic bronchitis B. bronchiectasis C. idiopathic pulmonary fibrosis D. chronic asthma C. When looking at pulmonary function data, the primary purpose is to differentiate between an obstructive defect and a restrictive effect. To determine if the patient is obstructive, we must look at flows. To determine if they are restrictive we must look at volumes. In this case the Fev1/FVC% is 79%. Most pulmonary function data requires 80% of predicted or higher to be considered normal. With Fev1/FVC%, 75% is considered normal. This is because it is not a predicted value but the actual value of the ratio. In other words, a person should be able to blow out 75% of their forced vital capacity in one second. This data indicates that the patient is not obstructive. Further examination shows a forced vital capacity of 62% of predicted. This is consistent with a restrictive pulmonary defect. Examining the options reveals three diseases that are obstructive in nature and only one that is restrictive - idiopathic pulmonary fibrosis.

Pulmonary function testing is done on a patient with a 65-pack-year history of smoking. The following pulmonary function data is recorded: Percent of Pred Actual value Fev1.0 58% FEF200-1200 75% FEF25-75 52% SVC 88% FVC 81% DLCO 18 CO/min/mmHg Which of the following most likely represents the patient's condition?

A. chronic bronchitis B. emphysema C. cystic fibrosis D. kyphosis B. These pulmonary function results show that the patient obstructed. This is indicated because the FEV1 is less than 80% of predicted. The SVC and the FVC are normal, or about 80% of predicted. Therefore, the patient can not be restrictive. When we look at the answers we see there are three obstructive diseases to choose from. When this happens we need more information. When we return to the pulmonary function test results we further see that the DLCO is reduced and is below normal. Normal DLCO is 25 CO/min/mmHg. That means that less than 80% of predicted would be anything less than 20 CO/min/mmHg. In this case, the DLCO is 18 CO/min/mmHg. There is only one obstructive disease that is associated with the poor DLCO - pulmonary emphysema.

The physician requests patient education for a newly diagnosed COPD patient being discharged home with continuous supplemental oxygen and inhalers. Which of the following are NOT indicated for the teaching plan?

A. cleaning the MDI spacer B. increasing quality of life C. recognizing signs of infection D. achieving normal lung functions D. It is not realistic for a respiratory therapist to expect to achieve normal lung functions within COPD patients, therefore patient instructions should not include such false hopes.

A patient with a tracheostomy is receiving supplemental oxygen via tracheostomy collar connected to a large volume nebulizer set at 40%. The respiratory therapist analyzes the FIO2 at the tracheostomy collar with a galvanic fuel cell analyzer. The analysis shows the FIO2 to be 55%. Which of the following could be the cause of the increase in FIO2?

A. clogged jet orifice in the nebulizer B. calibration error in the galvanic fuel cell C. the aerosol tubing is too short D. too much water in the circuit D. When administering oxygen by any device that has a venturi mechanism, back pressure on the venturi will slow the speed of gas, decrease room air entrainment, and result in an increase in FIO2. Of the options offered, only excess water in the tubing would cause this type of back pressure.

Which of the following is most likely the cause of pressure dampening in a pulmonary artery waveform obtained from a Swan-Ganz catheter?

A. clot in the distal port B. catheter is advanced too far C. catheter is not advanced far enough D. 90-degree kink in the catheter A. When pressure dampening is present in a pulmonary artery catheter, often called a Swan-Ganz catheter, there are two potential causes: blood clot in the distal port, and inadvertent occlusion of the distal port by the wall of the pulmonary artery. A potential clot may be resolved by first aspirating from that port and then flushing. If that does not resolve the problem the most likely cause is that the pulmonary artery catheter's distal port is being occluded by the wall of the pulmonary artery. This can be resolved by twisting the catheter. Withdrawing or advancing the catheter is not an appropriate method of solving pressure dampening.

A patient is recovering from abdominal surgery. During postoperative sustained maximal inspiration (SMI) therapy with a volume-type incentive spirometer, the respiratory therapist notices that patient is having difficulty staying awake and following directions. The therapist should recommend?

A. coach more aggressively to get the patient to wake up and follow directions B. IPPB hyperinflation therapy with an inflatable mask C. discontinue therapy until the patient is awake and alert D. administer narcotic-reversing medication B. A patient who is unable to perform a SMI (sustained maximal inspiration therapy) postoperatively due to sedation and sleepiness may be given IPPB therapy to accomplish the same objective. The purpose of these therapies is to ensure inflation and recruitment of alveoli which will prevent postoperative complications, including infection.

Which of the following is the best indicator of the adequacy of alveolar ventilation?

A. color B. PaCO2 from an arterial blood gas analysis C. respiratory rate D. tidal volume B. Adequate alveolar ventilation is best manifest by the exhalation of CO2 observed on an arterial blood gas. Tidal volume, respiratory rate, or color are not good indicators of the adequacy of alveolar ventilation.

Which of the following data cannot be determined from a comatose patient?

A. color B. SPO2 C. tidal volume D. dysphagia D. A comatose patient cannot report symptoms. Of the options offered dysphagia is the only symptom. Color, oxygen saturation, and tidal volume can all be measured by the practitioner regardless of the alertness and cooperation of the patient.

Arterial blood gases on a patient in the emergency department are as follows: pH 7.29 PaCO2 51 mmHg PaO2 70 mmHg HCO3- 25 mEq/L BE +4 mEq/L Which of the following represents an accurate interpretation of these results?

A. compensated metabolic alkalosis B. ketoacidosis with hypoxemia C. uncompensated metabolic acidosis with moderate hypoxemia D. uncompensated respiratory acidosis with mild hypoxemia D. A look at the CO2 reveals an elevation. When we glance at the pH we see that there is no compensation, thus making the problem acute uncompensated respiratory acidosis. Additionally, we see the patient is mildly hypoxic. This, mild hypoxemia and respiratory acidosis is the correct interpretation for this patient.

A 70-kg (154-lb) male patient is receiving mechanical ventilation. After increasing PEEP, the following data is available: Arterial blood gases reveal pH 7.32 PaCO2 47 mmHg PaO2 80 mmHg HCO3- 26 mEq/L BE +2 mEq/L C.I. 2.5 PAP 14 mmHg CVP 4 mmHg PCWP 8 mmHg The respiratory therapist should do which of the following?

A. decrease FIO2 B. remove 100 mL of deadspace C. decrease mandatory rate D. administer sodium bicarbonate B. In this problem the arterial blood gas reveals that the patient is under ventilating, or that the CO2 is too high. There are three ways to correct this. One way is to increase tidal volume, which is not offered in this case. Another way is to increase the rate. The option listed is to decrease the rate. That is a change in the wrong direction. The third way to decrease CO2 is to remove dead space. In this case that is the best answer. All other answers either cause a change in the wrong direction or are not related to the problem that should be addressed first.

A 65-kg (143-lb) male patient in the intensive care unit is being monitored by a pulmonary artery catheter. Current ventilator settings and clinical data include the following: Mode Assist/control Mandatory rate 16 Tidal volume 450 mL FIO2 0.70 PEEP 18 cm H2O pH 7.35 PaCO2 45 mm Hg PaO2 56 mm Hg HCO3- 26 mEq/L BE +2 mEq/L PAP 20 mm Hg CVP 5 mm Hg PCWP 12 mm Hg CI 1.8 L/min/m2 The respiratory therapist should recommend

A. decrease FIO2 to 0.6 B. switch to pressure control ventilation C. increase FIO2 to 1.0 D. increase PEEP to 22 cmH2O C. Arterial blood gas data indicates appropriate ventilation but also shows hypoxemia. To correct this we must raise either FIO2 or PEEP. Because FIO2 is already above 60% the best option is to increase PEEP. However, closer examination of the hemodynamic data shows cardiac instability. This is derived from an elevated PCWP and a decreased cardiac output. In this case, cardiac index is given but cardiac output may be estimated by doubling cardiac index. This would give us a cardiac output of 3.6 L/min., well below the normal 4 to 8 L. If cardiac instability is present an increase in PEEP is contraindicated. Increasing FIO2, therefore, is the only option.

A patient is receiving non-invasive positive pressure ventilation by mask with an IPAP of 12 cmH2O and an EPAP of 4 cmH2O. Corresponding blood gas data is as follows: pH 7.49 PaCO2 30 mmHg PaO2 87 mmHg HCO3- 24 mEq/L BE -0 mEq/L Which adjustment is most appropriate?

A. decrease IPAP and EPAP B. increase both IPAP and EPAP C. decrease IPAP D. increase EPAP C. In this blood gas the patient is over ventilating. The primary BiPAP control that affects CO2 is IPAP. A reduction in IPAP will decrease the distance between the IPAP and EPAP settings, which will result in less ventilation and will raise PaCO2

A patient who is being mechanically ventilated in the intensive care unit had a complete pulmonary function test one-month prior. SVC was 75% of predicted. FRC was 70% of predicted. The patient is now experiencing an increase in A-a gradient. The use of mechanical PEEP in this patient would

A. decrease MAP. B. increase FRC. C. not affect lung capacities. D. decrease oxygenation. B. The use of mechanical PEEP would increase functional residual capacity (FRC).

An 84-kg (185-lb) 6-ft 1-in (185-cm) male is receiving mechanical ventilation by a volume-controlled ventilator in the assist/control mode on the following settings: Mandatory rate 16 VT 600 mL FIO2 0.5 PEEP 10 cmH2O ABGs pH 7.46 PaCO2 34 mmHg PO2 100 mmHg HCO3- 24 mEq/L BE 0 mEq/L Which of the following represents the most appropriate action?

A. decrease PEEP B. add 50 mL deadspace C. decrease rate to 10 D. decrease VT to 500 mL B. In this problem the CO2 is slightly low. When we need to raise CO2 there are three choices - decreasing tidal volume, decreasing rate, or adding dead space. In this case, decreasing the rate from 16 to 10 to change CO2 by only a few points is too drastic. A tidal volume of 500 mL is less than the patient's range based on their predicted body weight. The best option is to add 50 mL of dead space.

A 75-kg (165-lb) male is receiving mechanical ventilation by a volume-controlled ventilator in the assist/control mode on the following settings: Mandatory rate 14 VT 400 mL FIO2 0.4 PEEP 8 cmH2O ABG results: pH 7.47 PaCO2 33 mm Hg PaO2 97 mm Hg HCO3- 25 mEq/L BE +1 mEq/L Which of the following represents the most appropriate action?

A. decrease VT to 300 mL B. increase rate C. decrease PEEP to 5 cmH2O D. add 50 mL deadspace D. When looking at the arterial blood gases the only problem noted is a decreased CO2. The patient is hyperventilating slightly. To correct this problem you can either decrease rate, decrease tidal volume, or add deadspace. In this case, because CO2 is only 2 mmHg off from its target, the best option would be to add 50 mL of deadspace. The option to decrease the tidal volume to 300 mL is wrong because it places the tidal volume below the patient's tidal volume range based on their ideal body weight.

A 70-kg (154-lb) female patient with myasthenia gravis is receiving mechanical ventilation by a volume-controlled ventilator in the assist/control mode on the following settings: Mandatory rate 10/min VT 600 mL FIO2 0.5 ABGs pH 7.47 PaCO2 33 mmHg PO2 81 mmHg HCO3- 23 mEq/L BE -1 mEq/L Which of the following represents the most appropriate action?

A. decrease VT to 500 mL B. decrease rate to 8 C. increase rate to 12 D. add 50 mL of deadspace D. The arterial blood gas reveals that the CO2 is slightly low. There are three ways in which we can increase CO2 when the patient is on a ventilator - we can decrease rate, decreased tidal volume, or add dead space. The best option is to add 50 mL of dead space because it will change CO2 by a very small increment which is needed. Increasing the rate would further decrease CO2, which is a move in the wrong direction. Decreasing rate from 10 to 8 is too drastic. Decreasing tidal volume to 500 mL would place the tidal volume lower than the patient's range based on their ideal body weight.

A respiratory therapist is asked to interpret the following arterial blood gas for an 11-year old patient on a mechanical ventilator: pH 7.21 PaCO2 56 mm Hg PaO2 54 mm Hg HCO3- 26 mEq/L BE -2 mEq/L The patient is being ventilated with an adult circuit and an HME is in-line. What should the therapist recommend?

A. decrease deadspace B. change to a pediatric circuit C. increase tidal volume D. replace HME with a heated-humidification system C. This patient is both hypoventilating and hypo oxygenating. Of these two problems hypoventilation should be addressed first. This is because fixing ventilation may also relieve hypoxemia. To increase ventilation either tidal volume or mandatory rate may be increased. Removing deadspace will also decrease CO2 but is usually insufficient when CO2 is more than 4 mm Hg off target. The best option would be to increase mandatory rate but this option is not offered. Therefore, the second best option is to increase tidal volume.

A patient receiving 0.5 mL Albuterol by small volume nebulizer complains of tingling fingers, nervousness, and heart palpitations. The therapist should

A. decrease dosage to 0.15 mL B. have the patient pause at the top of each breath C. instruct the patient in performing shallow inspirations D. reduce future treatment dosage to 0.25 mL D. When a patient is experiencing an adverse reaction from a breathing treatment or any other procedure, the first action should be to stop therapy. However, this is not offered in this question. The next option would be to modify the therapy to accomplish the same objective. In this case the 0.5 mL of albuterol may be reduced to another acceptable adult dosage, 0.25 mL. Keep in mind that you can reduce a dosage only if the new dosage is still in the acceptable therpeutic range. For instance, if the patient was receiving 0.25 mL of albuterol, it would be unacceptable to cut that dose in half. In this case, it is acceptable because the adult therapeutic range of albuterol is 0.25-0.5 mL.

The respiratory therapist has discovered the development of autoPEEP on a patient due to decreased dynamic compliance. Which of the following should the respiratory therapist do?

A. decrease expiratory time B. decrease inspiratory flow C. increase the I:E ratio D. decrease mandatory rate D. AutoPEEP is a condition whereby gases become trapped in the lungs due to an insufficient exhalation time before the next mechanical breath begins. AutoPEEP may be reduced by either decreasing the mandatory rate, which allows more time for exhalation on each cycle, or by increasing inspiratory flow which decreases inspiratory time and thereby lengthens expiratory time. In truth, this is primarily dealt with by increasing the flow. However, in this question the only suitable option is to decrease the mandatory rate. Of course, a decrease in the rate would result in a decrease in alveolar ventilation which must be compensated for in other ways.

A patient undergoes a change in PEEP from 15 to 20 cm H2O. Which of the following would indicate an adverse reaction to the change in PEEP?

A. decrease in C(a-v)O2 B. elevated PCWP, decreased C.I. C. elevated systolic BP, decreased diastolic BP D. elevated CVP, decreased PAP, normal C.O. B. As PEEP is increased, the patient is likely to experience decreased venous return and degradation in hemodynamic pressures. Of the options listed, an elevated PCWP in conjunction with a decreased cardiac index (which is another way of looking at cardiac output), is the most likely outcome. Additionally, an elevated pulmonary capillary wedge pressure in the presence of decreased cardiac output is indicative of our left heart problem. Although the actual source of the problem may not be the left cardiac muscle, excessive PEEP takes on this appearance.

Blood gas results for a patient receiving mechanical ventilation with an FIO2 1.0 and PEEP of 25 cm H2O are as follows: pH 7.37 PaCO2 43 torr PaO2 110 torr HCO3- 24 mEq/L BE -1 mEq/L The respiratory therapist should recommend

A. decrease in PEEP B. performance of an optimal PEEP study C. decrease in FIO2 D. administration of potassium chloride (KCL) C. Arterial blood gas results reveal adequate ventilation but over oxygenation. To correct this either PEEP or FIO2 may be decreased. When oxygen percentage is greater than 60%, FIO2 should be decreased first to below 60%. Then, PEEP may be decreased.

Which of the following would result in an increase in mean airway pressure (MAP) on a time-cycled ventilator?

A. decrease in frequency B. decrease in peak inspiratory pressure C. decrease in expiratory time D. decrease in inspiratory time C. On a time-cycled ventilator, because the inspiratory time is set, a reduction in expiratory time will result in an increase in rate. The primary and most significant cause of increased mean airway pressure is an increase in rate. Thus mean airway pressure on a time cycle ventilator would be increased most significantly by a decrease in expiratory time.

A 73-kg (161-lb), 178-cm (5-ft 10-in) male patient is receiving PC, A/C ventilation at the following parameters: FIO2 0.60 IP 42 cm H2O f 11/min VT (exhaled) 780 mL PEEP 5 cm H2O I-time 1:2.4 ABGs pH 7.53 PaCO2 27 torr PaO2 102 torr HCO3- 22 mEq/L BE +7 mEq/L A proper change in the ventilator parameters should include a

A. decrease in inspiratory flow rate. B. reduction in FIO2. C. decrease in inspiratory pressure. D. reduction in the mandatory rate (f) C. Q

Which of the following would decrease autoPEEP?

A. decrease inspiratory time B. decrease expiratory time C. decrease inspiratory flow rate D. increase rate A. utoPEEP is caused by insufficient time for exhalation before the inspiratory phase begins on a mechanical ventilator. When this happens air is trapped in the alveoli and autoPEEP develops. To decrease autoPEEP, more time must be allowed for exhalation. This can best be accomplished by decreasing inspiratory time, which is done by increasing inspiratory flow.

The physician orders a change in I:E ratio from 1:2 to 1:4 for a COPD patient receiving volume-cycled mechanical ventilation. Which of the following accommodates the physician's request if the mandatory rate is not changed?

A. decrease inspiratory time B. increase inspiratory time C. decrease inspiratory flow rate D. decrease expiratory time D. To decrease the I:E ratio, expiratory time must be increased by decreasing inspiratory time. This is done by decreasing inspiratory flow rate.

A 60-kg (132-lb) female patient is being weaned from mechanical ventilation. The following data and ventilator settings are available: Mode SIMV Mandatory rate 8 Total rate 24 VT 450 mL VT (spont) 200 mL FIO2 0.4 PEEP 5 cm H2O PS 5 cm H2O Which of the following represents an appropriate recommendation?

A. decrease mandatory rate B. discontinue mechanical ventilation C. increase pressure support D. increase mandatory rate C. This patient is in the beginning stages of ventilator weaning. When a patient is weaning it is most helpful to observe the total rate compared to the set rate, and the spontaneous tidal volume compared to the set tidal volume. In this case the patient has a set rate of 8/min but his breathing at a total rate of 24/min. This rate is excessive. Further examination reveals the patient's tidal volume is 200 mL, which is far less than the 5 mL per kilogram tidal volume needed to sustain life. Together, the high rate and low spontaneous tidal volume show an increased work of breathing. Ultimately, the patient will be unable to sustain independent ventilation. Pressure support is a way that we can assist the patient by increasing their spontaneous tidal volume which will cause a lower respiratory rate and preserve the patient's energy.

A patient who is receiving volume-controlled ventilation is showing signs of ventilatory distress. The low return volume alarm is sounding and there is excessive bubbling found in the water-seal compartment of a three-chamber chest tube drainage system. The respiratory therapist should first

A. decrease tidal volume B. decrease suction pressure at the wall C. clamp the chest tube at the patient's chest D. clamp the tubing between the collection chamber and the water-seal chamber C. When these two conditions are observed, low return volume and excessive bubbling in the water seal compartment, the primary concern is that there is a perforation in the lung tissue. However, the excessive bubbling may be coming from a leak in the chest tubes between the patient and the collection chambers. To determine if this is the case, one starts by clamping the chest tubes at the patient's chest and observing for a decrease in bubbling in the water seal compartment. If bubbling does not decrease, the clamp is moved towards the chambers to determine if there is a leak in other parts of the tubing. However, if bubbling stops or decreases when the clamp is placed on the tubes where they exit the patient's body, the leak must be inside the patient and would suggest a perforated lung.

A patient is receiving volume-controlled mechanical ventilation on the following settings: Mode A/C Mandatory rate 16 FIO2 0.5 Tidal volume 600 mL PEEP 5 cm H2O Insp flow rate 45 L/min The following ventilator graphic is associated with the current state of ventilation. Which of the following should the respiratory therapist recommend?

A. decrease tidal volume B. increase PEEP C. increase inspiratory flow rate D. switch to SIMV mode B. The key to this question is not to focus on oxygenation but rather the shape of the pressure-volume ventilator graphic. In a normal graphic, pressure and volume should increase at the same time. In this case, volume is increasing but pressure is not rising. This is shown by the flatness of the "football". This may be solved by increasing PEEP until pressure rises immediately with volume.

Which of the following will result in an increase in mean airway pressure for a patient receiving volume-controlled ventilation?

A. decreased I:E ratio B. increase inspiratory flow rate C. increase in tidal volume D. decrease in mandatory rate C. Mean airway pressure increases most significantly with an increase in mandatory rate on a ventilator. The second most significant cause of increased mean airway pressure is an increase in tidal volume. Increasing inspiratory flow rate, decreasing mandatory rate, and decreasing inspiratory time will all cause a decrease in mean airway pressure.

A patient is receiving VC AC ventilation with the following data: 3 PM 5 PM Plateau pressure (cm H2O) 33 33 Peak pressure (cm H2O) 39 46 VT (mL) 500 500 This data indicates

A. decreased static compliance B. development of autoPEEP C. increase airway resistance D. pulmonary overdistension C. This data shows increasing peak pressures while plateau pressures are remaining constant. This combination suggests that the patient does not experiencing a change in pulmonary compliance but rather decreasing dynamic compliance. This can be caused by many things, including secretions in the airway, an occluded ET tube, or airway resistance through bronchoconstriction, among others.

A 175-cm (5-ft, 9-in), 71-kg (156-lb) male patient with ARDS has the following ABGs while receiving PC, A/C ventilatory support: pH 7.46 PaCO2 34 mm Hg PaO2 60 mm Hg HCO3- 25 mEq/L BE +1 mEq/L Ventilator settings: FIO2 0.6 PEEP 15 cm H2O VT(exhaled) 650 mL IP 38 cm H2O IT% 33% (f) 14/min The patient is resting comfortably. The respiratory therapist should recommend

A. decreasing PEEP. B. reducing inspiratory pressure. C. reducing the rate. D. increasing FIO2. B.

Which of the following would indicate an increasing cardiac output?

A. decreasing alveolar deadspace B. decreasing C(a-v)O2 C. decreasing QS/QT D. increasing A-aDO2 B. Of the options listed, the gradient between the arterial content and venous content - C(a-v)O2 - is most pertinent. There is an inverse relationship between cardiac output and the C(a-v)O2. The larger the gradient, the lower the cardiac output. The lower the gradient, the more likely cardiac output is increasing.

Which of the following would be an indicator for the use of pressure control mechanical ventilation?

A. decreasing plateau pressure below 30 cm H2O B. pulmonary emphysema C. increasing compliance D. peak inspiratory pressure in excess of 50 cm H2O D. The use of pressure-control ventilation is primarily indicated when peak inspiratory pressures are in excess of 50 cmH2O. This, however, is only true if the increased peak pressures are a result of decreasing pulmonary compliance and are not caused by temporary conditions such as excess secretions in the airways or bronchoconstriction

The following information is available on the ventilator flow sheet of a 55-year old patient who is 3 days post appendectomy. Following the procedure, the patient sustained a pneumothorax and has been recovering in the intensive care unit. 8:00 AM 10:00 AM 12:00 PM Peak Pressure (cm H2O) 25 35 36 Plateau Pressure (cm H2O) 18 23 22 PEEP (cm H2O) 5 10 10 VT (mL) 500 500 500 The therapist can conclude which of the following is present?

A. decreasing pulmonary compliance, increasing airway resistance B. pulmonary stiffening, decreasing dynamic compliance C. increased pulmonary elastance D. increasing airway resistance, no change in pulmonary compliance D. In this data, increased airway resistance is manifested by an increase in peak pressure. It is tempting to assume decreased pulmonary compliance because plateau pressure is also rising. However when one looks closely, it is noted that PEEP has undergone a change during the study, which accounts for the change in plateau pressure. This will also accordingly change the peak inspiratory pressure but this data shows an increase greater than the amount of increase in the PEEP and therefore airway resistance is increasing.

A decreasing A-aDO2 is mostly closely related to which of the following?

A. decreasing pulmonary shunting B. increasing venous admixture C. hypoxemia D. increased atelectasis A. A widening alveolar-arterial oxygen gradient (A-a gradient) is associated with an increase in pulmonary shunting. Thus, as the gradient decreases, a decrease in pulmonary shunting must also be ocurring.

A patient receiving mechanical ventilatory support has the following capnographic waveform obtained from an infrared device. Which of the following is most consistent with this data?

A. defective infrared sensor B. hypoventilation C. condensate on the infrared sensor D. deadspace disease D. This saw-tooth pattern seen on the capnograph is associated with dead space disease. Notice the low end-tidal CO2 and the rapid rate.

Which of the following must be done to transition a patient with a fenestrated tracheostomy tube, who is receiving positive pressure ventilation, to a speaking configuration?

A. deflate the cuff, install the cap B. sever the pilot valve line C. insert the inner cannula D. change the tracheostomy tube A. A fenestrated tracheostomy tube is used to both provide positive pressure ventilation and to allow the patient to speak when temporarily off ventilatory support. The fenestrated tube comes with an inner cannula and has a cap or a button. These parts are used to configure the tube for speech or positive pressure ventilation. The first step to transition to a positive pressure ventilation configuration is to remove the button or cap. To transition to a speaking configuration the inner cannula must be removed, the cuff must be deflated, and the button must be placed on the tube. A hole in the outer cannula allows air to flow through the tube and pass over the vocal folds, allowing the patient to speak.

A respiratory therapist is alerted by a high-pressure ventilator alarm. Which of the following could be a cause of the alarm?

A. depleted humidifier reservoir B. dislodged temperature probe in the ventilator circuit C. increased lung compliance D. a pneumothorax D.

What is the primary purpose for a nitrogen washout test?

A. determine closing volume B. directly measure TLC C. determine FRC D. evaluate evenness of pulmonary gas distribution C. A nitrogen washout test is used to determine three different lung volumes: TLC, RV, and FRC. For the NBCR exam the most important of those volumes is the FRC. Thus, when asked what a nitrogen washout test is for (or helium dilution test), the correct answer is FRC.

The following data is observed on a patient receiving VC A/C ventilation: PEEP 12 cm H2O FIO2 0.60 VT 550 mL Mandatory rate 12 Laboratory data: SpO2 90% PvO2 29 mm Hg PaO2 70 mm Hg

A. determine current cardiac index B. increase FIO2 to 0.80 C. calculate PVR D. assess the A-aDO2 to determine treatment A. This data shows a remarkable difference between arterial oxygenation and venous oxygenation as listed in the PaO2 and PvO2, respectively. Normally, venous oxygenation should be about 25% less than arterial oxygenation. In this case, however, it is less than 50%. This suggests that tissues are consuming more oxygen which supports the conclusion that blood is flowing slower across the tissues. When blood flows slower across the tissues, it can be concluded that cardiac output is decreasing. Therefore, cardiac function should be assessed either by measuring cardiac output or by checking the cardiac index.

A respiratory therapist notices a widening black area between the visceral wall and the right lung periphery. The therapist should suspect (a)

A. developing pneumothorax. B. pulmonary contusion. C. growing cavitations. D. tension hemothorax. A. The growing dark area is an indication of a loss of adhesion between the lung and visceral wall (chest wall). This is the beginning stage of a pneumothorax.

Orders for short and long-acting bronchodilator medication administration by many physicians has been inconsistent and sometimes inappropriate for some patients. To best prevent this type of activity, the respiratory therapist should recommend

A. development of a bronchodilator therapy protocol. B. instructing pharmacy not to distribute medication until a respiratory assessment is done. C. determining which physicians are making such orders and report them to the department medical director D. restricting offending physicians from ordering bronchodilator therapy. A. In this question, there is an obvious that there is a broad problem among physicians to incorrectly order bronchodilator therapy. Although physicians could be spoken to individually, a more efficient method would be to create a bronchodilator protocol. This is a documented set of instructions that allows a physician to order bronchodilator therapy, without having to provide the specifics of how the therapy is given or what drug or dosage is used. This approach helps to standardize care and avoid problems with physician orders.

A patient involved in a bus accident has sustained multiple injuries, including a tibia/fibia fracture and a significant strike to the right thoracic cage, indicated by a bruised area over the right lung, which measures 8 inches by 6 inches. Six hours after the accident, the patient is beginning to demonstrate ventilatory distress and is complaining of sharp pleuritic pain on the right. Which of the following is most likely?

A. diffuse alveolar hemorrhage B. right-side pneumothorax (greater than 60%) C. pulmonary embolus D. pleural effusion D.

A patient is transferred to the post-surgical recovery unit after undergoing a partial lobectomy in the left lung. Three days after surgery, the patient is unable to reach their pre-operative incentive spirometry values, in spite of a report of feeling well with no significant pain. Which of the following could explain this observation?

A. diminished Hb level B. poor V/Q matching C. reduced total lung capacity D. increased FRC C.

One indication to change from administering a front door bronchodilator to a back door bronchodilator is

A. diminished lung sounds result in wheezing after the front door bronchodilator is administered B. nausea and anxiousness induced by the front door bronchodilator C. ineffectiveness of a side door bronchodilator D. the patient's heart rate does not increase with the front door bronchodilator B. Nausea, anxiousness, tingling of the digits, & excessive increase in heart rate are all examples of poor tolerance by front door (sympathomimetic) bronchodilators. It is appropriate to try either a side door or back door bronchodilator should poor tolerance occur. If diminished lung sounds result in wheezing after a bronchodilator is administered, it is an indication that increased air movement is occurring and the therapy should continue.

A patient receiving short-term mechanical ventilation in the post-op recovery room has an HME inline. Over the past 24 hours, the HME has become clogged twice with thick secretions. The respiratory therapist should

A. discontinue use of HME B. use a bubble humidifier C. install a new HME D. use heated humidification D. Even though it is appropriate to use an HME device for short-term ventilation, the presence of secretions is a direct indication to remove the HME device and switch to heated humidification.

A point-of-care blood gas analyzer produces results well outside the machines physiological reportable range. The respiratory therapist should

A. examine the patient for consistent clinical signs B. suspect a venous blood sample C. replace the sample cartridge D. perform a 2-point calibration D. When any blood gas analyzer produces erroneous results that are outside the reportable physiological range, the device requires calibration. These types of devices offer one-point, and two-point calibrations. In this case, a two-point calibration would be more appropriate so that the machine is calibrated to both the high and low aspect of the range.

Predicted Observed TLC (liters) 4.40 4.28 FRC (liters) 2.65 2.50 SVC (liters) 3.40 3.31 FEV1 (liters) 2.25 1.61 DLCO CO/min/mmHg 25 17 Which of the following represents the most appropriate interpretation of the preceding spirometry results?

A. emphysema B. ARDS C. pulmonary fibrosis D. asthma A. A slow vital capacity (SVC) is nearly normal, indicating the patient does not have a restrictive pulmonary defect. However, diminished flows found in the FEV1 indicate the patient is obstructive. When picking the most likely disease interpretation for this patient, we must pick an obstructive disease. The answers reveal two possibilities - asthma and emphysema. The key to differentiating between emphysema and other obstructive diseases is to look at the DLCO. Only emphysema has a poor DLCO. In this case, DLCO is less than 20 which is less than 80% of predicted, indicating the patient must have emphysema.

Which of the following conditions would benefit most from bronchoscopy?

A. emphysema B. foreign body aspiration C. chronic bronchitis D. ARDS B. A bronchoscopy will not help to resolve emphysema, chronic bronchitis, or adult respiratory distress syndrome. It is, however, helpful in removing a foreign body from the trachea or lungs.

The following flow volume loop is obtained from a 59-year-old farmer who works in a granary. Which of the following could represent the patient's diagnosis?

A. emphysema B. asthma C. chronic bronchitis D. pulmonary fibrosis A. The flow volume loop is short and fat indicating diminished flows but normal volumes. This is associated with patients who have obstructive lung defect and is commonly seen in patients with COPD. To find the correct answer, one must pick the disease that is obstructive. In this question in, emphysema is the only obstructive disease offered.

Which of the follow data, if observed, would indicate proper positioning of an oral endotracheal tube?

A. end of ET tube 2 cm below the aortic knob B. poor left chest symmetry C. radiologist reports the ET tube is 1 inch above the carina D. end of ET tube is 6 cm above the carina C. A properly placed endotracheal tube will cause even chest rise and symmetry during ventilation. ET tube markings will be in the low 20s, if an oral intubation. A chest x-ray will show the end of the tube 2 to 5 cm above the carina or level with the aortic arch. Thus, all options indicate inappropriate positioning of the endotracheal tube.

Which of the following emergency airway clearance techniques should be used on a female who is 7 months pregnant and who has complete airway obstruction by aspirated food?

A. endotracheal suctioning B. abdominal thrusts C. chest thrusts D. bronchoscopy C. Foreign body aspiration can result in partial or complete airway obstruction. If emergent, abdominal or chest thrusts are needed. In the case of advanced pregnancy, chest thrusts should be used.

Which of the following would NOT be included in the education process for a patient receiving instruction related to treatment of their asthma?

A. enforcement B. planning C. assessment D. evaluation A. Appropriate elements of an education plan include assessment, planning, and evaluation. Enforcement is not included.

While transferring a ventilator-dependent patient from bed to gurney, the respiratory therapist notices sudden resistance when manually ventilating a patient through a standard tracheostomy tube. The therapist is unable to pass a suction catheter. The therapist should NEXT

A. ensure adequate cuff inflation. B. provide ventilation by bag and mask. C. replace the tracheostomy tube. D. call for help. C. When a standard tracheostomy tube becomes clogged, the tracheostomy tube must be removed and replaced to ensure ventilation.

A home care patient calling in to the pulmonary rehabilitation center indicates that their oxygen concentrator is not operating even though the on/off switch appears to be in the ON positioned. The therapist should direct the patient to

A. ensure the machine is plugged in B. check flow rate C. discontinue oxygen use until technical help arrives D. increase flow rate A. When a home care patient complains that their oxygen concentrator is not working properly, the first step should be to get them to utilize a tank for oxygen while the source of the problem is determined. In this question that option is not offered. In this case we can have the patient perform some very simple troubleshooting. Ensuring the machine is plugged in is the most obvious and most appropriate. The other troubleshooting options are too complex for the patient and increasing the flow rate or discontinuing oxygen is also inappropriate with the order of a physician.

During a full cardiopulmonary arrest, the medical team is unable to initiate intravenous access for the purpose of epinephrine delivery. The respiratory therapist should recommend

A. establishing an indwelling arterial catheter for medication administration. B. instillation of the medication through the endotracheal tube. C. delivery of the medication through the oral gastric tube (OG tube). D. doubling the dose and deliver by the intramuscular route (IM). B. During a code, life-saving medications, such as epinephrine can be delivered through the endotracheal tube. Typically, the dose is doubled, the medication is instilled and then followed by vigorous manual ventilations by bag/valve.

A respiratory therapist has provided continuous aerosolized Albuterol to a patient in the emergency room for the last 90 minutes. The patient has bleeding gums. What would the respiratory therapist recommend now?

A. evaluate prothrombin time B. provide warm salt water for mouth rinse C. apply topical epinephrine to the gums D. discontinue Albuterol D. Bleeding from the mouth, nose, or gums is a rare but serious symptom of an allergic response to albuterol.

The disk fails to rise on a volume-oriented incentive spirometer in spite of a forceful exhalation into the device. The therapist should

A. examine the device for a leak B. switch to IPPB by mask C. switch to a flow-oriented device D. instruct the patient to inhale through the device D. With incentive spirometers, patients commonly confuse the maneuver and exhale into the device rather than inhale. When this happens the respiratory therapist must instruct the patient on the proper use of the device, which is to perform inhale deeply through the device.

A respiratory therapist is determining a patient's orientation to time, place, and person. Which of the following methods of communication should the therapist employ

A. examine the medical record B. speak to the patient's family and/or friends and visitors C. open-ended questioning D. pictograms C. When interviewing a patient to perform a clinical or historical assessment or to assess the patient's orientation and mental condition, open-ended questions should be asked. These are questions that may not be answered with a simple "yes" or "no". People with language barriers will often answer in the simple affirmative or negative even when they do not understand the question.

A 24-year-old firefighter is brought to the emergency department after being trapped for 10 minutes in a collapsed, burning building. The respiratory therapist notes marked inflammation around the face and oropharynx. Nasal hairs appear to be singed. The following arterial blood gas data is noted while the patient is breathing air: pH 7.52 PaCO2 25 torr PaO2 102 torr HCO3- 23 mEq/L BE 0 mEq/L The therapist should FIRST recommend which of the following?

A. hyperbaric oxygen therapy B. initiate aerosolized racemic epinephrine C. check COHb D. oral intubation D. A key concern for a patient who has had exposure to heated gases, such as might be seen in firefighters, is inflammation of the upper airway and potential complete obstruction. In this case, the presence of singed nasal hairs and inflammation visualize about the face and oropharynx is enough to suspect significant mucosal exposure to heated gases. Therefore, protection of the airway is appropriate through oral intubation.

A patient has an Hb level of 6.0 g/dL and the following blood gas results: pH 7.55 PaCO2 27 mmHg PaO2 110 mmHg HCO3- 24 mEq/L BE 0 mEq/L Which of the following is an accurate statement?

A. hyperoxemia is present B. hyperventilation secondary to hypoxemia is present C. metabolic alkalosis is present D. patient has respiratory acidosis B. The first thing we see on this patient is that the hemoglobin is low. When a patient's hemoglobin is low they suffer from hypoxemia regardless of their PaO2. In this case the patient's PaO2 is high. They are also over ventilating as a result of the hypoxemia that is present due to the low hemoglobin. The best interpretation of this condition is hyperventilation secondary to (caused by) hypoxemia.

Which of the following findings in a physical examination would indicate the need for an echocardiogram?

A. hypertension B. bilateral rales C. cardiac murmur D. digital clubbing C. The presence of a cardiac murmur upon auscultation would best indicate a need for further evaluation with echocardiography.

A patient has a pulmonary artery catheter in place. The dicrotic notch is visible on the PA waveform. The following data is obtained: PAP 22 torr CVP 10 torr PCWP 12 torr C.I. 4.0 L/min/m2 Which of the following is an accurate interpretation of this data?

A. hypervolemia B. hypovolemia C. pulmonary hypotension D. congestive heart failure A. In this problem all hemodynamic values are high. That means the right heart, the lungs, and the left heart are all OK. An elevation of all hemodynamic values indicate a hypervolemia.

Immediately after performing a 12-lead ECG, the respiratory therapist notices "flipped" T waves and elevation of S-T segments. The therapist should conclude the patient is experiencing

A. hypokalemia and ischemia B. premature ventricular contractions C. hyperkalemia and infarction D. myocardial ischemia and infarction D. When T waves on an ECG are flipped, or point in the opposite direction compared to P waves, cardiac ischemia is indicated. Furthermore, elevation of the ST segment is related to myocardial infarction. Therefore, myocardial ischemia and infarction appear to be present.

A patient in the cardiac intensive care unit has a Swan-Ganz pulmonary artery catheter in place. The following data is available: CVP 1 torr mPAP 10 torr PCWP 4 torr C.I. 1.7 L/min/m2 The therapist can conclude the presence of which of the following?

A. hypovolemia B. hypervolemia C. left sided heart failure D. increased right atrial filling pressure A. Examination of the hemodynamic values in this case show that all values are low. When all hemodynamic values are low, dehydration is present. Thus, hypovolemia is the best interpretation of this data.

A 60-year-old female receiving oxygen therapy by a nonrebreathing mask has the follow arterial blood gas results: pH 7.47 PaCO2 32 torr PaO2 50 torr HCO3- 24 mEq/L BE 1 mEq/L The respiratory therapist should document which of the following conditions in the patient's medical record?

A. idiopathic ventilatory failure B. compensated respiratory alkalosis C. refractory hypoxemia with mild hypocapnia D. intrapulmonary shunt secondary to hyperventilation C. This patient has profound hypoxemia in spite of maximum FIO2. This is known as refractory hypoxemia. Additionally, hypocapnia (reduced CO2) is present.

A patient has just self-extubated and is now experiencing inspiratory stridor. Vital signs are stable and SaO2 is 97% on a large-volume, non-heated aerosol device set at 30% oxygen. In addition to close monitoring, the respiratory therapist should also consider

A. immediately reintubating the patient B. examining the oropharynx with a tongue depressor C. administering aerosolized racemic epinephrine D. providing heated mist C. When a patient self-extubates, it is likely that the air in the endotracheal tube cuff was not evacuated. Pulling a tube with an inflated cuff can cause damage and inflammation of the vocal folds and upper airway tissues, resulting in inspiratory stridor. When this happens, the administration of racemic epinephrine is appropriate. If, however, inspiratory stridor is described as severe or marked, reintubation of the patient is indicated.

A COPD patient is receiving oxygen at 1 L/min when he was admitted to the hospital for suspected pneumonia. The physician orders a target SpO2 of 90%. The following clinical and laboratory data is obtained: RR 17 HR 101 BP 142/88 mm Hg SpO2 86% pH 7.35 PaCO2 68 torr PaO2 51 torr HCO3- 34 mEq/L BE +7 mEq/L The respiratory therapist should recommend which of the following FIRST?

A. implement NIPPV B. place a heated aerosol mask at FIO2 1.0 C. switch to an air-entrainment mask at 0.55 D. titrate the oxygen flow rate D. An SpO2 of 86% does not meet the physician's requirement to maintain oxygen saturation at 90%. Therefore, the titration of oxygen is required. Titration refers to a process of increasing or decreasing FIO2 in response to data to achieve a desired SpO2 or PaO2 value.

A left-to-right cardiac shunt, due to a leak in the septum wall of the heart, would be best indicated by an increase

A. in PCWP B. in mixed venous oxygen saturation C. in mPAP D. in arterial to venous oxygen content difference B. A left-to-right cardiac shunt would best be observed by examining the oxygen saturation of mixed venous blood.

Results of a quality control maneuver for a spirometer, using a 3.0 L calibration syringe, are as follows: Volume 1 Volume 2 Volume 3 3.05 L 2.98 L 3.01 L According to ATS Standards, the spirometer is

A. inaccurate B. not meeting ATS standards C. accurate and precise D. lacking in precision C. Results from the 3 L calibration syringe are within 2.85 L and 3.15 L and are therefore considered to be accurate. The results are also close together and are therefore considered to be precise. Results that are close together but are not within the proper range are considered to have precision but be lacking in accuracy.

A patient is receiving non-invasive positive pressure ventilation by mask with an IPAP of 16 cmH2O and an EPAP of 6 cmH2O. Corresponding blood gas data is as follows: pH 7.36 PaCO2 44 mmHg PaO2 62 mmHg HCO3- 23 mEq/L BE -1 mEq/L Which adjustment is most appropriate?

A. increase IPAP to 20 cmH2O B. increase EPAP to 10 cmH2O C. decrease EPAP to 4 cmH2O D. increase IPAP to 18 and EPAP to 8 cmH2O D. Arterial blood gases show a normal CO2, which means ventilation is appropriate. But, the patient is hypoxic. The primary control on the BiPAP machine that increases oxygen delivery is the EPAP setting. If EPAP is raised by itself oxygenation will likely increase but ventilation will be decreased inadvertently. This is because ventilation is determined by the distance between the EPAP and the IPAP settings. An increase in EPAP without an increase in IPAP decreases ventilation. The point here is that if you increase EPAP you must also increase IPAP by the same amount to keep ventilation the same.

A pressure-volume loop ventilator graphic shows no rise in pressure for the first 200 mL of delivered volume. The therapist should

A. increase PEEP B. decrease inspiratory flow rate C. increase inspiratory flow rate D. decrease tidal volume A. In this question the description of the pressure volume loop would indicate a flat bottom as manifested by no rise in pressure with the first 200 mL of delivered volume. We call this a "flat football". The solution is to increase PEEP to a level that the pressure begins to rise immediately as volume is introduced.

A pediatric male is receiving volume ventilation with an adult circuit. A Ventilator settings and arterial blood gas data are below: Mode SIMV FIO2 0.40 VT 250 mL Mandatory rate 14 Total rate 14 PEEP 5 cm H2O pH 7.50 PaCO2 30 torr PaO2 112 torr HCO3- 27 mEq/L

A. increase PEEP B. increase FIO2 C. decrease mandatory rate D. decrease tidal volume C. This patient is over ventilating and is over oxygenating. Of these two problems over oxygenation should be corrected first. However that option is not offered. Increasing FIO2 or PEEP would increase the problem of over oxygenation. The best option in this case is to decrease mandatory rate which will increase arterial carbon dioxide.

The respiratory therapist obtains the following blood gas data on a patient receiving mechanical ventilation in the assist/control mode at FIO2 0.6, PEEP 5 cmH2O and tidal volume of 450 mL. Corresponding arterial blood gas results are as follows pH 7.28 PaCO2 52 torr PaO2 45 torr HCO3- 24 mEq/L BE +1 mEq/L The therapist should recommend which of the following changes?

A. increase PEEP B. increase FIO2 C. increase alveolar ventilation D. switch to pressure-control mode C. Blood gases reveal poor ventilation as well as poor oxygenation. Ventilation should be addressed before oxygenation. To correct this, of the options given, alveolar ventilation should be increased. Other options either relate to oxygenation or the management of airway pressures.

A male patient who weighs 72-kg (170-lb) and is 5-ft, 4-in tall is receiving mechanical ventilator support on the following settings with the following corresponding blood gas values: Mode SIMV Mandatory rate 14 VT 400 mL FIO2 0.50 PEEP 5 cmH2O pH 7.32 PaCO2 47 mm Hg PaO2 70 mm Hg HCO3- 25 mEq/L BE +1 mEq/L The respiratory therapist should recommend:

A. increase PEEP to 8 cmH2O B. increase rate to 16 C. increase FIO2 to 0.6 D. increase tidal volume to 500 mL D. Arterial blood gases show mild hypoventilation and hypoxemia. Although increasing rate would correct ventilation, increasing tidal volume is most appropriate. Additionally, the CO2 is only off by a very small amount and a change in tidal volume is the better choice when making small changes in CO2 because it results in a smaller increase in mean airway pressure.

A COPD patient, orally intubated for bacterial pneumonia has been receiving volume-controlled ventilation for 15 days. Daily spontaneous breathing trials have not been progressing sufficiently toward cessation of mechanical ventilation. What may the respiratory therapist do to most effectively increase the patient's ability to wean from mechanical ventilation?

A. increase SBT intervals and duration B. decrease caloric intake C. insert a tracheostomy D. perform a BAL C. Insertion of a tracheostomy tube can lower airway resistance in a COPD patient just enough to decrease work of breathing and make it possible for ventilator liberation.

Which of the following benefits from continuous low-flow supplemental oxygen should be expressed to a patient with COPD who is participating in a pulmonary rehabilitation program?

A. increase ability to perform ADLs B. reverse lung disease C. return to normal life D. establish normal pulmonary function volumes A. When performing any kind of pulmonary rehabilitation or home care with the patient, it is important for the practitioner to understand they cannot reverse lung disease or return the patient to normal life. Patients with lung disease will never return to normal life because their condition is not reversible. Therefore, the appropriate goals and intentions of rehabilitation and education is to increase their ability to perform activities of daily living, generally increase exercise tolerance, reduce the incidence of infection and hospitalization, and generally improve their quality of life from their point of view.

A patient is receiving bi-level therapy to decrease work of breathing. Current settings and blood gas results are: IPAP 18 cm H2O EPAP 10 cm H2O FIO2 0.21 pH 7.32 PaCO2 48 torr PaO2 110 torr HCO3- 24 mEq/L BE 0 mEq/L The respiratory therapist should make which of the following changes?

A. increase both IPAP and EPAP by 2 cmH2O B. decrease EPAP to 8 cmH2O C. increase IPAP to 22 cmH2O D. increase EPAP to 12 cmH2O B. Blood gases reveal hypoventilation and over oxygenation. We can correct both of these problems with one change in the BiPAP settings. Because ventilation is determined by the distance between the IPAP and EPAP settings, we can simply lower EPAP. A decrease in expiratory positive airway pressure will decrease oxygenation but also increase ventilation because the distance between inspiratory pressure and expiratory pressure is increased.

A patient is receiving bi-level therapy to decrease work of breathing. Current settings and blood gas results are: IPAP 18 cm H2O EPAP 10 cm H2O FIO2 0.21 pH 7.32 PaCO2 48 torr PaO2 110 torr HCO3- 24 mEq/L BE 0 mEq/L The respiratory therapist should make which of the following changes?

A. increase both IPAP and EPAP by 2 cmH2O B. increase IPAP to 22 cmH2O C. increase EPAP to 12 cmH2O D. decrease EPAP to 8 cmH2O C. Moderate inspiratory and expiratory stridor may be treated with racemic epinephrine. This alpha-1 medication can constrict vessels and tissues and reduce inflammation. If stridor is considered to be severe or marked, racemic epi not the best choice. An airway should be established immediately. This action is preferred over racemic epinephrine because of the time it takes for the medication to have effect. Therefore, racemic epinephrine should be used in cases involving mild and moderate stridor.

A 74-kg (166-lb) 5-ft 8-in (173-cm) female is receiving mechanical ventilation by a volume-controlled ventilator in the assist/control mode on the following settings: Mandatory rate 14 VT 450 mL FIO2 0.5 PEEP 5 cm H2O ABGs pH 7.33 PaCO2 47 mm Hg PaO2 72 mm Hg HCO3- 23 mEq/L BE -1 mEq/L Which of the following represents the most appropriate action?

A. increase rate to 16 B. increase FIO2 to 0.6 C. increase VT to 550 mL D. add deadspace C. The PaCO2 is slightly high. One may reduce CO2 by doing one of three things - increasing rate, increasing tidal volume, or removing dead space. Increasing rate from 14 to 16 to blow off a few points of CO2 is too drastic and would increase mean airway pressure significantly. Adding dead space would increase CO2 further, which is a move in the wrong direction. The best option, therefore, is to increase the tidal volume to 550 mL.

A 75-kg (165-lb) male is receiving mechanical ventilation by a volume-controlled ventilator in the assist/control mode on the following settings: Mandatory rate 14 VT 550 mL FIO2 0.4 PEEP 5 cm H2O Arterial blood gas results: pH 7.32 PaCO2 47 mm Hg PaO2 101 mm Hg HCO3- 25 mEq/L BE +1 mEq/L Which of the following represents the most appropriate action?

A. increase rate to 18 B. decrease FIO2 to 0.3 C. increase VT to 700 mL D. remove deadspace D. Arterial blood gases show the patient is not ventilating and is under oxygenating. Because ventilation should be corrected first it is appropriate to either increase the tidal volume or increase the rate. In this case the PaCO2 is off by only 2 mmHg. A change in rate is too drastic. A change in tidal volume or a decrease in deadspace should be examined. The answers offered show an increase in tidal volume to 700 mL. This would exceed the maximum tidal volume range as determined by the patient's ideal body weight. Removing deadspace is the best option.

A 10-year-old child is receiving bronchodilator therapy by IPPB. The therapist notes the needle on the pressure manometer shows a significant negative deflection before each inspiratory cycle starts. Which of the following will help remedy this problem?

A. increase sensitivity B. decrease inspiratory pressure C. increase flow rate D. decrease flow rate A. When a significant negative deflection is seen on the needle during an IPPB treatment, the patient is having to work too much to trigger inspiration. The solution is to increase sensitivity.

A respiratory therapist is having difficulty effectively removing secretions through endotracheal suctioning of a 7-year-old child with cystic fibrosis. The ET tube size is 6.0-mm. The suction pressures is set at 75 mmHg and the suction catheter size is an 8 Fr. The therapist should do which of the following to increase the efficacy of suctioning?

A. increase suction during per pass B. increase suction pressure to 80 mmHg C. increase catheter size to 10 Fr D. instill 20 cc of normal saline prior to each suction attempt B. There are three ways to increase suction efficiency. In order: 1) increase suction catheter size 2) increase suction pressure 3) increase suction time. However, prior to any of these three one must first ensure adequate minimal pressure for the patient's age is being used. This is a pediatric patient and suction pressure range should be between 80 and 100 mmHg. However, the suction pressure is only set at 75 mmHg. So, this should be corrected first before considering any other action to improve suction efficiency.

While assisting a physician with a bronchoscopy, the respiratory therapist notices the bronchoscope is not applying suction even though the physician is depressing the suction button on the bronchoscope. The therapist should

A. increase suction pressure at the wall. B. flush the suction channel on the bronchoscope. C. change the suction tubing between the vacuum source and the suction canister. D. recommend a change to a new bronchoscope. B. The suction channel on a bronchoscope can easily become clogged. Flushing the channel would be an appropriate first step in resolving the problem.

A patient receiving volume-controlled ventilation has thick secretions that are difficult to suction. The patient has an 8.0-mm oral endotracheal tube and is being suctioned with a 10 Fr catheter at a pressure of 100 mmHg. Which of the following would be most helpful at increasing the efficiency of suctioning?

A. increase suction pressure to 120 mmHg B. increase suction time and frequency C. increase suction pressure to 110 mmHg D. switch to a 12 Fr suction catheter D. Increasing suction efficiency can be most improved by increasing the size of the suction catheter. However, the diameter of the suction catheter cannot be greater than one half of the internal diameter of the endotracheal tube. The patient currently has an 8.0 mm ET tube in. One half of 8 mm is 4 mm. We then must multiply it times three to obtain French units. 4x3 is 12 French. This means the maximum catheter size that may be used is 12. The current size being used is 10 French. This means we may benefit from increasing the catheter size from 10 to 12. This change is far more effective than increasing suction pressure or suction time.

The following arterial blood gas values are reported for a patient who is weaning from mechanical ventilation. There is no notable change in the patient's condition. 0800 hrs 1000 hrs pH 7.42 7.38 PaCO2 37 torr 32 torr PaO2 80 torr 41 torr HCO3- 26 mEq/L 26 mEq/L FIO2 0.40 0.40 Based upon this data, the respiratory therapist should

A. increase the FIO2 to 1.0. B. extubate the patient. C. increase the FIO2 to 0.50. D. repeat the arterial draw. D. The respiratory therapist should question all laboratory results to assure that they match the clinical scenario prior to reporting them. In this example, the patient's condition has not changed with the PaO2 of 41 torr. The sample might be a venous sample, and therefore should be redrawn.

An infant is receiving time-cycled ventilation through a 4.0 mm cuffless endotracheal tube. The respiratory therapist notices the ventilator is cycling properly but that the needle on the pressure manometer is not rising to the set pressure. The therapist should

A. increase the flow to the system B. increase the set peak inspiratory pressure (PIP) to compensate C. decrease the pressure pop-off limit D. replace the ET tube with a cuffed 6.0 mm tube A. Most time-cycled ventilators used on infants operate by having a continual flow through the machine. When the operating flow is insufficient, one observable characteristic is that the machine will fail to reach the set pressure.

During CPR, a self-inflating manual resuscitator fails to re-inflate properly between breaths. The patient has no spontaneous breaths, but has a pulse. In order to provide rescue breaths and resolve the problem, the respiratory therapist should

A. increase the flowrate. B. replace the PEEP valve. C. slow down the rate of manual breaths. D. replace the resuscitator bag. D. It is best to replace malfunctioning equipment, especially during an emergency. A respiratory therapist should avoid making repairs or applying work-arounds with equipment when the patient needs immediate care.

A respiratory therapist observes the following while reviewing the ventilator flow sheet on a patient receiving VC A/C ventilation: 1:00 PM 2:00 PM 3:00 PM PEEP (cm H2O) 7 7 7 Peak Pressure (cm H2O) 31 35 44 Plateau Pressure (cm H2O) 28 26 22 The therapist can conclude which of the following is present?

A. increasing airway resistance, increasing pulmonary compliance B. decreasing pulmonary compliance, increasing airway resistance C. alveolar collapse, decreasing static compliance D. pulmonary stiffening, decreasing dynamic compliance A. An increase in peak pressures suggests an increase in airway resistance, likely due to some transient issue such as secretions in the airway or bronchoconstriction. In addition to an increase in peak pressure, plateau pressures appear to be decreasing, suggesting a change in the stiffness of the lungs or an increase in pulmonary compliance.

The following information is available on the ventilator flow sheet of a 55-year old patient who is 3 days post appendectomy. Following the procedure, the patient sustained a pneumothorax and has been recovering in the intensive care unit. 8:00 AM 10:00 AM 12:00 PM Peak Pressure (cm H2O) 25 35 36 Plateau Pressure (cm H2O) 18 23 22 PEEP (cm H2O) 5 10 10 VT (mL) 500 500 500 The therapist can conclude which of the following is present?

A. increasing airway resistance, no change in pulmonary compliance B. decreasing pulmonary compliance, increasing airway resistance C. increased pulmonary elastance D. pulmonary stiffening, decreasing dynamic compliance A. In this data, increased airway resistance is manifested by an increase in peak pressure. It is tempting to assume decreased pulmonary compliance because plateau pressure is also rising. However when one looks closely, it is noted that PEEP has undergone a change during the study, which accounts for the change in plateau pressure. This will also accordingly change the peak inspiratory pressure but this data shows an increase greater than the amount of increase in the PEEP and therefore airway resistance is increasing.

A decreasing A-aDO2 is most closely related to which of the following?

A. increasing alveolar recruitment B. increase in tissue oxygen consumption C. refractory hypoxemia D. increasing pulmonary shunting A. When the gap between the alveolar oxygen tension and the arterial oxygen tension (A-a gradient) decreases, the patient is improving. The best interpretation or explanation for this, of the options given, is increasing alveolar recruitment. The other options are associated with a degradation of the patient. Increasing tissue oxygen consumption, for instance, is associated with a decrease in cardiac output. Refractory hypoxemia is not related. An increase in pulmonary shunting is associated with a widening alveolar-arterial oxygen gradient.

A 38-year old male is receiving ventilatory support by a high frequency jet ventilator (HFJV). Heart rate is 120/min and blood pressure is within normal limits. Arterial blood gas results on high frequency ventilation are as follows: pH 7.26 PaCO2 64 torr PaO2 82 torr HCO3- 26 mEq/L BE 0 mEq/L The respiratory therapist should recommend

A. increasing amplitude B. switching to pressure control ventilation C. increasing frequency D. switching to volume-controlled ventilation C. This patient, who is on a high-frequency ventilator, shows evidence of hypoventilation as manifested by high PaCO2. Oxygenation appears adequate. To correct the hyperventilation, the most appropriate action is to increase frequency. This is equivalent to increasing the mandatory rate.

An adult patient has a respiratory rate of 28, a pulse of 138, and a blood pressure of 85/65 mm Hg. Breath sounds are absent on the left and diagnostic percussion reveals hyperresonance on that same side. The respiratory therapist should FIRST recommend

A. insert a chest tube in the left chest B. insert a 14-guage needle into the left chest C. schedule a V/Q scan D. radiographic examination of the chest B. The data shown here, especially pulse and blood pressure, indicate an emergent condition. Hyperresonance and absent breath sounds is suggestive of a tension pneumothorax. Because the tension is likely building, the patient is experiencing hemodynamic degradation that is dangerous. This is manifested by the low blood pressure. This problem should be remedied in an emergent way. This can be done by inserting a 14-gauge needle into the left chest, in between ribs. This will allow immediate venting of air and minor amounts of fluid to be ejected from the lung space and will provide immediate hemodynamic improvement.

A patient who has a fenestrated tracheostomy tube that is configured for speaking, is in need of cardiopulmonary resuscitation. The therapist should FIRST do which of the following?

A. insert the inner cannula B. remove the inner cannula C. remove the cap D. inflate the cuff C. A fenestrated tracheostomy tube has two possible configurations. The first is a speaking configuration and the second is a positive pressure ventilation configuration. When transitioning from a speaking configuration to a configuration that allows positive pressure ventilation, one must be careful to do things in the right order. The proper order is one that allows the patient to breathe freely during the change. In this case, the first step to transition from a speaking configuration is to remove the cap.

A patient with an 8.0-mm fenestrated tracheostomy tube has been speaking and breathing spontaneously for the last hour. SpO2 has suddenly dropped below 92%. RR is 32/min, HR 129/min. In preparation for mechanical ventilation, the respiratory therapist should FIRST

A. insert the inner cannula. B. prepare an airway exchange catheter. C. replace the airway with a non-fenestrated type. D. remove the artificial airway. A.

An 81-kg (178-lb) adult male is orally intubated with a 6.5-mm endotracheal tube. The therapist reports difficulty removing secretions with the suction pressure set to 120 mmHg. The respiratory therapist will FIRST

A. instill Albuterol down the ET tube B. increase suction pressure to 130 mmHg C. decrease suction pressure to 100 mmHg D. switch to an 8.0-mm ET tube D. The first problem that should be recognized is that the patient's endotracheal tube is too small for their ideal body weight. And 81 kg male should have an endotracheal tube size of 8 mm to 8.5 mm. Having the proper endotracheal tube size will permit a larger suction catheter to be used. This will drastically improve the efficiency of suctioning.

Within 2 hours of abdominal surgery, a patient has a blood pressure of 70/45 mmHg while receiving 10 mcg/kg/min Dopamine HCL, IV. The patient has not awakened from anesthetics and is ashen in color. The respiratory therapist should suspect the patient

A. is in a state of shock B. is hemorrhaging C. has cor Pulmonale D. has left heart failure B. Low blood pressure, ashen color of the patient, and the fact that the patient has just been discharged for surgery, all indicate the likelihood of loss of blood. This is most likely occurring from internal hemorrhaging related to the surgical procedure. This patient should be stabilized with maximum supplemental oxygen and returned to surgery for exploration.

A 52-year-old patient is receiving VC SIMV ventilation due to myasthenia gravis. The patient has no history of pulmonary disease and is alert and oriented. The following data is observed: Mode SIMV Set Rate 6 Total rate 12 VT(spont) 425 mL PEEP 5 cm H2O FIO2 0.40 pH 7.42 PaCO2 38 torr PaO2 122 torr HCO3- 24 mEq/L BE 0 mEq/L MIP -42 cm H2O The respiratory therapist should recommend

A. institute PEEP B. increase FIO2 C. increase VT D. increase mandatory rate D. An examination of the arterial blood gas reveals that this patient is under-ventilating and is under-oxygenating. In such a case, ventilation should be addressed prior to oxygenation. Since carbon dioxide is too high and is off by more than 4 mmHg from the target, the most suitable action is to increase the rate. Increasing the tidal volume may also be helpful but the patient's current tidal volume is already near the top of the range. Additionally, adjustment of tidal volume should be reserved for changes of CO2 that are less than 4 mmHg.

A patient with COPD is found with shallow respirations and is disoriented to time and place. The patient is receiving oxygen by Venturi mask set at 45%. SpO2 is 99%. The therapist should first do which of the following?

A. institute mechanical ventilation B. switch to a nonrebreathing mask set at 12 L/min C. switch to a nasal cannula at 5 L/min D. decrease FIO2 to 0.28 D. A patient with COPD should not receive more than 28% oxygen, or more than 1-2 liters per minute by nasal cannula. Excessive oxygen delivery can result in suppression of the ventilatory drive, which can lead to disorientation, hypoventilation, and ventilatory failure. Oxygen saturation should be kept at or below 94%.

A respiratory therapist receives a phone call from a home care patient who is receiving oxygen by pulse-dose oxygen delivery system. The patient complains that she does not feel enough oxygen is coming out. The therapist should instruct the patient to

A. instruct the patient to call their physician B. switch the flow meter to continuous mode C. report to the emergency room D. discontinue oxygen use until a therapist arrives B. Any time a home care patient complains of not receiving enough oxygen, before troubleshooting or attempting to repair the situation, the patient should be instructed to switch to a different modality to ensure oxygen delivery. The patient should not be asked to find the problem or to troubleshoot the problem before switching to a modality that is working. In this case, a pulse-dose oxygen delivery device can be switched to continuous delivery mode.

A post-operative adult male patient has been instructed in the proper use of an incentive spirometer. He demonstrates a good technique, cooperates well, and has clear breath sounds. Inspiratory capacity is 75% of predicted. The flow indicator remains flat during inspiration. The respiratory therapist should

A. instruct the patient to take fast deep breaths. B. adjust the flow tubing. C. replace the spirometer. D. continue therapy. C. Even though a respiratory therapist may believe that malfunctioning equipment is working "well enough", it is never appropriate to continue using broken or malfunctioning equipment. In this case the incentive spirometer should be replaced because it is not working properly.

A 31-weeks of gestation infant is receiving mechanical ventilatory support with a time-cycled, pressure limited ventilator at the following settings: PIP 30 cm H2O Rate 24 FIO2 0.5 PEEP 2 cm H2O I-time 1.0 sec The respiratory therapist notices the pressure manometer is not rising to the set pressure during each inspiration. Which of the following is the mostly likely cause?

A. insufficient flow B. I-time is set to high C. rate is set too low D. PEEP is insufficient A. When the pressure manometer on a time-cycle ventilator is not rising to the set pressure, the most likely cause is that the operational flow of the ventilator is insufficient.

A respiratory therapist is having difficulty getting a 12 Fr. suction catheter to pass smoothly through an 8.0-mm ET tube. Which of the following is the most likely cause?

A. insufficient lubricant on the catheter. B. suction catheter is too large for the ET tube. C. suction catheter material is too flaccid. D. a sputum plug has formed the airway. A.

Which part of a pulmonary function testing machine is being evaluated when performing quality control with a 3-liter syringe?

A. internal electronic components B. pnemotachometer C. body box D. mouthpiece and circuit B. A 3-liter super syringe is used to calibrate the volume measuring devices on a pulmonary function testing machine.

A patient is receiving non-invasive positive pressure ventilator assistance at settings: IPAP 18 cm H2O EPAP 10 cm H2O FIO2 0.90 Arterial blood gases reveal pH 7.21 PaCO2 59 mm Hg PaO2 60 mm Hg HCO3- 25 mEq/L BE +1 mEq/L The respiratory therapist should recommend:

A. intubate and mechanically ventilate in assist/control mode B. increase EPAP to 12 and IPAP to 20 cmH2O C. increase FIO2 to 1.0 D. increase IPAP to 22 cmH2O A. In spite of receiving noninvasive positive pressure ventilatory support, this patient's blood gases are showing clear ventilatory failure. A pH of less than 7.25 in the presence of elevated CO2 is an indication of acute ventilatory failure. BiPAP is no longer sufficient. The patient is in need full mechanical ventilatory support.

A patient in the emergency department (ED) is breathing spontaneously after inhalation of kerosene. The following physiologic data is available: ABG pH 7.31 PaCO2 49 torr PaO2 74 torr HCO3- 23 mEq/L BE -2 mEq/L HR 110/min SpO2 92% The patient is lethargic but responds appropriately to commands. The respiratory therapist should recommend

A. intubating and beginning volume-controlled ventilation. B. placing the patient on a non-rebreather mask C. placing the patient on a heated, high-flow oxygen mask at FIO2 0.50 D. implementing non-invasive ventilation. D. Elevated PaCO2 and an uncompensated pH suggest that the patient is under-ventilating. Because pH is not below 7.25 (the threshold for ventilatory failure), they would likely benefit from a moderate amount of ventilatory support given through bi-level therapy (NIPPV) by mask.

A patient with asthma is taking Flovent TID. This medication A. is a decongestant. B. may cause yeast infections. C. should not be used with COPD patients. D. is effective for active wheezing.

A. is a decongestant. B. may cause yeast infections. C. should not be used with COPD patients. D. is effective for active wheezing. B. Flovent (fluticasone) is a corticosteroid used generally to prevent and relieve inflammation of airway walls. Inflammation is one of the components of asthma so corticosteroids are key in treatment. The frequency is twice per day or b.i.d. Flovent may cause Candidiasis, (oral yeast infection), which may be prevented by judiciously rinsing the mouth after inhaler use. Treat with Nystatin.

A 68 year-old female patient has the following pulmonary function values: % of predicted Actual value Fev1/FVC% 54% SVC 85% FEF25-75 75% FEF200-1200 79% DLCO 88% Which of the following could represent the patient's diagnosis?

A. kyphoscoliosis B. emphysema C. mesothelioma D. asthma D. When looking at pulmonary function data the primary purpose is to differentiate between an obstructive defect and a restrictive effect. To determine if the patient is obstructive, we must look at flows. To determine if they are restrictive we must look at volumes. In this case flows are obviously reduced with an Fev1/FVC% of 54%. A slow vital capacity shows the patient is not restrictive. When we look at the answers we see two obstructive diseases - asthma and emphysema. To determine which one is correct we must look back at the DLCO. Only emphysema has an abnormal DLCO. This patient has a normal DLCO which means the patient has asthma.

A chest radiograph that shows a tracheal tube that is not aligned with the spinal process is most indicative of

A. kyphoscoliosis B. improper positioning of the tracheal tube C. scoliosis D. improper patient rotation during the exposure D.

The addition of a 2.0 second inspiratory plateau facilitates the determination of which of the following physiological parameters?

A. laminar and turbulent airflow properties B. airway resistance C. pulmonary secretions D. static compliance D. The addition of an inspiratory hold or plateau helps to determine what is called the plateau pressure, which is used primarily to calculate and evaluate static compliance.

When evaluating a patient's pulmonary condition, the therapist should include which of the following in the assessment?

A. language skills and social status B. marital status and disease history C. insurance and disease history D. occupation and smoking history D. When evaluating a patient's pulmonary condition, occupation may reveal long-term exposure to pulmonary irritants. Smoking history would also be helpful. Marital status and language skills do not directly relate to a patient's pulmonary condition.

During the flow volume loop maneuver, the therapist notes a nearly round loop with virtually no inflection points. Which of the following may represent the patient's condition?

A. laryngotracheobronchitis B. laryngeal cancer C. obstructive pulmonary defect D. restrictive pulmonary defect B. A "round" shaped flow-volume loop is consistent with a fixed upper airway obstruction. Of the options offered laryngeal cancer and vocal cord paralysis would cause a fixed upper airway obstruction.

A respiratory therapist notes a blunted costophrenic angle in the lower right lung periphery when viewing a patient's PA chest radiograph. The patient is demonstrating ventilatory distress after being involved in a bicycle accident at high speed. What should the therapist recommend to further investigate the problem in the lower right lung periphery?

A. lateral decubitus chest radiograph B. AP chest radiograph C. BAL (bronchoalveolar lavage) D. ventilation/perfusion scan A. When suspecting a pleural effusion, the key radiographic tests is a lateral decubitus chest radiograph. A bronchoalveolar lavage is used to clean out the lungs and a ventilation perfusion scan will help determine blood flow adequacy and gas flow adequacy in the pulmonary vasculature and alveoli, respectively, and is not relevant in this case. Merely changing to an AP x-ray will not help diagnose a pleural effusion.

Excessive bubbling is noted in the water seal chamber of a chest tube drainage system. Which of the following could be the cause?

A. leak in the drainage tubing coming from the patient B. water seal fluid level is too high C. wall pressure is too low D. water seal fluid level is too low A. A leak in the drainage tubing coming from a patient's chest will cause the entrainment of additional air into the system which will show up as excessive bubbling in the water-seal compartment. Normally gentle bubbling should be observed. When excessive bubbling is present, a leak most likely exists somewhere between the waterseal compartment and the patient's lung tissue and may be caused from a perforation or hole in the lung tissue.

An adult male patient who is receiving volume-controlled ventilation has a chest tube drainage system in place with a chest tube in the right anterior lung. Suction pressure at the wall is set to 120 mmHg. The therapist notices profuse bubbling in the water seal chamber. The low volume alarm is sounding and the digital display on the ventilator indicates a return volume of 220 mL. Which of the following clinical conditions should the respiratory therapist suspect?

A. leak in the ventilator circuit B. perforation in the lung tissue C. leak in the tubing between the fluid collection and water seal chambers D. leak in the tubing between the patient and the fluid collection chamber B. Excessive bubbling in the water seal compartment, in addition to low return volume during mechanical ventilation, is an indication that a leak may be occurring through a perforation in the lung tissue - delivered volumes may be lost through a hole in the lung into the chest tube drainage system.

Auscultation of the chest on an adult patient intubated with an 8.0 mm endotracheal tube reveals diminished breath sounds on the left side. This is mostly likely caused by

A. left-sided fluid consolidation B. intubation of the right mainstem bronchus C. left-sided pneumothorax D. pleural effusion B. The most likely cause of breath sounds that are diminished only on one side, especially the left side, is most caused by inadvertent intubation of the right mainstem bronchus.

A physician has inserted a CVP line in a 78-year-old patient who has cor Pulmonale. The respiratory therapist should expect to see a placement radiograph that shows the end of the catheter positioned in which of the following areas?

A. level with the aortic knob or notch B. pulmonary artery C. right ventricle D. superior vena cava D. CVP is determined by measuring the pressure in the superior vena cava or right atrium.

A patient is receiving PC ventilation with the following data: 11 PM 4 AM Plateau pressure (cm H2O) 25 34 Peak pressure (cm H2O) 32 41 VT (mL) 500 500 PEEP (cm H2O) 5 5 This data indicates a

A. likely mucus plug B. decrease in pulmonary compliance C. decrease in dynamic compliance D. change in airway resistance B. This data shows increasing peak pressures as well as increasing plateau pressures. The plateau pressure appears to be rising at the same rate of the peak pressures, suggesting there is a decrease in pulmonary compliance.

A 52-year-old patient with polyneuropathy has received volume-controlled ventilation for 9 weeks. In an attempt to slowly wean the patient from dependence on mechanical ventilatory support, spontaneous breathing trials with a tracheostomy collar have been implemented over the last few days. The following data is available: 30 Minutes 1 Hour 2 Hours Pulse rate 89 98 119 SpO2 (%) 97 95 89 MIP (cm H2O) -30 -28 -18 Vital capacity (L) 1.7 1.6 0.8 Respiratory rate 20 24 32 The respiratory therapist should recommend

A. limit the next SBT to 1 hour in duration B. discontinue SBT trials for 3 days C. limit the next SBT to 2 hours in duration D. switch to a T-tube A. This patient is weaning from mechanical ventiltory support by undergoing spontaneous breathing trials. After a 30-min and 1-hour trial, the patient appears to be stable. However, after two hours, a significant degradation in the patient's status is noted. This would indicate that future trials should not exceed one hour.

The respiratory therapist is making a home visit to evaluate an oxygen-dependent patient. The patient has 3+ pitting peripheral edema and has not followed the physician's orders to limit fluid intake. You would expect which of the following hemodynamic values?

A. low mean arterial pressure B. elevated right sided preload with a normal to low right ventricular after load C. reduced cardiac index D. elevated PCWP with a low cardiac output B. Right-sided preload is another name for CVP (central venous pressure). Peripheral edema is one of the signs associated with right heart failure, which results in an elevated CVP with a normal or low pulmonary artery pressure (PAP), a.k.a. right ventricular afterload.

A patient with adult respiratory distress syndrome is receiving mechanical ventilation with a pressure-cycled ventilator. Which of the following alarms is most important?

A. low pressure alarm B. I:E ratio alarm C. high pressure alarm D. low-volume alarm D. A pressure-cycled ventilator delivers a breath at a specific set pressure in spite of the volume achieved. The machines focuses on pressure without regard to delivered volume. Therefore, the respiratory therapist should be focused on volume. The most important alarm on a pressure-cycled ventilator is a low return volume alarm.

If a patient's static compliance increases while receiving PCV, which of the following will also occur?

A. low-volume alarms would likely be triggered. B. tidal volume will increase. C. dynamic compliance will be unchanged. D. plateau pressures will increase. B. Pressure-control ventilation (PCV), does not have a constant set tidal volume. VT is a result of set peak inspiratory pressure and inspiratory time in this mode. If the lung has low compliance and is difficult to ventilate, the returned volumes will diminish. If the lung becomes easier to ventilate, the tidal volumes will increase.

An oxygen-dependent COPD patient is brought to the emergency room on 4 L/min oxygen by nasal cannula. From the paramedic documentation it is observed that respiratory rate has changed from 30 to 18 bpm. Heart rate has decreased from 120 to 90. Saturation is 92% and the patient's color has also improved. The therapist should do which of the following?

A. maintain current therapy B. discontinue oxygen therapy C. decrease flow to 1 lpm D. switch to a partial rebreathing mask C. When the COPD patient receives more than 2 L/min. oxygen by nasal cannula, or more than 28% by any other means, it is possible to suppress their hypoxic drive and reduce their total ventilation. In this question the patient has a decreased respiratory rate from 30 to 18. The solution is to lower the supplemental oxygen. An oxygen flow rate of 1 to 2 L/min. by nasal cannula is most appropriate.

Prior to providing manual resuscitation, the respiratory therapist tests the function of the flow-inflating resuscitator bag. The therapist notices the bag does not inflate during complete occlusion of the patient connector. The cause of the problem is most likely

A. malfunctioning blender B. open flow-control valve C. malfunctioning inlet valve D. insufficient oxygen flow B. If the bag does not inflate on a manual resuscitator, of the options given, there is most likely a problem with the flow-control valve. Essentially, gases must not be permitted to flow backwards when the patient-bag interface is occluded. This is especially true on a flow-inflated resuscitator bag.

Which of the following signs indicate the immediate need for intubation?

A. marked high pitched sound in upper airway B. barking cough in a toddler C. moderate use of accessory muscles D. moderate inflammation in the hypopharynx A. The word "marked" signifies a severe or emergent situation and must be responded to aggressively to establish an immediate airway. Subglottic edema seen in a toddler's neck x-ray indicates acute epiglottitis and also requires prompt intubation. In order to protect the airway of a drug overdose victim who is snoring with shallow respirations intubation should also take place. A barking cough by a toddler is more indicative of croup (larygotracheobronchitis), and rarely requires intubation.

Which of the following is a strategy that may help reduce or prevent autoPEEP in a patient with ARDS?

A. matching PEEP B. sedation of the patient C. increasing minute ventilation D. high flow rates D. The number one strategy to prevent autoPEEP from developing in a patient with noncompliant lungs is to decrease inspiratory time by using high flow rates.

The development of AutoPEEP is mostly likely caused by which of the following:

A. mechanical sighs B. excessive inspiratory flow rate C. insufficient inspiratory flow rate D. excessive expiratory time C. AutoPEEP develops because the patient has insufficient time to exhale. This may be corrected by decreasing inspiratory time. Inspiratory time may be decreased by increasing inspiratory flow rate.

A patient with a history of Status Asthmaticus arrives in the emergency room (ER) after having received multiple bronchodilator treatments. Paramedics report breath sounds have not improved. The physician orders supplemental oxygen. The respiratory therapist will recommend which of the following?

A. mechanical ventilator B. nonrebreathing mask C. Venturi mask D. nasal cannula B. The presence of status asthmaticus, specifically the lack of responsiveness to bronchodilator therapy, indicate the patient is in an emergent situation. Clinical emergencies involving the absence or reduction of the vital functions to life should be responded to by administering maximum FIO2 (100%). This can best be done by administering oxygen by a nonrebreathing mask.

Arterial blood gases on a patient in the cardiac intensive care unit are as follows: pH 7.19 PaCO2 30 mmHg PaO2 90 mmHg HCO3- 17 mEq/L BE -8 mEq/L Which of the following represents an accurate interpretation of these results?

A. metabolic acidosis B. respiratory compensated metabolic alkalosis C. compensated respiratory alkalosis D. respiratory acidosis A. When looking at a blood gas the first thing that should be looked at is the PaCO2. In this case the CO2 is low and therefore alkalosis should be present. However, when we look at the pH acidosis is present. This state cannot be caused by the CO2, or in other words by respiratory involvement. Respiratory acidosis can not be the answer. The source of the acidosis must be metabolic nature. When we look at the HCO3, we see that it is low, proving that the interpretation is metabolic acidosis. Incidentally, the CO2 is low because the body is trying to blow off the acidosis caused by low HCO3.

A COPD patient has a blood gas pH of 7.35, PaCO2 58 mmHg, PaO2 55 mmHg, HCO3- 28 mEq/L. Which of the following is most indicative of COPD?

A. metabolic compensation B. hypercarbia C. hypercapnia D. hypoxemia A. When we think of COPD patients, it is easy to think that the elevated CO2 is what defines their disease. While COPD patients are chronic CO2 retainers the real data that indicates they are COPD is the normalized pH. This indicates the chronic nature of their condition. In this question, the most appropriate answer is metabolic compensation because that is the means by which the pH is corrected.

What could cause a disparity between the SpO2 and actual SaO2?

A. methemoglobinemia B. elevated body temperature C. hypothermia D. low hemoglobin level A. Methemoglobinemia is an increased level of met hemoglobin, an altered form of hemoglobin that has less ability to bind with oxygen. In high enough levels, it can be deadly.

The following pulmonary function test results are reported for a 67-year-old female patient who weighs 75-kg (165-lb) and is 5-ft, 6-in (cm here) tall: % of predicted Actual Value FVC 76 SVC 78 Fev1.0/FVC% 75% Fev1.0 82 FEF 200-1200 79 FEF25-75 81 DLCO 20 Based on this information the patient has

A. mild restrictive defect B. mild diffusion defect C. moderate obstructive defect D. asthma A. A mild restrictive defect is indicated by an SVC of less than 80% of predicted. Because the Fev1/FVC% is at least 75%, and obstructive defect is not supported.

Due to an air leak, a respiratory therapist adds air to the cuff of an endotracheal tube inserted five hours prior in the emergency room. Cuff pressure following a minimal-seal technique is 32 mm Hg. The therapist should recommend

A. minimal-leak cuff management B. a larger tube C. a radiologic view of the tube D. advancing the tube 2 cm Because cuff pressure is higher than the limit after minimum-seal technique is used, the endotracheal tube is likely too small. B.

A respiratory therapist is monitoring a patient's maximal inhalation and exhalation repeatedly over a period of 15 seconds. The therapist is attempting to observe which of the following?

A. minute ventilation B. MVV C. alveolar ventilation D. inspiratory capacity B. Maximum voluntary ventilation (MVV) is determined by having the patient breathe maximally in and out for a period of either 12 seconds or 15 seconds and then multiplying that volume by five or by four respectively.

Which of the following types of indicators is used to show that a piece of nondisposable equipment did not go through a proper sterilization process?

A. misshapen equipment B. torn package C. chemical tape indicator D. biological spores indicator C. A chemical tape indicator found inside an enclosed package of nondisposable equipment can be used to determine if the equipment was processed through the sterilization procedure correctly. From this method, practitioners can peer into the package and determine the likelihood of a sterile condition. Keep in mind that the tape may only indicate if the process was followed but does not indicate the presence or absence of viable spores (living organisms).

Based on the pulmonary function testing below, which of the following represents the best interpretation of the results? Pre-bronchodilator Post-bronchodilator FEF 25-75% 80% 90% FEF 200-1200 81% 82% FVC 71% 75% MVV 74% 78%

A. mixed obstructive and restrictive defect B. normal spirometry C. mild obstructive defect, responsive to bronchodilator therapy D. restrictive defect D. Pulmonary function data shows normal flows. Volumes, however, are decreased. This indicates the presence of a restrictive defect.

A physician wants to check the accuracy of SvO2 readings from reflective oximetry for patient who has undergone open heart surgery for mitral valve replacement. Which of following blood samples would be most helpful and accurate?

A. mixed venous blood from the pulmonary artery B. venous blood from the right or left arm C. venous blood from the right atrium D. arterial blood from the femoral artery A.

In preparation for a patient who will be receiving oxygen by nasal cannula at 2 L/min at home, the respiratory therapist should recommend which of the following devices for primary oxygen delivery?

A. molecular sieve device B. bank and manifold of H tanks C. bulk liquid oxygen conversion system D. 3-5 E cylinders A. Patients who are in need of low-flow oxygen at home (between 1-6 L/min) are best served by using an oxygen concentrator. This is also called a molecular seive device.

During treatment of a patient who is demonstrating bilateral expiratory wheezing, the patient complains of nausea and tremors with beta-agonist medication. The respiratory therapist should

A. monitor carefully for the remainder of the treatment, notify the nurse and physician B. direct the patient to breathe at a slower rate C. immediately discontinue the treatment, document the finding, notify the physician D. reduce future doses to half-strength C. Nausea and tremors associated with the delivery of the beta agonist medication (like albuterol) is not considered a normal reaction, but is classified as an adverse reaction to the medication. As with any medication, when an adverse reaction is encountered, the very first step is to stop the therapy. Additionally the finding should be documented and the physician should be notified.

A patient presents in the emergency room with blood pressure of 95/60 mmHg. Skin turgor is poor and urine output is less than 10 mL/hr. Which of the following tests would be helpful in further evaluation of the patient?

A. multiple-wavelength spectrophotometry B. chest radiograph C. serum electrolytes D. arterial blood gases C. Serum electrolytes, especially potassium, can reveal the fluid status of the patient. Poor skin turgor and reduced urine output, as well as a low blood pressure all indicate a fluid status change in the patient. Serum electrolytes will help us to assess this problem.

A 33-year-old female has been rehabilitated from a motor vehicle accident where she suffered a spinal injury. She has been admitted to the emergency department with flu-like symptoms, is febrile, and is expectorating yellow sputum. Arterial blood gas results are: pH 7.27 PaCO2 52 torr PaO2 67 torr HCO3- 25 mEq/L BE -1 mEq/L The respiratory therapist should recommend

A. non-invasive positive pressure ventilation (NPPV) B. high frequency chest wall oscillation (HFCWO) C. intermittent positive pressure ventilation (IPPV) D. pressure support ventilation (PSV) A. Arterial blood gas reveal hypoventilation and hypoxemia. However, the patient is not quite yet demonstrating acute ventilatory failure. When CO2 rises and pH falls below 7.25, acute ventilatory failure is present and mechanical ventilatory assistance is indicated. In this case, the patient is just shy of acute ventilatory failure and may benefit from non-invasive positive pressure ventilation (NIPPV or BiPAP).

A patient in a body box performs a maneuver that shows airway resistance (Raw) to be 1.2 cmH2O/L/sec. Which of the following conditions most likely represents the patient's condition?

A. normal B. history of smoking C. ARDS D. myasthenia gravis A. An airway resistance of 1.2 cmH2O/L/sec is considered normal. Elevated airway resistance is associated with asthma.

While transporting at patient from the emergency department (ED) to the intensive care unit (ICU), the respiratory therapist suspects the oral endotracheal tube has changed position. The quickest way to assess the ET tube position would be to

A. obtain a chest x-ray B. visualize diaphragmatic excursion C. analyze end-tidal CO2 D. auscultate breath sounds D. There are many ways to determine the location or position of the ET tube. The question is asking for the quickest way. Of the options offered, auscultation of breath sounds would provide the most immediate pertinent data. If an option such as examination of equal and bilateral chest rise were offered, that option would be even better because it is visual in nature and therefore quicker.

A respiratory therapist notices the reservoir bag on a nonrebreathing mask collapses completely every time the patient inspires. The respiratory therapist should

A. obtain a larger mask B. switch to a high-flow Venturi mask C. switch to a partial rebreather mask D. increase flow to the mask D. When a patient is using a non-rebreather mask, the reservoir should collapse slightly with each breath. The collapse indicates that the seal between the mask and the patient's face is adequate. If the reservoir collapses completely, this indicates there is not enough flow of oxygen to the reservoir bag to keep up with the patient's inspiratory demand. Increasing flow to the reservoir, or mask, is most appropriate.

Immediately after placing a sterile tracheostomy tube in a patient who has maintained a stoma for 4 years, the patient coughs vigorously and expectorates the tube out onto the sterile napkin resting on the chest of the patient. The therapist should

A. obtain a new, sterile trach tube B. obtain a larger tracheostomy tube C. re-insert the expectorated trach tube D. immediately insert an obdurator into the stoma B. For a patient who has maintained an open stoma for several years and easily expectorates a new tracheostomy tube, the patient is most likely in need of a larger airway.

A patient with ARDS is receiving mechanical ventilatory support. An end-tidal CO2 monitor shows a PetCO2 of 59 torr. The therapist should

A. obtain an arterial blood gas B. increase minute ventilation C. recalibrate the capnography D. continue current therapy and ventilator settings B. Remember that the end-tidal CO2 data from a capnometer is usually 10 mmHg less than the corresponding arterial CO2 level. Thus, an end-tidal CO2 of 59 mmHg is consistent with an arterial CO2 of about 69 mmHg. The best solution to correct this problem is to increase ventilation.

A newborn has a one-minute APGAR score of 6. The respiratory therapist should

A. obtain an arterial blood gas B. monitor the infant, reassess in 4 minutes C. begin cardiopulmonary resuscitation D. administer supplemental oxygen D. An Apgar score can be used to determine the appropriate intervention of an infant without knowing the details of the score. A score from 0-3 indicates a need for cardiopulmonary resuscitation. A score from 4-6 indicates the infant requires supplemental oxygen, warming, and general stimulation. A score of 7-10 is normal in indicates routine care, which includes drying the infant and placing on a radiant warmer.

A newborn has a one-minute APGAR score of 6. The respiratory therapist should

A. obtain an arterial blood gas B. begin cardiopulmonary resuscitation C. monitor the infant, reassess in 4 minutes D. administer supplemental oxygen D. An Apgar score can be used to determine the appropriate intervention of an infant without knowing the details of the score. A score from 0-3 indicates a need for cardiopulmonary resuscitation. A score from 4-6 indicates the infant requires supplemental oxygen, warming, and general stimulation. A score of 7-10 is normal in indicates routine care, which includes drying the infant and placing on a radiant warmer.

A respiratory therapist observes a chest radiographic assessment that shows hyperlucency with no vascular markings on the right, and tracheal deviation to the left. There are no distinctive spaces between the vertebrae. The therapist should

A. obtain another chest-radiograph. B. recommend chest tube insertion on the left. C. order a CT scan of the spinal column. D. recommend chest tube placement on the right A. The lack of recognizable spaces between the vertebrae is an indication of a poorly exposed x-ray. Despite the obvious signs of a pneumothorax on the right, chest tubes should not be placed until confirmation by a properly exposed x-ray is obtained.

A neonate delivered 3 minutes ago has an APGAR score of 2. The therapist should FIRST do which of the following?

A. obtain umbilical cord blood gas values B. provide routine care C. begin cardiac compressions D. insert a chest tube C. Apgar scores of 4 to 6 indicate the administration of supplemental oxygen and general stimulation of the infant. Apgar scores higher than six require only routine care of the infant. Apgar scores of 0-3 indicate the need for CPR.

While auscultating the neck of a patient who is orally intubated, a slight high-pitched sound can be heard at the top of each inhalation. The therapist should respond by

A. obtaining a chest radiograph B. advancing the ET tube C. increasing cuff pressure D. replacing the ET tube C. The high-pitched sound auscultated over the trachea comes from air leaking around the ET tube cuff at the top of each breath. To prevent this, air must be added to the cuff, thus increasing the cuff pressure

The emergency room physician is concerned about a patient who was in involved in a multiple-vehicle accident. The patient is unconscious and has obvious signs of aspiration of gastric contents. A quick assessment reveals significant trauma to the neck and the area of the trachea. To protect the airway, the respiratory therapist should recommend which of the following?

A. oral intubation with a bronchoscope B. insertion of an oral airway C. insertion of a nasopharyngeal airway D. blind nasal intubation A. Because this patient is unconscious, and has obviously been vomiting, a primary concern is that of aspiration. This may be prevented by establishing an airway through oral intubation. Because the patient has obvious trauma to the neck and trachea this intubation would be considered complex and therefore should be done with the assistance of a bronchoscope or in surgery where an emergency tracheotomy could be done if needed.

A patient is sedated for a cardioversion and begins snoring. The respiratory therapist should

A. order a sleep study following the procedure. B. insert an oropharyngeal airway. C. check the pulse. D. assess breath sounds. B. Cardioversion should be done with resuscitation equipment available because the sedation may severely compromise a patient's airway. An oropharyngeal airway would prevent the snoring caused by soft tissue obstruction.

You are asked to instruct a patient being discharged home to use inhalers instead of small volume nebulizers. The patient is 8-years-old and was generally non-compliant with aerosol therapy during the hospitalization. Which of the following should NOT be included in the teaching plan?

A. order and timing of ordered medications B. MDI self-administration with a spacer C. instructions for the parents/legal guardians D. MDI self-administration without a spacer D. A spacer is indicated for a patient this age in order to reduce the need to rely upon coordination with an MDI, along with improving medication particle delivery. Including the family and taking multiple medications in their proper order are necessary points for teaching.

A 45 year-old patient is recovering in the intensive care unit after undergoing a surgical procedure to repair a hernia. The patient remains unconscious. To prevent soft tissue obstruction, the respiratory therapist should recommend a(n)

A. oropharyngeal airway B. double-lumen endobronchial tube C. bite block D. nasopharyngeal airway A. An oral pharyngeal airway is the primary mechanism used to prevent the time from falling back and obstructing the airway. Although a double lumen endobronchial tube would also do this, it is far too invasive and not necessary.

Quality control data plotted on a daily chart shows points above the mean followed by a group of points below the mean. All points are within 2 standard deviations from the mean. This data can best be categorized as

A. out-of-control B. a shift C. a variance D. a trend B. The quality control data plotted on a daily chart shows the ability for the blood gas machine to produce accurate results. If all points plotted are within the upper and lower control limits, the machine is said to be in control and may be used to report patient results. The closer the points are to the mean, or the middle, the more accurate and precise is the analyzer. The respiratory therapist must monitor these points plotted on a graph to ensure the machine's accuracy. If points on the graph are described as above the mean and suddenly below the mean, this is known as a shift. If points are described below the mean or above the mean and gradually move to the other side of the mean line, this is known as a trend. The machine must be monitored during trends closely to ensure it does not trend to an out-of-control state.

A 42-year-old patient who works in construction received a crushing injury to the chest during the movement of a large wooden beam. The respiratory therapist notes paradoxical chest movement on the right and generally labored respirations. The first step in caring for this patient will be

A. oxygen administration B. complete chest wall immobilization C. incentive spirometry D. insertion of chest tubes A. Paradoxical chest movement is associated with flail chest, or the breaking of one or more ribs in the rib cage. When this happens, that portion of the rib cage that is broken has a tendency to move in during inspiration rather than out. Thus, the movement is called "paradoxical". There are several methods to treat this situation but it is always appropriate to assure the patient's ventilation and oxygenation first. The most appropriate initial action is to administer oxygen.

During morning ventilator rounds, the respiratory therapist observes the following results of an arterial blood gas done 1 hour prior with the patient receiving volume-controlled ventilation. pH 7.53 PaCO2 39 torr PaO2 85 torr HCO3- 19 mEq/L BE +8 mEq/L FIO2 0.45 SaO2 97% The respiratory therapist should suspect an error in recording which of the following values?

A. pH B. BE C. HCO3- D. PaCO2 C. Because the arterial blood gas results show a PaCO2 of 39 mmHg but the pH shows alkalosis, the problem must be metabolic rather than respiratory. This would mean that the bicarbonate must be very elevated. However, when we look at the bicarbonate it is nearly normal at 26 mEq/L. This would suggest an error in the recording of the HCO3-.

A patient receiving mechanical ventilation has the following arterial blood gases: pH 7.29 PaCO2 55 torr PaO2 60 torr HCO3- 26 mEq/L BE 1.2 mEq/L After increasing minute ventilation, what new ABG would most likely be seen?

A. pH 7.32, PaCO2: 48 torr, PaO2 68 torr B. pH 7.25, PaCO2: 59 torr, PaO2 65 torr C. pH 7.34, PaCO2: 45 torr, PaO2 60 torr D. pH 7.30, PaCO2: 45 torr, PaO2 60 torr A. Increasing minute ventilation should reduce PaCO2, and increase pH. Because CO2 is an acid, the higher the level of the CO2 in the blood, the lower the pH. You will notice also that the PaO2 has increased. It is important to solve the problem of insufficient ventilation prior to insufficient oxygenation because as ventilation improves, oxygenation should also improve to some degree.

Which volume is recorded in the medical record after instructing a patient on the use of incentive spirometry?

A. pH 7.44 PCO2 38 mmHg PO2 82 mmHg B. pH 7.36 PCO2 42 mmHg PO2 79 mmHg C. pH 7.45 PCO2 37 mmHg PO2 81 mmHg D. pH 7.40 PCO2 40 mmHg PO2 78 mmHg C. The answer that shows a pH of 7.45 and a CO2 of 37 should be examined closely as these values appear to be outside the range of tolerance. If these results are accurate the arterial blood gas machine requires calibration and should not be used to report patient results until maintenance has occurred.

The following pulmonary function data is obtained from a 72-year-old male: Percent (%) of Predicted Actual value (%) SVC 81 FVC 79 FEV1.0 /FVC % 70 FEF25-75 69 FEF200-1200 63 DLCO 16 The respiratory therapist should interpret this data as

A. pulmonary emphysema B. asthma C. sarcoidosis D. severe diffusion impairment A. This pulmonary function data shows a slow vital capacity of 81% of predicted. This means the patient is not restrictive. However, the FEV1/FVC% is 70%. This is not a predicted value but rather an actual ratio. In other words, of the patient's vital capacity only 70% is exhaled in one second. A normal person should be able to exhale at least 75% of their vital capacity. This indicates the patient is mildly obstructive. The choices offered consist of two diseases that are obstructive - asthma and pulmonary emphysema. To distinguish between these two, one must look at the DLCO. Of the obstructive diseases, only emphysema has a poor DLCO. In this case, the patient's DLCO is 64% of predicted, proving that the patient has emphysema.

A respiratory therapist is reviewing the medical record of a 69-year-old patient admitted 2 hours prior. The record indicates the patient has chronic hypercapnia, emphysema, and an 80 pack-year history of smoking. Admission documentation shows the patient reported with moderate dyspnea. Currently, the patient is sleeping but arouses with significant verbal stimulation. The therapist should FIRST recommend

A. pulmonary function testing B. CO-Oximetry C. arterial blood gas analysis D. single-breath nitrogen elimination test (SBN2) C. An arterial blood gas will be most revealing in terms of assessing the patient's ventilation and oxygenation status.

The significant beak on a pressure-volume loop of a patient receiving volume-controlled ventilation is evidence of which of the following?

A. pulmonary over distension B. increased V/Q mismatch C. decreased venous admixture D. increased shunting A. A pressure volume loop on a ventilator that shows a significant beak indicates inappropriate ventilation. It indicates that a massive increase in pressure is required to deliver the final part of a tidal volume. This is undesirable because an increase in mean airway pressure has many negative affects on the patient, including decreased venous return, among other things. The solution is to sacrifice some of the tidal volume and therefore reduce pressure. A proper pressure volume loop will show a very tiny beak. This beak indicates the point of over distention of the lung and is a good indicator that we are filling the lung properly. Achieving this small beak will help to increase alveolar recruitment, aerate distal parts of the lungs, and help mobilize and remove secretions.

Fine crackles are auscultated at the bases of both lung fields. This is most consistent with the presence of

A. pulmonary secretions B. pleural friction rub C. atelectasis D. tuberculosis C. Fine crackles heard about the bases of the pulmonary tree are most closely associated with atelectasis.

Which of the following would be most helpful in determining if a patient regularly smokes tobacco products?

A. pulse oximetery B. end-tidal carbon dioxide C. DLCO D. COHb D. The most reliable data that indicates the degree to which the patient has recently been smoking comes from a COHb analysis.

Which of the following would be most helpful in continuously monitoring the arterial oxygen and carbon dioxide levels in a newborn?

A. pulse oximetry B. transcutaneous monitoring C. single-wavelength spectrophotometry D. arterial blood gas analysis B. The preferred method to provide continuous monitoring of arterial oxygen and carbon dioxide levels in a newborn is transcutaneous monitoring. This is a device that consists of a heated electrode that is placed on the skin of the infant. The electrode can detect minute perfused gases across skin and estimate PO2 and CO2. While it is not as accurate as arterial blood gas analysis, it does provide fairly precise data without removing precious blood volume from the infant.

Which of the following oxygen-delivery devices will most likely lead to an increase in infections?

A. pulse-dose B. reservoir cannula C. nasal cannula D. transtracheal catheter D. Because a transtracheal oxygen catheter is an invasive, internal device, the likelihood of infection is greater than any other device listed among the options.

Which of the following cannot be measured or evaluated in a comatose patient?

A. pupillary response B. tidal volume C. symptoms D. objective information C. If the patient is comatose, they cannot cooperate with procedures or follow commands. To answer this question one must pick the option that does not require compliance with verbal commands. Symptoms, coming from the root word sympathy, require the patient to tell you something. From the list only a tidal volume can be measured without help from the patient. But, the question is asking which of the following CANNOT be measured in a comatose patient. Therefore symptoms, vital capacity, maximum inspiratory pressure are all appropriate answers.

What is the primary purpose for a helium dilution test?

A. quantify alveolar deadspace B. determine FRC C. determine alveolar diffusion capacity D. directly measure TLC B. The purpose of the helium dilution test is to determine lung volumes such as TLC, RV, and FRC. On the NBRC exam the primary purpose of the helium dilution (and nitrogen washout) test is to determine FRC.

Three hours after endotracheal extubation, a patient demonstrates moderate stridor. Which of the following should the therapist administer?

A. racemic epinephrine B. viscous Xylocaine C. Albuterol sulfate D. topical Lidocaine A. Stridor is produced by inflammation in the upper airway and may be seen occasionally post extubation. The solution is dependent upon the degree of the stridor. Mild stridor may be treated by cool mist or racemic epinephrine. Moderate stridor requires racemic epinephrine at a minimum. Marked or severe stridor requires immediate intubation or other airway protection procedures.

An infant is delivered by C-section at 36 weeks of gestation. A maternal vaginal culture is positive for streptococcus. One hour after delivery, an umbilical artery blood gas reveals the following: FIO2 0.30 pH 7.44 PaCO2 38 torr PaO2 41 torr The respiratory therapist should prepare for

A. radial artery puncture B. intubation and time-cycled ventilation C. transcutaneous PO2 and PCO2 monitoring D. antimicrobial administration B. An umbilical artery PO2 of 41 is low, even compared to normal umbilical artery values. Although ventilation appears to be adequate, ventilatory support is indicated by the profound level of hypoxemia.

One hour after performing quality control on a blood gas analyzer, a respiratory therapist obtains arterial blood from a patient breathing supplemental oxygen at 2 L/min by nasal cannula. The following blood gas results are available: pH 7.41 PaCO2 39 torr PaO2 190 torr HCO3- 25 mEq/L BE -1 mEq/L The respiratory therapist should

A. recommend discontinuing supplemental oxygen based on these results B. perform a two-point calibration and repeat the analysis C. replace the Sanz electrode, repeat the analysis D. report the blood gas results B. An estimation of the alveolar oxygen tension reveals that on 2 L/min the maximum oxygen tension in the alveolo is about 147 mmHg. However, the oxygen tension in the blood is high at 190 mmHg, according to the blood gas results. This would be impossible. Arterial oxygen tension cannot exceed the maximum possible oxygen tension in the alveoli. The blood gas instrument should be recalibrated and the analysis should be repeated.

While performing minimal leak technique on a 70-kg (154-lb) male with a 7.0-mm tube, the therapist auscultates a high-pitched sound over the neck at the top of each mechanical breath. The therapist should

A. record results and monitor exhaled volume B. add air to the ET tube cuff C. reintubate with a larger ET tube D. remove air from the ET tube cuff A. When performing a minimum leak technique to establish proper inflation of the ET tube cuff, if properly performed, the practitioner should be able to auscultate a high-pitched sound over the trachea at the top of each mechanical breath. This sound is coming from a tiny amount of air passing around the cuff and is purposeful. Ensuring a leak is the best way to ensure minimal pressures are being placed on the trachea wall.

A patient in the intensive care unit has an arterial catheter in place. The respiratory therapist notices the blood pressure from the arterial line is 95/70 mmHg. Blood pressure taken by a sphygmomanometer reads 110/78 mmHg. The patient is asymptomatic. The therapist should

A. record the sphygmomanometer blood pressure B. discard the blood pressure cuff C. record the arterial line blood pressure D. calibrate the blood pressure cuff A. When blood pressure by sphygmomanometer (cuff) differs from the blood pressure obtained from an indwelling arterial line, the blood pressure taken by cuff is considered more accurate and reliable. Problems that may occur in the yard line include a clots in the line or bubbles in the transducer dome.

A patient with emphysema receiving volume-controlled ventilation has the following arterial blood results on the settings below: Mode Assist/control Mandatory rate 10 VT 450 mL FIO2 0.40 PEEP 5 cm H2O I:E 1:4 pH 7.29 PaCO2 65 torr PaO2 61 torr HCO3- 26 mEq/L BE +2 mEq/L Which of the following changes is most appropriate?

A. reduce inspiratory flow B. increase FIO2 C. increase rate to 12 D. maintain current settings C. This patient has both a problem with under ventilating and under oxygenating. Of the two problems under ventilation should be addressed first. This can best be done by increasing the mandatory rate. Reducing inspiratory flow will do nothing to change CO2 and increasing FIO2 does not address ventilation first.

A 75-kg (165-lb), 173-cm (5-ft 8-in) patient, who was intubated for accidental inhalation of kerosene, is on PC, A/C ventilation for the third week. The following is observed: FIO2 0.5 IP 30 cm H2O f 18/min PEEP 12 cm H2O ABGs pH 7.52 PaCO2 28 torr PaO2 80 torr Exhaled VT 755 mL The respiratory therapist should recommend

A. reduce inspiratory pressure. B. increase PEEP. C. change mode to SIMV. D. increase the rate. A. The central issue with this patient is the acid-base state of the blood gases. The elevated pH and low PaCO2 is consistent with hyperventilation. Additionally, exhaled VT is greater than 10 mL/kg. This is too high and indicates a need to lower ventilation. This can be done by decreasing inspiratory pressure, which will indirectly promote a reduction in tidal volume.

A patient receiving beta adrenergic medication complains of dizziness and tingling during the treatment. The respiratory therapist should

A. reduce the frequency of the treatment B. reduce the dose to sub-therpeutic level C. instruct the patient to pause the treatment for 1-2 minutes periodically D. switch to Xanthine-type medication C. Dizziness or tingling during a breathing treatment often comes from the excessive exhalation of CO2. The option that would reduce excessive discharge of CO2 is to instruct the patient to pause the treatment for a minute or two.

A patient's C(a-v)O2 has increased from 4.2 to 7.4 vol% following an increase in PEEP from 10 to 15 cm H2O. Which of the following best explains these data?

A. reduced cardiac output B. decreased ventilation C. alveolar overdistension D. increase cardiac index A. When the difference of oxygen in the artery compared to the oxygen in the venous system increases, it must be assumed that tissue oxygen consumption is increasing. Oxygen consumption at the tissue level will typically increase because of the speed of blood flow across the tissues. Since the speed of blood flow is shown through the metric of cardiac output, an increase in oxygen consumption at the tissue level must mean that there is a decrease in cardiac output.

Which of the following is a KEY indicator of hypovolemia?

A. reduced right atrial filling pressure B. reduced PCWP C. reduced PAP D. increased CVP A. Hypovolemia is indicated by a reduction in all hemodynamic values including CVP, PAP, PCWP, and cardiac output. However, of these four values CVP by itself is a primary indicator of the body's fluid status. CVP is known by many names, including right atrial filling pressure, right side preload, right ventricular end-diastolic pressure, and right atrial pressure. All of these terms are interchangeable with CVP. Dehydration or hypovolemia is indicated when these values are reduced. If CVP is increased, hypervolemia is present.

A 58-year-old patient with Crohn's Disease is receiving VC ventilation in the SIMV mode on the following settings: FIO2 0.50 PEEP 10 cm H2O (f) 12/min VT 550 mL Flow 35 L/min I:E 1:2.3 Laboratory values RBC 6.0 g/dL WBC 8,000 cu mm Hb 18.1 g/dL pH 7.46 PaCO2 34 torr PaO2 118 torr HCO3- 24 mEq/L SAT 0.99 The therapist should first consider

A. reducing PEEP. B. decreasing FIO2. C. increasing tidal volume. D. increasing the ventilator rate. A.

Immediately after the insertion of a nasopharyngeal airway (nasal trumpet), the patient begins to cough uncontrollably. The therapist should consider

A. reinserting the airway with a viscous lidocaine lubricant B. spaying the oropharynx with benzocaine C. replacing the airway with one that is shorter D. withdrawing the airway by 1 cm C. The patient's response to cough vigorously immediately after insertion of the nasal trumpet indicates the airway is likely too long. It should be replaced with a shorter one.

A respiratory therapist changes from a standard adult ventilator circuit to a heated-wire circuit. Arterial blood gases are as follows: pH 7.32 PaCO2 47 torr PaO2 78 torr HCO3- 24 mEq/L BE 0 mEq/L Which of the following changes is most indicated?

A. return to a standard circuit B. increase mandatory rate C. increase inspiratory flow D. add 50 mL deadspace B. In this blood gas the patient is demonstrating hypoventilation and hypoxemia. Of these two problems ventilation should be addressed first. Hypoventilation may be addressed by increasing respiratory rate, increasing tidal volume, or removing deadspace. Of the options offered, only an increase in mandatory rate will result in a decrease in arterial CO2. Use of a heated-wire circuit has no affect and increasing inspiratory flow rate will only change the I:E ratio. Adding deadspace will move CO2 in the wrong direction.

A patient with COPD has received mechanical ventilatory support for 2 days. Recent settings and clinical data include: Mode SIMV Mandatory rate 4 VT 500 mL FIO2 0.30 PEEP 5 cmH2O Total rate 12 VT (spont) 430 mL MIP -30 cm H2O The therapist should recommend:

A. return to full ventilatory support B. add pressure support of 10 cmH2O C. increase mandatory rate to 6 D. extubate the patient D. The patient in this question is obviously weaning. Because the rate is set to four. The patient appears to be adequately ventilating, the next step is to remove the patient from the ventilator. In this case, extubating the patient is the best option.

A patient is receiving volume-controlled ventilation and has a balloon-tipped pulmonary artery catheter in place. The following clinical data is available: 10 am 2 pm CVP (mm Hg) 4 5 PCWP (mm Hg) 12 28 C(a-v)O2 vol% 5 10 The patient is likely having a problem with the

A. right atrial filling pressure B. left heart C. right heart D. pulmonary system B. This hemodynamic data shows a drastic increase in PCWP and a decrease in cardiac output. The decrease in cardiac output can be assumed because of the increase in the C(a-v)O2. Remember, an increase in C(a-v)O2 is a direct indication of a decrease in cardiac output. The opposite is also true. Because cardiac output is decreasing and PCWP is increasing, a problem in the left heart is indicated. In other words, blood is having difficulty flowing freely through the left heart.

A respiratory therapist is performing diagnostic chest percussion on a 48-year-old male who is febrile. While percussing over the right chest, the therapist detects a dull note. Percussion of the left side reveals resonance. The therapist should suspect which of the following?

A. right-sided pneumonia B. pleural effusion C. pulmonary tuberculosis D. left-sided pneumothorax A. The presence of a dull percussion note on the right, combined with a fever, suggests pneumonia. A dull note is produced when the corresponding area of the lung is consolidated with secretions.

A respiratory therapist notices the dicrotic notch is missing from the pulmonary artery catheter waveform. After unsuccessfully attempting to resolve the problem by aspiration, the therapist should

A. rotate the catheter B. advance the catheter C. replace the catheter D. withdraw the catheter A. The absence of a dicrotic notch on a pulmonary artery waveform is an indication that the catheter is being occluded in some way. To correct this problem, the therapist should first aspirate the catheter. If this does not remedy pressure dampening, flushing the catheter is the next option. Finally, the catheter should be rotated. The only option given in this question that is appropriate is rotation of the catheter.

Which of the following conditions should result in patient instructions to decrease inspiratory time and increase expiratory time?

A. sarcoidosis B. COPD exacerbation C. asbestosis D. CHF B. A patient with COPD, chronic obstructive pulmonary disease, suffers with the inability to exhale gas from the lung. This results in air-trapping during a COPD exacerbation. Pursed-lips breathing, along with an extension of expiratory time is helpful. One may also decrease I-time or increase E-time to resolve air-trapping created by COPD.

A 22-year-old male patient with status asthmaticus is receiving VC A/C ventilation and is orally intubated with a 7.5-mm ET tube. The therapist notices a marked increase in peak airway pressures. BS are clear bilaterally but decreased on the right. The therapist should recommend

A. scheduling a ventilation/perfusion scan B. obtaining a portable chest radiograph C. performing a bronchoscopy D. inserting a chest tube in the right side, mid-clavicular line B. Decreased breath sounds on the right is most suggestive of a possible pneumothorax. A chest radiograph is most helpful in assessing for this possibility.

A spontaneously breathing, non-intubated patient has an appropriately sized oral pharyngeal airway in place. The patient repeatedly ejects the airway with her tongue. Which of the following actions is appropriate?

A. secure the airway to the patient B. sedate the patient with Versed C. use a smaller oral pharyngeal airway D. replace the oral airway with a nasal pharyngeal airway D. The patient who cannot tolerate an oral pharyngeal airway may be able to tolerate a nasal pharyngeal airway (a nasal trumpet).

A patient is showing signs of respiratory distress with a respiratory rate to 30/min and a spontaneous tidal volume of 800 mL. The patient is receiving oxygen at 60% by large volume nebulizer with the flow set at 10 L/min. The respiratory therapist should suggest to

A. sedate the patient B. decrease FIO2 to 0.50 C. increase FIO2 to 1.0 and the flow to 15 L/min D. increase flow to 15 L/min D. This question is about recognizing that the patient is not receiving adequate inspiratory flow, or that the inspiratory demanded the patient is not being met. Some simple math will show you that the patient's minute ventilation is higher than the total flow being received from the large volume nebulizer. 800 mL multiplied by 30 times a minute is equal to a minute ventilation of 24 L per minute. The ratio of air to oxygen at 60% is 1:1. 1 +1 = 2. Next, 2 x 10 L/min = total gas flow of 20 L/min., 10 L/min less than the inspiratory demand of the patient. Although the question does not ask this, the remedy for this situation would be to implement a tandem device, or another side-by-side large volume nebulizer. This would double the total gas flow to the patient and exceed the inspiratory demand, which is what we want.

A respiratory therapist is working in a 69-bed rural hospital with limited resources. In order to disinfect a re-usable plastic mouthpiece used for pulmonary function testing, the respiratory therapist should

A. soak in Cidex for 10 hours. B. soak in 60% alcohol for 10 hours. C. send for steam autoclave. D. soak in Sonacide for 30 minutes. A. Cidex (alkaline glutaraldehyde), will kill bacteria in 10 min, and everything in 10 hours. It works by using a caustic pH between 7.5 and 8.5. It should be changed every 14 days.

While administering postural drainage and percussion with the head of bed down 30 degrees, the therapist begins to note signs of distress including shortness of breath and PVCs. The therapist should immediately

A. sedate the patient, continue therapy B. perform postural drainage with the patient in prone position C. stop therapy, report findings to the physician D. raise the head of bed up by 15 degrees, continue therapy C. The first step in managing any complication with any procedure is to stop that procedure and report findings to the physician. Ultimately, the therapy must be modified. This means another therapy with the same objective must be implemented. Be careful not to modify a procedure in a dysfunctional way. For instance, if the patient requires their head of bed down 30° in order to drain the lower lobes, it would be inappropriate to raise the bed up to 15° to respond to their adverse reaction to the treatment. Raising the bed up to 15° would drain other areas of the lung. Therefore, it would be more appropriate to use something like PEP therapy, or vest therapy, or other airway clearance maneuvers.

A respiratory therapist is instructing a motorcycle accident victim who has no muscular activity below the neck and who is receiving ventilatory support through a tracheostomy tube. Which of the following airway clearance techniques should the therapist recommend?

A. self-administered IPPB B. vibratory PEP C. quad cough D. incentive spirometry C. A quad cough is a procedure that requires an additional person and consists of a coordination of the patient taking a deep breath and then exhaling while a helper compresses the chest during exhalation.

After a gastric bypass surgery, which of the following positions should the patient be placed in to facilitate effective ventilation?

A. semi-Fowler's position B. 1/4 turn to either side C. supine D. prone A. The lungs are most effectively ventilated when patients are in semi-Fowler's position. This, of course, only applies to patients who are in bed. Standing always provides the best gas distribution and ventilation of the lungs.

A 32-year old female, suspected of drug overdose, is on mechanical ventilation on the following settings: Mode SIMV Mandatory rate 16 Tidal volume 450 mL FIO2 0.40 Pressure limit 60 cm H2O I:E 1:1.5 The physician asks the respiratory therapist to make an adjustment to the ventilator to change the I:E ratio to 1:2. Which of the following will the therapist increase to comply with the request?

A. sensitivity B. flow C. mandatory rate D. base flow B. To increase expiratory time, inspiratory time should be decreased. This is done by manipulating the flow control on a ventilator, specifically by increasing the flow.

Which of the following pulmonary function test trails represents the best patient effort? Trial 1 Trial 2 Trial 3 FEV1 (L) 2.2 2.2 2.7 FVC (L) 2.7 2.7 2.3 IC (L) 2.4 1.9 2.1 FEF25-75% (L/sec) 1.8 1.3 1.3

A. set of trials do not meet ATS standards B. Trial 2 C. Trial 1 D. Trial 3 D. Determining the best patient effort in a pulmonary function test is done by adding the FEV1 and FVC. The trial with the highest sum is considered the "BEST TEST".

The following pulmonary function test results are reported for a 57-year-old male patient with a smoking history of 50-pack years. % of predicted Actual Value FVC 56 SVC 68 Fev1.0/FVC% 67% Fev1.0 62 FEF 200-1200 68 FEF25-75 68 DLCO 18 Based on this information the patient has

A. severe diffusion defect B. asthma C. emphysema D. restrictive defect C. This patient is demonstrating an obstructive defect as shown by an Fev1/FVC of 67%, which is less than the 75% required to be normal. Additionally, DLCO is also less than 80% of predicted indicating mild diffusion impairment. These two conditions, together, are associated with pulmonary emphysema.

A patient with asthma is taking Flovent TID. This medication

A. should not be used with COPD patients. B. is effective for active wheezing. C. is a decongestant. D. may cause yeast infections. D. Flovent (fluticasone) is a corticosteroid used generally to prevent and relieve inflammation of airway walls. Inflammation is one of the components of asthma so corticosteroids are key in treatment. The frequency is twice per day or b.i.d. Flovent may cause Candidiasis, (oral yeast infection), which may be prevented by judiciously rinsing the mouth after inhaler use. Treat with Nystatin.

Which of the following is a side effect of inhaled mucomyst (acetylcysteine)?

A. sickle cell anemia B. pernicious anemia C. rhinorrhea D. methemoglobinemia C.

A patient receiving volume-controlled ventilation is coughing uncontrollably. The high-pressure alarm is sounding continuously. The respiratory therapist should first

A. silence the ventilator alarm, calm the patient's anxiety B. sedate the patient C. provide the patient with manual ventilatory support D. suction the patient C. The first response to any ventilator alarm is to disconnect the ventilator from the patient and begin manual ventilation. Once ventilation and oxygenation of the patient are assured, troubleshooting for the source of the problem may begin.

A patient with acute epiglottitis is awaiting transport to surgery for a tracheotomy under anesthesia. SpO2 is 88% on room air. While waiting the patient would benefit most from which of the following?

A. simple mask at 6 L/min B. heliox therapy with 60%/40% mixture C. nasal cannula at 2 L/min D. high flow Venturi mask set at 50% B. The patient has an arterial oxygen saturation of 88% on room air. This is consistent with hypoxemia and indicates a need for supplemental oxygen. Because the patient has acute epiglottitis, Heliox therapy is appropriate to reduce airway resistance. Additionally, the patient would benefit from an adult therapeutic dose of oxygen, which starts at 40%. Therefore, 60-40% Heliox mixture is most appropriate.

A home care patient requires intermittent mechanical ventilatory support but is able to eat and speak when off the ventilator. Which of the following tracheostomy tube types is most appropriate for the patient?

A. single-cannula, cuffed B. bivona foam cuff C. single-cannula, uncuffed D. fenestrated D. For a home care patient, who does not require positive pressure ventilation constantly, a fenestrated tracheostomy tube will allow the patient to speak when not on the ventilator. A fenestrated tracheostomy tube has an inner cannula that may be removed and a hole in the outer cannula that allows gas to pass through the tube and over the vocal folds, permitting speech.

A 42-year-old patient has undergone a laryngectomy for the removal of laryngeal cancer. A laryngectomy tube is in place. For the first 12 hours after surgery, the therapist should expect which of the following?

A. slight hypoventilation B. atelectasis C. periodic expectoration of blood D. decreased RAW C. After the placement of a laryngectomy tube, the patient is most likely going to produce a clotted blood for several hours after surgery. This is considered normal.

The respiratory therapist observes a gradual increase in peak inspiratory pressure over the last 24 hours on a patient who is receiving mechanical ventilation. Additionally, the patient has become febrile and is producing purulent pulmonary secretions. The therapist should recommend which of the following?

A. sputum gram stain B. exploratory bronchoscopy C. bronchodilator therapy D. lung tissue biopsy A. The presence of fever and purulent pulmonary secretions suggests the possibility of an infection. A CBC would confirm the presence of an infection but a sputum gram stain would actually identify the class of offending pathogenic organism (gram-positive or gram-negative) Although not offered here, a more appropriate exam would be a sputum C&S which would provide specific organism information and suggest the most appropriate antimicrobial therapy. If a sputum gram stain is done and the class of organism is identified, a general class of antibiotics may be started until more specific pathogenic organism identification is done.

The result for a patient who has undergone a V/Q scan shows areas of the lungs where ventilation without perfusion is positively identified. This could be caused by a

A. sputum plug. B. pulmonary embolus. C. hemothorax. D. pleural effusion. B. Ventilation without perfusion suggests that gases are entering the alveoli at the lung periphery but are not being transferred adequately across the alveolar capillary membrane into the blood. This is mostly likely due to an absence of blood flow to the area, which could be caused by a clot in the pulmonary vasculature tree (an embolus).

A patient is demonstrating increased work of breathing and remains hypoxic in spite of supplemental oxygen by nonrebreathing mask. The patient is anxious. Which of the following should the respiratory therapist recommend?

A. start CPAP therapy B. non-invasive positive pressure ventilation (NIPPV) C. administer Ativan to the patient D. institute full mechanical ventilatory support B. Noninvasive positive pressure ventilation is often used to decrease work of breathing and to resolve hypoxemia by supplying continuous positive airway pressure.

While assisting a physician in a radiological cardiac catheterization procedure, the mechanical ventilator begins to alarm. From the control room, it is difficult to determine what alarm is sounding. The therapist's first action should be to

A. stop any procedure and manually ventilate the patient with a bag-valve B. enter the room with lead-lined protective clothing and troubleshoot the ventilator alarm C. watch oxygen saturation while the radiological procedure is occurring D. inform the physician of the problem and await recommendations A. When a ventilator alarm occurs during a radiological procedure, or any procedure that sequesters the patient, assuring the patient's ventilation and oxygenation status takes priority. The first action should be to stop any diagnostic testing and manually ventilate the patient.

A mechanically ventilated patient who with an 8.0 mm trachestomy tube in place has a strong cough, but is unable to expectorate thick, dry secretions. During an attempt to suction the patient, the tube becomes dislodged, but promptly placed back to its proper position. Following the procedure, the respiratory therapist notes dry crackles in the right upper lobe and neck upon auscultation. These breath sounds would most likely be associated with

A. subcutaneous emphysema B. retained secretions in the airway C. bleb emphysema D. pneumothorax A. In this scenario the respiratory therapist should assume that while the tube was dislodged, breaths delivered by the ventilator caused air to be forced into soft tissue areas under the skin outside of the trachea. This condition is known as subcutaneous emphysema. Auscultation reveals dry crackles and crepitus upon palpation.

To calculate alveolar minute ventilation, the respiratory therapist should

A. subtract anatomical deadspace from the VT, then multiply by the RR. B. measure with a metabolic cart study. C. multiply the RR by the VT. D. measure while the patient is receiving mechanical ventilation. A. Alveolar ventilation is calculated by subtracting the anatomical dead space from the tidal volume then multiplying by the respiratory rate. If the weight is known, the absolute value in pounds can be used to estimate anatomical deadspace. For example a patient weighing 130 pounds would have 130 mL of anatomical dead space. If weight is unknown, anatomical dead space should be estimated at 150 mL.

A patient has a sleep study (PSG) which shows an AHI of 50 and an average oxygen desaturation of 10%. Which of the following should the respiratory therapist recommend?

A. surgical insertion of a tracheostomy B. nocturnal supplemental oxygen C. CPAP/BIPAP nocturnal titration study D. weight loss C. A PSG is another name for a sleep study. AHI stands for apnea-hypopnea index. An index greater than 30 indicates severe sleep apnea. The gold standard treatment for this is nasal CPAP. But, a titration study should first be done to determine the appropriate level of positive airway pressure that should be prescribed for the patient.

An adult patient with asthma is receiving Xopenex by small volume nebulizer T.I.D. at a dosage of 0.63 mg. The patient complains of dizziness, tingling in his fingers, and anxiety with each treatment. The therapist should

A. switch to Albuterol B. switch to cromolyn sodium (Intal) C. switch to Atropine sulfate D. decrease dosage to 0.31 mg A. When the patient has an adverse reaction to a specific medication, the therapist must modify the therapy. Modifying therapy means to change to a different method or modality that accomplishes the same objective. In this case, switching to albuterol, which is another bronchodilator, is most appropriate. Decreasing the dose of Xopenex to 0.31 mg is inappropriate because that is a pediatric dose. Cromolyn sodium is not a bronchodilator and should specifically not be used when the patient has bronchoconstriction. Atropine sulfate, while it technically is a bronchodilator, is rarely used in this kind of situation.

While administering aerosolized bronchodilator therapy via mouthpiece to a patient in the emergency room, the respiratory therapist notices the patient repeatedly falls asleep and drops the nebulizer on to the bed. The therapist should

A. switch to IPPB with an inflatable mask B. stand at beside, hold the nebulizer and mouthpiece in the patient's mouth C. discontinue the therapy D. utilize an aerosol mask with an elastic strap D. One appropriate alternative to a nebulizer treatment with a mouthpiece is a mask that has an elastic strap. This will require no coordination by the patient.

A 24-year-old patient is receiving mechanical ventilation. The following pressure - volume graph is available. What can accurately be stated about this information?

A. this breath was patient-triggered B. inspiratory flow is set too high C. this breathing was machine-initiated D. sensitivity is set too high A. The presence of a "fishtail" on a pressure-volume ventilator graphic indicates the patient initiated a negative pressure prior to the machine administering the breath. This means the breath was patient-triggered.

A patient receiving volume-controlled ventilation shows a sudden decrease in cardiac output in response to an increase in PEEP from 25 cm H2O to 28 cm H2O. FIO2 is 0.60. After reverting to a PEEP of 25 cmH2O, the physician asks the respiratory therapist what should be done next to address the persistent hypoxemia. The therapist would recommend

A. switching to pressure-controlled ventilation B. utilizing an inverse I:E ratio breathing pattern C. reducing PEEP to 15 cm H2O D. increasing FIO2 D. When a patient is receiving high levels of PEEP, the hemodynamic stability of the patient should be monitored closely. In this case, an increase in PEEP resulted in a decrease in cardiac output. When this happens, it is paramount that the PEEP level be decreased to its prior setting. Beyond this, to address the patient's hypoxemia, FIO2 should be increased, even if it has to be increased above 60%.

Which of the following data, if observed, would indicate improper positioning of an oral endotracheal tube?

A. symmetrical chest rise B. ET tube markings at 29 cm C. chest radiograph shows tube 2 cm above the carina D. radiologist reports the ET tube is level with the aortic arch B. A properly placed endotracheal tube will cause even chest rise and symmetry during ventilation. ET tube markings will be in the low 20s, if an oral intubation. A chest x-ray will show the end of the tube 2 to 5 cm above the carina or level with the aortic arch. Thus, all options indicate inappropriate positioning of the endotracheal tube.

Which of the following defibrillator settings is appropriate for treating a patient with ventricular tachycardia?

A. synchronize to the P wave B. synchronize to the T wave C. synchronization set to OFF D. synchronize to the R wave C. When using a defibrillator, one setting that must be adjusted is the synchronization. When defibrillating a patient who has ventricular tachycardia, the synchronization must be set to OFF. Only when cardioversion is being done should the synchronization be set to ON or ACTIVE.

Which of the following clinical outcomes is a side effect of inhaled nitric oxide

A. systemic hypotension B. elevation of leukocyte count C. cardiac artery hardening D. contraction of smooth muscle tissue A. One of the side effects associated with inhaled nitric oxide is systemic hypotension. The purpose of nitric oxide is to dilate pulmonary vessels and relieve pulmonary hypertension. However, it also decreases systemic blood pressure. Therefore, systemic blood pressure must be monitored closely in patients who are taking inhaled nitric oxide

After transferring a morbidly obese patient with a tracheostomy, the respiratory therapist discovers the development of subcutaneous emphysema. Physical assessment findings most closely associated with this condition are

A. tachycardia with eupnea. B. auscultation of dry crackles and crepitus upon palpation. C. wet crackles throughout the lung. D. diminished breath sounds. B. The respiratory therapist should assume that the transfer of the patient resulted in the displacement of the tracheostomy tube, which then resulted in air being delivered outside of the trachea. The air has entered the soft tissues under the skin. This condition is known as subcutaneous emphysema, which produces dry crackles upon auscultation and a crepitus palpation.

A patient with asthma monitors their peak flow in the morning and documents that peak flow is 60% of his usual baseline. Based on the NAEP and the patient's asthma action plan, the patient should

A. take a short-acting beta 2-agonist, continue to monitor peak flow B. report to the hospital C. check peak flow again in 2 hours D. contact their physician A. According to the national asthma guidelines, a patient who is self-monitoring peak flow and is only able to achieve 60% of baseline should first take a short-term bronchodilator (a short acting beta-II agonist), and continued to monitor peak flows.

A patient is being weaned from mechanical ventilation after being placed on the ventilator 3 days prior due to complications from a pulmonary infection. Which of the following would be most helpful in determining the status of the infection prior to weaning?

A. temperature B. CBC C. ABG D. evaluate color of sputum B. To confirm the correction of an infection a complete blood count must be obtained. The complete blood count consists of a white blood cell count, among other data, that will show conclusive evidence regarding the presence and/or resolution of an infection. If the patient has an elevated white blood cell count ventilator weaning may not continue.

Prior to the removal of chest tubes, which of the following should be done?

A. the chest tube should be exposed to normal atmospheric pressure B. clamp the chest tube for 24 hours C. flush the chest tube with normal saline D. suction pressure should be increased temporarily B. There is a specific procedure that must be followed to discontinue chest tubes. Prior to removing the tubes the first step is to clamp the chest tubes for 24 hours and observe if drainage has adequately ceased

Parents of a newborn are complaining of sleep deprivation due to a frequently alarming apnea monitor at night. They indicate that when the apnea monitor alarms, the infant appears to have been breathing adequately. Which of the following should the respiratory therapist investigate?

A. the contact surfaces between the metal snaps on the leads and the electrode patch B. the parent's understanding of true apnea C. the internal battery level of the monitor D. the electrical outlet into which the monitor is plugged A. The most common source of erroneous alarming on an apnea monitor is the connection between the lead wire and the electrode patch. Often, the female portion of the snap does not maintain adequate contact with the male portion of the snap on the patch.

A patient is in the emergency room (ER) with an oxygen saturation of 86% on a non-rebreathing mask. The respiratory therapist observes paradoxical chest movement on the left side. Trachea is mid-line. Which of the following can the therapist conclude?

A. the patient has fractured ribs on the left side B. the patient has a pulmonary embolism C. the patient has a massive pulmonary shunt D. the patient has a partial pneumothroax A. Paradoxical chest movement is an indication that individual ribs are fractured in two or places, which causes the area of the rib cage to move in with inhalation while the rest of the chest moves out.

A patient in the emergency room shows signs of atelectasis as evidenced by scattered plate-like infiltrates in both lung fields. Which of the following would be most helpful?

A. thoracentesis B. IPPB treatment C. chest tubes D. corticosteriod therapy B. The primary treatment for atelectasis is positive pressure breathing. If the patient is on a ventilator, increasing PEEP would be the solution. Otherwise, achieving positive pressure through an IPPB treatment is appropriate.

The following airway graphic is observed for a patient receiving mechanical ventilation in the SIMV mode. Which is true concerning the ventilation of the patient?

A. the patient has triggered a mandatory breath B. the patient is receiving PEEP therapy C. the loop is indicative of bronchial cancer D. the flow-trigger sensitivity is set too low A. This pressure volume loop shows a "fishtail" pattern. This indicates a significant negative pressure deflection before the machine was triggered to administer a mandatory breath. Although it is normal to have some negative deflection of -1-3 cmH2O, this amount of deflection is too significant for the patient and will ultimately increase work of breathing. The presence of a "fishtail" is an indication that the patient has triggered the breath rather than the machine. If no fishtail exists, the suggestion is that the machine triggered the breath rather than the patient.

A respiratory therapist is monitoring a continuous flow CPAP device set at 8 cm H2O. With each inspiration from the patient the monitored pressure drops from 8 cm H2O to 3 cm H2O but then returns to 10 cm H2O during expiration. Which of the following is the best explanation for this observation?

A. the patient is in need of Bi-level therapy B. gas flow to the patient is insufficient C. leak in the mask D. normal operation B. CPAP is intended to deliver a continuous amount of positive airway pressure. When the pressure falls below the set pressure during inspiration, it is an indication that flow rate to the system is insufficient.

A patient is found sleeping in a car that has an apparent exhaust leak inside the vehicle cabin. COHb is 45%. Which of the following is true statement about this situation?

A. the patient smokes cigarettes B. the patient is hyperoxemic C. C(a-v)O2 is decreased D. CaO2 is decreased D. An elevated CO HB results in hypoxemia. But the hypoxemia is not derived from the PaO2 from the blood gas but rather is caused by the lack of hemoglobin available to carry oxygen. In fact, the patient may have plenty of hemoglobin but their oxygen-carrying capacity is diminished because the hemoglobin are bound with carbon monoxide. Consequently this results in a reduction of arterial oxygen content which results in hypoxemia.

During a cardiac stress test, the respiratory therapist notes no increase in heart rate or blood pressure after the incline of the treadmill has been increased. The patient appears to be pale in color. Which of the following can be concluded?

A. the patient's maximum exercise level has been exceeded B. the patient's minimum exercise level has been exceeded C. the treadmill should be inclined further D. the treadmill be declined by 5% A. Heart rate and blood pressure should increase during cardiac stress testing when workload is increased. Workload may be increased by inclining the treadmill. In this case, as workload is increased, the patient's blood pressure and heart rate have not responded. And, the patient is showing clinical signs of distress. This is evidence that the patient's maximum workload has increased above the patient's ability. This is also defined as the patient's maximum exercise level.

One important consideration when drawing an arterial blood sample is

A. the patient's potassium level B. the type of antibiotics currently used by the patient C. needle size D. heart rate C. There are many important considerations when drawing an arterial blood gas. However, of the options listed in this question, the best answer is needle size. Needle size is something the average respiratory therapist may not routinely consider, however, it is a basic recall matter that one should know.

A 48-year-old female completes the several FVL maneuvers. Both the expiratory and inspiratory side of the loops demonstrate a similar inverse pattern that is round in appearance. Which of the following could the respiratory therapist potentially conclude about this pattern?

A. the shape of the loop demonstrates normal pulmonary function B. a fixed upper airway obstruction is present C. a restrictive pulmonary defect is likely present D. an obstructive pulmonary defect is likely present B. In a normal flow volume loop, the inspiratory side of the loop (the topside), is vastly different than the bottom side of the loop. However, when upper loop resembles the bottom part of the loop in shape (this is often called a round loop) it indicates that the patient is having difficulty getting air in AND out. This is most commonly associated with a fixed upper airway obstruction or vocal cord paralysis or cancer.

A respiratory therapist is asked to assist in the development of a teaching plan for a 7-year-old asthmatic patient at home. The plan should avoid instructions regarding

A. the water-float technique to determine MDI canister content B. the proper technique for the use of MDIs C. improving the patient and family's understanding of infection control D. daily monitoring of peak-expiratory flow rates A. Proper MDI technique, peak-flow monitoring, and improving the understanding of infection control are all appropriate aspects of a teaching plan for a pediatric patient. Determining MDI canister content by floating the canister in water is never appropriate. The proper way to assess content is by actuating the MDI and observing the output.

A pulmonary rehab patient has the following arterial blood gas on air: pH 7.37 PaCO2 50 torr PaO2 62 torr HCO3- 30 mEq/L SaO2 87% COHb 6.5% What should the respiratory therapist suspect about this patient?

A. there is no long-term lung damage B. rehab is effective C. rehab should be discontinued D. recent exposure to cigarette smoke D. Because COHb is increased (more than 1-2%), the patient should be suspected for recent cigarette smoking.

A 38-week gestational age infant is receiving supplemental oxygen by oxyhood. An air/oxygen blender is set at 40% and the heated large volume aerosol is set at 100%. A capillary blood sample reveals the following values: pH 7.45 PcCO2 35 torr PcO2 47 torr HCO3- 22 mEq/L BE -2 mEq/L SpO2 is reading 97% and the patient appears to have good color. The therapist should conclude which of the following?

A. these are normal findings B. the air/oxygen blender should be set to 100% C. FIO2 should be increased D. nasal CPAP of 4 cmH2O should be implemented A. Infants have the same blood gas values as adults except for the PaO2. Close examination of the data provided shows that arterial CO2 and arterial oxygen is not what is being reported. The small "c" noted before the CO2 and O2 level indicate the blood was taken from the capillaries. When this is the case it must be remembered that capillary CO2 and the corresponding pH may be trusted but capillary oxygen levels are not accurate and may not be used. Therefore, this capillary blood gas reveals normal levels in term so ventilation. But the capillary oxygen data must be ignored entirely.

A respiratory therapist is assisting a patient who has an SpO2 of 85% while receiving oxygen by nonrebreather mask at 12 L/min flow. Laboratory results show a WBC of 10,000 cu mm and an Hb of 8.1 g/dL. The patient has a DNI order and an advanced directive that indicates no blood administration. Which of the following can be done to maximize arterial oxygenation?

A. turn the flow to the nonrebreather to flush B. one unit of packed RBCs C. NIPPV D. installation of a tracheostomy C. On this patient who has orders indicating 'DO NOT INTUBATE' and an advanced directive that includes no blood administration, the best and most effective way to increase arterial oxygenation is through the use of positive airway pressure. In this case NIPPV (noninvasive positive pressure ventilation). Installation of a tracheostomy is considered a form of intubation and would violate the patient's wishes. Administration of RBCs is a type of blood product and would go against the patient's advanced directives.

Documentation of inhaled Xopenex will include which of the following units of dosage?

A. ug B. uL C. mL D. mg D. Xopenex is a bronchodilator in which dosage is expressed in milligrams. Common dosages includes 1.25 mg, 0.63 mg, and 0.31 mg (pediatric dose).

What type of procedure is the suctioning of blood-tinged secretions from a patient with an artificial airway?

A. universal B. benign C. sterile D. clean C. All suctioning of a patient's airway is done with sterile technique, regardless of the type of secretions removed.

A patient who has been receiving negative pressure ventilation at home is admitted to the intensive care unit, intubated, and placed on positive pressure ventilation. What should be monitored closely as a result of the positive chest pressure?

A. urinary output B. electrolytes C. BUN D. glucose A. As positive pressure ventilation increases, venous return may be decreased. This may result in a decrease in urinary output. Therefore monitoring urinary output is appropriate when concerned about the negative effects of positive pressure ventilation.

A 62-year-old patient is brought to the emergency room (ER) from a local nursing care center where the patient has been in a vegetative state for 2 years. He routinely receives supplemental oxygen by trach collar and cool aerosol at 30%. He has a HR of 120 and an SpO2 of 83%. During a suction attempt, the therapist is unable to pass a suction catheter down the tracheostomy tube. What should the therapist do FIRST?

A. use a stiffer suction catheter B. switch to a nonrebreather mask C. replace the airway D. switch to an air-entrainment mask at 50% C. The inability to pass a suction catheter down the tracheostomy tube is suggestive of an obstruction. In this kind of circumstance, the respiratory therapist should immediately remove the obstruction or remove the airway. The use of a suction catheter should be primarily used to clear the obstruction. In this case, the therapist is unable and therefore should act more aggressively to secure a patent airway by replacing the airway altogether.

While suctioning a patient through the inner cannula of a 7.0-mm fenestrated tracheostomy tube, the therapist observes a foul order from the collected sputum. With which of the following is this most consistent?

A. use of Pulmozyme (Dornase alpha) B. pseudomonas C. bronchiectasis D. increased eosinophils B. A foul, and sometimes described as sweet, odor of sputum is associated with a Pseudomonas infection.

A 75-kg (165-lb) patient with mycoplasma pneumonia is receiving mechanical ventilator support by a PB 840 ventilator on the following settings with corresponding arterial blood gas values: Mode assist/control Mandatory rate 10/min VT 525 mL FIO2 0.6 pH 7.29 PaCO2 51 mmHg PaO2 78 mmHg HCO3- 23 mEq/L BE -1 mEq/L Which of the following actions is most appropriate?

A. use of expiratory retard B. decrease in inspiratory time C. decreasing inspiratory flow D. institution of a 1.0 sec inspiratory plateau B. Decreasing inspiratory time will lower the amount of time a patient is exposed to positive pressure and will therefore result in a decrease in mean airway pressure. Use of expiratory retard, increasing inspiratory flow, and institution of an inspiratory plateau will all lead to increased mean airway pressure.

Which of the following is most helpful at decreasing incidence of ventilator-associated pneumonia?

A. use of minimal leak technique for ET tube cuff inflation B. use of an HME device C. periodic discontinuation of sedation D. position the patient in prone position C. The incidence of ventilator-associated pneumonia, or VAP, is lowered by using a closed system suction catheter, periodically discontinuing sedation, keeping the patient and semi-Fowler's position, and proper handwashing among caregivers. Of the options given the most helpful item to decrease ventilator associated pneumonia is periodic discontinuance of sedation. This allows the patient to awaken, to move around, and to indirectly mobilize secretions.

A 12-year-old child is receiving IPPB therapy by mouthpiece. The machine continually fails to cycle on both the inhalation and exhalation phases. The first step the therapist should take to resolve the problem is to

A. utilize nose clips B. decrease sensitivity C. decrease inspiratory pressure D. switch to a mouth seal A. Patients who received IPPB therapy by a mouthpiece can sometimes have a tendency to leak at the corner of the mouth during a positive pressure breath. The solution for this is to switch to a mask. However, prior to trying a mask, it is possible the patient is leaking through their nose. Therefore, an easier solution, if offered, may be to try nose clips. This is especially true of pediatric patients who often have not learned how to close their nose internally.

Which of the following is NOT part of the suction procedure of an orally intubated patient receiving volume-controlled ventilation in the assist/control mode

A. utilize sterile technique B. switch to SIMV mode C. hyperoxygenate prior to suctioning D. hyperventilate the patient through the ventilator B. Normal suctioning procedure includes hyper oxygenation and hyperventilation of the patient prior to suctioning. A sterile technique is also used. Switching to an SIMV mode is not helpful.

A 22-year-old patient with bronchiectasis will be attending college away from home and family. Due to significant daily expectoration of pulmonary secretions, the patient is concerned about maintaining adequate pulmonary hygiene while away from family support. The respiratory therapist should recommend

A. vibratory PEP therapy B. IPPB with compressed air C. cromolyn sodium inhalation daily D. Spiriva (tiotropium bromide) Inhale A. For a patient who will be traveling away from home and away from family support and other caregivers, the performance of bronchial hygiene is a concern. Because the patient will not be able to rely upon others for such procedures as chest percussion, the patient should be given the ability to provide his or her own bronchial hygiene independently. Additionally, expensive equipment may be inappropriate. Therefore, of the options listed the use of a hand-held vibratory PEP device would be most appropriate.

A 22-year-old patient with bronchiectasis will be attending college away from home and family. Due to significant daily expectoration of pulmonary secretions, the patient is concerned about maintaining adequate pulmonary hygiene while away from family support. The respiratory therapist should recommend

A. vibratory PEP therapy B. Spiriva (tiotropium bromide) Inhaler C. cromolyn sodium inhalation daily D. IPPB with compressed air A. For a patient who will be traveling away from home and away from family support and other caregivers, the performance of bronchial hygiene is a concern. Because the patient will not be able to rely upon others for such procedures as chest percussion, the patient should be given the ability to provide his or her own bronchial hygiene independently. Additionally, expensive equipment may be inappropriate. Therefore, of the options listed the use of a hand-held vibratory PEP device would be most appropriate.

Over the last 24 hours, the respiratory therapist notices that plateau pressures on a patient receiving PC, A/C ventilation have decreased from 40 to 34 cm H2O. What can be concluded from this change?

A. worsening of a restrictive pulmonary defect B. a decrease in pulmonary shunting C. increased airway resistance D. increased pulmonary compliance D. A decrease in plateau pressures suggests that the lungs are becoming more compliant, and thus an increase in pulmonary compliance. A restrictive defect is related to decreasing pulmonary compliance. Shunting cannot be directly determined through plateau pressures alone and airway resistance is related to peak pressures, which are not present in this case.


संबंधित स्टडी सेट्स

Objective 3 Describe the origin and production of the formed elements in blood.

View Set

QUESTIONS Chapter 4 Carbohydrates

View Set

The Ultimate Psychology Guide (All in one)

View Set

Anatomy Test Chapter 7 (Nervous System)

View Set

Ch. 11 Test C (Forensic Science)

View Set

Medsurg 1 Module 3 Exam-Musculoskeletal Questions

View Set

Business Finance Chapter 6 Learnsmart

View Set

APUSH ch 12 quiz, APUSH Ch 13 quiz, APUSH Ch 14 quiz, APUSH ch 15 quiz

View Set